2ちゃんねる ■掲示板に戻る■ 全部 1- 最新50    

■ このスレッドは過去ログ倉庫に格納されています

スレッドを立てるまでもない質問スレッド Part 368

1 :名無しさん@英語勉強中 (ワッチョイW 2386-ZRN2):2023/10/01(日) 14:31:49.08 ID:V70/XNkx0.net
■ 質問する方へ
・常識的な内容なら何を質問してもOKです。
・できるだけ分りやすく、具体的に質問して下さい。ソースがあれば必ず書いて下さい。

■ 回答する方へ
・YAHOO!知恵袋のコピペを使った嫌がらせが後をたたないので、回答の前に、
 知恵袋に同じ質問が出ていないか、質問文の中のワードでググってみたほうが賢明です。
・回答する方は、質問者の立場になって答えてあげましょう。

■ 辞書
・単語や熟語の意味なら辞書で調べた方が速くて正確です。

 無料で引けるオンライン辞書(英英辞典) - おさんぽ英語 おすすめ散歩道 https://osanpo-english.com/site-book/online-dictionary.html

※前スレ
スレッドを立てるまでもない質問スレッド Part 367
https://lavender.5ch.net/test/read.cgi/english/1694329122/

2 :😷英ちゃん🤧 (ワッチョイW 2386-ZRN2):2023/10/01(日) 14:35:41.14 ID:V70/XNkx0.net
あげる!

3 :😷英ちゃん🤧 (ワッチョイW 2386-ZRN2):2023/10/01(日) 14:40:27.78 ID:V70/XNkx0.net
前スレ986の自称語源マニアの虚言がひどいので再掲いたします。参考URLは規制のため割愛いたします。
>いかにも雷が鳴っている様子をそのまま onomatopoeia としてしまったということがまるわかりです。

日本語で雷の音は「ドーン!」て表すからDonnerは雷の音だ!
学のない人の語源俗解ですねw

E thunder, G Donner > PG *þunraz > PIE *(s)ten-, *(s)tenh₂-

次の語と同様の音韻対応がある。
E three, G drei, > PG *þrīz > PIE *tréyes

4 :😷英ちゃん🤧 (ワッチョイW 2386-ZRN2):2023/10/01(日) 14:41:23.14 ID:V70/XNkx0.net
>ドイツ語は日本語と同じく、こういう感覚的な言葉が多いみたいで、擬声語も多いのです。

聞いたことがない話だったので調べたら、やっぱり虚言でした。

「ドイツ語のオノマトペでは„Duden CD-Rom“の記載数が541語に留まっている」
(※引用者注 英語1500, 日本語2200)
ドイツ語のオノマトペに関する一考察 : 音韻特徴と意味特徴を中心に

>ここで n が一つではなくて2つになっていますが、こういうふうに重なることによって、もしかしたら don という onomatopoeia の音の響きを強調したかったのかもしれません。

ドイツ語のド基礎を知らない珍説ありがとうございますw
複子音は、直前が短母音であることの印。Rote と Rotte で比較せよ。

>古い英語ではそんな問題はなかったはずです。
>ましてはドイツ語ではそんな問題はありません。

知りもしない言語のついて語るから、結局虚言になるのです。

5 :The OED Loves Me Not (ワッチョイ cb89-VEJP):2023/10/01(日) 14:53:02.21 ID:y+Y9w1JS0.net
新しいスレッドが光栄にも2つ並んで拮抗していますが、二つとも
存続させ続けていいのではないかと思います。こっちで喧嘩が始まれば、
向こうに避難することもできますからね。

それはともかく
https://oshiete.goo.ne.jp/qa/13601464.html
上のリンク先のような質問がありました。これと同じ質問が
知恵袋だと思いますが、そこでも提起されました。

これについて、どう思いますか?どうでもいいじゃないかとも思うでしょうけど。
(アクセス規制がいつ始まるかわからないので、ここで切ります。)

6 :The OED Loves Me Not (ワッチョイ cb89-VEJP):2023/10/01(日) 14:53:18.55 ID:y+Y9w1JS0.net
新しいスレッドが光栄にも2つ並んで拮抗していますが、二つとも
存続させ続けていいのではないかと思います。こっちで喧嘩が始まれば、
向こうに避難することもできますからね。

それはともかく
https://oshiete.goo.ne.jp/qa/13601464.html
上のリンク先のような質問がありました。これと同じ質問が
知恵袋だと思いますが、そこでも提起されました。

これについて、どう思いますか?どうでもいいじゃないかとも思うでしょうけど。
(アクセス規制がいつ始まるかわからないので、ここで切ります。)

7 :The OED Loves Me Not (ワッチョイ cb89-VEJP):2023/10/01(日) 14:58:35.81 ID:y+Y9w1JS0.net
この質問は、知恵袋
https://detail.chiebukuro.yahoo.co.jp/qa/question_detail/q11286346636
でも出ました。

これについて私は、

on a diet
on strike
on fire

のうち、on fire だけが Old English の時代からある言葉であり、
あとの二つは19世紀あたりになってやっと使われ始めたという背景が
主な理由だと思います。なお、on strike については、OED Online にも
書いてある通り、アメリカでは on a strike とも言うのです。

(アクセス規制を受けそうなので、ここで切ります。)

8 :英語職人 ◆azN58fWNzw (ワッチョイ 75f4-AvD6):2023/10/02(月) 11:10:05.36 ID:5yl/IG6G0.net
on a diet
on fire
on strike

なぜ、dietだけに[a]がつくのか? dietは、「減量のためのダイエット」という意味のとき可算名詞なのだから、aを付ける。
on fireの「火」は、「水」と同じく誰が見ても不可算名詞なのだから、aを付けるわけがない。(最近生まれた言葉だと仮定しても、aがつく余地はない)
※この時点で、言葉の発生期が理由になっていないことが証明できる。※このfireは火事という意味ではない。

strikeに関しては、ジーニアス英和辞典によれば、onの後ろは不可算として扱うと書いてある。そして、on strikeがふつう。
アメリカ人は「on a strike」をふつうに使うといのも間違い。下記のリンク先(3つめのブロック)で、アメリカ人が「we don't use "a" in that phrase」と言っている。on a strikeはごく一部で使われる非標準英語に過ぎない。
https://forum.wordreference.com/threads/did-our-mother-go-on-a-strike-or-something.2628512/

9 :英語職人 ◆azN58fWNzw (ワッチョイ 75f4-AvD6):2023/10/02(月) 11:15:13.24 ID:5yl/IG6G0.net
↑の訂正
(×)使うといのも
 (〇)使うというのも

10 :英語職人 ◆azN58fWNzw (ワッチョイ 75f4-AvD6):2023/10/02(月) 12:29:34.79 ID:5yl/IG6G0.net
on fireのfireが「火災」という意味ではない証拠

https://dictionary.cambridge.org/ja/dictionary/english-japanese/fire
このページの真ん中あたりに次のような記述があります。
----------------------
on fire
火( ひ)がついて
The house was on fire.
家が燃えていた。
-----------------------
このように「火災になって」としないで、「火がついて」としていることに着目。onの後ろのfireはもともと不可算名詞なのです。

11 :名無しさん@英語勉強中 (アウアウウー Sa89-YTgz):2023/10/02(月) 19:01:27.27 ID:dsNoXKiKa.net
>I would say that "Human Disqualified" sounds a bit weird grammatically. "The Disqualified Human" or what I've seen in the Donald Keene translation, "The Disqualified," sound more natural.
However "No Longer Human" is a very attention grabbing title and fits the themes of the book well. I think the difference is style, the meaning is basically the same. If you are disqualified from humanity, then you are no longer human.

redditに回答が付いてるのにスルー
都合の悪いことは全てNGする爺さん

12 :名無しさん@英語勉強中 (ワッチョイ 234f-5C2y):2023/10/03(火) 12:17:29.08 ID:+LxsKVEM0.net
New York Times

are on a strike 約800
are on strike 約3000

on a strikeはごく一部、非標準とまで言えるか?

13 :😷英ちゃん🤧 (ワッチョイW 2386-ZRN2):2023/10/03(火) 18:45:40.10 ID:hXN/px6p0.net
5chなんかいらね!教えてgooに移動したら良い!

という白人娘の creampie動画が好きなエロ爺さんが、教えてgooの質問を持ち込んでくるのはなぜです??

14 :😷英ちゃん🤧 (ワッチョイW 2386-ZRN2):2023/10/03(火) 18:47:28.58 ID:hXN/px6p0.net
>>8
>なぜ、dietだけに[a]がつくのか? dietは、「減量のためのダイエット」という意味のとき可算名詞なのだから、aを付ける。

diet が加算なのは diet が加算だから、というトートロジーになってる

15 :英語職人 ◆azN58fWNzw (ワッチョイ 75e0-AvD6):2023/10/03(火) 23:03:06.84 ID:T56fBpCn0.net
>>12
https://books.google.com/ngrams/graph?content=on+a+strike%2C+on+strike&year_start=1800&year_end=2019&corpus=en-2019&smoothing=3

on a strikeはいつの時代も底(使用例が極端に少ない)だよ。ジーニアス英和辞典の「onの後ろは不可算である」こそが標準英語。
I don't know nothihg.は、非標準英語だが、使う人は結構いる。非標準英語とは、めったに見ないという意味ではない。
on a strikeは、非標準英語。米国人ネイティブが「私たちは使わない」と言っているのだから、標準・略式でもない。

16 :英語職人 ◆azN58fWNzw (ワッチョイ 75e0-AvD6):2023/10/03(火) 23:08:34.23 ID:T56fBpCn0.net
>>14
「diet が加算なのは diet が加算だから」なんて言ってないんだが。🤠
「dietは可算・不可算両方の用法があり、「減量のためのダイエット」という意味のときは可算名詞扱いである」と言っている。

17 :英語職人 ◆azN58fWNzw (ワッチョイ 75e0-AvD6):2023/10/03(火) 23:14:32.44 ID:T56fBpCn0.net
試験で書いたら×になるような知識(非標準英語)を、さも、標準英語であるかのように主張するのはやめましょう。

18 :英語職人 ◆azN58fWNzw (ワッチョイ 75e0-AvD6):2023/10/03(火) 23:28:58.31 ID:T56fBpCn0.net
>>15の訂正

(×)I don't know nothihg.は、非標準英語だが、
(〇) I don't know nothing.は非標準英語だが、

19 :😷英ちゃん🤧 (ワッチョイW 2386-ZRN2):2023/10/03(火) 23:37:55.91 ID:hXN/px6p0.net
>>16
じゃあなんで減量の時のdietは可算扱いなの

20 :英語職人 ◆azN58fWNzw (ワッチョイ 75e0-AvD6):2023/10/04(水) 00:59:18.63 ID:doZ2vOZv0.net
>>19
>>7のリンク先の質問↓
-------------------------------------------
質問
on a diet(ダイエット中)は diet が可算名詞なので、a が必要だと思っています。
on strike(スト決行中)では、strike が可算名詞なのに a を付けません。 これはどうしてですか?
on fire(燃えている)ですが、fire は、一般論では不可算名詞、具体的特定対象では可算名詞ですが、目の前の火災でもa を付けないのですか?
------------------------------------------------
よって、「onの後ろのstrikeはジーニアスにあるように不可算名詞扱いなんだよ。on fireのfireは、「(具体的な)火災・火事」ではなく、「火」(物質名詞)だから、不可算だよ」と答えれば、それで済む話。質問者は、不可算を可算と勘違いしているだけだから。そもそも、なぜon の後ろのstrikeが不可算なのかなんて、「みんながそう使ってきたから」以外の理由はないだろう。on a dietも同様。

21 :😷英ちゃん🤧 (ワッチョイW 2386-ZRN2):2023/10/04(水) 01:16:00.61 ID:huVW3qd20.net
エロリストが話し相手欲しさに持ってきた問題なんてどうでもええねん

22 :名無しさん@英語勉強中 (ワッチョイW 9b66-65xA):2023/10/04(水) 02:38:18.29 ID:UkCzBk5N0.net
an eating planだから

23 :The OED Loves Me Not (ワッチョイ cb89-VEJP):2023/10/04(水) 11:42:41.81 ID:p2YhxMDY0.net
on a diet
on strike
on fire

またもやアクセス規制を受けるかもしれませんが、
もし隙があれば書き込みたいと思います。

私が数日前に言いたかったことは、
on fire は Old English の時代から(つまり 900 年以上も前から)
すでにあった idiom です。そのころには不定冠詞 (a, an) なんてものが
「基本的には」ありませんでした。それについては

Also, ★in Old English they generally had no indefinite article★ (although occasionally their word for "one" - ān could be translated into Modern English as "a" or "an"). So in speaking Old English, a noun with no article at all would often be the equivalent to a noun with an indefinite article in Modern English, for example hūs - "a house", and dēor - "an animal".
https://en.wikibooks.org/wiki/Old_English/Articles

上のようにネット上に書いてあります。

さて、Old English には不定冠詞 (a, an) が基本的にはなかったのだから、
"something is on fire" などと言うときに、a をつけたくてもそれに相当する
ものがその時代にはなかった(あるいは、そんなものがなくても
a fire という意味で fire と言いさえすればよかった)のだから、
on fire をという形をそのまま使い、現代に至っていると思われます。

しかし on strike (一部のアメリカ英語では on a strike とも言う) や
on a diet に関しては、19世紀あたりになって初めて出来上がった idiom なので
a がつくのです。しかし on a strike という言い回しがもともとはあったのかもしれませんが、
現代では標準英語としては on strike つまり a のない形が定着したのでしょう。
on a diet に関しては、そのまま a をつけているのです。

私は以上のように推測しています。

24 :😷英ちゃん🤧 (ワッチョイW 2386-OELq):2023/10/04(水) 11:58:14.36 ID:huVW3qd20.net
前から思考力のある人間ではなかったが、いよいよやばいな
Fireに関してはソフィ先生が全く正しかろう

25 :The OED Loves Me Not (ワッチョイ cb89-VEJP):2023/10/04(水) 11:59:30.69 ID:p2YhxMDY0.net
童謡に、on top という idiom があります。これには the がつかない。
それは、OED Online を見てもわかるように、on top という idiom が
Old English の時代から今までずっと使われてきたからだと思っていいように思います。

その一方、
on top of something
on the top of something
これには二種類あって、the があるときとないときがあります。
これについては、たぶん on the top of の方が後の時代になって出来上がったのかもしれないな
と思いながら覚えると覚えやすいんじゃないかと思います。

私がなぜこんなことを考えたりするかというと、こういう想像を巡らせて遊んでいるわけでもないし、
あるいはその道の学者でもないくせに偉そうに学説を打ち立てるふりをして自己満足するためでもなく、
ド素人のただのデタラメの説であっても構わないから、自分がこういう idiom や文法事項や単語を
覚えるときのコツとして使いたいからです。

語源なんかでも同じなのです。一つの単語が覚えにくいとき、学者たちがその語源を明らかにしていないなら、
たとえ ★こじつけ★ やデタラメでもいいから、自分でそれは自分なりのただの自己満足の語源説だと
百も承知の上で、自分の記憶の助けとしてそういう語源を考えてみるのです。

要は、一つでもたくさん単語を覚える必要があるのです。その助けとして語源の知識を活用し、時には
自分なりの説を自分の中だけで作ってみることもよいことだと言っているだけなのです。

それをみんなが必死で、何とかして私の言っていることは一から十まですべて間違いだと証明したくて躍起になっているだけです。

26 :The OED Loves Me Not (ワッチョイ cb89-VEJP):2023/10/04(水) 12:01:00.99 ID:p2YhxMDY0.net
なお、
on top of something
on the top of something
この二つには、使い方や意味が微妙に違うことは百も承知です。
下らない茶々を入れないようにお願いします。

27 :😷英ちゃん🤧 (ワッチョイW 2386-OELq):2023/10/04(水) 12:08:49.58 ID:huVW3qd20.net
エロリストのやってきた「比較的言語学」は、単語熟語を覚えるための方便かよ🤭

28 :The OED Loves Me Not (ワッチョイ cb89-VEJP):2023/10/04(水) 12:14:02.07 ID:p2YhxMDY0.net
simmer の語源については、すでにチラッと(掲示板が極めて重くて
書き込みづらかったときに)触れたように、

Online Etymology Dictionary
Wiktionary の語源欄
OED Online

には Middle English での simpere(n) から来ている
としか書いていないことも、最初から百も承知でした。

私が数日前に言っていたのは、研究社の「英語語源大辞典」によると
Middle English での simpere(n) の語源が擬音語であると書いている
ということでした。

そしてそれは、sim + -er (接尾語) というふうに出来上がったと書いています。
そしてこれと同じ辞書に -er という接尾語について、これまた詳しく書いてあったので、
それを私は数日前に紹介して、同時に sim- という語幹の部分については
どこにも誰も書いていないので、私が想像をたくましくして解説してみたのです。

研究社の「英語語源大辞典」は 1,700 ページほどにわたりますが、
略語などを駆使して、狭いページ上にぎっしりと情報が網羅されており、
OED Online における語源欄とはまた別の学説もたくさん紹介しています。

だから語源に関しては、OED Online とこの研究社「英語語源大辞典」とを
併用するのがよいのです。なお、

Online Etymology Dictionary
Wiktionary 語源欄

この二つは無料のオンライン辞書でしかないので、所詮は無料らしく情報に厚みがありません。
こんなものだけを引いて喜んでいてはいけないのです。

29 :The OED Loves Me Not (ワッチョイ cb89-VEJP):2023/10/04(水) 12:15:34.41 ID:p2YhxMDY0.net
今日は数日ぶりに気持ちよく書き込みが進みます。
またいつアクセス規制がかかるか、あるいは書き込みが万人にとって困難になる
かわかりません。

30 :😷英ちゃん🤧 (ワッチョイW 2386-OELq):2023/10/04(水) 12:22:25.13 ID:huVW3qd20.net
donnernの件は、エロリストの比較言語学に関する無知・無理解が凝縮されている。
よくこんなレベルで学問の名を出せたなと呆れた
近いうちに簡潔にまとめようと思う。

31 :😷英ちゃん🤧 (ワッチョイW 2386-OELq):2023/10/04(水) 12:22:35.58 ID:huVW3qd20.net
donnernの件は、エロリストの比較言語学に関する無知・無理解が凝縮されている。
よくこんなレベルで学問の名を出せたなと呆れた
近いうちに簡潔にまとめようと思う。

32 :The OED Loves Me Not (ワッチョイ cb89-VEJP):2023/10/04(水) 12:35:45.69 ID:p2YhxMDY0.net
simmer という単語の語源について、Middle English では simpere(n) と書いてあるのに
なんでそれが simmer になって、いつ真ん中の p が消えちゃったんだよ、と思う人もいるでしょう。

そのように m のあとに p とか b の音がくっつくことは、言語発達の歴史上、きわめて
自然に頻繁に起こることなのです。それはいろんな言語の方言も共通語もすべて
いろいろとかじっていれば、ごく自然にわかります。

例を挙げると、
assembler というフランス語があります。これは英語の中に入って assemble となりましたが、
これは vulgar Latin (俗ラテン語って言うんでしたっけ?) の assimulare から来ていて、それは
ad + simulare から来ていて、さらにそれは現代英語の simulate, similar, simile などと
似ていますが、すべて「似ている」という概念を表す言葉が中に含まれているのです。
だからこそ、same という英単語とも似ているのです。

それはともかく、assimulare という vulgar Latin の形があって、その assim のあとに b が
一つの訛りとして b が入ってしまって、結果として assimbulare みたいな形になり、
そのあとに assembler というフランス語ができ、そのあとに assemble とか assembly とかいう
英語が出来上がったと考えれば、実にこれらたくさんの単語が一つにつながり、
ややこしかった語彙大系がすっきり整理され、記憶しやすくなり、すでに記憶していたら
その記憶が長続きし、奥行きができるのです。

33 :The OED Loves Me Not (ワッチョイ cb89-VEJP):2023/10/04(水) 12:36:03.99 ID:p2YhxMDY0.net
そしてついでに、自分がすでに知っている英単語が実はラテン語やフランス語やイタリア語や
スペイン語やルーマニア語やカタロニア語や、そしてロシア語やトルコ語の中に入り込んだ
フランス語などにもつながり、世界中の言語が自分の頭の中でつながっていくのです。

そういう意味で、語源について勉強したり、我流でもいいから自分なりに、「この単語の語源については
学者は明らかにしていないけど、もし私が学者だったら、このように考えるかもしれない」と想像を
たくましくして、その語源説が仮にデタラメであったとしても、そのように考えた方が
自分としてはたくさんの単語が楽に覚えられるなら、それでいいのです。
(もちろん、自分のど素人としての語源説を学会で発表しても、無視されておしまいであることは百も承知です。)

要は、できれば数万語の単語を覚えたいけれども、そのためにはどうすればいいかと思ったとき、語源が役立つこともあるということです。

34 :The OED Loves Me Not (ワッチョイ cb89-VEJP):2023/10/04(水) 13:45:09.15 ID:p2YhxMDY0.net
前スレとか、さらにその前のスレを検索したいと思っても、掲示板の全体が重すぎて
検索できません。

前のスレで、確か
He seems + 形容詞句
(たとえば He seems in good condition. という言い方はできないとか何とかいう
話が出ませんでしたっけ?私の勘違いでしょうけど、ともかく今、そういう
用例を読書中に偶然に見つけたので、紹介します。

I again flashed the light to see if ★he seemed in any pain★.

(The Shunned House, The Library of America #155, p. 117 の冒頭より)

もしもこんなことはまったくそのスレでは論議しなかったのであれば、私の記憶違いです。
済みません。)

35 :The OED Loves Me Not (ワッチョイ cb89-VEJP):2023/10/04(水) 13:45:25.90 ID:p2YhxMDY0.net
前スレとか、さらにその前のスレを検索したいと思っても、掲示板の全体が重すぎて
検索できません。

前のスレで、確か
He seems + 形容詞句
(たとえば He seems in good condition. という言い方はできないとか何とかいう
話が出ませんでしたっけ?私の勘違いでしょうけど、ともかく今、そういう
用例を読書中に偶然に見つけたので、紹介します。

I again flashed the light to see if ★he seemed in any pain★.

(The Shunned House, The Library of America #155, p. 117 の冒頭より)

もしもこんなことはまったくそのスレでは論議しなかったのであれば、私の記憶違いです。
済みません。)

36 :😇英さま🤔 (ワッチョイ 2386-2pcI):2023/10/04(水) 22:06:34.31 ID:huVW3qd20.net
>>25
間違いを指摘されると、「語源なんかデタラメでいいんだ。おれは学者じゃねー」と開きなおる

>>28
他人には、無料のオンライン辞典なんか引いて満足してんじゃねーよ、という
デタラメでも記憶のための方便なんだからいいじゃなかった?

他人に厳しく自分の激甘の、虚言エロリストの平常運転でご愛敬である。
問題は次ね

37 :😇英さま🤔 (ワッチョイ 2386-2pcI):2023/10/04(水) 22:18:59.57 ID:huVW3qd20.net
>>32
> いつ真ん中の p が消えちゃったんだよ

これは脱落(elision)という現象

> m のあとに p とか b の音がくっつくこと

これは同化(assimilation)という現象

> その assim のあとに b が一つの訛りとして b が入ってしまって

これは挿入(Epenthesis)という現象

「pが消える」という現象を、「mとpがくっつく」と解説し、「bが増える」例をあげ
ている。

子音が消える現象に対し、子音が加わる例を挙げて、それで本人はわかった気になってる。

38 :😇英さま🤔 (ワッチョイ 2386-2pcI):2023/10/04(水) 22:21:01.11 ID:huVW3qd20.net
最近、エロリストの知能が著しく低下してない?
こんなにも抜けてたっけ?

39 :😇英さま🤔 (ワッチョイ 2386-2pcI):2023/10/04(水) 22:35:54.28 ID:huVW3qd20.net
>>23
これもエロリストの知能の低下を示す例

OEには不定冠詞がなかったから、on fire なのだという。
じゃあ不定冠詞があったら、on a fire だったのか?

> 可算名詞は、その表すイメージとして、「形あるもの」「境界線がハッキリしているもの」「壊すことがでできるもの」「半分にすることができるもの」「具体的なもの」などと説明される一方、不可算名詞は、逆に「形のないもの」「境界線がハッキリしていないもの」「壊すことができないもの」「半分にすることができないもの」「抽象的なもの」などと説明される。
『新英文法概説』p.99

火などは不可算名詞のもっとも典型的なものだろう

40 :😇英さま🤔 (ワッチョイ 2386-2pcI):2023/10/04(水) 22:42:50.45 ID:huVW3qd20.net
ワイの方もまとめるのが大変だから自重してほしいのだが😥

41 :😇英さま🤔 (ワッチョイ 2386-2pcI):2023/10/04(水) 22:52:37.63 ID:huVW3qd20.net
Online Etymology Dictionary
Wiktionary 語源欄

この二つは無料のオンライン辞書でしかないので、所詮は無料らしく情報に厚みがありません。
こんなものだけを引いて喜んでいてはいけないのです。

https://lavender.5ch.net/test/read.cgi/english/1696138309/28
-------

我流・デタラメで良いなら語呂合わせでもいいはずなのに、他人には、ネットの無料辞書で満足するな!と厳しく当たる。

42 :😇英さま🤔 (ワッチョイ 2386-2pcI):2023/10/04(水) 22:53:16.46 ID:huVW3qd20.net
>>41
誤爆😝

43 :英語職人 ◆azN58fWNzw (ワッチョイ 758c-AvD6):2023/10/04(水) 23:37:37.25 ID:doZ2vOZv0.net
>>25
> たとえ ★こじつけ★ やデタラメでもいいから、自分でそれは自分なりのただの自己満足の語源説だと
> 百も承知の上で、自分の記憶の助けとしてそういう語源を考えてみるのです。

自分の頭の中で思う分には自由ですが、さも、「これが真実だ!」というノリで、掲示板でそのデタラメを披露してはいけないのではないでしょうか?

>Middle English での simpere(n) から来ているとしか書いていないことも、最初から百も承知でした。
いや、知ってたら、sim sim simの擬音語とは書かないのでは? 知ってたら「simp simp simpの擬音語」と書いていたはずでは?

あと、物質名詞は、今も昔も、aを付けないのです。「火」は物質名詞です。
https://nebstudent.com/material-noun-definition-examples-and-rules/job
↑物質名詞の例を載せてるページ。ここの48番の例文 My cat is afraid of fire.参照。 物質名詞という文法事項を理解していないから、ズレた回答になってしまうのですよ。

44 :英語職人 ◆azN58fWNzw (ワッチョイ 758c-AvD6):2023/10/04(水) 23:49:57.71 ID:doZ2vOZv0.net
>>23
>しかし on a strike という言い回しがもともとはあったのかもしれませんが、
>現代では標準英語としては on strike つまり a のない形が定着したのでしょう。

下記リンクによれば、「on a strike」がスポットライトを浴びた時代は一度もありませんでした。常に日陰の存在でした。
https://books.google.com/ngrams/graph?content=on+a+strike%2C+on+strike&year_start=1800&year_end=2019&corpus=en-2019&smoothing=3

45 :名無しさん@英語勉強中 (アウアウウー Sa89-Kgno):2023/10/05(木) 00:22:20.06 ID:0jYQYau2a.net
語源の知識はデタラメ
英訳は機械翻訳みたいに不自然

60年間何を学んできたんだこの自慰さんは

46 :英語職人 ◆azN58fWNzw (ワッチョイ 758c-AvD6):2023/10/05(木) 00:45:53.32 ID:A8+BaAHp0.net
訂正
>>43
『いや、知ってたら、sim sim simの擬音語とは書かないのでは? 知ってたら「simp simp simpの擬音語」と書いていたはずでは?』は、次のように訂正します。

『でも、simperenを本当に知ってたら、「simper→simperen」という一番重要なプロセスを省略するとは思えないんだよなあ~🤠』

47 :😇英さま🤔 (ワッチョイ 2386-2pcI):2023/10/05(木) 00:46:47.39 ID:nDAwEVO20.net
>>28
>Online Etymology Dictionary
Wiktionary の語源欄
OED Online

には Middle English での simpere(n) から来ている
としか書いていないことも、最初から百も承知でした。

--------------

嘘つき!ほら吹き!虚言癖!

48 :😇英さま🤔 (ワッチョイ 2386-2pcI):2023/10/05(木) 00:47:41.45 ID:nDAwEVO20.net
simber (the modern form is from 1680s), an alteration of simperen (late 15c., simpring "simmering," in an alchemical text), which might be 【imitative】
https://www.etymonline.com/search?q=simmer

of possibly 【imitative origin.】
https://en.wiktionary.org/wiki/simmer#Etymology_1

一日一虚言どこじゃねーぞ!

49 :😷英ちゃん🤧 (ワッチョイW 2386-OELq):2023/10/05(木) 00:55:58.76 ID:nDAwEVO20.net
>>23
Wiktionary は信用できん!😤
と言っておきながら、自分はwikibooksをソースにする

自分には激甘ないつものです

50 :名無しさん@英語勉強中 (ワッチョイ 234f-5C2y):2023/10/05(木) 09:54:41.81 ID:LiJ1LQm30.net
>>0015
New York Times

are on a strike 約800
are on strike 約3000

それでは上記データをどう説明するのか?
日本人がアホな説明を日本語学習者にしているのをよく目にする
権威のないネイティブの説明などあてにならんよ

51 :名無しさん@英語勉強中 (ワッチョイ 234f-5C2y):2023/10/05(木) 10:10:03.21 ID:LiJ1LQm30.net
上記データを検証して間違いを指摘して下さい。

52 :名無しさん@英語勉強中 (ワッチョイ 2386-2pcI):2023/10/05(木) 13:59:06.31 ID:nDAwEVO20.net
The New York Times

"are on strike" 746
"are on a strike" 23

🤔?

53 :名無しさん@英語勉強中 (ワッチョイW 1b09-iTzj):2023/10/05(木) 18:55:23.47 ID:b59CaBZT0.net
on a hunger strike って言うから
on だから不可算という簡単な話ではないな

54 :名無しさん@英語勉強中 (ワッチョイ 052b-sSEZ):2023/10/05(木) 19:28:17.03 ID:FVNTje3M0.net
come up with 〜 が「〜を思い付く」「〜を考え出す」の意味で使われる場合
この 副詞の up は 「ぽんと」とか「ふと」といった上昇のニュアンスになりましょうか?
仮にそうだとすれば come up with はあまり苦労や難題感がない問題解決で用いられる
表現なのかな、と思いましてきいてみました

55 :名無しさん@英語勉強中 (ワッチョイ 052b-sSEZ):2023/10/05(木) 19:40:09.20 ID:FVNTje3M0.net
追記
それとも come up to なんかと同じ「距離感」を表す 副詞の up で
むしろ難問解決の場で使うほうが相応しい表現でしょうか?

56 :The OED Loves Me Not (ワッチョイ cb89-VEJP):2023/10/05(木) 19:49:05.13 ID:GI6Vlfc80.net
on a strike
on strike
に関して言えば

Over 100 Boggabri coal mine workers ★went on two strikes★ at the end of last month in a bid to restore fair pay and conditions at the mine, CFMMEU stated.
https://www.australianmining.com.au/idemitsu-rejects-claim-over-boggabri-fifo-arrangement/

上のような用例もありますね。

57 :名無しさん@英語勉強中 (ワッチョイ 2386-2pcI):2023/10/05(木) 20:33:08.13 ID:nDAwEVO20.net
【理想】

俺は違う。たとえ 99.9% の人間たちから馬鹿にされたままで、貧乏と、一見して孤独
のように見える状況の中で死んでいってもいい。俺は真実を愛する。その真実を
99.9% の人々が目もくれなくてもいい。

【現実】

虚言癖

58 :名無しさん@英語勉強中 (ワッチョイ 2386-2pcI):2023/10/05(木) 20:35:44.17 ID:nDAwEVO20.net
【理想】

僕は知識や真実を追求することを自分の使命だと感じているからだ。
真実のために生き、真実のために死ぬことが僕の使命なのだ。

【現実】

間違いを指摘されると「はいお前NG!」

59 :名無しさん@英語勉強中 (ワッチョイ 2386-2pcI):2023/10/05(木) 20:40:40.15 ID:nDAwEVO20.net
>>54
思い「浮かぶ」と同じで上昇だと思うけど、なぜ苦労感が出てないと言えるの?🤔

60 :名無しさん@英語勉強中 (ワッチョイ 2386-2pcI):2023/10/05(木) 22:20:30.53 ID:nDAwEVO20.net
エロリストは一見すると孤独だけど、実は違うの?🤔
こんな面白いレス書いてるなんて、もっと早く知りたかったよ

61 :英語職人 :2023/10/05(木) 23:24:36.14 ID:A8+BaAHp0.net
>>50
https://books.google.com/ngrams/graph?content=on+a+strike,+on+strike&year_start=1800&year_end=2019&corpus=en-2019&smoothing=3

これをみても、on a strikeが覚えてはいけない表現であることがわからないの? ジーニアス英和辞典が言うとおり、on strikeが標準英語だよ。
米国人ネイティブが「on a strike」とは言わないと言って、誰もそれに反論しなかった。それは、多くのネイティブがそれに賛同していることを暗示している。あきらめましょう🤠  試験で書いたら×になるようなことを言うのは有害です。

62 :英語職人 :2023/10/05(木) 23:31:13.55 ID:A8+BaAHp0.net
>>53
形容詞が付いたら、aが付くのは、当たり前。意味を変えて論じちゃだめ。理系で言ったら、不公平な環境で対照実験をするようなもの。
「on strike」か「on a strike」かの問題なのであり、余計な言葉を加えたら、条件が変わるのだから、答えも変わってくる。お話にならない。

>>56も同じ過ちを犯しています。意味を変えたら、答えは変わってくるのです。

63 :英語職人 :2023/10/05(木) 23:40:24.94 ID:A8+BaAHp0.net
私に反論している人たちの犯している間違い

私:地球の周りを回っている月は、「the moon」であって、「a moon」としてはいけないよ。
反論者:満月は、a full moonだよ。
私: そりゃ、fullを付けたら a がつくに決まってるさ。勝手に単語を加えるなって😄

64 :名無しさん@英語勉強中 :2023/10/06(金) 00:18:21.74 ID:ttPlkp+o0.net
>>63
The Earth has a moon.

65 :名無しさん@英語勉強中 :2023/10/06(金) 00:18:30.19 ID:ttPlkp+o0.net
>>63
The Earth has a moon.

66 :名無しさん@英語勉強中 (ワッチョイW 1b09-iTzj):2023/10/06(金) 00:24:16.98 ID:XlClQpRg0.net
on の後ろは不可算だとか
形容詞付いたら a がつくに決まってるとか
単純すぎる説明を書かれても困る

67 :英語職人 ◆azN58fWNzw (ワッチョイ 75e0-AvD6):2023/10/06(金) 00:30:16.44 ID:j093vLbY0.net
>>64 お、久々にいい反論だ🤠  使わせてもらうよ😃

>>63を訂正 私に反論している人たちの犯している間違い

私:地球の周りを回っている月に着陸したは、「The rocket landed on the moon」であって、「landed on a moon」としてはいけないよ。
反論者:The Earth has a moon.だよ。
私: そりゃ、意味を変えたら a がつくこともあるさ。勝手に意味を変えるなって😄

68 :英語職人 ◆azN58fWNzw (ワッチョイ 75e0-AvD6):2023/10/06(金) 00:35:45.13 ID:j093vLbY0.net
>>66
「onの後ろは不可算だ」と言ってるんじゃないぞ。
ジーニアス英和辞典が言っている通り、「on strike」のonの後ろのstrikeは不可算名詞であるのがふつうだと言っている。
>>64のような私が感心する反論をしてくれ。

69 :英語職人 ◆azN58fWNzw (ワッチョイ 75e0-AvD6):2023/10/06(金) 00:41:10.48 ID:j093vLbY0.net
誤解している人がいるようなので、ここではっきりさせておく。

私が言いたいのは、「on strike」と「on a strike」の2者では、前者のほうがふつうであると言っている。ただ、これだけが言いたい。
ここに形容詞のようななんらかの意味を加えたら答えも変わってくるので、これに余計な意味を加えて考えてはいけないと言っている。

70 :名無しさん@英語勉強中 (ワッチョイ 4d96-tvb5):2023/10/06(金) 09:28:43.99 ID:+TngeQyt0.net
お願いします。

The fast-food restaurant said it raised its prices because the price of ingredients had raisen a lot over the past month.

この文のit (said it raisedのit)がよくわかりません。

このitって何を指しているのか、どうしてitが必要なのか、said that they raised its prices ではだめなのか。
raiseをしている主体(レストラン)がitなのか、、。

どう理解すればよいでしょうか?

71 :名無しさん@英語勉強中 (ワッチョイW 2db9-9Y2f):2023/10/06(金) 09:46:57.54 ID:pIwtSyYW0.net
>>70
いや普通にrestaurant を指してるだけやで
said の後のthat省略な

72 :名無しさん@英語勉強中 (ワントンキン MMa3-LmEL):2023/10/06(金) 10:50:33.95 ID:Pi/EGPyHM.net
レストランが単数だからtheyはやや変かな
企業とか組織だったらそこに属してる人達がってことでweとかtheyを使うこともあるけどね
あとまあ人間だったらhe/sheの代わりにtheyって言ったりもするけど

73 :The OED Loves Me Not (ワッチョイ cb89-VEJP):2023/10/06(金) 11:36:52.83 ID:Iy/RdZw20.net
>>70
> The fast-food restaurant said it raised its prices because the price of ingredients had raisen a lot over the past month.

The restaurant が何か行動を起こすというとき、それを it で受けることはよくあります。

The restaurant said ★it★ has investigated the incident after a customer claims she was left feeling "physically sick"
https://www.liverpoolecho.co.uk/news/liverpool-news/restaurant-explains-reason-customers-found-27839597
https://www.liverpoolecho.co.uk/news/liverpool-news/restaurant-explains-reason-customers-found-27839597

上の it は The restaurant を指しています。こういうふうに、company, government, administration, the U.S. administration などの
組織を指して、その組織を代名詞で受けるときには it で受けるのが正式な英語であるように見えます。

しかし気楽な英文(特に口語での英語)では、it ではなくて they で受けることがよくあるようです。それは
restaurant を含めて、そういう組織は人間の集合体だから they で受けてしまうのだろうと思います。

The restaurant said ★they★ accepted the apology and that they would no longer pursue the issue with the police.
(Facebook より)

74 :The OED Loves Me Not (ワッチョイ cb89-VEJP):2023/10/06(金) 11:48:43.54 ID:Iy/RdZw20.net
>>70
よく似た例をもう一つだけ挙げておきます。

When the House says that ★it★ wishes to investigate something, it has a right to do that.
https://www.washingtonpost.com/news/post-politics/wp/2014/05/15/condoleezza-rice-says-there-are-unanswered-questions-about-benghazi/

この上の用例では、it とは the House のことで、その the House とは the U..S. House of Representatives のことですね。
it wishes だけを見て、それが the House says という意味だということがわかっても、日本語に直訳して考えると
「家が物を言うわけねえだろ?」と思ってしまいますが、この the House は組織全体のことであって「米国下院」(という組織の全体)を
指していて、その組織全体(そして直接的にはその米国下院の上層部)がそのことを wishes しているのだから、it wishes でよいわけです。

75 :The OED Loves Me Not (ワッチョイ cb89-VEJP):2023/10/06(金) 11:48:57.04 ID:Iy/RdZw20.net
>>70
よく似た例をもう一つだけ挙げておきます。

When the House says that ★it★ wishes to investigate something, it has a right to do that.
https://www.washingtonpost.com/news/post-politics/wp/2014/05/15/condoleezza-rice-says-there-are-unanswered-questions-about-benghazi/

この上の用例では、it とは the House のことで、その the House とは the U..S. House of Representatives のことですね。
it wishes だけを見て、それが the House says という意味だということがわかっても、日本語に直訳して考えると
「家が物を言うわけねえだろ?」と思ってしまいますが、この the House は組織全体のことであって「米国下院」(という組織の全体)を
指していて、その組織全体(そして直接的にはその米国下院の上層部)がそのことを wishes しているのだから、it wishes でよいわけです。

76 :名無しさん@英語勉強中 (ワッチョイ 859d-5C2y):2023/10/06(金) 12:29:51.90 ID:Ho/cVSQa0.net
「on strike」  ストライキ中
「on a strike」 数ある中の一つのストライキ中

77 :名無しさん@英語勉強中 (ワッチョイ 859d-5C2y):2023/10/06(金) 12:30:07.00 ID:Ho/cVSQa0.net
「on strike」  ストライキ中
「on a strike」 数ある中の一つのストライキ中

78 :英語職人 :2023/10/06(金) 21:36:01.39 ID:j093vLbY0.net
そろそろまとめに入るよ。 >>50の比較がいかに不公平なものかに気づいた人は頭がいい 🤠

”on strike” と”on a strike”で検索かけて数を比べて「on a strikeもそんなに少なくない」と勝ち誇ることが、いかに恥ずかしい行為であるかの説明↓

「on a strike」で検索をかけると「on a strike day」のような後ろに名詞が続く例までカウントされるのだ。「on a strike day」などがいかに多く使われている表現かがわかれば、>>50の比較を持ち出すことがいかに恥ずかしいことかが理解できるだろう。😃

79 :英語職人 :2023/10/06(金) 21:37:17.98 ID:j093vLbY0.net
■□■ まとめ ■□■
on strike と on a strike、どっちが標準英語?

私の意見: on strikeは標準英語。on a strikeは非標準英語。
反論者の意見: on strikeも on a strikeも標準英語だ。

検証 本場アメリカの権威のある英英辞典「American Heritage Dictionary」でstrikeを調べてみると、
https://www.ahdictionary.com/word/search.html?q=strike
==============================
Idioms:
on strike
Engaged in a work stoppage: Most of the employees were on strike.
==============================

on strikeしか載せていません。これは、on a strikeが載せる価値もない非標準英語であることを示しています。私の圧勝でしたね。🤠

80 :英語職人 :2023/10/06(金) 21:38:01.27 ID:j093vLbY0.net
文句がある人は、アメリカの英英辞典で「on a strikeとも言う」と明記している辞書を出してみなさい。日本の辞書でもいいよ。本場アメリカの辞書が載せてないものを「アメリカではふつうに使っているから正しいのだ」なんてデタラメ言ってはいけません。社会的に有害です。

81 :英語職人 :2023/10/06(金) 21:43:18.60 ID:j093vLbY0.net
まだ、納得いかない人は>>61のリンク先をもう一度見直しましょう。

82 :名無しさん@英語勉強中 :2023/10/06(金) 22:59:03.31 ID:SXzT3BQV0.net
Just a few month into letting their staff work from home over the internet, Campbill Technologies saw productivity increase by 30 percent.

Just a few month into letting their staff work from home over the internet

ここの意味がよく分かりません
文法的にどう解釈すればいいのでしょうか?

時間 into 名詞句 で
__ months into a trial separation
《be 〜》〔離婚を望む人が〕試験的[お試し]別居に入って_カ月がたつ[経過している]

という説明をネットで見ましたが、文法的にきちんと説明するためにはどうすればいいでしょうか?
参考サイトなどがあれば教えてください

83 :名無しさん@英語勉強中 :2023/10/06(金) 22:59:32.06 ID:SXzT3BQV0.net
Just a few month into letting their staff work from home over the internet, Campbill Technologies saw productivity increase by 30 percent.

Just a few month into letting their staff work from home over the internet

ここの意味がよく分かりません
文法的にどう解釈すればいいのでしょうか?

時間 into 名詞句 で
__ months into a trial separation
《be 〜》〔離婚を望む人が〕試験的[お試し]別居に入って_カ月がたつ[経過している]

という説明をネットで見ましたが、文法的にきちんと説明するためにはどうすればいいでしょうか?
参考サイトなどがあれば教えてください

84 :😤英ちゃん⛱ :2023/10/06(金) 23:22:33.35 ID:vmS4Z7CO0.net
>>82
説明間違ってます🫢
https://i.imgur.com/TaYfdxI.png

85 :😤英ちゃん⛱ :2023/10/06(金) 23:36:35.74 ID:vmS4Z7CO0.net
intoの直前は時間以外でも使えるという意味で間違ってると書いたけど、その文のケースでは問題ないね🙈

86 :名無しさん@英語勉強中 :2023/10/06(金) 23:37:17.89 ID:+TngeQyt0.net
>>71-75

ありがとうございました!勉強になりました。

87 :名無しさん@英語勉強中 :2023/10/06(金) 23:41:23.14 ID:kRKbblfm0.net
>>84
About ten minutes into the flight, the plane began to vibrate badly.
飛び立って10分ほどしたところで飛行機がひどく揺れ始めた。

>>82
Just a few month into letting their staff work from home over the internet,
Campbill Technologies saw productivity increase by 30 percent.
スタッフに在宅勤務をさせてほんの2〜3か月したところで当社の生産性は30%伸びた。

A into B 「Bに入って(を始めて)Bの期間が経った時」

88 :名無しさん@英語勉強中 :2023/10/06(金) 23:43:07.09 ID:kRKbblfm0.net
>>84
About ten minutes into the flight, the plane began to vibrate badly.
飛び立って10分ほどしたところで飛行機がひどく揺れ始めた。

>>82
Just a few months into letting their staff work from home over the internet,
Campbill Technologies saw productivity increase by 30 percent.
スタッフに在宅勤務をさせてほんの2〜3か月したところで当社の生産性は30%伸びた。

A into B 「Bに入って(を始めて)Bの期間が経った時」

89 :名無しさん@英語勉強中 :2023/10/06(金) 23:55:56.70 ID:kRKbblfm0.net
<類例>
Six months into his employment with the company, he was able to sell more clothes than any other employee.
入社して半年で、彼は他のどの社員よりも多くの服を売ることができた。

90 :名無しさん@英語勉強中 (ペラペラT SDa5-T1Gw):2023/10/07(土) 06:06:52.94 ID:LM6XcSPOD.net
質問させていただきます。

Lincoln reminded his audience, and all Americans since, of the high ideals behind the founding of this country.

訳は「リンカーンは聴衆と全てのアメリカ人に建国の背後にある高い理念を思い出させた」と解釈しました。
Americans の後ろにある since の意味(役割)が分からず困っております。

よろしくお願いします。

91 :名無しさん@英語勉強中 (ワッチョイ 7a80-6t2b):2023/10/07(土) 06:42:51.37 ID:b0dnlASd0.net
その訳であってるよ
sinceにさほど意味はなくて今に至るまでの全てのみたいな言い方をしてるんだけど日本語ならその訳くらいで充分だよ
その文章の中で言いたいのはそのワードの部分じゃないしね
書いてる人はリンカーンの言葉は今でも影響あるよねみなさんもご存じだよねくらいのニュアンスで書いたんでしょ

92 :The OED Loves Me Not (ワッチョイ 7689-GYY9):2023/10/07(土) 07:06:24.89 ID:TGT5Fgrb0.net
>>83
(1) 質問者の抱えている原文: >>Just a few month into letting their staff work from home over the internet

(2) ... ★hundreds of pages into the book,★ in (say) the 1980s pages, he quotes an interview with Maureen. In this interview, Maureen talks about something pertaining to the '30s.
https://www.google.co.jp/books/edition/Let_Me_Tell_You_How_I_Really_Feel_The_Un/zsB7DwAAQBAJ?hl=en&gbpv=1&dq=%22hundreds+of+pages+into+the+book%22&pg=PT256&printsec=frontcover&bshm=rime/1
(上のリンク先に表示されるページの、上から9行目。「この本を数百ページも書き進めたところで、著者は〜だと言っている」というような意味)

(3) go on working late [deep] into the night (ジーニアス英和、第6)

(4) He is well into his forties. (ジーニアス)

(5) She is very into jazz. (ジーニアス)

(6) He is into her for $100. (ジーニアス)
彼は、彼女に100ドル借りている。

こういうのはすべて、
He put his stick into the pond.
の into という言葉の持つ意味合いから派生しているだけだと考えたらいいと思います。

(続き)

93 :名無しさん@英語勉強中 (ペラペラT SDa5-T1Gw):2023/10/07(土) 07:06:33.86 ID:LM6XcSPOD.net
>>91
なるほど、リンカーンが演説した「その時」だけでなく、
演説の「その後、それ以来」全アメリカ人に影響したという意味だったのですね。
納得できました。ありがとうございました。

94 :The OED Loves Me Not (ワッチョイ 7689-GYY9):2023/10/07(土) 07:06:55.35 ID:TGT5Fgrb0.net
(続き)
棒を家に突っ込んでいるイメージ、つまり棒が池の中に入り込んでいるわけです。
(1) では、自宅勤務体制(という池)の中に従業員を数か月ほど突っ込んでいるわけです。
ということは、当然のことながら「従業員の自宅勤務体制を数か月ほど」続けたわけです。

(2) では、その本の中に数百ページ分くらい入り込んでいるわけです。だから「その本を最初から数百ページほど書いたころに」
という意味になるに決まっています。

(3) では、夜の中に深く入り込むところまで働いたわけです。

(4) では、四十代という年齢ゾーンに深く入り込んでいるのです。

(5) では、彼女がジャズという音楽世界に深く入り込んでいるのです。だから「それに夢中」なのです。

(6) では、彼女に、100ドル分だけ入り込んでいるのです。だから「彼女に100ドル借金している」という意味になるのです。

というわけで、into の「〜の中に入り込んでいる」というイメージが、あらゆるところにちゃんと生きているのです。

95 :The OED Loves Me Not (ワッチョイ 7689-GYY9):2023/10/07(土) 07:23:39.73 ID:TGT5Fgrb0.net
>>83
>>Just a few month into letting their staff work from home over the internet

すでに挙げたいろんな用例を見たらすぐわかるように、ここでの
数詞 + 名詞 into something
は、
He is six feet tall.
She is 17 years old.
と結局は似たような構造だということですね。こういうときの
six feet とか 17 years のことを「副詞的目的格」とでも言うのでしょうか?
あるいは単に「数詞と名詞とを合わせたもの」とでも言うのでしょうか?

six feet tall は "(*) tall by six feet" みたいな意味合い。
(もちろん、そんな英文はあり得ないでしょうけど、無理やりに six feet tall の
厳密な意味合いを英語で表現しろ、と言われたらそう言いたくなります。

17 years old も同じく、"(*)old by 17 years" とでも言いたくなります。
(もちろん、そんな変な英文はありません。17 years old と言えば済むからです。)

"a few months into something" も同じく、"into something for a few months" みたいな
意味合いだと思えばいいのです。「数か月のあいだ(数か月分だけ)、〜という活動に入り込んで」みたいな
意味合いだと思えばいいのです。

96 :😤英ちゃん⛱ (ワッチョイW da86-ISoK):2023/10/07(土) 12:26:52.43 ID:QUvxfItg0.net
いまの教科書にはリンカンて出てるはず
最初にリンカーンてカナ表記したのは誰だ😤

97 :名無しさん@英語勉強中 (ワッチョイW 311e-pzsk):2023/10/07(土) 12:34:37.64 ID:X/26DE1L0.net
えーーーリンカンとかなんかやだな

98 :名無しさん@英語勉強中 (ワッチョイ 959d-OOOs):2023/10/07(土) 12:57:01.59 ID:eV1JHqWd0.net
× リンカーン
○ リンカン、リンキン、リンクン、リンコン、リンケン

99 :名無しさん@英語勉強中 (ワッチョイ 7a69-lHjV):2023/10/07(土) 13:23:55.71 ID:kLsMBFzo0.net
エイブラハム=リンカン Abraham Lincoln(1809〜1865)
第16代アメリカ合衆国大統領(共和党、在任1861〜65年)。1861年からの南北戦争で北部を指導、63年に奴隷解放宣言を出した。第2期途中の65年に暗殺された。アメリカ合衆国では「偉大な解放者」であり、国家の統一と民主主義を守った大統領として尊敬されている。

https://www.y-history.net/appendix/wh1203-047.html

100 :名無しさん@英語勉強中 (ワッチョイ 6196-Hwqz):2023/10/07(土) 14:21:34.60 ID:dqq5V3lG0.net
The rescurers managed to get the driver out of the car wreck safely.

この文なんですが、carとwreckが逆なんじゃないかと思うのですが、この方が自然なんでしょうか?
というか、wreck carよりもwrecked carだといちばんわかりやすい気がするんですが。

101 :😤英ちゃん⛱ (ワッチョイW da86-ISoK):2023/10/07(土) 14:29:42.88 ID:QUvxfItg0.net
Hitlerはヒットラーの方が絶対いい
ヒトラーは原音に決して近くない上、日本語として発音しにくい

102 :😤英ちゃん⛱ :2023/10/07(土) 14:31:43.28 ID:QUvxfItg0.net
>>100
wreck 名詞

103 :The OED Loves Me Not :2023/10/07(土) 14:35:04.18 ID:TGT5Fgrb0.net
>>100
get him out of the wrecked car
get him out of the car wreck

この二つを比べると、確かに wrecked car の方が
出現頻度が高いように思います。しかし car wreck が
間違いというわけでもないでしょう。

https://books.google.com/ngrams/graph?content=out+of+the+wrecked+car%2Cout+of+the+car+wreck&year_start=1800&year_end=2019&corpus=en-2019&smoothing=3

上のグラフを見ていてもわかるように、car wreck は
それなりに使われているのですから、そのまま受け入れるしかないでしょう。

104 :The OED Loves Me Not :2023/10/07(土) 15:28:48.81 ID:TGT5Fgrb0.net
>>100
wreck という言葉を日本人の感覚だけで考えていると
the wrecked car の方が the car wreck よりも自然なような気がしてくるだけであって、
英英辞典で wreck という言葉の意味をきちんと調べれば、
get them out of the ship wreck
get them out of the car wreck
にはまるで問題がないどころか、きちんとした言い回しであることがわかります。

なお、a ruined or destroyed ship という意味での a wreck という言葉がまずは
広く使われ、それからずっと後になって a car wreck という言葉が流布するようになったのだろう
と推測しますので、a car wreck について理解する前に、まずは a wreck あるいは a shipwreck
という言葉の意味を(英和辞典ではなくて)あくまで英英辞典できちんと調べて理解する必要があります。

105 :The OED Loves Me Not :2023/10/07(土) 15:44:47.02 ID:TGT5Fgrb0.net
ここからはあえて anchor なしで投稿します。

wreck という言葉が歴史的にどのように意味が拡張発展していったかを
調べると、面白いだけでなくて、この言葉の意味合いが深く理解できるような気がします。
歴史的な意味範疇の発展と言えば、OED Online で調べるのが一番です。
この下に、wreck という単語やそれに似た単語の、最も古い意味からその後に分化発展した
意味合いについての解説を引用しますが、和訳や日本語での解説はしませんので、あくまで
英文だけを読み取ってご自分で理解してください。

wreck
Law. That which is cast ashore by the sea in tidal waters; esp. goods or cargo as thrown on land
by the sea from a wrecked, stranded, or foundered vessel. Occasionally wreck of the sea
(medieval Latin. wreccum maris, Anglo-Norman wrec or wreck de mer), †wreck of the king (medieval Latin. wrech regis). Cf. shipwreck n. 1, shipbreche n.
(wreck は、この意味では 1425年から使われてきたと書いてあります。)

A vessel broken, ruined, or totally disabled by being driven on rocks, cast ashore,
or stranded; a wrecked or helpless ship; the ruins or hulk of such. Occasionally also wreck of a ship.
(この意味合いで wreck という言葉が使われたのは、1500 年からだということです。)

それからずっと後になってから、船から自動車や列車の事故にもこの wreck が使われるようになったそうです。

II.9.c.
1912–
North American. A road or railway accident.

1979
On the drive back to Springville, she was dreaming away and got in a wreck. Nobody was hurt but the car.
N. Mailer, Executioner's Song (1980) i. xviii. 304

106 :The OED Loves Me Not :2023/10/07(土) 15:55:03.23 ID:TGT5Fgrb0.net
get someone out of the car wreck という一節の文脈の中での
a car wreck という言葉の意味合いを本気で理解しようと思ったら、
まずは a ship wreck あるいは a shipwreck という言葉の意味合いを考える必要があります。

shipwreck
1. What is cast up from a wreck; the remains of a wrecked vessel; wreckage.
(wreck という言葉は、こういう意味合いで Old English の時代から使われてきたそうです。)

そういう意味の a ship wreck や a shipwreck という言葉がまずはあって、それからおそらくは
何百年も経ってから、路上の馬車での wreck などに使われ、20世紀あたりになってから
列車での wreck という意味合いに発展していったのでしょう。

そういうことを踏まえれば、今回の
get someone out of the car wreck
は、あくまでも
get someone out of the wrecked car
というか
get someone out of the remains of the wrecked car
get someone out of what is left of the wrecked car wreckage
という意味なんだということがわかります。

107 :名無しさん@英語勉強中 :2023/10/07(土) 16:18:39.88 ID:b0dnlASd0.net
>>100
どっちでもいいよ どっちが自然とかもないし言ってることは全く同じ
car wreckってのは日本語で言えば車の事故って言い方ってだけ

108 :😤英ちゃん⛱ :2023/10/07(土) 17:26:37.65 ID:QUvxfItg0.net
そうそう
wreckは、OEDか研究社の語源辞典でちゃんと語源調べないと意味を把握できない難語だからな
やっすいオンライン辞書引いて喜んでる輩は反省するように😤😤😤😤

109 :名無しさん@英語勉強中 (ワッチョイ 7ace-lHjV):2023/10/07(土) 18:32:05.46 ID:kLsMBFzo0.net
(A) get the driver out of the car wreck
(B) get the driver out of the wrecked car

(A) 自動車事故からドライバーを救い出す
(B) 大破した自動車からドライバーを救い出す

(A) は事故現場を指し、(B) は自動車そのものを指す

110 :名無しさん@英語勉強中 :2023/10/07(土) 18:40:12.91 ID:kLsMBFzo0.net
Oxford Learners' Dictionary

4. wreck (North American English)
an accident in which a vehicle hits something, for example another vehicle, usually causing damage and often injuring or killing the passengers

- a car/train wreck

111 :😤英ちゃん⛱ :2023/10/07(土) 19:14:56.20 ID:QUvxfItg0.net
そういうことを踏まえれば、今回の
get someone out of the car wreck
は、あくまでも
get someone out of the wrecked car
というか
get someone out of the remains of the wrecked car
get someone out of what is left of the wrecked car wreckage
という意味なんだということがわかります。

------------

↑これはOEDの語源情報のなにを踏まえてる?🤔
何も踏まえてない結論に見えるし、そもそも語源情報を踏まえなくてもこんな意味だって分かるよね🤔

112 :名無しさん@英語勉強中 (ワッチョイW 85da-pRTu):2023/10/07(土) 20:33:17.91 ID:vXgd9VPM0.net
the car wreck と the wrecked car を比べると
the car wreck の方が車の原形をとどめていない感じがするかな

113 :100 (ワッチョイ 6196-Hwqz):2023/10/08(日) 00:11:04.50 ID:7HtPjd8q0.net
wreck carについて質問したものです。

みなさまありがとうございました。

114 :名無しさん@英語勉強中 (ワッチョイ 7a6b-fmck):2023/10/08(日) 00:28:12.43 ID:aE6XV0N10.net
回答者のレベルがあまりにも低いので
<< The OED loves 英語職人 >>
これら老人2人による自作自演の質問しか来なくなってしまった

115 :外人 :2023/10/08(日) 07:28:38.23 ID:tAaf10tgH.net
sup

116 :外人 :2023/10/08(日) 07:29:14.45 ID:tAaf10tgH.net
Lol it was giving me an error

117 :外人 :2023/10/08(日) 07:29:22.69 ID:tAaf10tgH.net
Lol it was giving me an error

118 :外人 :2023/10/08(日) 07:45:51.70 ID:tAaf10tgH.net
https://i.imgur.com/BLod848.jpg
前スレのブタ耳さんの質問への回答です

どうせ説明できないものだし、文法などのルールを覚えることで自然に話せるようになることはありません。
言語は真似でできちゃうものだから、理由とか関係なく、ネイティブが実際言うものをそのまま真似すればいいだけの話。

119 :名無しさん@英語勉強中 :2023/10/08(日) 10:28:22.46 ID:uJNg/iV70.net
自然とか真似よりも、意味が重要だろう

120 :名無しさん@英語勉強中 :2023/10/08(日) 12:38:52.40 ID:aE6XV0N10.net
>>118

ブタ耳は相手にするな!言っていることはいつもトンチンカンw
レスをするだけ無駄だ

121 :名無しさん@英語勉強中 (ワッチョイ 85c0-7tk0):2023/10/08(日) 13:17:36.90 ID:2QpxfpAp0.net
(North American English) (also crash British and North American English ) an accident in which a vehicle hits something, for example another vehicle, usually causing damage and often injuring or killing the passengers

a car/train wreck

謎は無かった

122 :The OED Loves Me Not (ワッチョイ 7689-GYY9):2023/10/08(日) 13:47:22.36 ID:SpvdgAoT0.net
a car wreck などは、確かに North American English ではそういう意味として使われるように
なってしまったってことですよ。

世界中に流布している British English では、それ以前の伝統的な意味合いで使われているということです。

勘違いしている人がたくさんいて、まるでそれですべてが解決しているかのように思っている人が主流を占めている
かのように見えますので、あえて一言、言っておきました。

馬鹿馬鹿しくて相手にする気にはなれなかったのですが。

123 :😤英ちゃん⛱ (ワッチョイW da86-ISoK):2023/10/08(日) 13:51:42.30 ID:kQANtds50.net
外人は777の亜型やな

124 :The OED Loves Me Not (ワッチョイ 7689-GYY9):2023/10/08(日) 15:52:53.58 ID:SpvdgAoT0.net
a wrecked car のみならず、a car wreck も「事故車」という意味で使うのが、本来の英語での用法だ
ということを示す証拠。

car wreck
(a) A wrecked car (now usually a motor car); (b) = car crash n.
(この上の (a) の項目で、はっきりと a car wreck が「事故車」という意味だと書いている。)

1877
A few days previous to his death on the terrible ★car-wreck★ at Ashtabula.
(ここでも、his death on the... car-wreck と書いているから、自動車事故ではなくて事故車だとわかる。)
Sunday School Journal September 223/1Citation details for Sunday School Journal

1961
He accuses them of..selling ‘written off’ ★car wrecks★.
(これは、はっきりと「事故車」という意味だということがわかる。)
Sunday Express 12 February 9/2Citation details for Sunday Express

2007
Employees who become romantically involved could be caught in a real ★car wreck★.
(こっちの方は、「事故車」なのか「自動車事故」なのかははっきりしないと思う。)
J. R. Ellig & W. J. Morin, Driving Career Highway 155Citation details for J. R. Ellig & W. J. Morin, Driving Career Highway

(上記の記事は、OED Online の 2012 年における改訂後のもの)

125 :名無しさん@英語勉強中 (ワッチョイ 7a5a-lHjV):2023/10/08(日) 17:53:53.68 ID:aE6XV0N10.net
The rescurers managed to get the driver out of the car wreck safely.

Google: 救助者らはドライバーを自動車事故から安全に救出することができた。
DeepL: 救助隊はなんとか運転手を事故車から無事救出した。
Weblio: 救助隊は、なんとか問題なくドライバーを自動車事故から取り出すことができました。

126 :名無しさん@英語勉強中 (ワッチョイ 7a5a-lHjV):2023/10/08(日) 17:58:56.76 ID:aE6XV0N10.net
The rescuers managed to get the driver out of the car wreck safely.

Google: 救助者らはドライバーを自動車事故から安全に救出することができた。
DeepL: 救助隊はなんとか運転手を事故車から無事救出した。
Weblio: 救助隊は、なんとか問題なくドライバーを自動車事故から取り出すことができました。

127 :名無しさん@英語勉強中 (ワッチョイ 7aa8-lHjV):2023/10/08(日) 20:06:28.30 ID:aE6XV0N10.net
He was rescued from the car wreck.

Google: 彼は自動車事故から救出された。
DeepL: 彼は事故車から救出された。
Weblio: 彼は、自動車事故から救い出されました。
Bing: 彼は自動車事故から救出された。
Mirai: 彼はその自動車事故から救出された。

128 :The OED Loves Me Not (ワッチョイ 7689-GYY9):2023/10/08(日) 22:32:47.12 ID:SpvdgAoT0.net
「自動車事故」と「事故車」と比べたら、「事故車」は堅くて
日常的にはあまり使わないから、それを機械翻訳が
避けようとするのは当たり前です。

今さっき私が問題にしてたのは、あくまで原文の意味がどちらで
あるかを言っているだけで、どちらの和訳の方が日常的な
次元の日本人にとっって耳触りがよいかという話ではありません。

意訳をしたら、結局はここでは日常的にどっちを日本人が選びそうかといえば、
「自動車事故」が勝つに決まっています。

本来の意味はどちらなのかを問題にしましょう。

129 :The OED Loves Me Not (ワッチョイ 7689-GYY9):2023/10/08(日) 22:41:23.16 ID:SpvdgAoT0.net
Good morning. という挨拶が、本来はどういう意味なのかを議論するとき、
それはおそらくは I wish you a good morning. か、あるいは
It's certainly a good morning. とか、あるいは
It's a good morning, is it not? とかいう挨拶を大昔はしていたのだろうから、
それが Good morning. と縮まったのだろうというのが妥当な考えだろう。

そういうときに、いやそれは「おはよう」という意味だ。なぜなら機械翻訳で
Good morning. を打ち込めば「おはよう」と出てくるからだ、
なんていうのは笑止千万だろう?

今の議論は、Good morning. という挨拶の本来の意味を言っている。
日本語の「おはよう」は、「貴殿のお出ましは、まことにお早う御座いますな」
とか、あるいはそれに似たような挨拶が大昔からあって、それが縮まったのだろうと
思われる。

その日本語と英語とをそのまま機械翻訳にかけて、
「Good morning の本来の意味はあくまで "お早う" という意味だ」
などと言うのは変だ。

130 :The OED Loves Me Not (ワッチョイ 7689-GYY9):2023/10/08(日) 22:43:47.70 ID:SpvdgAoT0.net
こんなことを言い始めると、またもや英語職人や英太郎が得意な
屁理屈どうしの平行線の意地の張り合いになるので、
ここでやめます。これについては、もう私は身を引きますので、
あとは好き勝手に言ってればよろしい。

131 :🫥英ちゃん😶‍🌫 (ワッチョイW da86-ISoK):2023/10/08(日) 22:54:11.95 ID:kQANtds50.net
【理想】

僕は知識や真実を追求することを自分の使命だと感じているからだ。
真実のために生き、真実のために死ぬことが僕の使命なのだ。

【現実】

自分に甘い助言だけを聞き、真実から目を背ける背徳のしもべ

132 :🫥英ちゃん😶‍🌫 (ワッチョイW da86-ISoK):2023/10/08(日) 23:04:15.20 ID:kQANtds50.net
Donnernの件の反応

【真実を追求する人】
わが浅学を恥じます。私めを正しい道へと導いてくださって有難うございます🙇

【欺瞞の下僕】
ボクに読んで欲しいからコテハン外した!
卑怯だ!

は?😐🖕

133 :名無しさん@英語勉強中 (ワッチョイ 7acb-lHjV):2023/10/09(月) 00:11:10.21 ID:CIL9rFWp0.net
DeepL だけが「事故車」という表現を使うのが不思議だ

134 :名無しさん@英語勉強中 (ワッチョイW 8dc0-CCpT):2023/10/09(月) 02:26:59.47 ID:NTvheLgT0.net
>>124 の通り「事故車」という意味で確かに使われている例があろのだし
人を事故から救うって英語で表現するとき
get 人 out of 事故 なんて言わないのでは?
当該英文の解釈はoedさんが正しいと思うな。

135 :名無しさん@英語勉強中 (ワッチョイW 7ac8-pRTu):2023/10/09(月) 02:37:08.63 ID:vp3v08AY0.net
「車の残骸」から運転手をなんとか助け出した。
の意だから、DeepL の方があってる

136 :The OED Loves Me Not (ワッチョイ 7689-GYY9):2023/10/09(月) 06:42:10.74 ID:tB75RAsp0.net
a car wreck と言うときに、これを確かに

(1) 自動車事故(北米でよく使われる用法)
(2) 事故車(の残骸)(イギリス英語を中心に世界中で使われているように見える用法)

という二通りの解釈があります。

その一方で、
I'm a wreck. (私は、悲しくてつらくてボロボロだ)
などという台詞が映画などでよく出てきます。

哀しい人が「私は事故(難破)だ」というような発想をするでしょうか?
これはあくまで「私は事故車(難破船)の残骸みたいにボロボロだ」という
意味で言っているのでしょう。

そういう、ごく日常的な表現の中にも、何百年もの大昔から難破船の残骸のイメージ
がイギリス人(そして後の新大陸の人々)の意識に定着し続けてきて、
それが現代でも話し言葉として使われているのだという気がします。

その延長線上に、今回の
get someone out of a car wreck
という表現もあるのだというのが、私の最初に感じたことだったのです。
だからこそここでは、a car wreck が「自動車事故」というような
抽象的な、手で触れないものを題材にして、私は事故だ、なんていう
発想を人はしないと思うのです。

つまり事故そのものは目に見えないものです。あくまで「私は、事故や難破のあとに
出てくる船や馬車や自動車の残骸みたい二無茶苦茶だ」という言葉を、
目の前にありありとそのイメージを空想しながら(少なくとも昔は)使っていたはずなのです。
現代でも、自動車事故のあとに無茶苦茶になった人間の死体とか自動車の残骸を
見たことのある人なら、I'm a wreck. という台詞を口にするとき、そういうイメージを
脳裏にまざまざと浮かべるはずです。get someone out of a car wreck も、その延長線上にあるのです。

137 :The OED Loves Me Not (ワッチョイ 7689-GYY9):2023/10/09(月) 06:57:26.43 ID:tB75RAsp0.net
なお、さらに検索を続けると、
get [someone] out of an (OR the) accident
のような言い回しも(正しい英語なのかどうかは知りませんが)ネット上で見つかります。

Normally, as a general rule, we say that there's a 10 minutes golden rule
that we have to play with this patient in order to ★get them out of the accident★
and so the 20 minutes for us was a goal.
https://www.7newsbelize.com/sstory.php?nid=40007

Our volunteers are put through special training to ★get someone out of an accident★,” said Barclay.
(上の用例のリンク先に行っても、その記事の全文をアクセスすることができません。会員制だろうと思います。)

get someone out of an accident という言い回しが仮に使われているとすれば、
get someone out of a car wreck の car wreck の部分が「事故車(の残骸)」ではなくて
「自動車事故」という意味で解釈しながら使っている人もいるかもしれない、という想定が成り立ちます。

ただ今のところは、それを裏付ける証拠が少なすぎます。

138 :The OED Loves Me Not (ワッチョイ 7689-GYY9):2023/10/09(月) 07:06:34.72 ID:tB75RAsp0.net
get someone out of a car wreck について考えるとき、
get someone out of an accident ならどうかを検討することは役立つと思って
検索を続けています。そして、次のような用例は見つかります。

Two adults have been ★rescued from an accident★ involving a truck and a trailer at the PWD Bus Stop in the Agege Motorway area of Ikeja, Lagos State.
https://punchng.com/two-rescued-in-lagos-road-crash/

ただしこれは、Nigeria の新聞での記事です。Nigeria 人やインド人は日本人の 5,000 倍くらい英語が上手ですが、
新聞紙上でも平気で間違い英語を書くこともありますので、注意が必要です。
あくまで純然たる英語圏での文献を対象にしないと、信頼性のある結論を出すことはできません。

だからこそ余計に、機械翻訳の結果などを元にしてこういう厳密な調査はできないのです。
機械翻訳はあくまで、「おおざっぱな、だいたいの意味がわかればそれで万歳」の世界です。

139 :The OED Loves Me Not (ワッチョイ 7689-GYY9):2023/10/09(月) 07:16:33.05 ID:tB75RAsp0.net
rescue someone from an accident という言い回しは、さっきの Nigeria の
新聞だけでなく、他のいくつかの Nigeria の新聞にも出てきます。さらには
Ecuador の新聞にも出てきます。そうすると、純然たる英語圏では出てこないのかと
思っていると、次のように、アメリカの新聞と思われるものからも出てきます。

2 ●rescued from overturned vehicle● in East Falls
Published May 2, 2016U.S.FOX 29 Philadelphia

EAST FALLS (WTXF) - Two people were ★rescued from an accident★ involving an overturned vehicle in East Falls early Monday morning.
https://www.fox29.com/news/2-rescued-from-overturned-vehicle-in-east-falls

上の記事の1行目では rescue someone from an overturned vehicle という
言い回しを(少し変えてありますが)使っています。だからやはり rescue someone from [乗物など]
という言い回しが主流なのかと思っていると、そのあとの本文では、rescue someone from an accident
という言い回しを使っています。

こうなると、accident という言葉が「事故」という抽象的な、手で触ることのできない現象を
指す意味から、「事故の結果としての車の残骸など」という具体的に、手で触れる即物的な
意味合いへとシフトすることもあるのかもしれないと思われてきます。

もし仮に rescue someone from an accident という言い回しがごく普通なのであれば、
get someone out of an accident もごく普通なのかもしれず、もしそうであるなら
get someone out of a car wreck の wreck が「事故」ではなくて「事故車の残骸」という
意味で英語ネイティブたちが解釈しているという考えが成り立つことになります。

140 :The OED Loves Me Not (ワッチョイ 7689-GYY9):2023/10/09(月) 07:40:13.89 ID:tB75RAsp0.net
>>accident という言葉が「事故」という抽象的な、手で触ることのできない現象を
>>指す意味から、「事故の結果としての車の残骸など」という具体的に、手で触れる即物的な
>>意味合いへとシフトすることもあるのかもしれないと思われてきます。
===

OED Online で確かめてみます。

accident

II.8.b.
1747–
A person injured as the result of a mishap.

1991
In 1847 the Secretary was asked to prepare a detailed return of such ★accidents★ admitted since 1844.
A. Rook et al., Hist. Addenbrooke's Hosp., Cambridge xv. 135

上の用例での accidents は、「事故や災害での被害者たち」を指しています。

II.8.e.
1850–
colloquial. An event which leads to an unplanned pregnancy, the pregnancy itself;
★a child conceived or born as a result of an unintended pregnancy.★ Cf. mistake n. d.

2006
His father took sadistic pleasure in reminding him that he was ★an ‘accident’★ and that his mother wanted to take him to an orphanage when he was born.
M. B. Frisch, Quality of Life Therapy i. iii. 36/2Citation details for M. B. Frisch, Quality of Life Therapy

この上の解説や用例では、「生むつもりはなかったのに生まれてきてしまった子供」という意味です。

141 :名無しさん@英語勉強中 (ワントンキン MMea-nQTY):2023/10/09(月) 07:41:17.79 ID:XT0z3vo3M.net
まあそんなカッカするなよ
それよりhuman disqualifiedはもうやめかね?

142 :The OED Loves Me Not (ワッチョイ 7689-GYY9):2023/10/09(月) 07:44:46.80 ID:tB75RAsp0.net
上に示したように、OED Online によれば、accident(s) が
「偶然に生れてしまった子供」とか「災害や事故で負傷した人たち」
という意味で使うことがあるということがわかります。しかし
「事故車(の残骸)」という意味は載っていません。

ですから、今のところは
rescue someone from an accident
という言い回しの accident が抽象的な「事故」ではなく
「事故車(の残骸)」を意味することもあるという
記述を、私は辞書などの中で見つけたことはありません。

もちろん、辞書にないからといって「accident という言葉を
そういう意味で使うことはない」とは断言できないことはもちろんです。

143 :英語職人 ◆azN58fWNzw (ワッチョイ 3de0-6t2b):2023/10/09(月) 08:47:59.48 ID:ngrkp1Dr0.net
>>130

おい、OED爺よ、屁理屈とはなんだ?私が言ってることはいつもまっとうな理論だ。それが屁理屈に見えるのは、OED爺の英語力がないからだろ。😠

144 :英語職人 ◆azN58fWNzw (ワッチョイ 3de0-6t2b):2023/10/09(月) 08:59:20.14 ID:ngrkp1Dr0.net
They managed to get the driver out of the car wreck safely.
&#9679;プロの翻訳者なら、これを「車の事故」とは訳さない。その理由を下記に示す。OED爺は的外れな冗長な屁理屈のような説明をながながとせずに、下記のことだけ言えばいんだよ。これは、語源なんか関係しない問題なのだから。

小学生の国語の問題 どっちが自然な日本語か?
A: 運転手を無事、車の事故から救出した
B: 運転手を無事、事故車から救出した。
小学生でもほぼ全員がBを選ぶぞ。

事故はすでに起きているので、どうやって事故から救出できるんだよ?
Aが成立するのは、タイムマシーンで事故前に戻り、事故を起こさないように歴史を変えた場合のみ正しい。🤠

145 :英語職人 ◆azN58fWNzw (ワッチョイ 3de0-6t2b):2023/10/09(月) 09:06:49.85 ID:ngrkp1Dr0.net
SF小説なら、>>144のAの訳もありうる。これに気づけた人は頭がいい。🤠

146 :英語職人 ◆azN58fWNzw (ワッチョイ 3de0-6t2b):2023/10/09(月) 09:14:46.43 ID:ngrkp1Dr0.net
>>127

DEEPL、すごい!えらい! 🤩

147 :英語職人 ◆azN58fWNzw (ワッチョイ 3de0-6t2b):2023/10/09(月) 09:54:58.06 ID:ngrkp1Dr0.net
>>144の追記

144のAが正しくなるケースをもう一つ追加。
現在事故が発生中の場合。たとえば、現在延焼中の家屋から人を救い出すのような、まだ事故が継続している場合は、「事故から救い出す」は可能。
しかし、車の事故のような一瞬で事故が終息するケースでは、「事故から救い出す」の表現は不適切。

148 :英語職人 ◆azN58fWNzw (ワッチョイ 3de0-6t2b):2023/10/09(月) 10:09:23.35 ID:ngrkp1Dr0.net
get the driver out of the car wreck safely.は、もし「衝突により車が燃えていてそこから救出する」という特殊な例なら、>>144のAの訳でもよいだろう。

149 :英語職人 ◆azN58fWNzw (ワッチョイ 3de0-6t2b):2023/10/09(月) 10:12:52.18 ID:ngrkp1Dr0.net
■□■ まとめ (最終見解) ■□■
They managed to get the driver out of the car wreck safely.の訳について。(これは、結構複雑な問題で、場合分けをして考えないといけないケースである。)

《衝突による自動車事故なら》
 (〇)運転手を無事、事故車から救出した。
  (×)運転手を無事、自動車事故から救出した。

《タイムマシーンで事故を起こさないように歴史を変えた》 SF小説の場合
  (×)運転手を無事、事故車から救出した。
  (〇)運転手を無事、自動車事故から救出した。

《衝突によって車が燃えだし、その燃えている最中にドライバーを助けた》
 (〇)運転手を無事、事故車から救出した。
 (〇)運転手を無事、自動車事故から救出した。

150 :名無しさん@英語勉強中 (テテンテンテン MM0e-OOOs):2023/10/09(月) 10:16:01.79 ID:DxzJWYQSM.net
どっちが自然な日本語訳になるかという問題以前に
car wreck が、「事故車」という意味として実際に使われるのかを検証することが先決であるに決まっている。

>>109-110 では、これは「自動車事故」という意味だと主張されている。
そして
>>137 では
get [someone] out of an (OR the) accident という言い回しが存在することが示されている。
これは、事故現場から救出するという意味であろう。

151 :名無しさん@英語勉強中 (テテンテンテン MM0e-OOOs):2023/10/09(月) 10:25:59.61 ID:DxzJWYQSM.net
だから語源的に遡って
car wreck が、「事故車」という意味として実際に使われることを示すことは重要。

私はOED氏の解釈が正しいと思う。
get the driver out of the car wreck safely
の car wreck は「事故車」という意味だ。
燃えていようがなかろうが、「事故車」と解すべきだ。

152 :英語職人 ◆azN58fWNzw (ワッチョイ 3de0-6t2b):2023/10/09(月) 10:36:45.72 ID:ngrkp1Dr0.net
>>150
やっぱりテテンちゃん、予想通り、がまんできなくなって再登場してきたね。😄

>「事故車」という意味として実際に使われるのかを検証することが先決
検証するまでもなく、英英辞典を見れば「事故車」「自動車事故」の2つの意味があることは明らかだろう。

>get [someone] out of the accident これは、事故現場から救出するという意味であろう。
「私は彼を事故現場から救出した。」は英作すると次のように言う。
 (〇)I rescued him from the scene of the accident.
  (〇)I rescued him from the accident cite.
勝手に意味を加えないように。

> 燃えていようがなかろうが、「事故車」と解すべきだ。
燃えている最中の車から救出したら、事故が発生中なのだから、「自動車事故から救った」という訳を否定できる論拠はない。>>149が正しいのだ。

153 :英語職人 ◆azN58fWNzw (ワッチョイ 3de0-6t2b):2023/10/09(月) 10:39:49.96 ID:ngrkp1Dr0.net
とにかく、『いかなる場合も、「自動車事故」と訳してはいけない』は、間違い。

154 :名無しさん@英語勉強中 (テテンテンテン MM0e-OOOs):2023/10/09(月) 10:45:19.11 ID:DxzJWYQSM.net
>>152
>>get [someone] out of the accident これは、事故現場から救出するという意味であろう。


では、>>137 の
Normally, as a general rule, we say that there's a 10 minutes golden rule
that we have to play with this patient in order to ★get them out of the accident★
and so the 20 minutes for us was a goal.

を訳してもらえますか?
事故現場から救出したという意味ではないのですか?

155 :名無しさん@英語勉強中 (テテンテンテン MM0e-OOOs):2023/10/09(月) 11:00:41.30 ID:DxzJWYQSM.net
訂正
×事故現場から救出したという意味ではないのですか?
○事故現場から救出するという意味ではないのですか?

156 :英語職人 ◆azN58fWNzw (ワッチョイ 3de0-6t2b):2023/10/09(月) 12:19:11.84 ID:ngrkp1Dr0.net
>>154
訳「患者を事故から救い出す(get them out of the accident)ためには、守るべき「10分間ルール」というものがある。」
どうみても、「車から引きずり出す」の意味で、現場から立ち去る時間じゃないよ。これは、car wreckも自動車事故と訳せるという根拠にはなるが、「事故現場を離れる」の根拠にはならない。これが、現場から立ち去る時間のことを言っている理由を言ってみてよ。
実際、下記より最後の患者が現場を立ち去ったのは35分後だからね。ちゃんと「left the sene」と使ってるからこれは現場を離れたの意味。

Reporter:"In terms of time and the response of different authorities, I counted, it's about &#9733;35 minutes&#9733; that the person who died, until that person left the scene, is that good enough?"

さらに訓練担当者は、つぎのように文を締めくくっている。
The 30 minutes, additional for taking out the patient, well that has nothing to with us, it's just the police and the fire department to get them out but, for us 20 minutes for 8 patients was very good."
「30分というのは、我々には関係がない。それは消防署員らが患者を現場から外へ連れて行く時間である。我々の20分のうちに8人を救助できたことはすばらしかった」

この記事の発言者(訓練担当者)は、患者が事故現場から外へ出される時間は、自分たちの功績とは関係がないと言い切っているのである。😃

157 :名無しさん@英語勉強中 (ワッチョイ 7aed-lHjV):2023/10/09(月) 12:48:45.32 ID:CIL9rFWp0.net
What Is the Legal Definition of a Car Wreck?
https://cummingsinjurylaw.com/car-accidents/what-is-the-legal-definition-of-a-car-wreck/

Cambridge Dictionary: <car wreck> collocation, meaning and examples of use
https://dictionary.cambridge.org/example/english/car-wreck

YouTube: Superman & Lois 3x10 Jordan saves sarah from a car wreck
https://youtu.be/evSxzZjLN3Y?si=gyv7ax7xZ0GIJK-U

映像で save 人 from a car wreck の意味が分かる

158 :英語職人 ◆azN58fWNzw (ワッチョイ 3de0-6t2b):2023/10/09(月) 12:56:54.32 ID:ngrkp1Dr0.net
■□■ まとめ (最終見解)修正版 ■□■
They managed to get the driver out of the car wreck safely.の訳し方について。

《衝突による自動車事故》 
(正解)運転手を無事、事故車から救出した。
(準正解)運転手を無事、自動車事故から救出した。※間違いであるとは言い切れない。

《タイムマシーンで事故を起こさないように歴史を変えた》 SF小説の場合
  (不正解)運転手を無事、事故車から救出した。
  (正解)運転手を無事、自動車事故から救出した。

《衝突によって車が燃えだし、その燃えている最中にドライバーを助けた》
 (正解)運転手を無事、事故車から救出した。
 (正解)運転手を無事、自動車事故から救出した。

159 :名無しさん@英語勉強中 (ワッチョイ 7ac6-lHjV):2023/10/09(月) 14:30:29.87 ID:CIL9rFWp0.net
wreck (noun) a vehicle or ship that has been destroyed or badly damaged
- Her car was a worthless wreck.

どうやら日本語の「事故車」は英語では a wreck 1語で表現できるようだ。
あるいは a damaged car, accident car, wrecked car とか表現できる。

つまり、a car wreck は「自動車事故」の意味であり、
「事故車」の意味はないようだ。

160 :英語職人 ◆azN58fWNzw (ワッチョイ 3de0-6t2b):2023/10/09(月) 23:15:52.74 ID:ngrkp1Dr0.net
>>159
https://pixta.jp/photo/70193170

ここの画像のタイトルは、「Rusty old car wreck」だ。これは、「事故車」じゃなくて、「自動車事故」って訳すのでしょうか?🤠
car wreckには、「事故車」と「自動車事故」の2つの意味があることは英英辞典から明白なのだ。

《get the driver out of the car wreck》
追突で燃えている最中の車から助けるときは、「火からも助ける」という意味を出したほうが適切なので、「自動車事故から救出」と訳すとよい。
でも、ふつの衝突事故なら、「事故車から助け出す」と訳すとよい。
「自動車事故から救出」は(>>137のリンク先の記事を読むまでもなく)「事故車から救出」の意味を少なからず内在している。よって、SF小説でもない限り、「自動車事故から救出」を完全には否定できない。
要は、話し手が事故車に焦点を当ていると思われるなら「事故車から救出」、車が炎上中のような自動車事故全体に焦点を当てているなら「自動車事故から救出」と訳したほうが良い。つまり、>>158が正しい。

161 :🫥英ちゃん😶‍🌫 (ワッチョイW da86-ISoK):2023/10/10(火) 00:58:04.66 ID:qM4E4X060.net
テテンがちゃっかり帰って来て草

162 :名無しさん@英語勉強中 (ワッチョイW 5af0-JtJs):2023/10/10(火) 01:17:43.01 ID:V3ORYExE0.net
LRがいまだに聞き分けられないんですけど皆さんどうしてますか?

した対策などでおすすめのものがあれば教えてください

163 :The OED Loves Me Not (ワッチョイ 7689-GYY9):2023/10/10(火) 04:33:51.73 ID:aWF39PAB0.net
L と R とが聴き分けられない人は、たぶんその二つを
発音し分けることができないんだろうと思います。
発音し分けられないのに、聴き分けることだけが
できる人はいないんじゃないかという気がします。

さらには、英語を習い始めたのが通常は 12 歳だとすると、
15 歳までの3年間に NHK の英語関係の番組を
真剣に毎日毎日、見続け、ネイティブの発音を真似し続けないと、
絶対に英語発音の基礎は身に付かないと思います。

そういう経験がなかった人は、おそらく
20歳以降にいくら頑張っても L や R のみならず、
英語ネイティブの発音を曲がりなりにも真似たり聴き分けたり
することは死ぬまで無理だと諦めて、読み書きだけに特化した方が
得策だと思います。やっている本人が苦しいばかりなのです。

いろんな人に発音を教えて来た経験上、そのように思っています。

164 :The OED Loves Me Not :2023/10/10(火) 05:18:07.34 ID:aWF39PAB0.net
>>159
>>124 での OED Online の記述をお忘れですか?
それとも、OED Online の記述は間違いだというご指摘ですか?

165 :名無しさん@英語勉強中 :2023/10/10(火) 11:09:07.28 ID:Z723lpx701010.net
>>162
LR以外は聞き分けられるの?
LRの発音の仕方を知っているの?
発音の仕方についての本などを読んだことあるの?

166 :名無しさん@英語勉強中 :2023/10/10(火) 12:02:03.68 ID:mhEW25tJM1010.net
>>156
「事故現場を離れる」という意味とは言っていないつもりです。
おっしゃる通り、そういう意味でないことは、前後の文脈を見れば明らかです。

そして
I rescued him from the scene of the accident.
I rescued him from the accident cite.
も、「事故現場を離れる」という意味はないでしょう。

get [someone] out of the accident は「事故現場から救出する」という意味だと言いましたが
具体的には、車の残骸から救出したり、、投げ出されて何かの下敷きになっていればそこから救出するといった
意味でしょう。

>>144
>事故はすでに起きているので、どうやって事故から救出できるんだよ?
>Aが成立するのは、タイムマシーンで事故前に戻り、事故を起こさないように歴史を変えた場合のみ正しい。
というコメントがあったので
get [someone] out of the accident は直訳すれば「事故から救出する」といったものになるが
「事故現場から救出する」という意味合いなら、タイムマシーン云々は関係なく意味が通るという旨で書きました。

167 :名無しさん@英語勉強中 :2023/10/10(火) 14:40:13.04 ID:RTX++JxX01010.net
a car wreck は「自動車事故」の意味であり
「事故車」の意味はない。

ただし、wreck には「残骸、廃墟」の意味があり、
Rusty old car wreck は「自動車の残骸、廃墟」の意味であり
この状態から人間を救出することは不可能である。

https://pixta.jp/photo/70193170

168 :名無しさん@英語勉強中 :2023/10/10(火) 15:48:23.79 ID:gCYleIgp01010.net
名詞wreckには@「事故(an accident in which a moving vehicle hits something and is damaged or destroyed)」という意味もあるし、A「残骸(the remains of something that has been destroyed)」という意味もある。

その前に名詞carを前に付けたら、名詞carの形容詞的用法で、@「車の事故」あるいはA「車の残骸」になる。無論、いずれも可。

これに対し日本語のB「事故車」は、逆に「事故」が「車」を修飾している形になり(「事故った車」)、そう言う種類の車を意味する語になる。こっちの方がむしろ「厳密」には「car wreck」の和訳として直接対応していない意訳になる。

そもそも日本語の「事故車」というのは単に交通事故を起こした車を指すのではなく、もっぱらヤバい事故歴のある(フレーム部分を損傷し損傷部を交換または修理した経歴のある)中古販売車両のことを意味する。

169 :名無しさん@英語勉強中 :2023/10/10(火) 18:30:41.77 ID:RTX++JxX01010.net
He was rescued from the car wreck.

Google: 彼は自動車事故から救出された。
DeepL: 彼は事故車から救出された。
Weblio: 彼は、自動車事故から救い出されました。
Bing: 彼は自動車事故から救出された。
Mirai: 彼はその自動車事故から救出された。

この中で DeepL だけ間違えたのは、
a wreck だけで「事故車、ポンコツ車」の意味になるので、
それにとらわれて誤訳したと思われる。

170 :英語職人 :2023/10/10(火) 19:39:57.32 ID:RA8jLQ3r01010.net
>>168 >>169
car wreckは、正確には、「自動車事故」という意味の他に、「難破船」ならぬ「難破車」という意味がある。>>124参照
衝突して破損した車は、「難破車」(事故で破損した車)と言える。別に錆びついた残骸車でなくてもいい。だから、「難破車から救出する」は十分ありえる英語だ。

証拠
https://www.dailymail.co.uk/news/article-10378143/Leichardt-Sydney-Shocking-moment-woman-screams-pain-shes-pinned-beneath-overturned-Toyota.html
↑ここに「A 34-year-old woman had to be cut out of the car wreck by Fire and Rescue NSW officers」
これは明らかに「難破車」の意味で使っている。「自動車事故」の意味ならbe cut out of ~ (~から車体を切断することで取り出される)という表現は使わない。

この難破車のことを、これまで「事故車」と呼んできたが、厳密には間違いではない。(下記リンク先に「事故で被害にあったクルマ」とある)
https://www.weblio.jp/content/%E4%BA%8B%E6%95%85%E8%BB%8A
しかし、誤解を与えるので、「事故車」よりも「破損した車」のほうがよいのかもしれない。

171 :英語職人 :2023/10/10(火) 19:43:28.61 ID:RA8jLQ3r01010.net
■□■ まとめ (最終見解)修正版2 ■□■
They managed to get the driver out of the car wreck safely.の訳し方について。

《衝突による自動車事故》 
(正解)運転手を無事、事故で破損した車から救出した。
(準正解)運転手を無事、自動車事故から救出した。※間違いであるとは言い切れない。

《タイムマシーンで事故を起こさないように歴史を変えた》 SF小説の場合
  (不正解)運転手を無事、事故で破損した車から救出した。
  (正解)運転手を無事、自動車事故から救出した。

《衝突によって車が燃えだし、その燃えている最中にドライバーを助けた》
 (正解)運転手を無事、事故で破損した車から救出した。
 (正解)運転手を無事、自動車事故から救出した。

172 :英語職人 :2023/10/10(火) 20:04:53.07 ID:RA8jLQ3r01010.net
>>167

>>170を読んで、復習しておきましょう😃


それにしても、いつまで、こんなサーバ状態つづくの? 書き込むだけで、大変なんだが・・・🤠

173 :英語職人 :2023/10/10(火) 20:08:05.89 ID:RA8jLQ3r01010.net
こんな書き込むのが大変な状況で、アンカーの矢印が全角だったよ😭

修正版

>>167

>>170を読んで、復習しておきましょう😃


それにしても、いつまで、こんなサーバ状態つづくの? 書き込むだけで、大変なんだが・・・🤠

174 :名無しさん@英語勉強中 :2023/10/10(火) 22:16:55.51 ID:RTX++JxX0.net
<< The OED loves 英語職人 >> の最終見解まとめは毎回お笑いレベル
誰かタイムマシーンに乗る必要はないと指摘してあげる親切な人はいないのかな?
これらご老人二人がいる限り、まともな質問や回答もまったく期待できない。

175 :名無しさん@英語勉強中 :2023/10/10(火) 22:46:42.28 ID:so87zYndM.net
>>160
>《get the driver out of the car wreck》
>追突で燃えている最中の車から助けるときは、「火からも助ける」という意味を出したほうが適切なので、
>「自動車事故から救出」と訳すとよい。

>車が炎上中のような自動車事故全体に焦点を当てているなら「自動車事故から救出」と訳したほうが良い。

ちょっとよくわからないのですが
話者が、the car wreck を事故車、破壊された車という意味で使っていても
そのように訳すべきと言っているのですか?

それとも、事故に焦点を当てているなら、
話者はthe car wreckを自動車事故という意味で使っているはずだということですか?

176 :英語職人 :2023/10/10(火) 22:49:03.33 ID:RA8jLQ3r0.net
>>174 は、私より歳行ってる老人のくせして、よくいうよ。 

>誰かタイムマシーンに乗る必要はないと指摘してあげる親切な人はいないのかな?

あのさ、この世には、SF小説の翻訳もあるんだよ、ほんと恥ずかしいご老体だな。
まあ、こんなに簡単に論破される174爺には、>>171が正しいなんて所詮理解できるわけがない。少しは、私をうならせる反論してみなよ。
寒くなってきたから、174爺はももひき履いて寝なさいね😃

177 :英語職人 :2023/10/10(火) 23:19:38.73 ID:RA8jLQ3r0.net
>>175

>話者が、the car wreck を事故車、破壊された車という意味で使っていても
車が炎上している場合、話者が「自動車事故総体から救出した」と言いたいのか「事故で破損した車から救出した」と言いたいのか、どっちの意味で使っているかなんてわかるはずもないでしょう。

だから、車が炎上中の場合は、どちらでも良いと>>171で言っています。(ただし、「自動車事故から救出した」のほうがより理にかなっていると思うが、これは個人的な感想なので無視してもらってもいいです)
話し手が「炎上中でも破損した車から救出したと言う意味で使った」と明言するようなことが仮にあるとすれば、炎上した場合も「破損した車から救出した」だけが正解になるけども、そんことを明言するわけもないので、結局はどちらでも正解だという結論に落ち着くのです。

178 :名無しさん@英語勉強中 :2023/10/11(水) 01:09:34.90 ID:OtLNVu9n0.net
ロングマン英和辞典
wewck
He was killed in a car wreck.
彼は自動車事故で死んだ。

COB
wreck
He was killed in a car wreck.

179 :名無しさん@英語勉強中 :2023/10/11(水) 01:19:40.47 ID:OtLNVu9n0.net
a car wreck 自動車事故(米)
wreckを事故は米語法

180 :名無しさん@英語勉強中 :2023/10/11(水) 01:22:57.89 ID:OtLNVu9n0.net
COBだとmainly AMだね

181 :名無しさん@英語勉強中 :2023/10/11(水) 01:28:37.63 ID:OtLNVu9n0.net
BBCではa car wreckを事故車の意味でも使ってるみたいだね

182 :The OED Loves Me Not (ワッチョイ 7689-GYY9):2023/10/11(水) 07:45:18.51 ID:IdmzrM9I0.net
https://oshiete.goo.ne.jp/qa/13621195.html
この上の「教えて goo」での質問を見てください。
写真に示されている参考書で

... feels that much more valuable.
とあって、that が O (目的語) だと書いています。

こういうのはケアレスミスではなく、根本的な間違いを
参考書の作成者が犯していると思うのですが、この程度の
間違いは、中学高校生が日常的に使う学習参考書では
たまにはあるものでしょうか?

たまにうっかり間違うことは誰にでもありますので、
編集者やチェッカーが読んで、少しずつ草稿を校正していけばいいのですが、
教科書と違って参考書というものは、あまり途中で校正作業なんてものは
コストがかかるのでやらないようにしようとするのでしょうか?

183 :名無しさん@英語勉強中 (ワッチョイW 5af0-JtJs):2023/10/11(水) 10:53:01.28 ID:2LMVXm2C0.net
>>163
>>165
LRの発音は出来ます
彼女がアメリカ人なんですが実際何度か実験してみて発音は完璧だと言われました
実際アメリカ人にアメリカ人だと間違われたこともあるので個人的にも発音は割と良いと思ってます

LRの発音は毎回聞き分けられないというより聞き分けられない時があるという感じです
あまり詳しくないのですがLRの発音の中にもさらに細かいカテゴリーがあって特定の音が聞き分けられないのかな?と思っています

184 :名無しさん@英語勉強中 (ワッチョイW 5af0-JtJs):2023/10/11(水) 10:54:45.21 ID:2LMVXm2C0.net
>>165
ちなみに発音の仕方の本は読んだことないです
発音は実際にアメリカ人から教えてもらう・YouTube・学校の授業以外でやった経験はないです

185 :😇英さま🤔 (ワッチョイ da86-iLfk):2023/10/11(水) 10:54:49.74 ID:SjcMd4rl0.net
エロリストも間違った語源談義を訂正しないじゃん
5chもyoutubeも簡単に訂正コメント出せるのに、コストがかからなくてもやらないじゃん

186 :名無しさん@英語勉強中 (ワッチョイ da4f-OOOs):2023/10/11(水) 11:03:22.43 ID:OtLNVu9n0.net
ネイティブのオンライン英会話が安くなってきてるので、LとRの発音
はそこでなんとかしましょう。
DMMかCamblyかな

187 :名無しさん@英語勉強中 (ワッチョイW 5af0-JtJs):2023/10/11(水) 11:05:33.38 ID:2LMVXm2C0.net
>>186
そんなんで解決するならとっくに彼女との会話とかで解決してる気がするんだよね 

何というかナチュラルに聞き分けられる人たちが助けになるとあまり思えない

188 :名無しさん@英語勉強中 (ワッチョイ da4f-OOOs):2023/10/11(水) 11:10:36.49 ID:OtLNVu9n0.net
>>0187
ネイティブオンラインで解決できますよ
毎日マンツーマンで矯正するんだから

189 :名無しさん@英語勉強中 (ワッチョイW 5af0-JtJs):2023/10/11(水) 11:15:04.55 ID:2LMVXm2C0.net
>>188
矯正って発音じゃないの?何を矯正するの?
発音は完璧なんだよね

もしそれで解決できるなら普通に彼女に頼むかなぁ

190 :名無しさん@英語勉強中 (ワッチョイ da4f-OOOs):2023/10/11(水) 11:17:41.35 ID:OtLNVu9n0.net
heardとhard, earとyear, lice とrice, thinkとsinkなんかを
まとめて毎回アメリカ人に直してもらったよ。

191 :名無しさん@英語勉強中 (ワッチョイW 5af0-JtJs):2023/10/11(水) 11:20:05.77 ID:2LMVXm2C0.net
>>190
それはぶっちゃけ完璧に発音できるし聞き分けもできるんだよね

Riceとliceくらい分かりやすいのは分かるんだけど単語の真ん中にある初見の単語とか名前が全然聞き分けられないんだよね..

192 :名無しさん@英語勉強中 (ワッチョイ da4f-OOOs):2023/10/11(水) 11:20:52.82 ID:OtLNVu9n0.net
正しく発音できれは聞き取れますよ
彼女以外で完璧というネイティブはいますか?

193 :名無しさん@英語勉強中 (ワッチョイW 5af0-JtJs):2023/10/11(水) 11:25:23.24 ID:2LMVXm2C0.net
>>192
LRの発音に関しては聞けばみんな完璧って言ってくれる
仕事柄外国人としか話さないんだけど初対面の人にはかなりの確率で海外育ちなの?とか聞かれるから発音良い方だとは思う
非ネイティブっていうのは2分以上くらい話せばバレる感じ

ネットでも発音できれば聞き分けられるというような記事が散見してるから困ってここで聞いてみた

194 :名無しさん@英語勉強中 (ワッチョイ da4f-OOOs):2023/10/11(水) 11:31:46.31 ID:OtLNVu9n0.net
>>0193
ご健闘をお祈りいたします。
あなたのタイプは私の手には負えませんw

195 :名無しさん@英語勉強中 (ワッチョイW 5af0-JtJs):2023/10/11(水) 11:37:25.93 ID:2LMVXm2C0.net
>>194
まぁ難しいだろうね〜

正直この辺はアツ英語とかが受けてるガチの教授かアクセントコーチとかじゃないと解決できない気もしてる

196 :名無しさん@英語勉強中 (ワッチョイW 5af0-JtJs):2023/10/11(水) 11:41:47.97 ID:2LMVXm2C0.net
一応わかる人がいた時のために書いておくと

・文頭にLRがある場合・知っている単語の場合は聞き分けられる

・初見の単語・名前などが会話に出てきた時で単語の途中にLRある場合確信が持てない(当然周りのアメリカ人は初見の場合でも聞き分けできる)

と言った感じです
同じような症状の方で解決した方がいれば対策を教えていただけると幸いです

197 :名無しさん@英語勉強中 (ワッチョイ bdfe-XtdT):2023/10/11(水) 11:52:47.79 ID:b/ndqsla0.net
>>196
>単語の途中にLRある場合確信が持てない(当然周りのアメリカ人は初見の場合でも聞き分けできる)

その単語の綴りを知ってるのでもなければ
途中のは普通にネイティブでもできない場合が多い。

198 :名無しさん@英語勉強中 (ワッチョイW 856d-LWaA):2023/10/11(水) 12:08:32.65 ID:U/bmisiw0.net
>>182
間違いではない

こちらが正確
https://detail.chiebukuro.yahoo.co.jp/qa/question_detail/q13287289948

199 :名無しさん@英語勉強中 (ワッチョイ 0d9d-iLfk):2023/10/11(水) 12:14:12.25 ID:IH4JUbOV0.net
>>191
知らない単語や方言を普通の話速や滑舌、やや悪い滑舌で話されたら聞き取れないのは日本語でも同じだからなぁ。単語を知ってることでの補完はある

200 :名無しさん@英語勉強中 (ワッチョイW 316a-pzsk):2023/10/11(水) 12:18:46.90 ID:eLAjBBKl0.net
>>182
誰の何ていう本よ

201 :名無しさん@英語勉強中 (ワッチョイ da4f-OOOs):2023/10/11(水) 13:16:44.34 ID:OtLNVu9n0.net
In the following sentence, what does ”that" mean? As a result, our time together feels that much more valuable when we connect in real life.
ChatGPT
In the given sentence, "that" is used as a demonstrative pronoun to refer to a quality or degree. In this context, "that" emphasizes the degree of value or importance of the time spent together when they connect in real life. It suggests that the value of their time together feels significantly higher or more significant under those circumstances

202 :名無しさん@英語勉強中 (ワッチョイ da4f-OOOs):2023/10/11(水) 13:18:56.94 ID:OtLNVu9n0.net
"that" is used as a demonstrative pronoun

203 :名無しさん@英語勉強中 (ワッチョイ bdfe-XtdT):2023/10/11(水) 13:27:39.10 ID:b/ndqsla0.net
>>198
思いっきり間違ってる

204 :名無しさん@英語勉強中 (ワッチョイ bdfe-weeI):2023/10/11(水) 13:42:33.79 ID:b/ndqsla0.net
that
指示副詞
1《口語》 [数量・程度を示す語を限定して] それほど,そんなに.
・She can't go that far. 彼女はそんなに遠くまでは行け[そこまではしてくれ]ません.
・He only knows that much. 彼はそれだけしか知っていない.
・It's not really quite that interesting. それほどおもしろいってもんじゃない.


ChatGPT
You are correct, and I appreciate your clarification. In the sentence, "that" is used as an adverb to modify the degree of value. It indicates "to the extent" or "to that degree." The sentence can be rephrased to make this clearer:

"As a result, our time together feels [to that] much more valuable when we connect in real life."

In this rephrased version, it becomes evident that "that" functions as an adverbial modifier, indicating the extent to which "our time together" feels more valuable. Thank you for pointing out the correct part of speech for "that" in this context.

ChatGPTもあんま当てにならん。

205 :名無しさん@英語勉強中 (ワッチョイ da4f-OOOs):2023/10/11(水) 14:17:31.08 ID:OtLNVu9n0.net
重要なのはこの部分で、内容把握についてはこのAIが信用できるのは周知の事実

In this context, "that" emphasizes the degree of value or importance of the time spent together when they connect in real life. It suggests that the value of their time together feels significantly higher or more significant under those circumstances

206 :名無しさん@英語勉強中 (ワッチョイ da4f-OOOs):2023/10/11(水) 14:29:11.49 ID:OtLNVu9n0.net
demonstrative adverbなんて37万語にしか載ってないな

207 :名無しさん@英語勉強中 (ワッチョイ 7aad-STj1):2023/10/11(水) 14:35:58.73 ID:HXKxsTyN0.net
もちろん that は副詞です

that (adverb)
[as much as suggested]:
- It cost at least that much, if not more.
- The movie really wasn't that good (= was not very good).
- She's too old to walk that fast.
- It wasn't (all) that (= very) good.

Our time together feels (that) much more valuable when we connect in real life.

that much more [形容詞/副詞] よりも
so much more [形容詞/副詞] の方が
もっと普通に使われ、だいたい意味も同じです。

Our time together feels (so) much more valuable when we connect in real life.

208 :名無しさん@英語勉強中 (ワッチョイ 7a08-STj1):2023/10/11(水) 15:22:56.02 ID:HXKxsTyN0.net
この feel は自動詞で SVC の形です

feel (verb)
[to experience something physical or emotional]:
- My eyes feel really sore.
- I never feel safe when Richard is driving.
- My suitcase began to feel really heavy after a while.
- Our time together feels more valuable when we connect in real life.

209 :名無しさん@英語勉強中 (ワッチョイ 6ef6-BH9i):2023/10/11(水) 15:33:40.81 ID:Syg6F8MH0.net
>>182
スマホだと、SNS などを通して、友人と絶えず接続している状態になる。
一方で、ガラケーだと、友人との接続に空白の時間が生じる。
この空白の時間があるからこそ、現実の生活で繋がった時に、
一緒に過ごす時間が、その空白の分だけより大切に感じられる。

210 :名無しさん@英語勉強中 :2023/10/11(水) 16:25:19.74 ID:HXKxsTyN0.net
when we connect in real life

これはスマホやパソコンを使って virtual な状況で会う
のではなくて、real な状況で会うこと
つまり、現実世界で直接会うということ

211 :名無しさん@英語勉強中 :2023/10/11(水) 18:31:07.76 ID:HXKxsTyN0.net
>> 209 in real time と in real life を混同しています

212 :名無しさん@英語勉強中 :2023/10/11(水) 19:28:54.36 ID:Syg6F8MH0.net
>>211
ガラケー論者が、
ガラケーだとスマホと違ってお互いの状況が分からない時間が生じるからこそ、
実際に繋がって一緒に過ごすことにより一層の価値が出て来る、
これもガラケーの利点の一つだ、
と主張している、
という解釈だと間違いだということ?

213 :名無しさん@英語勉強中 (ワッチョイ da4f-OOOs):2023/10/11(水) 21:23:52.43 ID:OtLNVu9n0.net
>>0207
demonstrative adverbといいます。

214 :名無しさん@英語勉強中 (ワッチョイW 3134-pzsk):2023/10/11(水) 21:25:45.27 ID:eLAjBBKl0.net
やっと普通に見たり更新できるようになってきたかな…良かった

215 :名無しさん@英語勉強中 :2023/10/11(水) 23:08:54.19 ID:I5psrVMo0.net
いい機会だから5ちゃんから離れればいいのに
5ちゃんが機能してほっとした、とか依存しすぎでしょ

216 :名無しさん@英語勉強中 (ワッチョイ 7a08-STj1):2023/10/12(木) 00:20:58.90 ID:cDQ6J/Bx0.net
それは慧眼だな
自分も夜中に5chanを見る習慣ができてしまった
本当はもっと他のことに時間を使うべきだろう
この5chanのサーバダウンが
多くの人がもっと生産的な新しい生活を始める
きっかけになるといいと思う

217 :名無しさん@英語勉強中 (ワッチョイ aeae-OOOs):2023/10/12(木) 00:22:48.46 ID:0zfcg8qM0.net
This is one of the buildings we offer in our housing subsidy program which is part of our benefits package.
この文ですが
which is a part of〜 と 単数形の a が必要だと思うんですが、必要ない理由を文法的に説明できる人は教えてください

218 :名無しさん@英語勉強中 (ワッチョイ 7a08-lHjV):2023/10/12(木) 00:30:28.18 ID:cDQ6J/Bx0.net
>必要ない理由を文法的に説明できる人は教えてください

この上から目線の質問にはまいったな
この答えは英和辞典で part を見れば書いてあるわけだし、
このスレの住民の知能レベルを試しているのでしょう

219 :名無しさん@英語勉強中 :2023/10/12(木) 01:55:07.53 ID:dJDH5yrL0.net
>>217
ただの語法です

220 :名無しさん@英語勉強中 (ワッチョイ 7a3b-iLfk):2023/10/12(木) 06:54:31.09 ID:U0l7mAMx0.net
>>217
理由っていうかどっちの言い方もするしaって言っても言わなくても同じ意味だからどっちでもいいよ

221 :名無しさん@英語勉強中 :2023/10/13(金) 10:41:19.38 ID:smKUi4UJ0.net
A. This is one of the buildings we offer in our housing subsidy program which is a part of our benefits package. B. This is one of the buildings we offer in our housing subsidy program which is part of our benefits package.

a part of は単なる全体の1部 part of は全体と統合された部分という ニュアンスの違いが出る。 従ってpart ofの方は重要不可欠な感じになる。

222 :名無しさん@英語勉強中 :2023/10/13(金) 10:52:20.03 ID:smKUi4UJ0.net
ケースバイケースでほとんど同じ意味の場合もあります。

223 :名無しさん@英語勉強中 :2023/10/14(土) 13:54:37.25 ID:OsNdv9PcF.net
院生もようやく改心したんか

224 :名無しさん@英語勉強中 :2023/10/14(土) 14:12:07.66 ID:9m8Kpij/C.net
Hamas tells civilians to ignore an Israeli Defense Forces call for 1.1 million civilians to evacuate to northern Gaza for southern areas within 24 hours “for their own safety.

この文章のto evacuateより後の部分が分からないのですが、to northern Gaza以降をどう訳せばいいのか、文法的にはどう解釈すればいいのか教えて欲しいです

225 :名無しさん@英語勉強中 :2023/10/14(土) 14:28:30.26 ID:JHVod+s80.net
https://www.voanews.com/a/israel-to-un-northern-gaza-should-evacuate-within-24-hours/7309227.html

Hamas tells civilians to ignore an Israeli Defense Forces call for 1.1 million civilians to evacuate to northern Gaza for southern areas within 24 hours for their own safety.

確かにニュース記事はこうかいてありますね

to evacuate <to> northern Gaza <for> southern areas

しかし、私がテレビでニュースを見ている限りでは
「ガザ北部から南部に移動しろ」とイスラエル側が言っているので
次のようにしないと話は通じないと思います

to evacuate <from> northern Gaza <to> southern areas

226 :名無しさん@英語勉強中 :2023/10/14(土) 14:42:42.15 ID:p73atLit0.net
>evacuate northern Gaza

to要らない。

227 :名無しさん@英語勉強中 :2023/10/14(土) 14:48:15.90 ID:JHVod+s80.net
Hamas tells civilians to ignore an Israeli Defense Forces call for 1.1 million civilians to evacuate from northern Gaza to southern areas within 24 hours for their own safety.

ハマスは民間人に対し、身の安全のため110万人の民間人に24時間以内にガザ北部から南部地域へ避難するよう求めるイスラエル国防軍の呼びかけを無視するよう指示した。(Google)

<しかし、不思議なのは原文のままでも Google は正しく翻訳します>

Hamas tells civilians to ignore an Israeli Defense Forces call for 1.1 million civilians to evacuate to northern Gaza for southern areas within 24 hours for their own safety.

ハマスは民間人に対し、身の安全のため24時間以内に110万人の民間人にガザ北部から南部地域へ避難するよう求めるイスラエル国防軍の呼びかけを無視するよう指示した。(Google)

228 :名無しさん@英語勉強中 :2023/10/14(土) 15:21:29.27 ID:nJzts8EY0.net
英語の多読はどこから始めるのがおすすめですかね
TOEIC810なんですが
ペンギンブックスとかgraded readerでいうとどこからが良いでしょう

229 :名無しさん@英語勉強中 :2023/10/14(土) 16:53:09.37 ID:y5BfZz4s0.net
自分で買って読んでみるがよろし
と常識的な答えを通りすがりの者がします

230 :名無しさん@英語勉強中 :2023/10/15(日) 00:30:29.73 ID:N/xYXNvQr.net
先日donnernの件。私は今、ひどく混乱している😵

OED 第2版
https://i.imgur.com/OSCvFmE.jpeg

英語語源辞典
https://i.imgur.com/N9VF30k.jpeg
https://i.imgur.com/VBEcHps.jpeg

おまけ DWB
https://i.imgur.com/pJ6hGiK.jpeg

いずれもゲルマン語以上の広がりに言及している。いくらエロ垢を自己開示するドジっ子爺さんでも、これを読んでいたら「いかにも雷が鳴っている様子をそのまま onomatopoeia としてしまった🧐」などとは言わなかったろう。

231 :名無しさん@英語勉強中 :2023/10/15(日) 00:32:03.45 ID:N/xYXNvQr.net
可能性1.
thunderの語源欄を読んでいなかった。「donnernは英語の thunder に相当する動詞」と言っているが、この二つが同語源であることに気づけなかった。

可能性2.
語源欄を読んだが、理解できなかった。事実として、英語の/θ/音は、ドイツ語の/d/音に先行する。しかし、エロリストはドイツ語が「ゲルマン語の古い形態を残す」と妄信しており、その偏見を捨てることが出来なかった。

どちらの仮説もあまりにレベルが低く、私を混乱させるのである…😔

232 :名無しさん@英語勉強中 :2023/10/15(日) 00:33:21.23 ID:N/xYXNvQr.net
なお、印欧祖語までさかのぼって、その語幹がオノマトペだと言う本を一つだけ見つけた。

Klein's Comprehensive Etymological Dictionary of the English Language
https://i.imgur.com/lvEHbOO.jpeg

233 :名無しさん@英語勉強中 :2023/10/15(日) 00:34:27.07 ID:N/xYXNvQr.net
著者(Ernest Klein)はセム語の専門家らしいから、どこまで信用できるのかは分からない。

仮にこの説が正しいとして、これはten-がオノマトペだと言っているのである。また、/t/はダイレクトに/d/に変化したわけでなく、/t/→/θ/→/ð/→/d/と変化したのである。

ドーン!でオノマトペ丸わかりなら、オリジナルに近いテーン!だとより一層丸わかりのはずだが、果たしてそうだろか😂

234 :名無しさん@英語勉強中 :2023/10/15(日) 00:37:21.35 ID:N/xYXNvQr.net
反対意見があるとエロリストがすぐにその相手をNGに🖤したことにする🖤のは、「自分は間違ってない。💙自分は反論できないのではなく、そもそもNGで見えないのだ💙」という言い訳を用意するためである。

本当はNG登録などしていない。その証拠もあるが、何よりエロ動画漁りと5chリロードしか楽しみのない「一見して孤独
のように見える」老人が、自分から楽しみを逃すことをするわけがない🤭

それは百も承知だが、このレスはあえてコテを外して書くのは、NGで見えないという言い逃れを通用させないためである。😏

235 :🤭英ちゃん🫢 :2023/10/15(日) 00:39:01.17 ID:N/xYXNvQr.net
>>230-234
以上!
NGで見えてません、なんて言い訳はできません😉

236 :🤭英ちゃん🫢 :2023/10/15(日) 00:45:27.66 ID:8U7k5Sz50.net
>>231
>この二つが同語源であることに気づけなかった。

お得意のフランス語のtonnerreも思い出せなかったようである

237 :🤭英ちゃん🫢 (ワッチョイW 1386-oEXR):2023/10/15(日) 03:36:08.04 ID:8U7k5Sz50.net
>>230
画像の訂正

先日donnernの件。私は今、ひどく混乱している😵

OED 第2版
https://i.imgur.com/OSCvFmE.jpeg

英語語源辞典
https://i.imgur.com/N9VF30k.jpeg
https://i.imgur.com/VBEcHps.jpeg

おまけ DWB
https://i.imgur.com/pJ6hGiK.jpeg

238 :🤭英ちゃん🫢 (ワッチョイW 1386-oEXR):2023/10/15(日) 03:38:37.99 ID:8U7k5Sz50.net
きゃーーー🙈
失敗!!🥳
もっかい訂正

OED 第2版
https://i.imgur.com/OSCvFmE.jpeg

英語語源辞典
https://i.imgur.com/N9VF30k.jpeg

おまけ DWB
https://i.imgur.com/pJ6hGiK.jpeg

Klein's Comprehensive Etymological Dictionary of the English Language
https://i.imgur.com/lvEHbOO.jpeg
https://i.imgur.com/VBEcHps.jpeg

239 :名無しさん@英語勉強中 :2023/10/15(日) 06:32:47.78 ID:L3d2JmXLd.net
この動画で塾の経営者が生徒の指導法について「調整点をオミットする」と言っています。オミットするとはどういう意味か分かりますか?
https://youtu.be/FPlTv2O-o-w?si=CZ1kK1eXyGLt5DyT

240 :名無しさん@英語勉強中 :2023/10/15(日) 06:36:22.89 ID:L3d2JmXLd.net
動画の6分22秒辺りの発言です。

241 :The OED Loves Me Not (ワッチョイ 8b89-pCU2):2023/10/15(日) 08:31:11.82 ID:nkTPfX4P0.net
>>239
「オミット」は英語では omit のはずであって、その意味は英語としては

omit
(1) SVOOを〔…から〕(故意に)省く, 除外する, (うっかり)抜かす(leave out)〔from〕

(2) 名簿に名前を入れないでおく[入れ忘れる]《◆「作成済みの名簿から削除する」はdelete, eliminate, erase》.
#SV to do(故意に)…しない, (うっかり)…することを忘れる

(ジーニアス英和、6版)

という感じ。そして外来語として日本語の中に入り込んでいる「オミット」の意味もやはり

オミットとは
オミットとは、「省く」「抜かす」という意味の英単語【omit】から来たカタカナ用語で、ビジネスシーンでは、「除外する」「省く」「却下する」といった意味で使われています。
https://news.mynavi.jp/article/20190221-773338/#:~:text=%E3%82%AA%E3%83%9F%E3%83%83%E3%83%88%E3%81%A8%E3%81%AF,%E3%81%A7%E4%BD%BF%E3%82%8F%E3%82%8C%E3%81%A6%E3%81%84%E3%81%BE%E3%81%99%E3%80%82

上のような意味です。

しかしこの YouTuber さんは、「オミットする」を「決定する、決める、設定する」
みたいな意味合いで使っているような気がします。だからまあ、この言葉の意味を少し勘違いされているような気がしますね。

242 :名無しさん@英語勉強中 (ワッチョイ d965-VGqS):2023/10/15(日) 12:23:07.89 ID:m1V2H02l0.net
>>224

自分も当初は>>225氏と同じように、northern と southern を勘違いで逆にして
しまったのだろうと思った。

が、>>226氏の書き込みを見て考え直した。
なるほど、to が余分に加えられているミスと考えた方が話が早い。
evacuate northern Gaza for southern areas
なら「ガザ北部から(ガザ南部に向けて)避難する」の意ですんなり意味が通ります。

northern と southern を勘違いで逆にしたと考えた場合は、for southern areas
を from southern areas に訂正しなければならなくなります。

243 :名無しさん@英語勉強中 (ワッチョイ 5370-T9H3):2023/10/15(日) 12:55:02.08 ID:rdM04LDF0.net
Hamas tells civilians to ignore an Israeli Defense Forces call for 1.1 million civilians to evacuate to northern Gaza for southern areas within 24 hours for their own safety.

確かにニュース記事はこうかいてありますね

to evacuate <to> northern Gaza <for> southern areas

しかし、私がテレビでニュースを見ている限りでは
「ガザ北部から南部に移動しろ」とイスラエル側が言っているので
次のようにしないと話は通じないと思います

to evacuate <from> northern Gaza <to> southern areas

244 :名無しさん@英語勉強中 (ワッチョイ fb4a-SGKv):2023/10/15(日) 13:29:03.23 ID:kXmKIhDQ0.net
他動詞として用いて「evacuate 場所」で
「場所から脱出する」という意味があるのだから
to を省くだけで十分じゃね?

If people evacuate a place,
they move out of it for a period of time,
especially because it is dangerous.

ex.
The fire is threatening about sixty homes,
and residents have evacuated the area.
https://www.collinsdictionary.com/dictionary/english/evacuate

leave A for B と同じ感覚だろう。

ex.
On April 10, 1912 the Titanic left Southampton for New York.

245 :名無しさん@英語勉強中 (ワッチョイW 599d-Wop/):2023/10/15(日) 13:31:32.21 ID:4bre0ref0.net
>>243
call for someone to ~

246 :242 (ワッチョイ d965-VGqS):2023/10/15(日) 13:32:45.15 ID:m1V2H02l0.net
evacuate は他動詞の用法があり、だから前置詞の to とかはなしで目的語の
名詞を置けますよ。

実際の用例も多数です。↓

・Palestinians struggle to evacuate northern Gaza amid ...
ABC7 Chicago
9 時間前 — Palestinians struggle to evacuate northern Gaza amid growing Israeli warnings of ground offensive. An estimated 1 million people have been ...

・Israel orders evacuation of 1 million in northern Gaza in 24 ...
PBS
https://www.pbs.org › world › israel...
2 日前 — JERUSALEM (AP) — Israel's military told some 1 million Palestinians on Friday to evacuate northern Gaza and head to the southern part of the ...

・As war looms Israel calls for 1.1m people to evacuate ...
The Economist
2 日前 — As war looms Israel calls for 1.1m people to evacuate northern Gaza. America may want Egypt to take in large numbers of Palestinian refugees.

・Thousands of Palestinians flee northern Gaza after Israel ...
Australian Broadcasting Corporation
1 日前 — UN officials in Gaza said the Israeli military told them to evacuate northern Gaza in 24 hours; 1.1 million people live in northern Gaza ...

・Israel Tells 1.1 Million Civilians to Evacuate Northern Gaza
Wall Street Journal
2 日前 — Israel Tells 1.1 Million Civilians to Evacuate Northern Gaza · U.N. asks Israeli military to reconsider order, warning of devastating ...

247 :242 (ワッチョイ d965-VGqS):2023/10/15(日) 13:37:14.07 ID:m1V2H02l0.net
おや、>>244氏とかぶってしまった。 (´·ω·`)

248 :名無しさん@英語勉強中 (ワッチョイ 5370-T9H3):2023/10/15(日) 13:56:37.94 ID:rdM04LDF0.net
もし他動詞だとすると、こうなりますが

evacuate northern Gaza <for> southern areas

このような for の使い方はあるのでしょうか
to, toward ならあるでしょう

249 :名無しさん@英語勉強中 :2023/10/15(日) 14:12:26.02 ID:rdM04LDF0.net
「ガザ北部から南部の地域に向けて避難しろ」

Google: Evacuate from northern Gaza to southern areas.
DeepL: Evacuate northern Gaza toward southern areas.
Bing: Evacuate from northern Gaza to southern areas

250 :名無しさん@英語勉強中 :2023/10/15(日) 14:20:23.65 ID:9czQVXOm0.net
実際の発言はこれでしょ
"Evacuate south for your own safety and the safety of your families and distance yourself from Hamas terrorists who are using you as human shields."

251 :名無しさん@英語勉強中 :2023/10/15(日) 14:34:22.36 ID:gROJ9s2t0.net
>as human shield
えげつねぇな…

252 :242 :2023/10/15(日) 14:45:34.91 ID:m1V2H02l0.net
>>248

なるほど確かに evacuate A for B(AからBに向けて避難する)という
言い方はなさそうですね。

正式な言い方は
1. evacuate A to B
2. evacuate from A to B、または evacuate to B from A
の2つということになるのだろうか。
(to は toward でもよさそう)

記事に即して考えると、原文の 〜 to evacuate to northern Gaza for
southern areas の訂正の仕方は、(ガザ北部から南部への避難指示
である事実は動かせないので)

1.evacuate が自動詞の場合
〜 to evacuate to southern Gaza from northern areas
もしくは、〜 to evacuate from northern Gaza to southern areas

2.evacuate が他動詞の場合
〜 to evacuate northern Gaza to southern areas

(以上の前置詞の to は toward でも可?)

のどちらかである、と、こういうことなのか。 (´·ω·`)
(今日の書き込みはここまでとさせて頂きます)

253 :名無しさん@英語勉強中 (スッップ Sd33-F/Li):2023/10/15(日) 16:20:49.51 ID:L3d2JmXLd.net
『イスラエル軍は、パレスチナのガザ地区北部に住む100万人以上の人々に対し24時間以内に地区の南部に退避するよう通告しました。』とnhk ニュースにはありますね。
https://www.google.com/amp/s/www3.nhk.or.jp/news/html/20231013/amp/k10014223621000.html

254 :名無しさん@英語勉強中 (ワッチョイ 535a-T9H3):2023/10/15(日) 18:36:27.20 ID:rdM04LDF0.net
The Israel Defence Forces (IDF) have issued an appeal to Gaza Strip residents urging civilians to evacuate to the south for their own safety and warning them that the army will carry on with their strikes against the Hamas terrorist group.

もともとのアラビア語の原文がどうなっていたか分かりませんが、
イスラエルがガザ北部に「南に避難しろ、死にたくなかったら」という
内容のビラを上空から撒いたということで、ガザ北部の人に向かって
「ガザ北部から」という文句は言う必要がないので

الإخلاء إلى الجنوب من أجل سلامتك.

Evacuate <to> the south <for> your own safety.

こんな原文が想像できる。

255 :名無しさん@英語勉強中 :2023/10/15(日) 22:08:50.45 ID:2zShePyV0.net
今ちょうどニュース番組でイスラエルの軍人?が呼びかけてる動画が流れたけど、多分
evacuate southern gaza
って言ってたぞw

北で戦争になるってのは現地ではみんなわかってることなのかね?
その辺知らんかったら混乱するだろw

256 :名無しさん@英語勉強中 :2023/10/15(日) 22:41:10.45 ID:kXmKIhDQ0.net
>>224 の英文は箇条書きになっている The latest の上から 5 番目にあるが
上から 3 番目の英文では他動詞として用いている。

Israeli airstrikes have killed 70 and wounded 200 people
evacuating northern Gaza, according to Gaza’s Interior Ministry.
https://www.voanews.com/a/gazans-scramble-as-clock-ticks-down-on-israeli-evacuation-orders/7310676.html

CBC や Reuters では leave が使われている。

Israel calls for civilians to leave Gaza City
https://www.youtube.com/watch?v=QMbQ1aZ7PRg

Israel tells civilians to leave Gaza City
https://www.youtube.com/watch?v=bznSJHfWc5Q

257 :名無しさん@英語勉強中 :2023/10/15(日) 23:29:28.79 ID:rdM04LDF0.net
Hamas tells civilians to ignore an Israeli Defense Forces call for 1.1 million civilians to evacuate to northern Gaza for southern areas within 24 hours for their own safety.

確かにニュース記事はこう書いてありますね
to evacuate <to> northern Gaza <for> southern areas

これは変なので<to>を削除して、他動詞にしてみても
to evacuate northern Gaza <for> southern areas
こんな表現はありません

根本的に前置詞の使い方が間違っているので
to evacuate <from> northern Gaza <to> southern areas
このようにしないと話は通じません

他動詞として使う場合も
to evacuate northern Gaza <to/toward> southern areas
このようにする必要があります

>>249 これを参考にしてください

258 :名無しさん@英語勉強中 (ワッチョイ fb4a-SGKv):2023/10/16(月) 00:31:23.51 ID:ef1FLeoe0.net
Even though the army evacuated the area for a string of forts farther west in 1853,
residents John Peter Smith opened a school, Julian Field a flour mill,
and Henry Daggett and Archibald Leonard department stores.
http://plainshumanities.unl.edu/encyclopedia/doc/egp.ct.023

evacuate A for B 「B を目指して A から離れる」で十分意味は通じると思うけどな。

VOA の英文は恐らく evacuate の辺りを書き直したときに
to が残ってしまっただけだと思うけど。

まあ、for がどうしても気に入らないのであれば、
VOA のライターに文句を言ってくれ。
お前の英語は間違ってる、ってな。

259 :名無しさん@英語勉強中 (ワッチョイ 5315-T9H3):2023/10/16(月) 01:15:18.66 ID:WMe8YSFh0.net
そうではなくて、VOAに指摘すべきは
evacuate to northern Gaza
「この部分が間違っている」でしょ

260 :名無しさん@英語勉強中 (ワッチョイ 5315-T9H3):2023/10/16(月) 01:43:20.70 ID:WMe8YSFh0.net
いろいろ例文を見て
他動詞の場合と、自動詞の場合の
だいたいの傾向が分かってきた

「避難すべき場所」だけの場合、圧倒的に他動詞

そのビルから避難しなさい。
- Evacuate that building.

「避難すべき場所」と「避難先」がある場合、自動詞も多くなる

そのビルから近くの公園に避難しなさい。
- Evacuate from the building to a nearby park.

261 :名無しさん@英語勉強中 (ワッチョイ 3196-iZeJ):2023/10/16(月) 01:52:17.54 ID:Nj5gbrZu0.net
An advanced reader of English is one on whom nothing is lost.


チャットGPT様のおかげで意味はわかったんですが、文の構造、とくにon がある理由がわかりません。
元々どういう熟語があって、onなのか。

is one who never lost anythingでもいいんじゃないかとか考えてしまいます。

解釈の仕方をおしえてください。

262 :🤭英ちゃん🫢 (ワッチョイW 1386-oEXR):2023/10/16(月) 02:12:58.02 ID:uIUGWIbS0.net
be lóst on [upon] A
A〈人〉に理解されない, A の注意を引かない; A に効果がない.

nothing is lost on A なら
Aに理解されないものはない=なんでも分かる

でしょう?🤔

263 :🤭英ちゃん🫢 (ワッチョイW 1386-oEXR):2023/10/16(月) 02:19:56.61 ID:uIUGWIbS0.net
A is lost on me.
Aは私において失われている

Nothing is lost on me.
何も私において失われていない
=なんでも分かる
=なんでも効果テキメン

264 :🤭英ちゃん🫢 (ワッチョイW 1386-oEXR):2023/10/16(月) 02:23:39.75 ID:uIUGWIbS0.net
DeepL
Nothing is lost on me.
何も迷うことはない。

ん~😐

265 :🤭英ちゃん🫢 (ワッチョイW 1386-oEXR):2023/10/16(月) 02:29:03.17 ID:uIUGWIbS0.net
'To be lost on' can mean either a) to have no effect or influence on someone. " Her tears were lost on him." = he wasn't affected by her tears or b) to not be understood by or beyond the comprehension of someone. " His explanation was too technical and lost on us" = We didn't understand his explanation because it was too technical.
The negative 'To not be lost on' as in your example therefore means to be understood by or not beyond the comprehension of someone. 'It's not lost on me' = I have understood.
https://www.italki.com/en/post/question-115495

わたくしの理解で正しかろう🤔

266 :The OED Loves Me Not :2023/10/16(月) 03:00:38.16 ID:nJGRCmUI0.net
>>261
be lost on somebody
if something is lost on someone, they do not understand or want to accept it
The joke was completely lost on Chris.
https://www.ldoceonline.com/dictionary/be-lost-on-somebody

267 :The OED Loves Me Not :2023/10/16(月) 03:04:30.73 ID:nJGRCmUI0.net
>>261
be lóst on [upòn] O O〈人〉に理解されない, わかってもらえない;Oに効き目がない
All my advice was lost on her.
私の忠告はすべて彼女に効き目がなかった.
(ジーニアス英和、第6版)

268 :名無しさん@英語勉強中 :2023/10/16(月) 09:08:33.71 ID:Nj5gbrZu0.net
>>262-263
>>266-267

ありがとうございました!

269 :242 :2023/10/16(月) 12:07:24.20 ID:eDW7Njf10.net
>>252の続き

結局自分の結論としては evacuate A for B でも大丈夫ということに落ち着いた。
確かに今回の場合そういう用例は見当たらないようだけど、この for はいわゆる
「方向の for 」に相当し、これ自体は珍しい用法ではない。文法的に間違いとは
言えない。
だから、原文を訂正すべきは、evacuate to の余分な to を省くだけでよいという
のが結論です。
>>258氏に先に同じ主旨を書かれてしまったけど)

もちろん訂正の仕方は他に 〜 evacuate from northern Gaza to southern areas
もありますが。

270 :242 :2023/10/16(月) 12:11:35.18 ID:eDW7Njf10.net
242
それから念のために補足しておきます。

>>250で、 Evacuate south という表現について触れられていますが、
この south は副詞です。「南へ・南に」という意味。自動詞の Evacuate
を修飾しています。全体で「南の方へ避難せよ」ということ。
「自動詞の evacuate + 副詞の south」の組み合わせです。

これを
「他動詞の evacuate + その目的語の名詞の south」という風に勘違い
してはいけません。「南から避難せよ・南を脱出せよ」の意味ではない
のです。

こういうのを老婆心というのであろう。わかっている人はわかっている
だろうけど。 (´·ω·`)

271 :名無しさん@英語勉強中 :2023/10/16(月) 12:23:18.57 ID:WMe8YSFh0.net
for でもいいと思った人はおそらく動詞 leave から類推していると思う
leave A for B
leave for B
(Bに向かって出発する)
しかし、evacuate にこのような for の用法はない

272 :242 :2023/10/16(月) 13:47:23.49 ID:eDW7Njf10.net
>>271
>for でもいいと思った人はおそらく動詞 leave から類推していると思う

いや、leave とは関係ないですよ、少なくとも自分の場合は。
「方向の for」はごく普通の用法ですから。

ジーニアス英和辞典第5版から引くと、

11. [方向](ある場所に)向かって、
dash for[to]the door ドアに向かって[まで]突進する
(for は方向、to は到着点を示す)
He left London for Paris early in the morning. 彼は早朝ロンドンを発ってパリへ向かった。

など。
どの辞書にも書いているし、英文を読んでて普通に出会う使い方です。

だから、evacuate の後に for + 場所を表す言葉 が来れば、ごく自然に
「 〜 (の方向)へ避難するんだな」と思います。普通の感覚の持ち主なら
皆そう考えると思います。

273 :名無しさん@英語勉強中 :2023/10/16(月) 14:25:16.98 ID:WMe8YSFh0.net
いみじくもその leave の解説の中に書いてあるように
(for は方向、to は到着点を示す)
for と to では意味が違うんです。

使い分けのヒントを言うと
evacuate to the south と言うが
evacuate for the south とは言わない

274 :名無しさん@英語勉強中 :2023/10/16(月) 17:36:25.06 ID:zl5lI8BO0.net
辞書に載っている表現か確認しながら読書するべきですか?
辞書に載ってなかったら、間違った表現と判断するべきですか?

275 :名無しさん@英語勉強中 :2023/10/16(月) 18:01:12.21 ID:iBByp+l30.net
https://i.imgur.com/Izfn32n.jpg
よろしければお試しください
ik..tk N-G用

276 :名無しさん@英語勉強中 :2023/10/16(月) 18:05:27.90 ID:kLL0yw5v0.net
i am not〜って即座に否定していますが
本当に英語なんですか
アメリカでアメリカ人が使っている文法ですか
即否定するならそれ以降しゃべる必要ないと思うのですが?

277 :名無しさん@英語勉強中 :2023/10/16(月) 19:08:11.21 ID:mMbRURFb0.net
>>275
これは良かったよなあ

278 :名無しさん@英語勉強中 :2023/10/16(月) 20:12:39.12 ID:Nj5gbrZu0.net
I think we are all born with the gift for enjoying beautiful things, but that we are indifferent to many of them
because our attention was never called to them in childhood.

これをどう解釈すべきかおしえてください。子どもの頃、どうだったんですか?

・美に注意を向けなかった(子どもから見て能動)
・美に惹かれなかった(子どもから見て受動。美の方が子を相手にしなかった)
・美に注意を向けるよう仕向けられなかった(子から見て受動。親が美を与えなかった)

どれでしょうか?

279 :🤭英ちゃん🫢 :2023/10/16(月) 20:22:34.47 ID:uIUGWIbS0.net
難しいこと考える人がいるね😵

280 :The OED Loves Me Not :2023/10/16(月) 21:11:33.85 ID:nJGRCmUI0.net
>>278
> I think we are all born with the gift for enjoying beautiful things, but that we are indifferent to many of them
> because our attention was never called to them in childhood.

(1) 美に注意を向けなかった(子どもから見て能動)

まず、これではないと思います。もしこの (1) に相当する英文を書こうと思ったら、
We never paid attention to them (= the beautiful things) in childhood.
となるはずではないかと思います。

(2) 美に惹かれなかった(子どもから見て受動。美の方が子を相手にしなかった)

「美の方が子供を相手にしなかった」のではなくて、子供の注意をそれに振り向けてくれる
環境やきっかけがなかった、ということではないかと思います。

>(3) 美に注意を向けるよう仕向けられなかった(子から見て受動。親が美を与えなかった)

「親が美を与えなかった」とは限らず、子供に美しいものを見せてくれるような環境やきっかけが
なかったということだと思います。

なお、言うまでもないことかもしれませんが、our arttention was never called to them
の部分については、次のような辞書の定義文を参考にしました。

call (somebody’s) attention to

a) to ask people to pay attention to a particular subject or problem
May I call your attention to item seven on the agenda?

b) to make someone notice someone or something
I wanted to shout out to Ken, but I didn’t want to call attention to myself.

https://www.ldoceonline.com/dictionary/call-somebody-s-attention-to

281 :名無しさん@英語勉強中 :2023/10/16(月) 23:03:13.71 ID:Nj5gbrZu0.net
>>280
ありがとうございました!
ご回答を何度も反芻します!

282 :ブタ耳 :2023/10/17(火) 10:19:20.24 ID:cVyYvOML0.net
https://www.phind.com/search?cache=e0om1uzkfjt9kbjopni0jlg3

# prompt
"[>>278のテキスト]"

In this situation, what was it that did not call our attention to beautiful things?

# phind (GPT-4)
The text doesn't explicitly mention what it was that didn't call our attention to beautiful things during childhood. It simply states that our attention "was never called to them." This phrase could be interpreted in different ways depending on the context.

One interpretation could be that the adults or educators in our lives did not point out or emphasize these beautiful things. This could be due to their own lack of awareness, or perhaps they had different priorities in terms of what they thought was important to teach or show us.

Another interpretation could be that societal norms or expectations didn't encourage us to appreciate certain beautiful things. For example, certain forms of art or nature might not be widely appreciated in a particular culture, and so children growing up in that culture might not have their attention drawn to these things.

Yet another interpretation could be that we were too preoccupied with other things as children (like playing, studying, etc.) and simply didn't notice the beautiful things around us.

Again, these are just interpretations. The text doesn't provide a specific reason why our attention wasn't called to beautiful things during childhood.

283 :🤭英ちゃん🫢 :2023/10/17(火) 10:44:19.69 ID:KSapVtir0.net
文法上の態と、意味上の態とゆーのか能動・受動がごっちゃになってる質問ですわ

284 :名無しさん@英語勉強中 (ワッチョイ 599d-AzOG):2023/10/17(火) 11:10:38.15 ID:6APVkG2q0.net
>>278
この文のポイント
weに、この文を読んでいる人(自分)を含める
最初の二つのbe動詞が現在形で、三つ目が過去形

285 :名無しさん@英語勉強中 (ワッチョイ 599d-AzOG):2023/10/17(火) 12:14:45.34 ID:6APVkG2q0.net
AIは、美についてどんなに上手に言うことが出来ても
美を実際に体験することが出来ないので、本当は美とは何かを知らない

286 :名無しさん@英語勉強中 :2023/10/17(火) 12:24:09.97 ID:6APVkG2q0.net
しかし、AIは、美について、いろいろな人のいろいろな考えを知っている

287 :名無しさん@英語勉強中 :2023/10/17(火) 12:24:59.14 ID:0zbxjQ810.net
http://oshiete.goo.ne.jp/qa/13625491.html

他の掲示板のことでごめんけど、東山さんの最初の回答の最初に言ってることがよく分からない
remain in は使えないってこと?

288 :🤭英ちゃん🫢 :2023/10/17(火) 12:47:50.49 ID:KSapVtir0.net
意味を問う問題でchatbot使うってアホだと思うの

289 :The OED Loves Me Not (ワッチョイ 8b89-pCU2):2023/10/17(火) 12:51:32.50 ID:+VJTjsT80.net
>>287
「東山さん」って、私のことかな?そしてもしかして、あなたはこの下の英文が
文法的に成り立つと思っているのですか?

もちろん、この英文のままでも通じることは通じます。つまり、間違った英文でも
言いたいことはわかる、という意味で「通じる」のです。でも文法的には間違いだと言っているのです。

「教えて goo」での質問者が提案した英文
>>孤立してしまうからといって納得のいかないことを受け入れる気にはなれなかった。
>>I never felt like accepting anything question marks remained in for fear of losing friends.

290 :名無しさん@英語勉強中 (ワッチョイ 53ca-T9H3):2023/10/17(火) 17:08:06.82 ID:3UZyIEzj0.net
なぜ「東山」という名前に反応するだろう

もしかしたら「東山修」と「太宰治」は同一人物かもしれない

291 :名無しさん@英語勉強中 (ワッチョイ 53ca-T9H3):2023/10/17(火) 17:09:44.68 ID:3UZyIEzj0.net
なぜ「東山」という名前に反応するだろう

もしかしたら「東山修」と「太宰治」は同一人物かもしれない

292 :名無しさん@英語勉強中 :2023/10/17(火) 18:05:09.68 ID:wVpVw0QE0.net
「近頃、日本では野球をする子供たちがますます減ってきている」
の英訳は以下の文でいいでしょうか?
The number of children who play baseball in Japan
has been decreasing lately.

293 :The OED Loves Me Not (ワッチョイ 8b89-pCU2):2023/10/17(火) 18:15:25.55 ID:+VJTjsT80.net
>>292
(1) 原文: 近頃、日本では野球をする子供たちがますます減ってきている。

(2) 英訳案: The number of children who play baseball in Japan has been decreasing lately.

<< 一つ目の問題点>>

原文の「ますます」という意味合いが、(2) の英訳には込められていないと思いませんか?
(2) は単に、「〜が減ってきている」と言っているだけだと私には感じられます。

<< 二つ目の問題点 >>

原文では「日本では、こういう状況が起こっている」という感じで、
「日本では」を前に押し出しています。

しかし英訳では、「日本でこういうことをする子供たちの数は、どうのこうの」と言っていて、
「日本で」という意味合いが仲に隠れています。

日本語原文の意味合いを生かそうと思うと、In Japan, S V. という形を取った方がいいと
思いませんか?

294 :The OED Loves Me Not (ワッチョイ 8b89-pCU2):2023/10/17(火) 18:16:59.55 ID:+VJTjsT80.net
>>292
(1) 原文: 近頃、日本では野球をする子供たちがますます減ってきている。

(2) 英訳案: The number of children who play baseball in Japan has been decreasing lately.

<< 一つ目の問題点>>

原文の「ますます」という意味合いが、(2) の英訳には込められていないと思いませんか?
(2) は単に、「〜が減ってきている」と言っているだけだと私には感じられます。

<< 二つ目の問題点 >>

原文では「日本では、こういう状況が起こっている」という感じで、
「日本では」を前に押し出しています。

しかし英訳では、「日本でこういうことをする子供たちの数は、どうのこうの」と言っていて、
「日本で」という意味合いが仲に隠れています。

日本語原文の意味合いを生かそうと思うと、In Japan, S V. という形を取った方がいいと
思いませんか?

295 :名無しさん@英語勉強中 (ワントンキン MM4b-ELGK):2023/10/17(火) 18:20:07.64 ID:/gZ8Uxc/M.net
>>292
いいよ

296 :名無しさん@英語勉強中 (ワッチョイ 3196-iZeJ):2023/10/17(火) 19:30:41.92 ID:OOaiUU0h0.net
of について、感覚がよくわかりません。

He persuaded me of its truth. このofは何のof?どういう働きでどういう意味?文法書の何というページを見るといいのでしょうか?

I myself am fully convinced that the best explanation an artist can give of
his aims and ability is afforded by his work.

この文の、一行目最後のofもまったく意味や働きがわかりません。

腑に落ちるような解説、この英語の感覚がわかるような理解のしかたってないでしょうか?

297 :名無しさん@英語勉強中 :2023/10/17(火) 19:50:21.56 ID:oVVNJw850.net
ofはついてやろ

298 :The OED Loves Me Not :2023/10/17(火) 19:53:10.46 ID:+VJTjsT80.net
>>296
> He persuaded me of its truth. このofは何のof?どういう働きでどういう意味?文法書の何というページを見るといいのでしょうか?

済みませんが、なぜこの程度のことにいちいち文法書を開く必要があるんですか?
全体を読めば、何となく of が about みたいな意味だろうなと想像できるでしょう?
それだけのことだと思いませんか?

別のスレッドでも誰かが言っていましたが、
I am glad of [something].
などの場合の of も about みたいな意味だということです。

learn of [something]
tell of [something]
inform someone of something
know of something

こういうときの of も、すべて about みたいなもんだろうなと思えばおしまいです。
まあ、もしかしたら英語学に詳しい人は、もっと微妙な解釈を加えるかもしれませんが、
素人の私たちにとっては、こういうときの of は about みたいなもんだと思っておけば、
さほど大きな間違いではないでしょう。

> I myself am fully convinced that the best explanation an artist can give ★of★
> his aims and ability is afforded by his work.

この上の文の中には、
An artist can give the best explanation ●of● his aims and ability.
という phrase が埋め込んであると思っていいですよね。この場合の
of も、about みたいな意味だと思えばおしまいではないでしょうか?
about の代わりに、concerning とか on とかでもいいかもしれません。

299 :名無しさん@英語勉強中 :2023/10/17(火) 20:37:17.90 ID:/gZ8Uxc/M.net
>>296
働きってほどのなんかはなくて、単にpersuade 誰それ ofてのが説得するみたいな意味の言い方で、give of 何それってのは分け与えるっていう意味なんだけど
こういうのはもうそういうもんだからで別に分かる必要ないよ へーと思って終わりで全然だいじょうぶ

300 :The OED Loves Me Not (ワッチョイ 8b89-pCU2):2023/10/18(水) 07:32:58.52 ID:qauKRSA50.net
persuaded me of its truth などに出てくる of については、「ジーニアス英和、第6」
には、次のような網羅的な解説がありますね。

of
([関連]

a) …について(の), …に関して(の)《◆堅い言い方》
stories of adventure
冒険の話 / A Tale of Genji 『源氏物語』
speak of [about] it
そのことを話す→語法
I've never heard of her name.
彼女の名前は聞いたことがありません
suspect him of stealing
彼に盗みの疑いをかける→語法
I think highly of him.
彼を尊敬している
Of his family I know very little.
彼の家族について私はほとんど何も知らない《◆of 句の前置可能》
What became of the picture that used to be on the wall?
壁に掛かっていた写真はどうしたんだ(→WHAT of O?).

語法

1 この意味でofがよく使われる動詞:approve, beware, boast, conceive, dream, hear, know, learn,
smell, speak, talk, taste, think, tell, warn など.

2 SVO1 of O2でよく使われる動詞:
accuse, advise, assure, convict, convince, inform, notify, ★persuade★, reassure, remind, suspect, tell, warn など.

(ジーニアス英和、第6)

301 :The OED Loves Me Not (ワッチョイ 8b89-pCU2):2023/10/18(水) 07:34:19.75 ID:qauKRSA50.net
今回のように、persuade *** of *** に出てくる of などについて悩んだら、
ともかく「ジーニアス英和」のその項目について
の解説を読むことですね。ただし of などの基本語についての解説があまりに長くて、
自分が探している情報にたどり着くのが大変だと思いますけど、それくらいの
苦労はしないといけないのです。

あるいは、紙じゃなくて電子版の辞書なら、今回の場合だったら
of をまずは検索し、その of の項目の解説を表示した上で、次に
persuade を検索すれば、この上で★の印をつけたところにあるような
persuade に of が付く場合についての解説が見つかります。

302 :The OED Loves Me Not (ワッチョイ 8b89-pCU2):2023/10/18(水) 07:42:18.74 ID:qauKRSA50.net
> I myself am fully convinced that the best explanation an artist can give ★of★
> his aims and ability is afforded by his work.

この上の of については、十分に解説したつもりでしたが、それでも誤解し続ける人が
いるようなので仕方なく追加して言いますが、ここでは例の give of [something] という
熟語とは関係がないですね。

熟語として出てくる give of something は「〜を分け与える」という意味で、give のあとに
目的語はつかず、of のあとにだけ目的語がつきます。

しかし今回の場合は、すでに言ったように

An artist can ●give● the best explanartion ★of★ his aims and ability.

という文が土台になって、それが変化したようなものなのです。ここでの of も about みたいなものであり、
give を無視して
the best explanation ◆of◆ his aims and ability
を考えて、of は about みたいな意味だと考え、その about はその前の explanation にかかっていて、
"his aims などについての説明" というような意味になっているのですね。こういうふうな
"[something] of [something]"
の形についても、この上の方で引用したジーニアス英和にもちゃんと解説があって、

>>a) …について(の), …に関して(の)《◆堅い言い方》
>>stories of adventure
>>冒険の話 / A Tale of Genji 『源氏物語』

上のように書いていますね。

303 :名無しさん@英語勉強中 :2023/10/18(水) 12:59:48.11 ID:Dv/H7nI80.net
>>296

江川泰一郎『英文法解説』の前置詞の章の418 〜 423ページを参照
すればいいと思う。

304 :名無しさん@英語勉強中 :2023/10/18(水) 14:23:29.91 ID:lkBZgtRA0.net
>>296
僕なら時吉師匠の『英文法の鬼100則』のP.368を推すなぁ。

305 :名無しさん@英語勉強中 (ワッチョイ d155-AzOG):2023/10/18(水) 15:26:23.10 ID:m3ZhQYSs0.net
数学で使われる英語表現について質問です。

an interval around 0

はどう訳すのが自然でしょうか?

306 :名無しさん@英語勉強中 (ワッチョイ d155-AzOG):2023/10/18(水) 15:28:39.17 ID:m3ZhQYSs0.net
「interval」は区間です。

例えば、 -1 < x < 1 をみたす実数 x の集合は区間です。

307 :名無しさん@英語勉強中 (ワッチョイ d155-AzOG):2023/10/18(水) 15:29:41.78 ID:m3ZhQYSs0.net
0 の周りの区間では意味が通じません。

おそらく、 0 を含む区間という意味ではないかと思いますが、合っていますか?

308 :名無しさん@英語勉強中 (オッペケ Sr05-dk25):2023/10/18(水) 15:57:32.51 ID:XNjJ1THjr.net
フードデリバリーの配達員してるのですが、この頃外人に多く配達するので
配達に使える慣用表現や使いまわしの利く表現などをいくつか知りたいです

309 :The OED Loves Me Not (ワッチョイ 8b89-pCU2):2023/10/18(水) 16:02:33.22 ID:qauKRSA50.net
>>307
数学について詳しくは知りませんが、そういう意味だと思います。

310 :名無しさん@英語勉強中 (ワッチョイ d155-AzOG):2023/10/18(水) 16:05:02.13 ID:m3ZhQYSs0.net
>>309
ありがとうございました。

311 :ブタ耳 (ワッチョイW d17e-S405):2023/10/18(水) 16:50:38.89 ID:dMz1VQgX0.net
>>308
prompt:
フードデリバリーの配達員してるのですが、この頃外人に多く配達するので、配達に使える慣用表現や使いまわしの利く表現などをいくつか知りたいです

bing (GPT-4,創造的):
https://sl.bing.net/fniukdFvxI
(抜粋)
まず、お客様に電話をかける場合は、以下のような表現があります。

- Hello, this is (your name) from (delivery service name). I'm on my way with your order. (こんにちは、(デリバリーサービス名)の(あなたの名前)です。ご注文の品をお届けに向かっています。)

- Hello, this is (your name) from (delivery service name). I'm outside your building. Could you please come down to pick up your order? (こんにちは、(デリバリーサービス名)の(あなたの名前)です。お客様の建物の外にいます。ご注文の品を受け取りに来ていただけますか?)

- Hello, this is (your name) from (delivery service name). I'm having trouble finding your address. Could you please give me some directions? (こんにちは、(デリバリーサービス名)の(あなたの名前)です。お客様の住所が見つかりません。道順を教えていただけますか?)

次に、お客様に商品を渡す場合は、以下のような表現があります。

- Here is your order. Please check the items and the receipt. (こちらがご注文の品です。商品とレシートをご確認ください。)

- Thank you for choosing (delivery service name). Enjoy your meal! ((デリバリーサービス名)をご利用いただきありがとうございます。お食事をお楽しみください!)

- I'm sorry for the delay. There was a lot of traffic. Thank you for your patience. (遅れて申し訳ありません。渋滞がひどかったんです。ご理解いただきありがとうございます。)

最後に、チップをもらう場合は、(ry

312 :ブタ耳 (ワッチョイW d17e-S405):2023/10/18(水) 17:02:28.60 ID:dMz1VQgX0.net
phind (GPT-4):
https://www.phind.com/search?cache=ksl63eerqjgk7xql0k29w8ye

(抜粋)
1 配達の到着を伝える:
"I have a delivery for you."(あなた宛の配達物があります)
"Your order is here."(ご注文の商品が到着しました)

2 確認を求める:
"Could you please confirm your name?"(お名前を確認させていただけますか?)
"Is this the correct address?"(こちらが正しい住所でしょうか?)

3 時間について伝える:
"Your delivery will arrive in about 30 minutes."(配達は約30分後に到着します)
"I'm sorry for the delay. Your order will arrive soon."(遅延して申し訳ありません。ご注文の商品はすぐに到着します)

4 ありがとうと言う:
"Thank you for your order."(ご注文ありがとうございます)
"Have a great day!"(良い一日を!)

313 :名無しさん@英語勉強中 (ワッチョイ 7381-yGvl):2023/10/18(水) 22:22:03.49 ID:jSM7FqgF0.net
モンキーに英語なんて難しいが使えない日本語よりましだろう

314 :296 (ワッチョイ 3196-iZeJ):2023/10/18(水) 22:50:33.02 ID:AZWXwv5L0.net
>>296です。
ご回答いただいた皆様ありがとうございました。
たいへん勉強になりました。

315 :名無しさん@英語勉強中 :2023/10/18(水) 23:36:22.23 ID:pWYj74lX0.net
At an evening party the guestd were asked to
take part in a game in which everybody was to
make a face, the one who made the worst face to
win the race.
なんですが、the one... to win the raceの
ところなんですが、分詞構文みたいなもので
不定詞構文というのがあるのかな?と思ったの
ですが、文法書を見たところそういう項目が
なかったのですが、こういう例とかあれば
教えてください。解説もよろしくお願いします。
the one to win the raceのところについてだけですが。

316 :名無しさん@英語勉強中 :2023/10/18(水) 23:41:57.09 ID:pWYj74lX0.net
-1 < x < 1 をみたす実数 x の集合は区間です。
The set of real values such as -1<x<1 is an interval.

317 :The OED Loves Me Not :2023/10/18(水) 23:46:10.88 ID:qauKRSA50.net
>>315
... the one who made the worst face to win the race.

それは、すでに誰でも知っている文法事項を二つほど組み合わせたらおしまいの
構文ではないですか?

At an evening party the guestd were asked to
take part in a game in which everybody was to
make a face, the one who made the worst face to
win the race.

この原文を
At an evening party the guestd were asked to
take part in a game in which everybody was to
make a face. ★And★ the one who made the worst face ●was● to
win the race.

上のように二つに分けたとして、その二つ目の文を分詞構文にして

At an evening party the guestd were asked to
take part in a game in which everybody was to
make a face, the one who made the worst face ★being★ to
win the race.

と書いて、そのあとに being を省略すればいいのです。

318 :名無しさん@英語勉強中 (ワッチョイ fb23-nU4p):2023/10/19(木) 00:30:25.23 ID:/4Uztq360.net
>>317
ありがとう。よくわかりました。

319 :名無しさん@英語勉強中 (ワッチョイ fbc7-nU4p):2023/10/19(木) 02:34:18.28 ID:/4Uztq360.net
>>317
思いつきませんでした。ありがとう。

320 :名無しさん@英語勉強中 :2023/10/19(木) 17:24:43.54 ID:IW0uQdOq0.net
質問させていただきます。
At the house of Lady Adeline Amundeville and her husband, Lord Henry Amundeville, the narrator informs that Don Juan's hostess, Lady Adeline, is "the fair most fatal Juan ever met", the "queen bee, the glass of all that's fair,...
という一文のthe fairという名詞はそのまま公正さと解釈してよいのでしょうか?どうもしっくり来ないのですが。

321 :名無しさん@英語勉強中 :2023/10/19(木) 17:25:43.98 ID:IW0uQdOq0.net
質問させていただきます。
At the house of Lady Adeline Amundeville and her husband, Lord Henry Amundeville, the narrator informs that Don Juan's hostess, Lady Adeline, is "the fair most fatal Juan ever met", the "queen bee, the glass of all that's fair,...
という一文のthe fairという名詞はそのまま公正さと解釈してよいのでしょうか?どうもしっくり来ないのですが。

322 :名無しさん@英語勉強中 :2023/10/19(木) 17:55:06.23 ID:u5PxfqNv0.net
「英文解釈新装版」p64にある文章です。

The incident shows that what people believe the duties of men and women to be differs from one country to another.

この文内のdiffersですが、語尾のsは三単現のsだと思います。
でも、dutiesと複数形になっているので、おかしいのではないか、正しくはdifferなのではないかと思うのですがいかがでしょうか?

たくさんの頭のいい人たちが目にしてきた伝統的な本なので間違いのわけないという気もしますが、どうしてdiffersになっているのかわからないんです。

323 :The OED Loves Me Not (ワッチョイ 8b89-pCU2):2023/10/19(木) 18:01:23.93 ID:5CMIg2ZM0.net
>>321
> At the house of Lady Adeline Amundeville and her husband, Lord Henry Amundeville,
the narrator informs that Don Juan's hostess, Lady Adeline, is "the fair most fatal Juan ever met",
the "queen bee, the glass of all that's fair,...

the fair は、「公正さ」という意味ではないと思います。ここでは、the fair most fatal" は "the most fatal fair (lady)"
という意味だと思います。そういう語順じゃなくて the fair most fatal としたのは、Lord Byron がその詩において
リズムを重視してあえてそのような語順にしたのだろうと思います。

the most fatal fair (lady) とは、「最高に致命的な(命取りになるほどの)美女」という意味だと思います。
fair は beautiful みたいな意味合いの、少し古い言い回しなんだろうと思います。少し昔の文章においては、
fair をそういう意味で使っていることが多いように思います。

324 :名無しさん@英語勉強中 (ワッチョイW d118-nmXS):2023/10/19(木) 18:05:45.70 ID:1Pe5pgby0.net
>>322
what節はふつう単数扱いになります
複数にしてもいいみたいだけどね

325 :The OED Loves Me Not (ワッチョイ 8b89-pCU2):2023/10/19(木) 18:08:27.67 ID:5CMIg2ZM0.net
>>322
(1) The incident shows that --- The incident は、that 以下のことを示している。

(2) what people believe the duties of men and women to be --- that 以下の節の長い主語
what S V O C という構文。
「人々が男女の義務はこういうものだと信じている事柄(そしてこれは単数を受ける)

(3) differs from one country to another.--- that 以下の節の述語

主語は what... to be なので、単数を受ける。だから、differs はそのままでよいのです。

326 :The OED Loves Me Not (ワッチョイ 8b89-pCU2):2023/10/19(木) 18:13:09.34 ID:5CMIg2ZM0.net
>>324 さんの言う通りですが、付け加えると、

https://www.google.com/search?q=%22our+duties+are+a+%22&rlz=1C1FQRR_enJP949JP949&oq=%22our+duties+are+a+%22&gs_lcrp=EgZjaHJvbWUyBggAEEUYOTIHCAEQIRigATIHCAIQIRigATIKCAMQIRgWGB0YHjIMCAQQIRgPGBYYHRgeMgwIBRAhGA8YFhgdGB4yDAgGECEYDxgWGB0YHjIHCAcQIRiPAjIHCAgQIRiPAtIBCTE3Mzg1ajBqN6gCALACAA&sourceid=chrome&ie=UTF-8&bshm=rime/1

この上のリンク先を見ると
Our duties are a
という phrase を含む用例がたくさん並びます。つまり
Our duties are...
となるとその補語は必ず複数形になるかというと、そういうわけではなく
a のついた単数形が来ることも大いにあるのです。

そういうわけで、duties が主語になっても、そのあとの補語は複数形でも単数形でもいいのです。
そして今回の場合は、補語にあたる what は単数形として扱われているわけです。

327 :名無しさん@英語勉強中 (ワッチョイ 3196-iZeJ):2023/10/19(木) 18:30:03.51 ID:u5PxfqNv0.net
>>324-326
納得できました。ありがとうございました。

328 :The OED Loves Me Not (ワッチョイ 8b89-pCU2):2023/10/19(木) 18:46:03.72 ID:5CMIg2ZM0.net
>>321 について、私は >>323 にて
>>the fair most fatal としたのは、Lord Byron がその詩において
>>リズムを重視してあえてそのような語順にしたのだろうと思います。

と言ってしまいましたが、実はそうじゃなくて、フランス語の語順に従って
英語を書いただけなのかもしれません。

もともと fatal という言葉を女性について言うときには、
femme fatale
というフランス語の言葉があります。それをそのまま英語の中で使う
こともあるだろうし、それを英語風に変えて
the fair fatal
などというときもあるのかもしれません。そして fatal に most をつけるときには
the fair most fatal
と書いていたのかもしれません。

ところで、femme fatale の fatale という女性形の形容詞に le plus つまり
most という言葉をつけると、

femme la plus fatale
となりますが、これは実際に使われているようです。現に

Mais qui, de la douairière qui manipule le mariage de sa petite-fille
ou de l'aventurière aux sens chauds, est ★la femme la plus fatale★ ?
https://www.payot.ch/Detail/une_vieille_maitresse-jules_barbey_daurevilly-9782080709554?fp=1

上のように、ちゃんと使われています。

329 :The OED Loves Me Not (ワッチョイ 8b89-pCU2):2023/10/19(木) 18:47:28.33 ID:5CMIg2ZM0.net
>>321 について、私は >>323 にて
>>the fair most fatal としたのは、Lord Byron がその詩において
>>リズムを重視してあえてそのような語順にしたのだろうと思います。

と言ってしまいましたが、実はそうじゃなくて、フランス語の語順に従って
英語を書いただけなのかもしれません。

もともと fatal という言葉を女性について言うときには、
femme fatale
というフランス語の言葉があります。それをそのまま英語の中で使う
こともあるだろうし、それを英語風に変えて
the fair fatal
などというときもあるのかもしれません。そして fatal に most をつけるときには
the fair most fatal
と書いていたのかもしれません。

ところで、femme fatale の fatale という女性形の形容詞に le plus つまり
most という言葉をつけると、

femme la plus fatale
となりますが、これは実際に使われているようです。現に

Mais qui, de la douairière qui manipule le mariage de sa petite-fille
ou de l'aventurière aux sens chauds, est ★la femme la plus fatale★ ?
https://www.payot.ch/Detail/une_vieille_maitresse-jules_barbey_daurevilly-9782080709554?fp=1

上のように、ちゃんと使われています。

330 :The OED Loves Me Not (ワッチョイ 8b89-pCU2):2023/10/19(木) 18:51:01.72 ID:5CMIg2ZM0.net
>>321
Lord Byron の時代には、イギリス人は何でもフランスの真似をしようとしていたので、
the fair most fatal がフランス語的な言い回しを真似ていたのだと考えても
不思議はありません。

331 :英語職人 ◆azN58fWNzw (ワッチョイ 2962-/4nw):2023/10/19(木) 19:51:07.17 ID:BVXSVDlF0.net
>>321
Lady Adeline, is "the fair most fatal Juan ever met", the "queen bee, the glass of all that's fair,
この部分を「現代の日本の小説家ならこう書くだろうな」というものを書いてみました。 🤠
===============================
レディ・アデリーヌは、「ファンが出会った中で最も運命的な美女」と形容され、その妖艶な姿は人間の枠を超えた魅惑的な女性として浮かび上がる。彼女は 女王蜂であり、美のすべてを映し出す魅惑的な存在である。
===============================

>>323
>致命的な(命取りになるほどの)美女
命取りになる美女ってなんだ?🤔 ここでは、「宿命的な」って意味ですよ🤠

332 :名無しさん@英語勉強中 (ワッチョイW 7b71-vzyT):2023/10/19(木) 20:27:07.91 ID:Tx2PtZ4b0.net
ごめんなさい
質問させてくだだい
All joking aside, Kagura bachi is getting this kind of meme reaction because there is no horizon for industry heavy hitter manga anymore.
冗談はさておき、カグラバチはミームになった
なぜなら~の先の意味がしっくりきません
もう業界のヒットマンガの限界がないから
そういう訳でいいんですか?

333 :名無しさん@英語勉強中 (ワッチョイW 7b71-vzyT):2023/10/19(木) 20:27:57.01 ID:Tx2PtZ4b0.net
ごめんなさい
質問させてくだだい
All joking aside, Kagura bachi is getting this kind of meme reaction because there is no horizon for industry heavy hitter manga anymore.
冗談はさておき、カグラバチはミームになった
なぜなら~の先の意味がしっくりきません
もう業界のヒットマンガの限界がないから
そういう訳でいいんですか?

334 :名無しさん@英語勉強中 (ワッチョイ 13e2-edRi):2023/10/19(木) 20:51:20.13 ID:QCYZ1MK30.net
>>333
これから先、その他には(かつては沢山あった)大ヒットするような作品が出る展望が無い(漫画オタクにとってはお先真っ暗)ってことでしょう

335 :名無しさん@英語勉強中 (アウアウウーT Sadd-oXAT):2023/10/19(木) 20:57:16.66 ID:TlbvW1FNa.net
大航海時代に船で大洋を進んでいて、水平線にポツンと島が点のように見えてきたら目的地が近づいてきている証左で嬉しい希望の光的な感じじゃないかね。

・on the horizon=地平線(上)に、〔〜の〕兆しが見えて

・There is little sign of an economic recovery on the horizon. : 景気回復の兆しはほとんど見えない。
〔災難などが〕差し迫って

・There is a big disaster on the horizon waiting for you. : 大きな災難があなたに迫っている。

・There is hope on the horizon. : この先に希望がある。

336 :英語職人 ◆azN58fWNzw (ワッチョイ 2962-/4nw):2023/10/19(木) 23:39:14.23 ID:BVXSVDlF0.net
>>321  追記です。
"the fair most fatal Juan ever met"について補足。
※ここでのfairは、古い英語で「美女」(名詞)という意味があります。fairには、他にも古い英語で「美しさ」「美」という名詞の意味もあります。
https://eow.alc.co.jp/search?q=fair
↑ここの「名詞」参照

A: She is the fair most fatal (that) Juan (has) ever met.
B: She is the most fatal fair (that) Juan (has) ever met.
Aは基本的にBと同じ意味であるが、Aは古い文学的表現で散見されるもので、「fair」(美人)を強調したいときに使われます。現代英語では、Aは使われません。

337 :The OED Loves Me Not (ワッチョイ 8b89-pCU2):2023/10/19(木) 23:44:13.32 ID:5CMIg2ZM0.net
>>331
>>323
>致命的な(命取りになるほどの)美女
命取りになる美女ってなんだ?🤔 ここでは、「宿命的な」って意味ですよ🤠

私はそういう意味で言ってたのです。「宿命的な」も、もともとはそういう意味だったでしょう?
私の言葉の使い方が悪かったなら、それは認めますがね。現代ではやはりあなたの言う通りの意味で使っているはずですから。
ともかく、いちいち揚げ足をg取らないでください。

338 :The OED Loves Me Not (ワッチョイ 8b89-pCU2):2023/10/19(木) 23:55:53.33 ID:5CMIg2ZM0.net
>>333
他のお二人の仰る通りだと思います。そして industry heavy hitter は
文字通りの意味はたぶん「業界での大ヒット作」とかいう意味でしょうね。

339 :The OED Loves Me Not (ワッチョイ 8b89-pCU2):2023/10/19(木) 23:57:40.23 ID:5CMIg2ZM0.net
>>338 は蛇足でした。すでに >>333 が同じことを書いていましたね。ごめんなさい。

340 :英語職人 ◆azN58fWNzw (ワッチョイ 2962-/4nw):2023/10/20(金) 00:07:05.98 ID:bd4l60BZ0.net
>>337
「致命的な」と「宿命的な」は、どう見ても別の意味です。揚げ足取りではなく、致命的な間違いです。
明らかな間違いを「訂正しないでくれ」って、それは公共の利益から見て問題ですよ🤠

基本的に私は、訂正行為などしたくはないのです。お願いですから、今後は間違わないでください。間違わなければ私は出てこなくてすむのですから。

341 :名無しさん@英語勉強中 (ワッチョイ 5368-T9H3):2023/10/20(金) 00:07:47.00 ID:1n2mQita0.net
<< The OED loves 英語職人 >>
この2人のコンビは素晴らしい。お笑いコンビを結成して
M1参加も検討してほしい!

342 :名無しさん@英語勉強中 (ワッチョイ 599d-AzOG):2023/10/20(金) 11:11:09.45 ID:aiCY5oin0.net
>>333
ミームとは、人から人へと変化しながら伝えてゆく考えなど
つまり、業界のヒットマンガにおいては、国境がなくや伝達方法も捉え方も限界がないということ

343 :名無しさん@英語勉強中 :2023/10/20(金) 11:54:56.13 ID:wZBuOJIv0.net
>国境がなくや伝達方法も捉え方も限界がないということ

ちがう。カグラバチが「ミーム」になるほどの人気作だという話。

ただし、一昔前なら他にも鬼滅やら何やら大作がひしめいていて
当時ならカグラバチ程度の作品がそこまで(ミームになるほど)殊更持て囃されることはなかっただろうに、
翻って今の漫画界の状況はそれだけ良作が乏しく悲惨だ、ということ。

becauseの用法。

344 :名無しさん@英語勉強中 :2023/10/20(金) 11:55:12.49 ID:wZBuOJIv0.net
>国境がなくや伝達方法も捉え方も限界がないということ

ちがう。カグラバチが「ミーム」になるほどの人気作だという話。

ただし、一昔前なら他にも鬼滅やら何やら大作がひしめいていて
当時ならカグラバチ程度の作品がそこまで(ミームになるほど)殊更持て囃されることはなかっただろうに、
翻って今の漫画界の状況はそれだけ良作が乏しく悲惨だ、ということ。

becauseの用法。

345 :名無しさん@英語勉強中 :2023/10/20(金) 11:57:47.83 ID:wZBuOJIv0.net
boards.4channel.org/a/thread/258544974/all-joking-aside-kagura-bachi-is-getting-this
All joking aside, Kagura bachi is getting this kind of meme reaction because there is no horizon for industry heavy hitter manga anymore.
No future Hero academy, demon slayer, no SNK, no actual new naruto, no new HxH, no CSM, no jjk, even big seinens like berserk, etc, etc, all are ending/ended, fell off, or both, or something else.
One piece is one piece. You might think some of these are shit, and you might be right, but they're undeniably cultural mainstays that carried the industry and garnered global attention.
I unironically believe JJK's explosion in popularity is because its the only manga that filling the need for a big 3 esque shounen.
Whats gonna happen to manga? Where are all the new, ambitious, long form, upcoming manga? Is it literally over? What the fuck is going on? Did they literally give up?

346 :名無しさん@英語勉強中 :2023/10/20(金) 11:57:53.79 ID:wZBuOJIv0.net
boards.4channel.org/a/thread/258544974/all-joking-aside-kagura-bachi-is-getting-this
All joking aside, Kagura bachi is getting this kind of meme reaction because there is no horizon for industry heavy hitter manga anymore.
No future Hero academy, demon slayer, no SNK, no actual new naruto, no new HxH, no CSM, no jjk, even big seinens like berserk, etc, etc, all are ending/ended, fell off, or both, or something else.
One piece is one piece. You might think some of these are shit, and you might be right, but they're undeniably cultural mainstays that carried the industry and garnered global attention.
I unironically believe JJK's explosion in popularity is because its the only manga that filling the need for a big 3 esque shounen.
Whats gonna happen to manga? Where are all the new, ambitious, long form, upcoming manga? Is it literally over? What the fuck is going on? Did they literally give up?

347 :The OED Loves Me Not :2023/10/20(金) 12:42:54.74 ID:IrvuHerc0.net
>>333 に関して、>>344 >>366 の話は、私にとって勉強になった。

今回の話と関連してるかどうかは私にはわからないが、次のようなサイトの英文に、私は注目している。

Meaning of There Is No Horizon by Ulcerate
Ulcerate

★The song "There Is No Horizon" by Ulcerate delves into themes of despair, submission, and the absence of hope.★
The lyrics paint a bleak picture of a world consumed by fear and anguish, where individuals are burdened by their own suffering and weariness.
https://www.songtell.com/ulcerate/there-is-no-horizon

●horizonless●
in American English
ADJECTIVE
1. lacking or without a horizon
2. ★without hope; hopeless★
https://www.collinsdictionary.com/dictionary/english/horizonless#:~:text=Definition%20of%20'horizonless'&text=1.,without%20hope%3B%20hopeless

348 :The OED Loves Me Not (ワッチョイ 8b89-pCU2):2023/10/20(金) 13:16:35.34 ID:IrvuHerc0.net
>>333
There is no horizon for [something].
が、本当に「〜には希望がない」という意味なのかどうかについては、
まだ私には断言できないけど、もし仮にそういう意味だとすると、
そういう意味での

(1) no horizon
(2) There is no horizon.
(3) There is no horizon for [something].

という言い回しは、まだ一般の辞書には出ていないようだし、
ネット上で一生懸命に用例を探しても、まだかなり少ないように見える。
つい最近になって使われ始めたのだろうか?


My beautiful mom , Diana , Iran, the 60’s. She was free and had a dream to marry her friend neighbor…
I imagine little girls in Iran dreaming but today ★there is no horizon for their dreams★… my dreams go to women life freedom
https://www.instagram.com/p/Co-b2osLIk0/?img_index=1

この上のリンク先には、かなりの美少女の写真が載っていて、そこにこの caption がついている。
前後の関係から、どうやらこれも「〜には希望がない」みたいな意味合いらしいということがわかる。

349 :名無しさん@英語勉強中 (ワッチョイ 599d-AzOG):2023/10/20(金) 13:18:11.26 ID:aiCY5oin0.net
horizonとは、空と大地との境界線であり
また、horizonがあるとは、遠くまでよく見えるということでもある

350 :The OED Loves Me Not (ワッチョイ 8b89-pCU2):2023/10/20(金) 13:22:55.24 ID:IrvuHerc0.net
>>333
From East-Jerusalem to Jenin in the West Bank, from Rafah to Beit Hanoun in the Gaza Strip,
from Jerash camp to Zarqa in Jordan, from Ein El-Helweh to Nahr El-Bared in Lebanon,
and from Dera’a to Aleppo in Syria, Palestine Refugees experience profound anxiety
about their future. ★There is no horizon for them★, neither political nor personal.
https://www.unrwa.org/newsroom/official-statements/new-horizons-sustainable-funding-unrwa

この上の一節でも、There is no horizon for ... は、「〜には希望がない」という意味であるように思える。

351 :The OED Loves Me Not (ワッチョイ 8b89-pCU2):2023/10/20(金) 15:27:45.25 ID:IrvuHerc0.net
>>333
この下の There is no horizon for... もやはり、「〜には希望がない」という意味のように見える。

There is no horizon for me.
I think it’s my time to end it. I try to be a good person and help my family
and friends but it’s not enough. I always feel like an outsider even being included in activities.
(この後にも長く文章が続きますが、省略します。)
https://www.reddit.com/r/SelfHate/comments/13l07nw/there_is_no_horizon_for_me/

352 :The OED Loves Me Not (ワッチョイ 8b89-pCU2):2023/10/20(金) 15:30:58.00 ID:IrvuHerc0.net
この下のものは、新聞記事。

“Biden will do nothing to change the existing reality,” he said.
“★There is no horizon for★ the Palestinian-Israel conflict.”
https://jp.reuters.com/article/us-palestinians-usa-biden-idAFKBN2OC3LB

353 :The OED Loves Me Not (ワッチョイ 8b89-pCU2):2023/10/20(金) 15:36:07.75 ID:IrvuHerc0.net
"There is no horizon for..." の用例は少ないと言ってしまったけど、実はけっこうありそうだ。
新聞記事らしきものにも、続々と見つかる。

こんなにも広く使われているように見えるのに、いまだに辞書などに出てこないのは不思議だ。
ネイティブなど、英語のよくできる人にとっては当たり前のことのように思えるので、
いちいち辞書には紹介しないということかもしれないし、あるいはつい最近に
使われるようになったからかもしれない。

We have come to the streets under the slogan ‘There is no free homeland, without free women,’
to redefine the concept of emancipation in our homeland, and to emphasize
that ★there is no horizon for★ national liberation without justice, dignity and freedom for each of us.
https://peoplesdispatch.org/2019/09/30/in-photos-there-is-no-free-homeland-without-womens-freedom/

354 :英語職人 ◆azN58fWNzw (ワッチョイ 290b-/4nw):2023/10/20(金) 15:51:21.03 ID:bd4l60BZ0.net
horizonの隠喩としての意味。隠喩なので辞書には載っていないが、ネイティブは次の意味で、horizonを使うことがあるよ。🕊

地平線の内側にあるものは何だろう?私たちの現在の居場所であり、何が起こるかを十分知っているあたりまえの既存の場所である。しかし、地平線の向こうには未知の世界が広がっている。そこには、問題を解決してくれるなにか、退屈を吹き飛ばしてくれるなにか、苦痛を癒してくれるなにか、あるいは人をハッピーにしてくれるなにかなど、私たちを喜ばせワクワクさせてくれるお宝が眠っているかもしれない。これこそが、英語におけるhorizonの象徴としての隠喩としての意味なのである。Horizonとは、眠っているお宝と出会う尖端である。そこに到達したとき、未知なるものが既知となり、また新たなhorizonが生まれてくるのだ。

なので、「もうhorizonがない」は、「もうこれ以上は、新境地(人を驚かすような刺激的なお宝など)を期待できない」という意味なのである。🤠

355 :名無しさん@英語勉強中 (ワッチョイW d1aa-nmXS):2023/10/20(金) 17:26:53.14 ID:5CtC/Pqq0.net
>there is no horizon for industry heavy hitter manga anymore.

↑no horizonより、これのforの後の構造ってどうなってんの?
みんな構造わかってる上で訳を言ってるの?
それとも「なんかよくわからんけどまぁ多分こんな感じっしょ」くらいな感じで言ってるの?

356 :英語職人 ◆azN58fWNzw (ワッチョイ 290b-/4nw):2023/10/20(金) 18:30:49.63 ID:bd4l60BZ0.net
>>355
え、わかんないの、そこ? 😃

英語には「industry heavy hitter」(業界の重鎮)という決まり文句があるんだよ。
それをmangaに修飾させて、「漫画業界のかなめとなる漫画作品」という意味を出している。

picture bookのように名詞が名詞を修飾するのは基本的に合法であるが、しかし、「industry heavy hitter manga」なんて、プロの文筆家なら使わない、こなれていない表現なので、覚えてほしくない英語表現ではある。

357 :英語職人 ◆azN58fWNzw (ワッチョイ 290b-/4nw):2023/10/20(金) 18:35:24.76 ID:bd4l60BZ0.net
「industry heavy hitter」(業界の重鎮)の使用例

https://www.kestrafinancial.com/newsroom/news-releases/industry-heavy-hitter-john-amore-joins-kestra-financials-executive-team

https://energycentral.com/news/industry-heavy-hitter-appointed-project-development-director-salamander-floating-wind-farm

358 :名無しさん@英語勉強中 :2023/10/20(金) 19:07:26.25 ID:5CtC/Pqq0.net
なんだ、それで一つの言い方なのか
それならok

359 :名無しさん@英語勉強中 :2023/10/20(金) 19:09:01.73 ID:5CtC/Pqq0.net
>>356
>「industry heavy hitter manga」なんて、プロの文筆家なら使わない、こなれていない表現なので、覚えてほしくない英語表現ではある。

↑そういう言い方があるんなら、むしろこの言い方は普通やな

360 :名無しさん@英語勉強中 :2023/10/20(金) 19:26:47.77 ID:uiHa1Ho/0.net
>地平線までの距離は?
>観察者からの距離は成人(目の高さ1.5m)で約4.4kmほどである。

361 :名無しさん@英語勉強中 (ワッチョイW aa11-DZK0):2023/10/21(土) 01:26:12.50 ID:wZSLycES0.net
Nihon国の場合Nihon人はなんて表記になるの?

362 :名無しさん@英語勉強中 (ワッチョイ 659d-dytz):2023/10/21(土) 11:30:11.46 ID:uGRlsZYQ0.net
horizon
【考古】 文化的広がり, 文化ホライズン
《(1) 広範囲の地域にわたってみられる同一の特徴の型をもつ遺物・遺跡の広がり
(2) それによって定義される文化(圏)・時代》
(リーダーズ・プラス)

there is no horizon for の表現は、これから派生してるかも

363 :名無しさん@英語勉強中 (ワッチョイ 659d-dytz):2023/10/21(土) 12:02:06.50 ID:uGRlsZYQ0.net
『失われた地平線』(原題: Lost Horizon)ジェームズ・ヒルトンによる長編小説。
内容(「BOOK」データベースより)
正体不明の男に乗っ取られた飛行機は、ヒマラヤ山脈のさらに奥地に不時着した。
偶然通りかかった謎の集団が導く先には、西洋の技術文明と東洋古来の精神文化が組み合わさった不老不死の楽園があった。

364 :名無しさん@英語勉強中 (ワッチョイ 1ace-Nl27):2023/10/21(土) 12:23:10.01 ID:1LNzgYc50.net
この原文の中で、no horizon を no future, no new, all are ending と言い換えている

>>345 Kagura bachi is getting this kind of meme reaction because there is no horizon for industry heavy hitter manga anymore. No future Hero academy, demon slayer, no SNK, no actual new naruto, no new HxH, no CSM, no jjk, even big seinens like berserk, etc, etc, all are ending/ended, fell off, or both, or something else.

365 :名無しさん@英語勉強中 (ワッチョイ 1a31-Nl27):2023/10/21(土) 18:30:56.10 ID:1LNzgYc50.net
>>334 これから先、その他には(かつては沢山あった)大ヒットするような作品が出る展望が無い(漫画オタクにとってはお先真っ暗)ってことでしょう

この解釈で正しいと思います。

366 :名無しさん@英語勉強中 :2023/10/21(土) 21:19:21.20 ID:PTroXzjH0.net
「2つの枕を左右にではなく前後に重ねて並べ置く」って
英語でどう表現したらいいでしょうか

367 :名無しさん@英語勉強中 :2023/10/21(土) 22:00:40.03 ID:1LNzgYc50.net
2つの枕を左右にではなく前後に重ねて並べ置く
(Google): put two pillows front and back instead of left and right

368 :ブタ耳 :2023/10/21(土) 22:05:24.42 ID:aJ9IdRAL0.net
https://www.bing.com/images/create/two-pillows-placed-up-to-down-along-the-center-lin/6533cb73688b490aa9e170cf5c4065f5?id=nH0M9L%2bF6qPQEdiDWqDxkw%3d%3d&view=detailv2&idpp=genimg&idpclose=1&FORM=SYDBIC

こりゃ駄目だ

369 :The OED Loves Me Not :2023/10/21(土) 22:19:19.50 ID:t+OTlp0S0.net
>>366
> 「2つの枕を左右にではなく前後に重ねて並べ置く」

put two pillows one in front of the other, not side by side, with the two in contact with each other

370 :名無しさん@英語勉強中 :2023/10/21(土) 23:13:53.62 ID:1mvEOF360.net
>>369
それだと、上から見た場合を排除できないので、こんな感じ↓として伝わってしまう。
https://www.bing.com/images/create/put-two-pillows-one-in-front-of-the-other2c-not-sid/6533dc1279894716b5a4d254a6c911fd?id=rpPVrsnCAG%2f0bBeLS1FS3w%3d%3d&view=detailv2&idpp=genimg&FORM=GCRIDP&mode=overlay

371 :英語職人 :2023/10/22(日) 00:40:35.86 ID:ZpHmlMGJ0.net
>>366
「2つの枕を左右にではなく前後に重ねて並べ置いてください」は、英語でつぎのように言うとよいでしょう。
 → Please position (the) two pillows vertically, with their backs touching each other.
   (2つの枕を立てた状態で、枕が互いに背中合わせになるように置いてください。)

372 :英語職人 ◆azN58fWNzw (ワッチョイ 8d77-xUKY):2023/10/22(日) 00:55:28.17 ID:ZpHmlMGJ0.net
>>366
>>371の場合、枕の長い辺を底にするのか、短い辺を底にするのか迷う人もいると思うので、イラストで説明するのがベストでしょう。

英語で言う場合は、「The long sides of the pillows need to be at the bottom.」となります。

373 :The OED Loves Me Not :2023/10/22(日) 06:58:20.91 ID:QddtuyRm0.net
>>370
久しぶりに建設的な批評を頂きました。ありがとうございます。

>>366
再び挑戦。実は、夜中に寝床で考えていたんです。

なお、イラストや写真を添えて英文を示した方がいいですが、今回の質問は、わざとイラストなしでどれくらいに厳密で
正確な英文が書けるかをテストするような、おそらくは理科系の学部あたりの英語関係の問題だと思っています。

2つの枕を左右にではなく前後に重ねて並べ置く。(命令文だと想定します。)
Put two pillows one in front of the other on a horizontal plane (such as on a floor),
with their longitudinal centerlines not aligned but in parallel with each other,
and with their lateral sides touching each other.

後でまた考え直し、さらにいくつかの代案を示していくかもしれません。簡単そうでいて、実は難しい問題だと思います。

374 :The OED Loves Me Not :2023/10/22(日) 07:02:41.45 ID:QddtuyRm0.net
>>366
>>2つの枕を左右にではなく前後に重ねて並べ置く。

ここでは、長い方の辺を下にして、枕を立てるような形にしないといけないという
ことを言っているのですか?

私自身は、そうではなくて人が寝るときのように枕を置くのだと思い込んでいましたけど。
イラストか何かで、それぞれの回答者がどういうものをイメージしているかを
示しあうことができればいいんですけどね。

375 :名無しさん@英語勉強中 :2023/10/22(日) 07:14:47.93 ID:srW06gMz0.net
>>366
そういう前後左右ならline them up vertically, not horizontallyみたいに言うよん
arrangeって言ってもおっけ

376 :The OED Loves Me Not :2023/10/22(日) 07:24:21.11 ID:QddtuyRm0.net
>>371 >>375
このお二人は vertically という言葉を使っていますが、
私の理解ではそれは垂直方向つまり人間が直立しているときの
頭から脚までの方向を言うのだと思っていますが、
そういうふうに垂直に枕を重ねろ、という問題なのですか?

私はそういう意味ではないと思っていたのですが。

377 :The OED Loves Me Not :2023/10/22(日) 07:29:22.89 ID:QddtuyRm0.net
>>366
この日本語文を Image Creator で作らせると
https://www.bing.com/images/create/efbc92e381a4e381aee69e95e38292e5b7a6e58fb3e381abe381a7e381afe381aae3818fe5898de5be8ce381abe9878de381ade381a6e4b8a6e381b9e7bdaee3818f/6534503ef5cc4bcfbb2133342ccfe5b5?FORM=GENCRE
上のような画像が出てきますが、私がこの日本語文から考えていた並べ方とはまったく違います。

もしかして、それぞれの回答者は互いに全く違う並べ方を頭に置いているのではないかと思います。

378 :名無しさん@英語勉強中 :2023/10/22(日) 07:58:14.94 ID:srW06gMz0.net
verticallyってのは縦にって意味だよ
なんて言ったらいいんだろう人から見た上下っていうか
例えばcut it verticallyって言ったら縦に切ってって意味なんだけど
見た目の上下の他にも重力に対しての垂直とか図形に対しての垂直って意味でも使えるよん
並べるんじゃなくて上下に積むって言うんだったら細かいけどそもそもstackって言うかな

379 :名無しさん@英語勉強中 :2023/10/22(日) 08:37:19.30 ID:4ljdPcOr0.net
>>377
being ですか
僕はマイクロソフトのアカウントを持ってるけど
Skypeでbeingが使えない。

380 :ブタ耳 :2023/10/22(日) 08:39:25.10 ID:jzGTDTSO0.net
>>377
DALL-E3は、DALL-2より相当進化してるそうですが、物体の配置指定に関してはまだまだ能力不足であるように感じます。
以前DALL-2に「うつ伏せに置かれたコップ」のイメージを生成させようとして苦労しましたが、このイメージはDALL-E3でも相変わらず難題であるようです。

件の枕の配置については、私の次のように解釈しています。
{===}を真上からみた枕だとして、

左右に並べる
{===}{===}

縦に並べる
{===}
{===}

381 :英語職人 :2023/10/22(日) 08:57:52.63 ID:ZpHmlMGJ0.net
>>376
verticallyには、uprightの意味があるから、「まくらをそれぞれ立てて置く」という意味が出せるのです。間違いではないけれども、そのように誤解を受ける可能性があるのであれば、verticallyをuprightと置き換えればよいのです。🤠

証拠
https://www.wordreference.com/definition/vertically

382 :英語職人 :2023/10/22(日) 09:00:25.89 ID:ZpHmlMGJ0.net
verticallyにuprightの意味があることの証拠2

https://www.merriam-webster.com/dictionary/vertically

383 :英語職人 :2023/10/22(日) 09:05:48.21 ID:ZpHmlMGJ0.net
改訂版
>>366
「2つの枕を左右にではなく前後に重ねて並べ置いてください」は、英語でつぎのように言うとよいでしょう。
 → Please position (the) two pillows upright, with their backs touching each other.
   (2つの枕を立てた状態にして、枕が互いに背中合わせになるように置いてください。)

枕の長い辺を底にするのか、短い辺を底にするのか迷う人もいると思うので、イラストで説明するのがベストでしょう。
英語で言う場合は、「The long sides of the pillows need to be at the bottom.」となります。

384 :名無しさん@英語勉強中 :2023/10/22(日) 09:08:56.97 ID:LqViH9CC0.net
DeepLのレイアウトが数日前からいきなりおかしくなってしまい治りません。
MACではOS 11以降でないとこうなるのですか?

web版だとなんとかいけるのですが

385 :名無しさん@英語勉強中 :2023/10/22(日) 09:09:02.13 ID:LqViH9CC0.net
DeepLのレイアウトが数日前からいきなりおかしくなってしまい治りません。
MACではOS 11以降でないとこうなるのですか?

web版だとなんとかいけるのですが

386 :英語職人 :2023/10/22(日) 09:09:35.09 ID:ZpHmlMGJ0.net

ここでのuprightは副詞で、in an upright positionの意味を持ち、「直立させて」と訳すことができます。

https://www.merriam-webster.com/dictionary/upright

387 :英語職人 :2023/10/22(日) 09:32:29.49 ID:ZpHmlMGJ0.net
『Please position the two pillows upright, with their backs touching each other. The long sides of the pillows need to be at the bottom.』を、DEEPLに訳させてみました。
DEEPL「2つの枕を背中が触れ合うように立ててください。枕の長い辺が下になるようにしてください。」

完璧だね!🤠

388 :The OED Loves Me Not :2023/10/22(日) 10:06:34.98 ID:QddtuyRm0.net
「縦に」というのは、少なくとも正式には longitudinally であって、
vertically は「垂直に」つまり人間が直立したときの頭から足への
方向を言うはずなんですけどね。少なくとも理科系の人たちは
そういう意味で使っているはずです。それについては、
Cambridge あたりの英英辞典を引けばすぐに詳しくわかります。

ただし一般の人は、そこら辺をいい加減に使っているだろうけど

389 :The OED Loves Me Not (ワッチョイ d689-JD/u):2023/10/22(日) 10:27:56.29 ID:QddtuyRm0.net
だから、みんな互いに全く違う意味で日本語原文を理解していて、それに応じた
英文を提示しているだけなのだから、どれが正解だなんてことは、今のところはわかりません。

390 :The OED Loves Me Not (ワッチョイ d689-JD/u):2023/10/22(日) 10:32:07.99 ID:QddtuyRm0.net
>>382
「証拠」どころか、vertically の本当の意味はそれだけですよ。
それ以外の意味はないはずです。

たとえば「ベッドに枕を縦(の方向)に置く」というと、
「ベッドの長さ方向に」というわけだから
longitudinally とか
in longitudinal direction of the bed
とかいうはずです。

391 :英語職人 ◆azN58fWNzw (ワッチョイ 8de0-xUKY):2023/10/22(日) 10:37:44.67 ID:ZpHmlMGJ0.net
>>389
そんなバカな。>>366を見れば、まともな国語力があれば、誰でも、>>380の意味だと認識しますよ。🤠

392 :The OED Loves Me Not (ワッチョイ d689-JD/u):2023/10/22(日) 10:40:06.09 ID:QddtuyRm0.net
>>380
ブタ耳さんの解釈は、まさに私の解釈とまったく同じです。
どうやら、他の人たちはそれとは違う意味で解釈しているようです。

393 :The OED Loves Me Not (ワッチョイ d689-JD/u):2023/10/22(日) 10:42:09.51 ID:QddtuyRm0.net
>>391
ブタ耳さんは、

左右に並べる
{===}{===}

縦に並べる
{===}
{===}

この図も、真上から見た様子だと解釈しているものと
私は考えています。それならば、私と同じ解釈です。

そして職人さんたちは、それとは違う解釈をしておられるように思えてしまいます。

394 :英語職人 ◆azN58fWNzw (ワッチョイ 8de0-xUKY):2023/10/22(日) 10:56:08.14 ID:ZpHmlMGJ0.net
□□□ まとめ □□□
>>366
「2つの枕を左右にではなく前後に重ねて並べ置いてください」は、英語でつぎのように言うとよいでしょう。
 → Please position (the) two pillows upright, with their backs touching each other.
   (2つの枕を立てた状態にして、枕が互いに背中合わせになるように置いてください。)

状況に応じて、「The long sides of the pillows need to be at the bottom.」(長い辺を下にしてください)を追加します。

※backは正確には枕の「背面」のことですが、表裏がない枕なら、「面」の代用として使用できます。
 &#9679;まくらの背面が「back」である証拠
https://www.pinterest.jp/pin/266205027943990166/
 &#9679;正面をbackと表現していそうな例
https://www.flickr.com/photos/shaebay/5695835753/
https://www.pinterest.jp/pin/497436721313308629/

395 :ブタ耳 (ワッチョイW c15a-flU9):2023/10/22(日) 10:57:20.08 ID:jzGTDTSO0.net
>>393
そうです、真上から見た様子です。

議論が錯綜してしまいそうですが、あえて二つ疑問点を提出してみます。

1) 縦に並べる(=前後に重ねて並べる?>>366)場合でも、"side by side"と表現できるのでは? 枕を長方形として考えた場合、その長辺も"side"と言えるのではないか。

2) この場合の「縦」を、verticalと表現することもあるのではないか?

(2)については、AIに次のように質問してみたところ、答えはYESでした。

If the x-axis and y-axis are orthogonal to each other on the same plane, and the x-axis is considered horizontal, can the y-axis be described as vertical?

396 :英語職人 ◆azN58fWNzw (ワッチョイ 8de0-xUKY):2023/10/22(日) 10:59:04.10 ID:ZpHmlMGJ0.net
>>393
私もブタ耳さんと同じ解釈ですよ。これ以外に解釈する人なんているわけないでしょ。🤠

397 :名無しさん@英語勉強中 (ワッチョイW 5d9d-R2Gb):2023/10/22(日) 11:02:12.82 ID:WpXl4GK20.net
Put two pillows in a bundle. (If you do anything strange, I'll beat you up.)

398 :英語職人 ◆azN58fWNzw (ワッチョイ 8de0-xUKY):2023/10/22(日) 11:05:56.46 ID:ZpHmlMGJ0.net
わかった。😃
ブタ耳さんのまくらが水平におかれているか、立てて置かれているかで意見が分かれているのかもね。
私は、立てて置かれていると考えたわけ。もし、ブタ耳さんが、まくらを寝かせた状態で、>>380を書いたのなら、私の意見はブタ耳さんと違う。

>>366の「前後に重ねて並べ」って、これはどうみても、枕が立っている状態でしょ?寝かせた枕なら、「重ねて」にならないでしょ?

399 :The OED Loves Me Not (ワッチョイ d689-JD/u):2023/10/22(日) 11:11:23.75 ID:QddtuyRm0.net
>>398
そうです。職人さんたちは、枕を垂直方向に立てて(つまり、危なっかしい姿勢で)置いていると
解釈しているようでウ。

そして私は、枕を水平方向において、つまり普通の人が寝るときに枕を置いた時の様子として
解釈しているのです。

だから、なんで vertically なんていう言葉を使うのかと、ずっと不思議に思っていました。

400 :英語職人 ◆azN58fWNzw (ワッチョイ 8de0-xUKY):2023/10/22(日) 11:12:27.40 ID:ZpHmlMGJ0.net
https://www.costco.co.jp/medias/sys_master/images/ha7/h8c/171466241048606.webp

>>366はこういう状態を言っていると自分は判断。

401 :The OED Loves Me Not (ワッチョイ d689-JD/u):2023/10/22(日) 11:13:16.89 ID:QddtuyRm0.net
枕を「重ねて」って言われていても、水平方向に二つ並べていれば、
「重ねた」ことになるかもしれない、と私は思ったのです。

第一、日本語をそんなに厳密にみんなが使っているとも限らないですから。

402 :英語職人 ◆azN58fWNzw (ワッチョイ 8de0-xUKY):2023/10/22(日) 11:14:26.69 ID:ZpHmlMGJ0.net
>>399

OEDさんの解釈では、>>366の「前後に重ねて並べ」の「重ねて」の意味を失っていると思いませんか?

403 :名無しさん@英語勉強中 (ワッチョイ f196-iKuA):2023/10/22(日) 11:55:44.07 ID:smfH5SDL0.net
お願いします。

The man and his wife rushed out.(彼と妻は急いで外に出た)

これを倒置で表現すると
Out rushed the man and his wife. 
となるそうなんですが、rushedよりも前にoutが出てくるというのが感覚的にしっくりきません。
Rushed out the man and his wife.ではダメなんでしょうか?

また、倒置においては動詞よりもさらに先に副詞が前に出る、という決まりがあるのでしょうか?

404 :ブタ耳 (ワッチョイW c15a-flU9):2023/10/22(日) 12:07:14.23 ID:jzGTDTSO0.net
AI用にこういうprompt考えた。
なかなかの傑作だと思う。

Suppose the surface of a bed is represented on an Excel sheet.
The area of the bed is from A1 to E12.
You put two pillows on C1 and C2.
If you want to someone else to place two pillows like this on a real bed, what instruction will you give?

405 :The OED Loves Me Not (ワッチョイ d689-JD/u):2023/10/22(日) 12:49:24.48 ID:QddtuyRm0.net
>>403
「しっくり来な」くても、それでもそのまま受け入れるしかないのです。
それが異文化を受け入れることです。
(*)Rushed out the man and his wife.

なんていう言い方はしないはずです。一般に

副詞 + 動詞 + 主語.
というのが定式のようですが、それだからこそ
次のような言い回しが巷に溢れかえっているのです。

Here comes the man.
Here comes the judge.
Here comes the bus.
There gores 主語.
In the village, lived a simple good-natured fellow named Rip Van Winkle.
Around seven to six million years ago lived what is known as the Ardipithecus group.

406 :The OED Loves Me Not (ワッチョイ d689-JD/u):2023/10/22(日) 12:52:49.72 ID:QddtuyRm0.net
>>404
確かに傑作です。

407 :名無しさん@英語勉強中 (ワッチョイ 659d-dytz):2023/10/22(日) 13:04:28.57 ID:konWI9kG0.net
>>344
ジャンプ新連載『カグラバチ』ネットミームで異例の国際的ヒット? 欧米圏の読者から大反響
https://realsound.jp/book/2023/09/post-1444578.html
9月18日から週刊少年ジャンプ(集英社)にて連載が開始された漫画『カグラバチ』の様子が何やらおかしい。
いや、おかしいのは作品そのものではなく、それを取り囲むネットの反応なのだ。
なぜか今、主に欧米圏のジャンプ読者たちによって本作が異例の大盛り上がりを見せている。

408 :ブタ耳 (ワッチョイW c15a-flU9):2023/10/22(日) 13:08:56.03 ID:jzGTDTSO0.net
>>406
改めて見直すと、want to のto が余計ですね。

このプロンプトに対するphindの応答から結論だけ抜き出すと、
"Place the first pillow at the top of the bed, about one-third of the way from the left side. Place the second pillow immediately below the first one in the same vertical alignment."

C1とC2の位置関係を"vertical"と捉えているようです。

409 :名無しさん@英語勉強中 :2023/10/22(日) 13:38:42.16 ID:0CAAaTfw0.net
単語学習アプリで日本語→英語の練習できるのってありますか?
和英辞書じゃなくてレベル別の学習アプリが欲しいのです
自分の検索能力では見つけられなかったのでここで質問させて下さい

410 :名無しさん@英語勉強中 :2023/10/22(日) 14:02:27.76 ID:konWI9kG0.net
>>403
out(外に)を強調した表現だよ

411 :名無しさん@英語勉強中 :2023/10/22(日) 14:07:25.31 ID:Of0J+fa80.net
>>366
Objects placed in a row
https://u.osu.edu/mclc/2023/06/07/objects-placed-in-a-row/

412 :名無しさん@英語勉強中 :2023/10/22(日) 16:31:51.40 ID:4ljdPcOr0.net
>>404
囲碁の最初の手を答えよみたいなプロンプトじゃないですか?
そんな好みの手を答えさせることにどんな意味がありますか?

413 :名無しさん@英語勉強中 (ワッチョイ f196-iKuA):2023/10/22(日) 18:11:50.63 ID:smfH5SDL0.net
>>405
ありがとうございました。

414 :ブタ耳 (ワッチョイW c119-flU9):2023/10/22(日) 18:36:03.63 ID:jzGTDTSO0.net
>>412
これ(>>404)は、>>366氏の質問にある「2つの枕が左右にではなく前後に重ねて並べて置かれた状況」を、ベッドの表面をExcelのシートに見立てて規定したプロンプトですよ。
C1とC2に枕が並んでるイメージが思い浮かびますか?
この配置を日常言語でどう表現するかをAIに尋ねているのです。

ところで、私は366氏の質問を、幾何学的な配置に関する英語表現上の工夫に関する興味に基づくものだと受け取った上で、>>380のような配置を前提にしてあれこれ表現を模索してきました。
けれども366の問いは、枕の配置に関する実用上の必要性から成されたものであって、その際366氏の念頭にあったのは、英語職人氏の提示した>>400のような配置だった可能性が高いと今では考えています。
C1とC2に枕を並べても実用的な意味は無いですよね。

415 :🤭英ちゃん🫢 (ワッチョイW 7a86-gqR+):2023/10/22(日) 18:39:06.13 ID:DmVXOHW90.net
囲碁じゃないが
枕の3Dの位置関係の言語表現を聞きたいのに、それを2Dのエクセルで図示して、どこが傑作なのか判らなかった🤔

416 :ブタ耳 (ワッチョイW c119-flU9):2023/10/22(日) 18:45:20.34 ID:jzGTDTSO0.net
>>415
私が問題にしてきたのは専ら2Dの位置関係です。

417 :🤭英ちゃん🫢 (ワッチョイW 7a86-gqR+):2023/10/22(日) 18:52:09.81 ID:DmVXOHW90.net
399も相当抜けてる
この人はホテルに泊まったことないのかしらね

418 :🤭英ちゃん🫢 (ワッチョイW 7a86-gqR+):2023/10/22(日) 19:03:49.57 ID:DmVXOHW90.net
366を読んでベッドメイキングの指南だと思った
ヨーロッパでは一人2個枕は当たり前
日本のお高めホテルも最近はそうだろう
Marriott hotel bed ででも検索しなさい

立てて置くか、積んで置くか、個人によるが、前者を「危なっかしい姿勢で」なんて言う人、はじめて見た🤭

419 :🤭英ちゃん🫢 (ワッチョイW 7a86-gqR+):2023/10/22(日) 19:14:58.89 ID:DmVXOHW90.net
>>373
>今回の質問は、わざとイラストなしでどれくらいに厳密で正確な英文が書けるかをテストするような、おそらくは理科系の学部あたりの英語関係の問題

↑からの↓

>>389
>みんな互いに全く違う意味で日本語原文を理解していて

>>401
>第一、日本語をそんなに厳密にみんなが使っているとも限らないですから。

この人バカじゃない??

420 :🤭英ちゃん🫢 (ワッチョイW 7a86-gqR+):2023/10/22(日) 19:31:10.83 ID:DmVXOHW90.net
ホテルのベッドに枕がたててあるの見たら、「危なっかしい!!😱😱😱」って言って、慌てて駆け寄って寝かせるのかなぁ…

頭スキャンしてもらった方がいいと思う…

421 :🤭英ちゃん🫢 (ワッチョイW 7a86-gqR+):2023/10/22(日) 19:56:35.57 ID:DmVXOHW90.net
>>394
1つ目はキティちゃんの背面だからじゃん
3つ目は刺繍した表に対して、刺繍のない方をbackにしてるじゃん

422 :🤭英ちゃん🫢 (ワッチョイW 7a86-gqR+):2023/10/22(日) 19:58:46.77 ID:DmVXOHW90.net
>>394
2つ目も刺繍が趣味の人みたいだから、刺繍の表に対して、裏だろ

423 :ブタ耳 (ワッチョイW c119-flU9):2023/10/22(日) 20:06:02.31 ID:jzGTDTSO0.net
意図通りの画像生成に1枚成功。
https://www.bing.com/images/create/image-of-two-pillows-on-a-bed2c-one-of-which-is-pro/65350155b21a4a6983ad106d8ac75594?id=UB3q8Vn2xIDhLjMKTb%2biCQ%3d%3d&view=detailv2&idpp=genimg&idpclose=1&FORM=SYDBIC&ssp=1&darkschemeovr=1&setlang=ja-JP&safesearch=off

424 :ブタ耳 (ワッチョイW c119-flU9):2023/10/22(日) 20:26:32.56 ID:jzGTDTSO0.net
改良版
https://www.bing.com/images/create/23imagine-realistic-photograph-image-of-two-white-/653506682b07450c9383c0468336df5f?id=lgwiX7aOVGJVsjlHSGgIhA.srSNth14RxH7oBWMfGSypQ&view=detailv2&idpp=genimg&idpclose=1&form=SYDBIC&ssp=1&darkschemeovr=1&setlang=ja-JP&safesearch=off

425 :🤭英ちゃん🫢 (ワッチョイW 7a86-gqR+):2023/10/22(日) 20:35:44.62 ID:DmVXOHW90.net
frontがあるからbackがあるんだよ
表裏がなかったら、面の意味で使える?

ソフィ先生とエロリストの悪いとこは、自分の領分を変えた部分を適当に創作するとこです😔

426 :英語職人 ◆azN58fWNzw (ワッチョイ 8d99-xUKY):2023/10/22(日) 22:11:57.08 ID:ZpHmlMGJ0.net
>>425
3年英太郎ちゃん、また恥をかきに出てきたの?(^O^)
じゃあ、聞くけど、表と裏が同一のまくらがあった場合、どっちがbackなんだよ? どっちでもbackになりえるだろ。
同じ柄なら、backと言えないのかな? そんなわけはない。同じ柄なら、どっちもbackになりえるのだ。どっちがbackなのか使ってる人もよくわからないなら、どっちの面もbackになりえるだろ。だから、そういう状況では実質「面」という意味で使うことになる。
もっと、物事は深く考えましょう。🤠 IQが低くなければこの私の反論に納得できるのだが・・・果たして3年ちゃんは、わかるかなあ~?

>>422
>>394の下ふたつのリンク先の画像は、99%裏表同じ模様だぞ。表がきれいなのに平凡な裏をわざわざ写真に撮って見せるかよ。よく考えろって。😆 なんでそんなこともわからない? しかも投稿者のコメントは「この生地が大好き。この生地は刺繍にぴったりだと思う」としか言っていない。勝手に「まくらに刺繍をしてる」なんて創作するなって。どっちが創作癖なんだか。>>394下のふたつのリンクは、表と裏の柄が同じ可能性が高い。それは、IQが高めの人であれば、誰でも予想できることだと思うよ。🤠

427 :🤭英ちゃん🫢 (ワッチョイW 7a86-gqR+):2023/10/22(日) 22:21:39.39 ID:DmVXOHW90.net
>>426
嘘つき😠
The fuckers are... が表だろが😡

428 :英語職人 ◆azN58fWNzw (ワッチョイ 8d99-xUKY):2023/10/22(日) 22:27:23.65 ID:ZpHmlMGJ0.net
裏・表同じ柄で、使っている人もどっちが裏か表か分からない枕があったとする。

ある日、私がネイティブにその枕のある面を「back」(背面)と表現し、その翌日に別の面を「back」と言ったときに、そのネイティブは、「どっちが表か裏か分からないのだから、backと言ってはいけない」と言うのだろうか?

言うわけないだろ!!! どっちも「back」で事実上問題ないのだ。この状況下では、backはもう数学的に「面」という意味しか持ちえないのである。

証明おわり

429 :🤭英ちゃん🫢 (ワッチョイW 7a86-gqR+):2023/10/22(日) 22:29:01.14 ID:DmVXOHW90.net
>>426
https://www.flickr.com/photos/shaebay/5695836081/in/photostream
これが表だ
嘘つきブラザーめ😤

430 :英語職人 ◆azN58fWNzw (ワッチョイ 8d99-xUKY):2023/10/22(日) 22:31:06.29 ID:ZpHmlMGJ0.net
>>427
下2つのリンク先の枕の反対側に刺繍がある証拠を出してみなさい。刺繍なんかあるわけないだろ😆  

431 :英語職人 ◆azN58fWNzw (ワッチョイ 8d99-xUKY):2023/10/22(日) 22:34:03.05 ID:ZpHmlMGJ0.net
>>429
あ、ほんとだ。よく見つけたね。 それは、私の負けだわ~(^O^)

432 :英語職人 ◆azN58fWNzw (ワッチョイ 8d99-xUKY):2023/10/22(日) 22:36:06.39 ID:ZpHmlMGJ0.net
でも、>>428により、「表裏がなかったら、面の意味で使える?」という3年ちゃんの反論は論破したから、今回は引き分けってことで・・・(^-^)

433 :🤭英ちゃん🫢 (ワッチョイW 7a86-gqR+):2023/10/22(日) 22:39:53.38 ID:DmVXOHW90.net
論破❓🤔💬

434 :🤭英ちゃん🫢 :2023/10/22(日) 23:01:26.96 ID:DmVXOHW90.net
エロリストは、飛行機のアナウンスがLadies and gentlemenからdear passengersになったことも知らなかったし、ほぼ引きこもりな人生だろな

あと、Skype が~とか言ってて、ホントに友だちいないんだ~😢と思った(笑)

435 :英語職人 :2023/10/22(日) 23:02:26.35 ID:ZpHmlMGJ0.net
>>433
枕の裏の刺繍の件は、ホントごめんね m(__)m 反省してます。 

話は変わり、では、>>428に反論してみて。
「どっちが裏か表か分からない枕なら、どっちの面もbackと呼んでOK」が私の主張だからね。これに反論できるならやってみて。

436 :🤭英ちゃん🫢 :2023/10/22(日) 23:10:44.46 ID:DmVXOHW90.net
最初に表が存在し、表が存在した上で裏が存在できるのだ

いわば裏は有標であって、複数個の裏表どっちでもいいものに対し、「裏と裏を合わせる」はひねくれた言い方

以上です。これ以上ソフィ先生の言い訳は聞きません🙇

437 :🤭英ちゃん🫢 :2023/10/22(日) 23:29:34.19 ID:DmVXOHW90.net
追記

ジェスチャー込みでソフィ先生の訳で十分通じると思うので、それで別に良いと思います。語学教師をはじめてから、通じりゃいいじゃん派にますます傾いて来てる今日この頃😌

あれみて理系の問題だと思った人は救いがたいアホだと思いますが(笑)(笑)

438 :名無しさん@英語勉強中 (ワッチョイW c18b-hgvi):2023/10/23(月) 10:28:47.77 ID:gFQGrisI0.net
>>197
>>191

遅レスすいません
なるほどですね!ある程度はそういうものだと思っていいと思うと少し気が楽になりました

439 :名無しさん@英語勉強中 (ワッチョイ a554-/eJP):2023/10/23(月) 16:59:03.75 ID:LsJgn7UJ0.net
? Language as we know it is a human invention .
 「私たちが知っているような言語は、人間が発明したものである」
? Language as we know is a human invention .
「私たちが知っている言語は、人間が発明したものである」

?のasは「接続詞」で ?のasは「制限用法の関係代名詞」だと思いますが、
?は日本語訳において一般に「ような」を入れて訳をあてることからも
?と?は日本語の上ではニュアンスが異なるように思うのですが、
これはあくまでも「日本語の問題」であって、英語での思考においては
?も?も事実上意味は同じと考えてよいでしょうか?

「接続詞のas」は「非制限用法の関係代名詞のas」との比較で
意味の違いを示して解説されているものはよく見かけるのですが、
「制限用法の関係代名詞のas」との比較では意味(ニュアンス)の
違いを解説しているものはあまり見かけないので質問してみました

440 :名無しさん@英語勉強中 (ワッチョイ 1a07-ANn9):2023/10/23(月) 17:28:08.80 ID:rUOaJxIB0.net
>>439
というか2番目は英語的にちょっとヘンなのでそもそもそういう風には言わないかな

441 :The OED Loves Me Not (ワッチョイ d689-JD/u):2023/10/23(月) 17:48:45.87 ID:rGUauAUK0.net
>>439
? Language as we know is a human invention .
「私たちが知っている言語は、人間が発明したものである」

誰がその和訳を作り出したのかは知りませんが、それはたぶん誤訳でしょう。
実際には、as we know は文の全体にかかっていて、

Language as we know is a human invention.

Language, as we know, is a human invention.
Language - as we know - is a human invention,
As we know, language is a human invention.
Language as we know is a human invention, as we know.
と同じような意味になるのだと私は理解しています。

しかし (1) の
Language as we know it is a human invention.
の as we know it は、あくまでその直前の Language だけにかかっています。

442 :🤭英ちゃん🫢 (ワッチョイW 7a86-gqR+):2023/10/23(月) 17:50:04.57 ID:AZbNh94f0.net
2もいいはず!

443 :The OED Loves Me Not (ワッチョイ d689-JD/u):2023/10/23(月) 17:51:18.43 ID:rGUauAUK0.net
>>441 にて、書き間違いをしました。正しくは、次の通り。(訂正した部分は、★の部分です。)

Language, as we know, is a human invention.
Language - as we know - is a human invention,
As we know, language is a human invention.
★Language is a human invention, as we know.★
と同じような意味になるのだと私は理解しています。

444 :🤭英ちゃん🫢 (ワッチョイW 7a86-gqR+):2023/10/23(月) 17:53:31.78 ID:AZbNh94f0.net
新英和大
as conj. 6b

The origin of English as we know it is commonly traced back to the eighth century

445 :🤭英ちゃん🫢 (ワッチョイW 7a86-gqR+):2023/10/23(月) 17:54:56.62 ID:AZbNh94f0.net
あ、これじゃねー

446 :🤭英ちゃん🫢 (ワッチョイW 7a86-gqR+):2023/10/23(月) 17:59:55.86 ID:AZbNh94f0.net
the origin of school as we know them

themが入ってたわ
|彡サッ

447 :名無しさん@英語勉強中 :2023/10/23(月) 20:13:45.29 ID:LsJgn7UJ0.net
(擬似)関係代名詞のas は「制限用法」では
実際に用いられることはまずない
と考える方が妥当でしょうか?

448 :The OED Loves Me Not (ワッチョイ d689-JD/u):2023/10/23(月) 21:24:31.95 ID:rGUauAUK0.net
>>447
そのことは、
"language as we know is "
という phrase を検索し、
https://www.google.com/search?q=%22langauge+as+we+know+is+%22&rlz=1C1FQRR_enJP949JP949&oq=%22langa&gs_lcrp=EgZjaHJvbWUqBggAEEUYOzIGCAAQRRg7MgYIARBFGDkyBwgCEC4YgAQyBwgDEAAYgAQyBwgEEC4YgAQyBggFEEUYQDIJCAYQABgKGIAEMgcIBxAAGIAE0gEIMzY0NmowajeoAgCwAgA&sourceid=chrome&ie=UTF-8&bshm=rime/1#ip=1
上のような検索画面にて、片っ端からその phrase の前後を読んでいけばいいのです。
そうすれば、質問者さんなりの予想が立てられるはずです。

こんな質問をして、その回答者たちの回答をたくさん得たとしても、
その回答者たちの回答が正しいのかどうかを、質問者さんは
どのように検証するおつもりですか?

そんなのできないでしょう?仮に権威ある課のように見える大学教授100人に尋ねたとしても、
その人たちが正しいことを言うかどうかなんて、わからないでしょう?
ましてや、こんな匿名掲示板でたくさんの回答者の回答を読んでも、それぞれの
回答者がどこのどういう人で、どれくらいに英語を知っているかなんて、
どうやって確かめられのでしょうか?いい加減なことを言っていないということを
どうやって確かめるのですか?

それなら、自分で実際の英文をたくさん読んで、自分なりの見識を持つしかないと
思いませんか?

私なりの見解は、すでに述べました。そしてその見解は、
私が今まで英文を読んできた経験と、
さらにはこの上に挙げたリンク先のたくさんの英文の前後関係を
読み取った結果とを総合して引き出したのです。

449 :英語職人 ◆azN58fWNzw (ワッチョイ 8de0-xUKY):2023/10/23(月) 22:35:03.52 ID:g8xgMqUm0.net
>>442 >>444

3年英太郎ちゃんの悪いとこは、自分の領分を変えた部分を適当に創作するとこです😔

450 :🤭英ちゃん🫢 (ワッチョイW 7a86-gqR+):2023/10/23(月) 22:47:39.97 ID:AZbNh94f0.net
創作してませんが😗
自分ですぐに気づける自浄能力付です

451 :🤭英ちゃん🫢 (ワッチョイW 7a86-gqR+):2023/10/23(月) 22:48:00.08 ID:AZbNh94f0.net
*領分を超えたとこ

452 :英語職人 ◆azN58fWNzw (ワッチョイ 8de0-xUKY):2023/10/23(月) 23:16:34.44 ID:g8xgMqUm0.net
>>450
冗談なので、気にしないでね 🤠

453 :英語職人 :2023/10/23(月) 23:22:58.62 ID:g8xgMqUm0.net
>>451

>>425に、「領分を変えた部分」とあるから仕方ない 😃

454 :英語職人 :2023/10/23(月) 23:40:43.10 ID:g8xgMqUm0.net
>>447
疑似関係詞asは、suich, the same, as, soと一緒なら制限用法で使えるよ。
(正)I have never heard such a story as he told us.

455 :英語職人 :2023/10/23(月) 23:44:11.83 ID:g8xgMqUm0.net
■□■ 上記の枕の英作問題について 追記 ■□■
※この問題は場合分けが必要
① 表と裏が明確である枕の場合は、「2つの枕を左右にではなく、一方の枕の背面に他方の枕の前面がくるように、前後に重ねて並べ置いてください」と言うべきだと思われる。これは、英語でどう表現したらよいだろうか? 英語では、次のように書くとよい。
『Please position one pillow with its back touching the front of the other pillow, both standing upright.』
状況に応じて、「The long sides of the pillows need to be at the bottom.」(長い辺を下にしてください)を追加します。

②表と裏が同じ柄の枕で、表と裏を区別できない場合。この場合、どっちも背面に成り得るので、
Please position the two pillows upright, with their backs touching each other.
で、問題ない。
Please place the two pillows back to back, both standing upright.でも良い。

③表裏が明確な枕で、「裏表関係なく、適当に前後に重ねて」という意味を出したい場合。こんな感じで書くとよい。
『Please position one pillow with its front/back touching the front/back of the other pillow, both standing upright. (The long sides of the pillows need to be at the bottom.)』

456 :英語職人 :2023/10/23(月) 23:48:36.05 ID:g8xgMqUm0.net
訂正
>>447
疑似関係詞asは、such, the same, as, soと一緒なら制限用法で使えるよ。

(正)I have never heard such a story as he told us.

457 :名無しさん@英語勉強中 (ワッチョイ a5b9-/eJP):2023/10/24(火) 01:37:40.64 ID:vAOI9f/Q0.net
「天気予報では山頂付近で夕方から雪が降ると言っていたように、
われわれが下山するやいなや大雪が降り始めた」
を時制を正確に、接続詞を使って一文の英語で書くなら
どういう表現になりますでしょうか?

458 :The OED Loves Me Not :2023/10/24(火) 04:27:37.61 ID:Y3w6nC720.net
>>457
> 「天気予報では山頂付近で夕方から雪が降ると言っていたように、
> われわれが下山するやいなや大雪が降り始めた」
> を時制を正確に、接続詞を使って一文の英語で

(1) なるべく短く書くと
As the weather forecast said that in the evening it would begin snowing near the mountaintop,
heavy snow began to fall on our arrival at the foot.

(2) 少し長く書いてもいいなら
As the weather forecaster said that it would begin snowing near the mountain summit in the evening,
we began to have heavy snow as soon as we arrived at the foot.

459 :名無しさん@英語勉強中 :2023/10/24(火) 09:25:18.23 ID:93lq+h7U0.net
「グルメ」というフォルダ名をネイティブな人が使ってるようなカッコいいフォルダ名に変更したいです。
「Foodie」とかは意味的に合ってますか?

460 :名無しさん@英語勉強中 :2023/10/24(火) 10:27:58.16 ID:F7mkAzC9M.net
なんか笑ったw ちなみにそのフォルダは何のフォルダなの?
人? 店? 料理? つか画像フォルダなのか?

461 :ブタ耳 :2023/10/24(火) 11:02:46.09 ID:l1j9Qq0K0.net
lengthwiseという語の用法を調べています。

Longmanの定義は、
"in the direction or position of the longest side"
・Lay the bricks lengthwise

この例文のDeepL訳は、
「レンガを縦に並べる」なのですが、厳密に言うと「レンガを横向きに並べる」が正しいのでは?

[===]をレンガとして

[===] [===] [===] [===] [===]
[===] [===] [===] [===] [===]
[===] [===] [===] [===] [===]
[===] [===] [===] [===] [===]

また、地面に1個のレンガを
[===]
のように、立てるのでなく寝かせて置くことを
Lay the brick lengthwise
と言えるのでしょうか?

462 :ブタ耳 (ワッチョイW c186-flU9):2023/10/24(火) 11:26:54.72 ID:l1j9Qq0K0.net
bingに>461のレスをそのままコピペして質問したら、詩まで作ってくれた。
https://sl.bing.net/iaLYiyIBJvw

463 :ブタ耳 (ワッチョイW c186-flU9):2023/10/24(火) 11:32:55.33 ID:l1j9Qq0K0.net
リンク修正
https://sl.bing.net/e6GGE2meNuS

464 :名無しさん@英語勉強中 (ワントンキン MMfe-pyZ0):2023/10/24(火) 11:44:47.55 ID:F7mkAzC9M.net
よく知らねーけどレンガってそう積むことあんの?w なんつーかアホが積みそうなレンガだな
つか、どう言われようとレンガ積めって言われたら普通、フツーのレンガの積み方すんじゃね?
lay them lengthwiseって言われたところで「あーこの人はあれをlengthwiseって表現するんだな」程度で終わるっていうか
そんな深く考えないと思うよ

465 :ブタ耳 (ワッチョイW c186-flU9):2023/10/24(火) 11:58:36.52 ID:l1j9Qq0K0.net
じゃ、AA修正

[===] [===] [===] [===] [===]
==][===] [===] [===] [===] [==
[===] [===] [===] [===] [===]
==][===] [===] [===] [===] [==
[===] [===] [===] [===] [===]

466 :🤭英ちゃん🫢 (ワッチョイW 7a86-gqR+):2023/10/24(火) 12:10:56.23 ID:HwDnIecn0.net
発達障害の人多くない?🫢

467 :名無しさん@英語勉強中 (ワッチョイ 659d-dytz):2023/10/24(火) 12:15:20.70 ID:LSMMJfFr0.net
>>461
レンガの個々の向きではなくて、複数のレンガを並べる方向だと思う
レンガ以外の対象に対して縦方向に並べる
辺の長さが等しいレンガもある
レンガの個々の向きは言及してない

468 :ブタ耳 (ワッチョイW c186-flU9):2023/10/24(火) 12:34:11.23 ID:l1j9Qq0K0.net
>>467
レンガ3個を、lengthwiseに、
縦2m横4mの壁に置くとしたら、
[===][===][===]
縦4m横2nの壁に置くとしたら、
[===]
[===]
[===]
ということですよね?

しかし現実世界で壁全体にレンガを置く場合、縦2m横4mだろうが縦4m横2mだろうが、全面をレンガで埋め尽くすとすれば、壁の縦横は無関係なのではないでしょうか?

469 :ブタ耳 :2023/10/24(火) 12:53:46.57 ID:l1j9Qq0K0.net
Excelのセルみたいなレンガ積み方は、実際にもあって、stack bondというらしいです。
https://www.thespruce.com/masonry-brick-bond-common-types-2736655

念のため言っておきますが、私の関心は「実生活上どのようにレンガを置くか」にはありません。
「何らかの物体をlengthwiseに置くとはどういう意味なのか」ということにあります。

470 :名無しさん@英語勉強中 :2023/10/24(火) 13:26:09.06 ID:vAOI9f/Q0.net
>>458
ありがとうございます!
すいませんが、この接続詞のas を
「時」「比例」「理由」 「直前の名詞を限定」 「様態」 「譲歩」
みたいな分類に当てはめると何になりますでしょうか?

471 :名無しさん@英語勉強中 :2023/10/24(火) 13:35:02.99 ID:93lq+h7U0.net
>>459
グルメ関係の資料です。

472 :名無しさん@英語勉強中 (ワントンキン MMfe-pyZ0):2023/10/24(火) 13:47:04.84 ID:F7mkAzC9M.net
だからさ、lengthwiseって言葉には長さ方向って意味があるだけでそれ以上でも以下でもねーんだよ
どう置くかなんてのはその言い方でイメージが伝わるかどうかっていうまた別の話だろうよw
別に英語に限った話じゃなくね?

473 :The OED Loves Me Not (ワッチョイ d689-JD/u):2023/10/24(火) 13:52:11.64 ID:Y3w6nC720.net
>>470
済みませんが、それは各人が勝手に考えればおしまいの話であって、
私はそれをいちいち説明する気はありません。

前後の関係を読み取ればわかる話だと思います。

474 :名無しさん@英語勉強中 (ワッチョイ a552-/eJP):2023/10/24(火) 14:07:28.60 ID:vAOI9f/Q0.net
as が「理由」ではない以上、この as はどれに属するかが
どうしてもわからいので質問してみました

475 :🤭英ちゃん🫢 (ワッチョイW 7a86-gqR+):2023/10/24(火) 14:57:25.23 ID:6dYKOUKH0.net
実際に使う場がないと、外国語は呪物化するね
水を飲むためにあるコップが聖杯にでも見えてるみたいな

476 :名無しさん@英語勉強中 (ワッチョイ a575-/eJP):2023/10/24(火) 15:41:57.46 ID:2c+vMbAJ0.net
>>473
申し訳ございません
ただ、この日本文に対する英訳にはたして
as が使えるものかどうかが知りたいだけです

477 :名無しさん@英語勉強中 (ワッチョイW c182-CNxV):2023/10/24(火) 15:55:09.40 ID:FWWrqIYc0.net
>>476
そもそもね、>>457の日本語自体が本当はちょっとおかしいのよ
>>458の英語も本当は変。文法的に間違ってるとかではなく論理の話ね
自分でも「このasっていいのか…?」みたいに思ってるから質問してるんだと思うけど、その理由は論理的にちょっとおかしいから

478 :名無しさん@英語勉強中 (ワッチョイ 7ae2-fb7B):2023/10/24(火) 16:03:09.73 ID:gJWdbRKS0.net
>>470
「様態」だけど、その各分類でasをちゃんと理解してるなら明白じゃない?

479 :名無しさん@英語勉強中 (ワッチョイ 7ae2-fb7B):2023/10/24(火) 16:08:39.66 ID:gJWdbRKS0.net
・Do exactly as I told you. : 私があなたに言ったように正確にやりなさい。
・Like I said last year, we're not moving from here. 去年言ったように,ここから引っ越すつもりはありません

6. CONJUNCTION B1
You use as to say how something happens or is done, or to indicate that something happens or is done in the same way as something else.
I'll behave toward them as I would like to be treated.

・Today, as usual, he was wearing a three-piece suit.
・The book was banned in the U.S., as were two subsequent books.

Synonyms: in the way that, like, in the manner that

480 :名無しさん@英語勉強中 (ワッチョイ a575-/eJP):2023/10/24(火) 16:12:37.04 ID:2c+vMbAJ0.net
>>478
As the weather forecast said, ならば「様態」でわかるのですが
that in the evening it would begin snowing near the mountaintop と
言った内容を具体的に示した時点で「様態」にはなりえないのではないか
と思うのですが  それは間違いでしょうか?

481 :名無しさん@英語勉強中 (ワッチョイ a575-/eJP):2023/10/24(火) 16:16:58.03 ID:2c+vMbAJ0.net
>>478
As the weather forecast said, ならば「様態」でわかるのですが
that in the evening it would begin snowing near the mountaintop と
言った内容を具体的に示した時点で「様態」にはなりえないのではないか
と思うのですが  それは間違いでしょうか?

482 :The OED Loves Me Not (ワッチョイ d689-JD/u):2023/10/24(火) 16:56:50.68 ID:7SF+s8VQ0.net
>>457
「様態」とか何とかいう文法用語なんてどうでもよくて、他の回答者の言う通り、日本文の原文も、それを忠実に
訳したつもりの私の英訳も、変なんだろうと思います。だから

(1) 元の英訳:
As the weather forecast said that in the evening it would begin snowing near the mountaintop,
heavy snow began to fall on our arrival at the foot.

上の英文は破棄して、次のようにでも書けばおしまいじゃないですか?

(2) The weather forecast was right in saying
that in the evening it would begin snowing near the mountaintop.
Heavy snow did begin to fall on our arrival at the foot.

(3) The weather forecaster was right. In the evening heavy snow did begin to fall
near the mountaintop on our arrival at the foot.

元々の日本語の原文が気持ち悪かったけど、「すべてを一文にせよ。しかも時制を正確に訳せ」という指示にそのまま
従って、結局は損をしてしまいました。

483 :名無しさん@英語勉強中 (ワッチョイ 7ae2-fb7B):2023/10/24(火) 17:03:37.30 ID:gJWdbRKS0.net
>>480
日本語でも別に:

・彼の言うように、彼はもうそれ以上食べられない。
・彼も『満腹だ』と言っているように、彼はもうそれ以上食べられないよ。

どっちもあるでしょ。

484 :名無しさん@英語勉強中 (ワッチョイ a575-/eJP):2023/10/24(火) 17:30:38.79 ID:2c+vMbAJ0.net
No sooner had I got back to my station than it started to rain,
as the weather forecast had said it would in the afternoon.
「天気予報が午後は雨になると言っていたように、駅に着くとすぐに雨が降り始めた」
某予備校の先生の英語本にこういう例文が書いてあって
「ん? この as はいったいなんなんだろう??}
とずっと思っていたのでここで聞いて見ました
どうやらあまり自然ではない日本語&英語のようですね

485 :名無しさん@英語勉強中 (ワッチョイ a575-/eJP):2023/10/24(火) 17:32:15.60 ID:2c+vMbAJ0.net
No sooner had I got back to my station than it started to rain,
as the weather forecast had said it would rain in the afternoon.

すいません rain が抜けてました

486 :The OED Loves Me Not (ワッチョイ d689-JD/u):2023/10/24(火) 18:23:43.08 ID:7SF+s8VQ0.net
>>484
その予備校の本にあった英文は、自然だと思います。
今回の質問にあった日本文や、それに忠実に従った
つもりの私の元々の英訳とは、違っています。

487 :The OED Loves Me Not (ワッチョイ d689-JD/u):2023/10/24(火) 18:25:44.93 ID:7SF+s8VQ0.net
>>484
ごめんなさい。言い間違いました。予備校の本にある
日本文には英文は忠実には従わず、英語らしく
書き直してあり、英訳としては自然だと思います。

488 :The OED Loves Me Not (ワッチョイ d689-JD/u):2023/10/24(火) 18:33:23.06 ID:7SF+s8VQ0.net
>>485
おっと、>>484
as ... said it would
だけなら自然かもしれないと思いましたが、
as ... said it would rain
のように予備校の本に書いてあったなら、
ひょっとしたら不自然かもしれません。

よくわからなくなってきました。連投していながら、
「わからない」なんていってしまって、すみません。

489 :英語職人 :2023/10/24(火) 22:03:46.84 ID:ZPG9mzGN0.net
>>482
>結局は損をしてしまいました。

ワロタ 🤠

490 :英語職人 :2023/10/24(火) 22:33:57.70 ID:ZPG9mzGN0.net
>>484
その英語は、自然で、正しいよ。(rainはあってもなくても正しい) 覚える価値があります。

ただ、日本語訳がおかしい。「駅」ならmy stationとは言わないよ。駅の所有者じゃないよね?😃 どこの予備校だよ?w
正しい訳「天気予報が午後は雨になると言っていたとおり、自分の持ち場に戻るやいなや雨が降り始めた。」

asは「~とおり」「~ように」の意味を持つ様態のasだね。ここでの「station」は「持ち場」「部署」または「軍の駐屯地」の意味。

>>487 英訳として自然? 「駅」と訳してはダメなのです。🤠

491 :英語職人 :2023/10/24(火) 22:37:06.09 ID:ZPG9mzGN0.net
修正
>>484
その英語は、自然で、正しいよ。(wouldの後ろにrainを書いても省略してもどちらでもよい)

覚える価値がある英文です。

492 :英語職人 :2023/10/24(火) 22:39:17.04 ID:ZPG9mzGN0.net
>>484 「言っていたように」とか「駅」とか、訳がひどい。

493 :英語職人 ◆azN58fWNzw (ワッチョイ 8de0-xUKY):2023/10/24(火) 23:16:44.72 ID:ZPG9mzGN0.net
>>484

訳がひどいと書きましたが、その本の著者はたまたまこの文の訳だけを間違ったのでしょうから、大目に見てあげてくださいね。🤠

494 :外人@5chは終わっている ◆a0V.EBsnGU (ワッチョイ 659d-dytz):2023/10/25(水) 00:21:23.01 ID:MFoLAfO+0.net
>>484
間違っているとは言わないけど、このthanの部分がおかしな英語で気持ち悪い感じがします。現実世界よりもフィクションの方で使われるみたいな英語。
誰もそういう風に話しません。
もっと一般的な言い方にすると

"Just like the weather forecast said it would, it rained in the afternoon just as I got back to my post"
or
"Just as I got back to my post it started raining just like the weather forecast said it would in the afternoon."

原文の「station」は駅のことと間違えやすいのでpostにします。

ちなみにイギリスから書き込み不可になったよ。もう終わってるねこのサイト・・・

495 :外人@5chは終わっている ◆a0V.EBsnGU (ワッチョイ 659d-dytz):2023/10/25(水) 00:21:31.57 ID:MFoLAfO+0.net
>>484
間違っているとは言わないけど、このthanの部分がおかしな英語で気持ち悪い感じがします。現実世界よりもフィクションの方で使われるみたいな英語。
誰もそういう風に話しません。
もっと一般的な言い方にすると

"Just like the weather forecast said it would, it rained in the afternoon just as I got back to my post"
or
"Just as I got back to my post it started raining just like the weather forecast said it would in the afternoon."

原文の「station」は駅のことと間違えやすいのでpostにします。

ちなみにイギリスから書き込み不可になったよ。もう終わってるねこのサイト・・・

496 :外人@5chは終わっている ◆a0V.EBsnGU (ワッチョイ 659d-dytz):2023/10/25(水) 00:23:33.25 ID:MFoLAfO+0.net
お決まりの連投!
Never change, 5ch! ;)

497 :外人@5chは終わっている ◆a0V.EBsnGU (ワッチョイ 659d-dytz):2023/10/25(水) 00:26:15.49 ID:jJncme4T0.net
ちなみに前に話した固定回線vpnを使って投稿したけどどうやらこのスレにいる他の方も同じやつを使ってて、ワッチョイが同じになった!

498 :名無しさん@英語勉強中 (ワッチョイ 79a1-dytz):2023/10/25(水) 01:19:59.26 ID:MvJG041f0.net
my stationって普通に言うでしょ

・I missed my stop.(乗り過ごしてしまいました)
・I missed my station.(駅を乗り過ごしてしまいました)
https://hapaeikaiwa.com/blog/2013/07/17/%E3%80%901%E6%97%A51%E5%88%861%E3%83%95%E3%83%AC%E3%83%BC%E3%82%BA%EF%BC%81%E3%82%A2%E3%83%A1%E3%83%AA%E3%82%AB%E3%83%B3%E9%9B%91%E8%AB%87%E9%9B%86%E3%80%91%E3%80%8C%E4%B9%97%E3%82%8A%E9%81%8E/

499 :英語職人 ◆azN58fWNzw (ワッチョイ 8de0-xUKY):2023/10/25(水) 01:30:12.54 ID:2OCE1BzV0.net
>>494
日常会話ではたしかに、No sooner ---thanは、誰も使わないだろうね。
でも、日本の大学受験の英語の試験によく出るので、日本の受験生にとっては覚えなければならない重要な表現なのだ。日本の英語教育は、実用英語を飛び越して、レアな文語表現(文学的表現)を優先しているところがある。🤠

500 :英語職人 :2023/10/25(水) 01:41:51.52 ID:2OCE1BzV0.net
>>498
missedなら駅でいいよ。 しかし、get back to my stationは、「持ち場」「部署」で解釈するのがふつう。
そもそも、missed my stationのstationは「持ち場」で訳せない。

日本語でも、「私が降りるべき駅はここです」を「私の駅はここです」と言うでしょう。しかし、「私は私の駅に戻った」とは言わない。
日本語でも動詞次第で「私の駅」と言えたり言えなかったりする。英語も同様。

501 :英語職人 :2023/10/25(水) 02:04:43.76 ID:2OCE1BzV0.net
いま、ざっとみたけど、got back to my stationを「駅」の意味で使ってるネイティブの文章は見つからなかった。全部「持ち場」「部署」「支局」という意味だった。get back to my stationは、ふつうは、「自分の持ち場」「自分の所属する部署」とかいう意味なのである。


https://www.letscookthatbook.com.au/books/webers-greatest-hits
One night we were very busy and I had to leave my station to get more ingredients from the walk-in refrigerator. For some reason, it was locked. By the time I found the key and &#9733;got back to my station&#9733;, the fire had completely burnt ten whole chickens and four whole ducks.
。ある夜、我々はとても忙しかったので、私は大型冷蔵庫から食材を取り出すために自分の持ち場を離れなければならなかった。その冷蔵庫にはなぜか鍵がかかっていた。鍵を見つけて自分の持ち場に戻った時には、火は鶏10羽とアヒル4羽を完全に焼き尽くしていた。

502 :名無しさん@英語勉強中 :2023/10/25(水) 02:53:09.88 ID:MvJG041f0.net
↓この人はアメリカ人
— The two guys helped me get back to my station and get on the correct train too.

https://www.reddit.com/r/JapanTravel/comments/8n9d3v/long_trip_report_8_days_in_tokyo_osaka_kyoto/
https://www.reddit.com/r/AskAnAmerican/comments/8qdbjm/those_of_you_who_traveled_to_japan_how_were_your/?ref=readnext

"How am I going to get back to my station?" "The damn train didn't make my stop, I'm not going to pay another fare to get back."

https://books.google.co.jp/books?id=rcNHDwAAQBAJ&pg=PA147&lpg=PA147&dq=%22back+to+my+station%22%22train%22&source=bl&ots=aGkDWRxuxi&sig=ACfU3U3ADeQGISKLBJocKrHJYO7n4hAL6g&hl=en&sa=X&ved=2ahUKEwiLs9Hmmo-CAxVKxGEKHZSqAMY4FBDoAXoECAIQAw#v=onepage&q=%22back%20to%20my%20station%22%22train%22&f=false

503 :名無しさん@英語勉強中 (ワッチョイW 659d-cqAW):2023/10/25(水) 05:24:45.03 ID:sTzMS/dj0.net
最近よく
wait to you
というのを聞くんだけど、よく使うのか

504 :名無しさん@英語勉強中 (ワッチョイW 5d9d-0Us4):2023/10/25(水) 05:37:42.36 ID:6WjjTePp0.net
あなたに仕えます

505 :名無しさん@英語勉強中 :2023/10/25(水) 06:14:51.81 ID:y/Eq9EM60.net
より年を取って見えるようにデジタル的に変えた自分自身の画像を大学生に見せた結果、
大学生達は勉学に励んだり試験勉強を先延ばしにしたりすることが少なくなった、という流れで

Making our future selves feel less like strangers can actually help us accomplish things in present.

という文章の日本語訳が

未来の自分自身を他人のように感じることが少なくなれば、私たちが現在の物事を成し遂げるのに実際役立つかもしれない。

でした

makeは使役動詞と捉えるなら「未来の自分自身に他人のように感じさせる」となるはずなので、
ここのmakeは使役動詞ではないと思うのですが、そうだとすれば文法的にはどうしてこのような訳になっているのでしょうか?

506 :名無しさん@英語勉強中 :2023/10/25(水) 07:06:26.15 ID:CuKy8qSk0.net
>>510
コミュニカティブな教育を受けてたのは
何歳くらいまでだろう?25くらい?
昔はそうじゃなかった。
今の方が英語が得意な人が多い気がする。
大学進学率も低かったから昔の教育に
戻れば英語が得意な人が増えるというのは
違うだろうな

507 :The OED Loves Me Not (ワッチョイ d689-JD/u):2023/10/25(水) 08:02:08.33 ID:9jthM56i0.net
>>498
my station は、外人さんの言うように「私の(職場などでの)持ち場」などの
意味合いと、「私が電車から降りないといけない駅」という意味との
二重性があるんでしょうね。特にイギリス人はそう思うのでしょう。

しかしアメリカ人はこういうときに my station をごく普通に使うのかもしれないと
私はチラッと思ったりしています。

https://books.google.co.jp/books?id=pvAhCwAAQBAJ&pg=PT12&lpg=PT12&dq=%22that%27s+my+station%22&source=bl&ots=jCXRtycLNe&sig=ACfU3U1PPaNtXT0rWvDvOioj9xTQjSWqgw&hl=en&sa=X&ved=2ahUKEwjlyduG4Y-CAxVrzTQHHWb3BF8Q6AF6BAgREAM#v=onepage&q=%22that's%20my%20station%22&f=false

この上のリンク先には、小説の一節が出てきますが、リンク先に表示されるページの
上から 15 行目に、Dam, that's my station. とあります。metro に乗っていて、
「いけね。俺は(本来なら)ここで降りないといけないんだが」とかいうような意味だろうと思います。

さらに、Party of Five というアメリカの昔の青春テレビドラマの中で、登場人物の女の人が
自分の働いているレストランにて、
That's my station.
と言っているのを聞いたことがありますが、これは「(客席内の)そこのテーブルは私の持ち場だよ」
というような意味らしいと感じました。別の店員に対して、「私の持ち場だから、余計なことするな」
というような文脈でそう言っていました。

508 :The OED Loves Me Not (ワッチョイ d689-JD/u):2023/10/25(水) 08:17:13.61 ID:9jthM56i0.net
>>505
>>Making our future selves ★feel★ less like strangers can actually help us accomplish things in present.
>>未来の自分自身を他人のように ●感じる● ことが少なくなれば、私たちが現在の物事を成し遂げるのに実際役立つかもしれない。
>>「未来の自分自身に他人のように感じさせる」となるはずなので、

ここでの feel は「感じる」ではなくて「感じられる」という意味です。だから
making our future selves feel less like strangers は
making our future selves feel (to us) less like strangers
つまり「現在の私たち(今の自分たち)に、未来の自分が他人のように感じられるように仕向けること」
というわけで、
「将来の自分が、現実にそうなりそうに思えてくるように仕向けること」
みたいな意味合いです。

This towel feels soft. (このタオルは、手触りが柔らかい)
これは
This towel feels soft (to you).
つまり(厳密に言えば)「(人にとって、人から見て)柔らかく感じられる」というわけであって、
タオルがまるで人間みたいに soft だと感じるわけではなく、タオルという存在の
手触りが、人間にとって柔らかく感じられるのです。

making this towel feel soft
と言えば、
「このタオルの手触りを柔らかくなるように仕向けること」
みたいな意味合いになります。

こういうときの feel の使い方を理解していない日本人がたくさんいるようで、ネット上で
いくら辞書の記述をきちんと引用しながらくどくどと説明してあげても、頑固に
「いや、feel はあくまで "感じる" という意味なんだから
This towel feels soft. なんていう英語は変だ、と言い続けたりする人もいます。

509 :The OED Loves Me Not (ワッチョイ d689-JD/u):2023/10/25(水) 08:19:07.98 ID:9jthM56i0.net
>>505
こういうときの feel は、seem みたいなものだと
思いながら読むと、わかりやすく感じられるのではないかと
思います。

510 :The OED Loves Me Not (ワッチョイ d689-JD/u):2023/10/25(水) 08:31:47.59 ID:9jthM56i0.net
>>505
... ★You feel soft to me★ . " Nestling her body against his , he bent his head and murmured against her ear . " Am I ' your man ' ? ” Lucia leaned back in his arms to look at him . She only had a moment to admire his face , though ; then he ...
https://www.google.co.jp/books/edition/The_Veiled_Web/9liIEAAAQBAJ?hl=en&gbpv=1&dq=%22you+feel+soft+to+me%22&pg=PT156&printsec=frontcover

上のリンク先の 5 行目あたりに、上のような一節が見られます。ここでの
You feel soft to me.
は、決して「君は俺に対して soft だと感じている」とか何とかいう意味ではなく、
「君の肌は柔らかいね」
とかいう意味のはずです。恋人同士なので、男が女の首筋を指でそうっと撫でながら
言っている台詞です。

feel の主語が無生物なら、「〜が〜のように感じられる」という意味であることは明らかですが、
この上の一節のように、feel の主語が人間である場合は、「感じる」という意味か
「(手触りが)〜のように感じられる」という意味なのかが(慣れていない日本人にとっては)
すぐにはわからないかもしれませんね。

511 :英語職人 ◆azN58fWNzw (ワッチョイ 8d6e-xUKY):2023/10/25(水) 09:22:42.78 ID:2OCE1BzV0.net
got back to my stationを「駅」という意味で使わないとまでは言っていないよ。「持ち場」と解釈するほうが一般的だと言っている。
特に教材では、一般的な意味のほうで訳さないとダメなので、あそこは「駅」と訳さないほうがいいんだよ。

どうしても駅にしたいのなら「got back to the station」に直すべき。
>>484の訳は間違いとは言えないよ。でも、教材としてはどうなの?って話ね。

512 :英語職人 ◆azN58fWNzw (ワッチョイ 8d6e-xUKY):2023/10/25(水) 09:29:34.11 ID:2OCE1BzV0.net
>>507
その文脈なら使うでしょう。
今は、「got back to my station」の話をしているのです。この文脈では、「駅」よりも「持ち場」で訳すほうが多いと私は言っているのです。(駅と訳せないとは言っていない) 特に、信頼できる記事では、「持ち場」「駐屯地」「部署」「~局」などと訳すが普通です。

513 :英語職人 ◆azN58fWNzw (ワッチョイ 8d6e-xUKY):2023/10/25(水) 09:46:41.02 ID:2OCE1BzV0.net
https://www.google.com/search?q=%22I+got+back+to+my+station%22&sca_esv=576306854&tbm=bks&sxsrf=AM9HkKmEVvJzE2rt4-AHAivYZFco3_1xnA%3A1698194039395&ei=d2I4Zf_pF5T9hwPamYfAAQ&ved=0ahUKEwi_genj-Y-CAxWU_mEKHdrMARgQ4dUDCAk&uact=5&oq=%22I+got+back+to+my+station%22&gs_lp=Eg1nd3Mtd2l6LWJvb2tzIhoiSSBnb3QgYmFjayB0byBteSBzdGF0aW9uIkjRE1C8DliVEXAAeACQAQCYAVygAYwCqgEBM7gBA8gBAPgBAYgGAQ&sclient=gws-wiz-books

このリンク先に「I got back to my station」が出てくる本が10冊表示されます。その中で「駅」という意味で使われているのは、たった1冊しかありません。
よって、教材に使う訳としては、「駅」は(間違いだとは言わないまでも)望ましくないのです。

514 :The OED Loves Me Not (ワッチョイ d689-JD/u):2023/10/25(水) 09:50:19.05 ID:9jthM56i0.net
>>512
>>507 のわたしの書き込みは、決して職人さんに反駁するために
書いたのではないのです。あくまで、関連する話題として書いただけですよ。

515 :英語職人 ◆azN58fWNzw (ワッチョイ 8d6e-xUKY):2023/10/25(水) 09:59:24.89 ID:2OCE1BzV0.net
>>514
了解しました 🤠

516 :The OED Loves Me Not (ワッチョイ d689-JD/u):2023/10/25(水) 10:03:22.08 ID:9jthM56i0.net
"no sooner had I"
という phrase を検索すれば、そのあとにはたぶん "... than S V"
が来て、例の有名な国文が出てくるんだろうなと思いますが、

https://books.google.com/ngrams/graph?content=no+sooner+had+i%2Cno+sooner+had+he%2Cno+sooner+had+she%2Cno+sooner+had+we&year_start=1800&year_end=2019&corpus=en-2019&smoothing=3

を見ると、私がふだん読書していて感じる通りのことが確認できます。
つまり、現代ではかなりこれは使われなくなってはいますが、19世紀くらいまでは
大いに使われていたのですね。実際、現代の小説を読んでいるとこういうのは
私自身はまったく見たことがありませんでしたが、100年以上前の小説を
読んでいると、ちゃんと使われているので、少しびっくりしました。
受験英語なんて役に立たないと非難されることが多いけど、少し古い文献を
読めば、受験英語だらけなのです。"lest ... (should)" も、19世紀までは
大いに使われていたらしいということを、私自身も古い英文を読んでいて感じます。

なお、上に挙げた Ngram Viewer のグラフを、1600年代から現代までに変えると

https://books.google.com/ngrams/graph?content=no+sooner+had+i%2Cno+sooner+had+he%2Cno+sooner+had+she%2Cno+sooner+had+we&year_start=1600&year_end=2019&corpus=en-2019&smoothing=3

このように、1650 年に、どういうわけか爆発的に頻繁に使われていたのです。
なぜか知りませんが。

まあともかく、1600年代から 1900 年くらいまでの300年間ほどは、
この構文は大いに使われていたらしいことがこのグラフでもわかります。

517 :名無しさん@英語勉強中 (ワッチョイ 79a1-dytz):2023/10/25(水) 10:16:57.58 ID:MvJG041f0.net
>「駅」ならmy stationとは言わないよ。駅の所有者じゃないよね?😃 どこの予備校だよ?w

なんかここからずいぶん後退したように見えるけど、まあいいや

518 :ブタ耳 (ワッチョイW c1ae-flU9):2023/10/25(水) 10:17:01.48 ID:RX08HZzV0.net
こういうプロンプト考案した。
You have found a fragment of text that reads:
"No sooner had I got back to my station than it started to rain,as the weather forecast had said it would in the afternoon."

Add your own text before and after this fragment to create a natural story .

試行結果:
ChatGPT(GPT-3.5):
https://chat.openai.com/share/2a84d429-9f2c-443e-9190-4f33b4bcc0be

bing (GPT-4,創造的):
https://sl.bing.net/hVzvt8kcODk

phind (GPT-4):
https://www.phind.com/search?cache=v6v775a01k7xgwr1gy97d6p1

519 :ブタ耳 (ワッチョイW c1ae-flU9):2023/10/25(水) 10:19:08.68 ID:RX08HZzV0.net
あ、1行抜けてた。
プロンプト冒頭に、
You are a professional nonfiction writer.
追加。

520 :名無しさん@英語勉強中 (ワッチョイ 1a44-f0DH):2023/10/25(水) 11:09:14.63 ID:RG+LaFCY0.net
すみません、前後の文脈を書き忘れていました。
私は個人経営のガソリンスタンド (gas station) を持っていて、
アルバイト1人を雇ってなんとか2人で運営しています。

ある日、ちょっと用事ができたので、アルバイト1人を残して
1キロ先の郵便局まで、傘を持たずに出かけました。
天気予報で雨が降ると言っていたのは知っていました。

As soon as I got back to my station, it began to rain.

ぎりぎりで雨に降られずに済みました。しかし、
ガソリンスタンドに戻ってみると、アルバイトの男がいません。
そして店の売上金もすべてなくなっていました。

521 :ブタ耳 :2023/10/25(水) 14:58:34.12 ID:RX08HZzV0.net
"a natural story”の部分は形容詞変えた方がいいな。
realistic とか?

522 :ブタ耳 :2023/10/25(水) 15:04:14.71 ID:RX08HZzV0.net
大して変わらない。
https://chat.openai.com/share/b43e913e-f04b-46e5-b1bd-a9b77185298e

523 :名無しさん@英語勉強中 :2023/10/25(水) 16:34:21.08 ID:dB+pfmV70.net
「雨の匂い」のような視覚とそれ以外みたいな複数の感覚を跨いだ表現を表す修辞の呼び名があったと思うのですが
思い出せません
英語でなんって言いますか?

524 :名無しさん@英語勉強中 (ワッチョイ 8d49-XYY0):2023/10/25(水) 16:44:53.11 ID:dB+pfmV70.net
あw 思い出しましたw

525 :名無しさん@英語勉強中 (ワッチョイW c182-CNxV):2023/10/25(水) 16:50:43.06 ID:fk9zzqmZ0.net
>>524
何だよそれ気になる!!!

526 :The OED Loves Me Not (ワッチョイ d689-JD/u):2023/10/25(水) 17:13:52.58 ID:9jthM56i0.net
>>503
wait to you なんて見聞きしたことがないと思ったけど、かなり使われているみたいですね。

(1) I'm gonna wait to you.
という言い回しなど、"wait for someone" という形が広く使われているようです。
これはおそらく、wait for someone という意味だろうと思います。

(2) Wait to you see what we'll do next.
みたいな言い回しも見つかりますが、この場合には
Wait until you see what we'll do next.
という意味のような気がしています。

上のような二通りの意味合いで "wait to someone" という phrase が
使われているように見えます。

Don't wait to someone to make you happy, drink with me.
(Tik Tok 上の書き込み。この wait to は wait for という意味のようです。)

hi ! while that i wait to someone to comment my 14 day challenge drawing i made this in 1 hours , hope you'll find what's wrong with it
https://www.reddit.com/r/istebrak/comments/v6h2u6/hi_while_that_i_wait_to_someone_to_comment_my_14/

Reddit にて、上のような書き込みも見つけました。While S V の代わりに、While that S V と書いています。
さらには、I wait to someone to comment... と書いていますが、これは I wait for someone to comment... という意味のようです。

527 :The OED Loves Me Not (ワッチョイ d689-JD/u):2023/10/25(水) 17:22:52.52 ID:9jthM56i0.net
>>526
このような

(1) I wait to someone to do something. (wait for someone to do something という意味か?)
(2) I wait to someone does something. (wait until someone does something という意味か?)

のような構文について誰か解説していないかどうか調べてみましたが、私には見つけ出せませんでした。
OED Online によると、
wait to someone or something
という言い回しは 15 世紀から 16 世紀にのみ使われ、今では廃用となったそうです。しかも、
「(誰それに対して)奉仕する、念入りに世話する」みたいな意味だったようです。

to wait to ——
c1440–1508
† To keep watch upon, attend carefully to. Obsolete.
c1440
Sirs, waites to þer wightis þat no wiles be wrought.
York Mysteries xxxiii. 253Citation details for York Mysteries
c1449
And skile whi al this is trewe may be perceyued weel ynowȝ of a diligent considerer, waiting weel to ech word which is now bifore here seid.
R. Pecock, Repressor (1860) 405Citation details for R. Pecock, Repressor
1508
And wayte well to the sewer how many dysshes be couered.
Boke of Keruynge (de Worde) sig. A.ivCitation details for Boke of Keruynge

もしかしてこのような言い回しが、現代でもどこかの地域で方言として残っていて、
その wait to someone という言い回しが Tik Tok や Instagram などの social media において
流行っているのでしょうかね。

528 :名無しさん@英語勉強中 :2023/10/25(水) 18:43:32.68 ID:RG+LaFCY0.net
>>525

The Smell of Rain Is More Complex Than Previously Thought
https://www.pinterest.jp/pin/794252084272487808/
Did you ever wonder what causes that earthy smell that rises after a light summer rain? That mysterious scent has been called "petrichor".

petrichor (noun)
https://www.merriam-webster.com/dictionary/petrichor
a distinctive, earthy, usually pleasant odor that is associated with rainfall especially when following a warm, dry period and that arises from a combination of volatile plant oils and geosmin released from the soil into the air and by ozone carried by downdrafts.

529 :名無しさん@英語勉強中 :2023/10/25(水) 19:11:26.57 ID:CuKy8qSk0.net
>>528
東京だととりわけ雨上がりは気持ちいいよね

530 :ブタ耳 :2023/10/25(水) 19:54:10.02 ID:RX08HZzV0.net
>>523-524
synesthetic?

531 :名無しさん@英語勉強中 (ワッチョイ b148-HMmB):2023/10/25(水) 21:27:05.74 ID:tw9dgOjE0.net
>>321
>>331
the fairの解釈について最初に質問致しました者です。
当時の英語やそれだけでなくてフランス語との関係までもを意識して理解に務めなくてはならなかったのですね。
大変参考になりましたしおかげさまでとても理解が深まりました。
お2人方ともわざわざ実例まで添えてご親切に回答くださって本当にありがとうございました。
またそこまでして頂いたのにも関わらずに返信が遅れてしまいまして申し訳ありませんでした。

532 :名無しさん@英語勉強中 (ワッチョイ b148-HMmB):2023/10/25(水) 21:29:05.92 ID:tw9dgOjE0.net
>>531
ああ失礼致しました。
>>323
>>331
でした。

533 :名無しさん@英語勉強中 (ワッチョイ f196-iKuA):2023/10/26(木) 01:37:37.71 ID:CJtnnmtV0.net
お願いします。
He was picking cherries in a cherry-tree, when a branch broke, and he fell head over heels to the ground.

この英文の訳は「彼がサクランボを摘んでいたとき枝が折れ、まっさかさまに地面に落ちた」となるそうなんですが、
「枝が折れたとき、彼はサクランボを摘んでおり、……」ではないのが不思議です。

when a branch brokeとなっているのに、なぜ「枝が折れたとき」という解釈にはならないのでしょうか?

534 :名無しさん@英語勉強中 (ワッチョイW 5d9d-0Us4):2023/10/26(木) 01:53:32.60 ID:tmvWsPlP0.net
継続用法のwhenだから前の文節のときってなる

535 :The OED Loves Me Not :2023/10/26(木) 06:34:32.21 ID:ygd4ieXv0.net
>>533
後ろから訳して前へと訳していくという「約仕上げ」ではなくて、
前から順番に訳していく「訳し下げ」(訳しおろし)という手法もあるのです。
https://www.makocho0828.net/entry/setuzokusi-yakusiorosi-0422

気の利いた翻訳家(翻訳者)や通訳は、なるべく訳し下ろし(前から訳していく方法)を
使うのです。

536 :The OED Loves Me Not :2023/10/26(木) 06:36:21.60 ID:ygd4ieXv0.net
>>533
漢字変換を間違えました。

後ろから訳して前へと訳していくという「★訳し上げ★」ではなくて、

537 :The OED Loves Me Not :2023/10/26(木) 10:09:49.17 ID:ygd4ieXv0.net
5ちゃんねる英語板にて、「私が今これを書いている時点では」という意味の
"as I write" という表現について質問があったが、そういう意味での
phrase は決して1種類ではなく、何通りもあって、著者によっていろいろ
好みがあるように見える。すでに英語板にて私が紹介したものでも、次のようなものがある。

(1) ★As this book was going to press★, in June 2019, President Trump,
frustrated by China's endless stalling tactics, announced trade tariffs on China.
(Robert Spalding, Stealth War, 2019)

(2) It is full of its golden flowers this July, ★as I write (1981)★.

出典:
Franklyn Grace Lyo, ?M.A. Lyons ・ 2008 ・ ?Biography & Autobiography
https://books.google.co.jp/books?id=4T6LAAAAQBAJ&pg=PA491&lpg=PA491&dq=%22as+i+write++1981%22&source=bl&ots=1dS_Dhm_mp&sig=ACfU3U30J4F4uvVz3t65IqKY_BASLx8WTQ&hl=en&sa=X&ved=2ahUKEwjirMvZmKH8AhVbpVYBHRY_BTgQ6AF6BAgJEAM#v=onepage&q=%22as%20i%20write%20%201981%22&f=false

(3) John Denvir ・ 2019 ・ ?History
I see by the Catholic Directory there are ★at the time I now write★ thirty-nine churches and chapels in Liverpool. The schools have increased in a like ...

******************

さて、これに似た意味合いの phrase を、再び今さっき読みかけていた本の中で見つけたので、紹介する。

●As of this writing,● a host of regulatory reform pro-posals are making the rounds, all of them hotly debated and all ofthem focusing squarely on the structure of Wall Street.
https://www.scribd.com/document/33498558/Chasing-Goldman-Sachs-by-Suzanne-McGee-Excerpt

この本は 2010 年に出たのだが、日本語訳はつい最近になって(2023年10月になって)、やっと出たみたいだ。
https://www.trannet.co.jp/works/view/13997

Wall Street Journal などいろんなところで絶賛されているそうだ。

538 :名無しさん@英語勉強中 :2023/10/26(木) 13:26:09.38 ID:KnUjHutF0.net
>>533
昔似たような疑問を聴いたことがあります。
その時どなたかが下の『ニック式英会話』という
you tubeを紹介していました。ご参考にいかが?

https://www.youtube.com/watch?v=tR-d3kD9ruA

539 :名無しさん@英語勉強中 (ワッチョイ f196-iKuA):2023/10/26(木) 18:24:31.40 ID:CJtnnmtV0.net
>>535-536>>538
ありがとうございました。難しいですね。

540 :名無しさん@英語勉強中 (ワッチョイ f196-iKuA):2023/10/26(木) 18:28:35.82 ID:CJtnnmtV0.net
似たような「英語の感覚がよくわからん」という質問ですが、よろしくお願いします。

It is not surprising that a man of such persistent and untiring energy should have exercised so great an
inflluence over the thought of mankind for many hundred of years.

この文なんですが、so great an influenceという部分がしっくりきません。anは必要なんでしょうか?
great influence なら形容詞+名詞として違和感がないのですが、great an influenceというのはへんな気がします。

great influence
great an influence この両者にはどんな違いがあるんでしょうか?

541 :🤭英ちゃん🫢 (ワッチョイW 7a86-gqR+):2023/10/26(木) 18:49:26.34 ID:n3C/qYpv0.net
ニックが言ってるのは、日本語直訳で中学生でとも腑に落ちるから違わない??🤔

542 :The OED Loves Me Not (ワッチョイ d689-JD/u):2023/10/26(木) 18:57:31.12 ID:ygd4ieXv0.net
>>540
辞書で so の項目にある、so の使い方の解説を読んでください。
必ず詳しく書いてあるはずです。

such a great influence
= so great an influence

ということです。このことは、英語を読み書きする上において
ぜひとも知っておかないといけない基本的なことです。
どこかで質問者さんも勉強したはずだけれども、忘れてしまっただけだと思います。

543 :英語職人 :2023/10/26(木) 19:49:07.77 ID:Sc9Kspwd0.net
>>533
whenのまえにコンマがあって、なおかつ、接続詞のwhenの意味で解釈すると矛盾がでるときは、このwhenは「関係副詞whenの非制限用法」です。
関係副詞のwhenの非制限用法は、「, and then (そしてその時)」と交換ができます。

He was picking cherries in a cherry-tree, when a branch broke, and he fell head over heels to the ground.
=He was picking cherries in a cherry-tree, and then a branch broke, and he fell head over heels to the ground.

ですから、このwhenの文は直訳では、「~を摘んでいました。そしてその時、枝が折れ~地面に落ちた」と訳すべきものです。
これを意訳したものが、「摘んでいたときに、枝が折れ~地面に落ちた」なのです。
このwhenは、接続詞「~したとき」ではなく、関係副詞「そしてその時」で解釈すべきものなのです。
ってか、質問者は、もういないか 🤠

544 :英語職人 :2023/10/26(木) 19:56:55.43 ID:Sc9Kspwd0.net
>>540
「これは、大きすぎるボールだ」を英作してみましょう。

どれが正解ですか?
① This is a too big ball.
② This is too a big ball.
③ This is too big a ball.

①番にしたいところですが、英語では③を使うのです。
so(とても)の場合も同様です。influenceはgreatなどの形容詞がつくと可算名詞になりますから、aを消すわけにはいきません。

545 :🤭英ちゃん🫢 :2023/10/26(木) 20:29:46.49 ID:n3C/qYpv0.net
①もいいんじゃない

546 :名無しさん@英語勉強中 :2023/10/26(木) 20:54:26.14 ID:qfVxfWcM0.net
そもそも不可算名詞や複数形には使はないのでは

547 :名無しさん@英語勉強中 (ワッチョイ 1a16-ptOG):2023/10/26(木) 21:04:03.46 ID:XlUisYFA0.net
>>545-546 お二人とも正しいです
ジーニアス英和辞典によると

(o) too long a story
(o) a too long story
(x) too hot days
(x) too bad meat

548 :英語職人 ◆azN58fWNzw (ワッチョイ 8d1a-xUKY):2023/10/26(木) 23:39:00.69 ID:Sc9Kspwd0.net
>>547
これだから、素人さんは・・・😔  

深い英語の知識があれば、>>544の③のみが堂々と自然だ言える選択肢だとわかるんだけどね。どのネイティブも 「a too big ballも自然な言い方だ」なんて言ってないぞ。証拠↓
https://english.stackexchange.com/questions/362412/too-big-a-vs-a-too-big
https://forum.wordreference.com/threads/too-old-a-man.909303/
https://books.google.com/ngrams/graph?content=a+too+big%2Ctoo+big+a&year_start=1800&year_end=2019&corpus=en-2019&smoothing=3

「a too big ballも、too big a ballと同じくらい正しいよ」と言っているネイティブがいるなら、連れてきなさい 😄

ジーニアスは、「時に a too long storyも」とか言ってるけど、ネイティブのほぼ全員が不自然だと思うものをあたかも正しいかのように言うのはよくないな。次の改定で修正しましょう🤠  

549 :英語職人 ◆azN58fWNzw (ワッチョイ 8d1a-xUKY):2023/10/26(木) 23:47:48.30 ID:Sc9Kspwd0.net
ウィズダム辞書は、ジーニアスよりかなり正しく、「まれに、a too 形容詞 名詞になることもある」と書いてるね。
「まれ」なら、まあ許してしんぜよう 🤠 

でも
https://books.google.com/ngrams/graph?content=a+too+big,too+big+a&year_start=1800&year_end=2019&corpus=en-2019&smoothing=3

これをみればわかるように、a too big ballの語順は「ほぼ誰も使ってないだろ」ってレベルなのである。

>>547はデマを書いてはいけませんよ 🤠

550 :英語職人 ◆azN58fWNzw (ワッチョイ 8d1a-xUKY):2023/10/26(木) 23:54:50.23 ID:Sc9Kspwd0.net
>>546
influenceはgreatなどの形容詞がつくと可算名詞になるのだ。

反論する人は、ちゃんと調べてから反論しましょう。>>545もね 🤠

551 :英語職人 ◆azN58fWNzw (ワッチョイ 8d1a-xUKY):2023/10/27(金) 00:08:32.20 ID:WQJ7WW2P0.net
英英辞書でtooを引くと、絶対に、「a too big ball」の語順は載ってない。


https://www.merriam-webster.com/dictionary/too
======================
too
2 a : to an excessive degree : excessively
too large a house for us
======================

英英辞典で、a too big ballの語順を許容しているものがあれば、出してみてよ。そんなのないから。

552 :名無しさん@英語勉強中 (ワッチョイ 8e02-dytz):2023/10/27(金) 00:11:18.76 ID:2O3g8dLE0.net
This is something only possible where every employee benefits directly from the compny's profitability.

このwhere が if みたいな使われ方をしています。
文法的にどのように解釈をしたらいいでしょうか?

例文や説明サイトなどを交えてわかりやすく教えていただければ幸いです

553 :名無しさん@英語勉強中 (ワッチョイW 7ab0-xqvO):2023/10/27(金) 00:40:30.89 ID:x3/HHQD70.net
>>505です

>>508-510
makeの方の問題ではなくてfeelの方の問題だったのですね
確かにseemと同じ様に考えると分かりやすいです

とても分かりやすく説明してくださってありがとうございました

554 :540 (ワッチョイ f196-iKuA):2023/10/27(金) 01:37:28.35 ID:LOLDT2450.net
>>542
>>544-550
ありがとうございました。
一生物の知識になるかと思います。

555 :The OED Loves Me Not (ワッチョイ d689-JD/u):2023/10/27(金) 02:44:14.44 ID:h9ZID0jq0.net
>>552
その場合の where は in cases where みたいなもんだと思えばいいと思います。
元々は where は in places where という意味で使うもんだったんでしょうけど、
だんだんと in cases where とか under circumstances where みたいな
意味合いでも広く使われるようになったんではないかと勝手に私は推測しています。

まあともかく、今回のような in cases where という意味での where は、
英語ネイティブが書いたりしゃべったりする英語の中でよく出てきますね。

in cases in which なんていうのは堅いから、in cases where になり、
そのあとに in cases が省略されていったのかもしれません。

556 :名無しさん@英語勉強中 :2023/10/27(金) 08:09:48.98 ID:yHefhT9c0.net
>>552
the place whereでthe placeが省略されている

557 :名無しさん@英語勉強中 (ワッチョイW c1c8-CNxV):2023/10/27(金) 11:41:32.01 ID:9/+M8ORN0.net
>>552
というか明らかにifではないやろw

558 :名無しさん@英語勉強中 (ワッチョイ 659d-dytz):2023/10/27(金) 12:23:23.43 ID:gjfZhTgN0.net
>>544
Too big.
This is too big.
This ball is too big.
This is too big a ball.
This is a ball too big.

559 :名無しさん@英語勉強中 (ワッチョイ 1a02-ptOG):2023/10/27(金) 12:29:49.60 ID:RIRH1uza0.net
This is something only possible where every employee benefits directly from the company's profitability.
「これは、従業員一人ひとりが会社の収益性から直接利益を得ている<場合に>のみ可能なことである。」

where [conjunction] in a case, or situation

It was inconceivable for Rousseau to imagine a democracy in a large state. He would have said it could not exist because popular free government relies on people sharing a culture and being willing to sacrifice themselves for one another, something only possible where states are small enough for citizens to know and appreciate one another.

Richard Whatmore "The History of Political Thought: A Very Short Introduction"

560 :名無しさん@英語勉強中 :2023/10/27(金) 18:42:46.66 ID:RIRH1uza0.net
A democrasy is something only possible where states are small enough for citizens to know and appreciate one another.
「民主社会とは、国民が互いを知り、理解し合えるほど国家が小さい<場合に>のみ可能なものだ。」

561 :名無しさん@英語勉強中 :2023/10/27(金) 18:45:10.07 ID:RIRH1uza0.net
democrasy → democracy

562 :名無しさん@英語勉強中 :2023/10/27(金) 19:05:41.51 ID:EJKzwtYx0.net
>>559
onlyはpossibleにかかってるから、
ーーの場合に可能であるに過ぎないものだ
と訳した方が良くありませんか?

563 :名無しさん@英語勉強中 :2023/10/27(金) 19:06:44.83 ID:EJKzwtYx0.net
possible only whereと>>559さんの日本語
だとなりそうですが

564 :名無しさん@英語勉強中 (ワッチョイ 5521-xAmZ):2023/10/27(金) 19:37:31.82 ID:PiKJqupm0.net
with a too large pasty face (Graham Greene)
like a boy on a too large horse (Margaret Mitchell)
like a horse preparing to haul a too heavy load (Louise Erdrich)
indeed a too hard-working ally (Yasunari Kawabata / Edward Seidensticker)
any of the cumbersome luggage of a too formal education (J. D. Salinger)
through a too literal interpretation of the text (Bertrand Russell)
after a too long waiting (Albert Camus / Stuart Gilbert)
after a too hearty meal (Thomas Mann / H. T. Lowe-Porter)
in a too long overcoat (Isaac Bashevis Singer / A. H. Gross)

565 :名無しさん@英語勉強中 (ワッチョイ 1a89-ptOG):2023/10/27(金) 20:18:25.67 ID:RIRH1uza0.net
>>562-563

確かに only は possible の前にありますが、

This is something only possible where A is B.
「これはAがBである場合にのみ可能なことだ。」

... something which is only possible where ...
「〜の場合のみ可能なこと」の意味です。

566 :The OED Loves Me Not (ワッチョイ d689-JD/u):2023/10/27(金) 20:37:39.18 ID:h9ZID0jq0.net
>>565
その通りですよね。「ジーニアス英和、第6版」にも、書き言葉や
正式な英語では only を、それが修飾する単語の直前に置くけれども、
そうでないときもよくあって、特に口語ではずいぶん離れたところに
only をつけることがよくあるというようなことを書いています。

I only have eyes for you.
という決まり文句がありますが、have の前に only があるからと言って、
「持っているに過ぎない」という意味ではなくて、for you あるいは you に
only がかかっていて、

「君にしか俺は興味がない。君だけに夢中だ。俺の目は、君だけを見てるんだ」

みたいな意味合いですよね。

567 :名無しさん@英語勉強中 (アウアウウーT Sa09-ArJN):2023/10/27(金) 20:46:49.06 ID:/FF2Hwfca.net
>>564
錚々たるメンツだね(笑)。

>>544
>influenceはgreatなどの形容詞がつくと可算名詞になりますから、aを消すわけにはいきません。

「so 形容詞 a/an 名詞」の形はそもそも冠詞を挟むことが必須であり、だから不可算名詞や複数形と共には用いられないという話。
「influenceに形容詞がつくと可算名詞になります『から』aを消すわけにはいきません」とは関係ない。

568 :名無しさん@英語勉強中 (アウアウウーT Sa09-ArJN):2023/10/27(金) 20:46:54.82 ID:/FF2Hwfca.net
>>564
錚々たるメンツだね(笑)。

>>544
>influenceはgreatなどの形容詞がつくと可算名詞になりますから、aを消すわけにはいきません。

「so 形容詞 a/an 名詞」の形はそもそも冠詞を挟むことが必須であり、だから不可算名詞や複数形と共には用いられないという話。
「influenceに形容詞がつくと可算名詞になります『から』aを消すわけにはいきません」とは関係ない。

569 :🤭英ちゃん🫢 (オッペケ Sr05-gqR+):2023/10/27(金) 20:49:24.05 ID:17kcDGjUr.net
5chが1日の上限レス数作るそうだから、ソフィ先生の反撃連投も見納めかも

570 :名無しさん@英語勉強中 :2023/10/27(金) 21:09:25.83 ID:RIRH1uza0.net
>>565

We can only fully understand a culture when we compare it with other cultures.
[only は when 節を修飾]

I'll only dance if you promise not to laugh at me.
[only は if 節を修飾]

This is something only possible where A is B.
[only は where 節を修飾]

571 :英語職人 :2023/10/27(金) 22:25:54.51 ID:WQJ7WW2P0.net
>>564
だから、文学的レトリックとしてごく稀に例外的にその語順が使われる事例があるのはわかってるんだよ。 
しかし、その語順は、全体的に見たら、あくまで稀であり特例なんだ。誤解を与えるレスはやめましょう。

どの日本の辞書(ジーニアス・ウィズダムを含む)にも「通例、too 形容詞 a 名詞の語順で」と書いてあるだろ。通例の意味わかる?「ふうつはこう書くよ」と意味だ。

ましてや、This is too big a ball.という文学的意味のないふつうの日常会話レベルの英文で、その語順を使うネイティブはいないんだよ。残念だったね🤠

572 :英語職人 :2023/10/27(金) 22:42:01.34 ID:WQJ7WW2P0.net
深い英語の知識があれば、>>544の③のみが堂々と自然だ言える選択肢だとわかるんだけどね。

「(文学作品でもない)日常会話で a too 形容詞 名詞 の語順は自然な言い方だ」と言ってるネイティブはいないんだよ。いい加減、理解しないさい。😄

https://english.stackexchange.com/questions/362412/too-big-a-vs-a-too-big

ここでネイティブが「tooという単語の前に冠詞の'a'がつくことはほとんどない。例えば、'That was too big a piece of pie'は正しいですが、'That was a too big piece of pie'は私にはおかしく聞こえます。」と言っている。これが実情だよ。読み手を誤解させる情報を流すのはやめなさい。

573 :名無しさん@英語勉強中 :2023/10/27(金) 22:44:20.47 ID:RIRH1uza0.net
"a too long story"
これを Google で検索してどれだけヒットするか見るとよい
最初はノンネイティブのものばかりかと思ったら
英米人もかなり普通に使っている
もちろん "too long a story" の方がヒット数は多いが

574 :名無しさん@英語勉強中 :2023/10/27(金) 22:48:11.09 ID:9/+M8ORN0.net
ネイティブも変な語順だなぁと思ってるんやろな

575 :英語職人 :2023/10/27(金) 22:58:59.56 ID:WQJ7WW2P0.net
私に反論してる人の間違いは、私が文学的表現ではなく、日常会話レベルの話をしていることに気づいていないのだ。

① This is a too big ball.
② This is too a big ball.
③ This is too big a ball.

こんな日常会話レベルの表現で、「This is a too big ball.」なんていうネイティブはいないんだよ。だから、③のみが正しいとする私の意見は、正しい。
証拠
https://books.google.com/ngrams/graph?content=a+too+big,too+big+a&year_start=1800&year_end=2019&corpus=en-2019&smoothing=3

しかし、全体からみれば「まれ」であるが、文学的表現の中には「a too 形容詞 名詞」はないわけではない。だから、「文学的表現において時に a too 形容詞 名詞になる」という説明なら納得できる。しかし、まるで日常会話でも時々使われるような説明をしてはダメなのだよ。

576 :英語職人 :2023/10/27(金) 23:05:45.28 ID:WQJ7WW2P0.net
>>573
「a too long story」なんて使ってる作家誰もいないよ。↓
https://books.google.com/ngrams/graph?content=too+long+a+story%2Ca+too+long+story&year_start=1800&year_end=2019&corpus=en-2019&smoothing=3

いい加減なデタラメを言うのはやめなさい 

577 :🤭英ちゃん🫢 :2023/10/27(金) 23:20:36.74 ID:17kcDGjUr.net
それ言うなら、This is too big a ball. だって日常会話でそうそう言わないんじゃ?🤔

> I am not sure if this answers your question, but 'too big a place' is much more poetical
https://english.stackexchange.com/questions/362412/too-big-a-vs-a-too-big

578 :英語職人 :2023/10/27(金) 23:21:05.59 ID:WQJ7WW2P0.net
反論者に教えてやるが、実は、「a too 形容詞 名詞」になりやすい形容詞というものが文学界には存在する。
たとえば、「a too vivid」である。これは、文学上その語順が許容される形容詞なのである。

証拠
https://books.google.com/ngrams/graph?content=too+vivid+a+%2Ca+too+vivid&year_start=1800&year_end=2019&corpus=en-2019&smoothing=3

しかし、bigをこの語順で書く作家はほぼ皆無なのである。だから、>>544では③のみが自然であるという結論に至り、私の>>544の正しさが証明されるのである。

579 :英語職人 ◆azN58fWNzw (ワッチョイ 8d13-xUKY):2023/10/27(金) 23:34:39.80 ID:WQJ7WW2P0.net
>>577 いや、ふつうに言うでしょ。🤠 

そこじゃなく、そのネイティブの「For example, 'That was too big a piece of pie' is correct, but 'That was a too big piece of pie' sounds funny to me.」という発言が重要。

580 :名無しさん@英語勉強中 (ワッチョイ 4e96-Ez9F):2023/10/27(金) 23:36:57.67 ID:vImY6XYA0.net
>>577
確かに。
This is too big a ball. を検索してもヒット件数はゼロですね。

581 :名無しさん@英語勉強中 (アウアウウー Sa09-J27o):2023/10/27(金) 23:39:00.58 ID:LT/BEB+pa.net
深い英語の知識(ChatGPT)

582 :名無しさん@英語勉強中 (ワッチョイ 1a3a-ptOG):2023/10/27(金) 23:43:51.88 ID:RIRH1uza0.net
これはもう英語職人の負けです。
あきらめなさい!

583 :英語職人 ◆azN58fWNzw (ワッチョイ 8d13-xUKY):2023/10/27(金) 23:49:37.20 ID:WQJ7WW2P0.net
>>580
「これは大きすぎるボールだ」もヒット数ゼロだよ。でも、正しい日本語だよね。それは、単にそういうことを書く必要性がないから誰も書かないんだよ。

https://www.heritageprofessional.com/blog/the-summer-of-yes

このページの8段落あたりに、「If this is too big a question」という英語がある。必要があれば、This is too big a ball.にしてよい。

584 :英語職人 ◆azN58fWNzw (ワッチョイ 8d13-xUKY):2023/10/27(金) 23:56:07.79 ID:WQJ7WW2P0.net
>>582
どこが負けてるんだよ。言ってみなよ。私が論破できない優秀な反論などひとつもないぞ。🤠  
また、今回も私の勝利でしたね (^-^)  反論者は、負けなんだから、いい加減あきらめましょう。

■■■ まとめ ■■■

私の意見: ふつう、a too big ballという言い方はしない。too big a ballという。
反論者の意見: a too big ballという言い方も日常会話でよく使われる

さあ、どっちが正しいでしょうか🤠

585 :名無しさん@英語勉強中 (ワッチョイW 819d-8DLG):2023/10/28(土) 00:14:16.49 ID:2cA1UUos0.net
そんなバカでかい玉があるのに悠長にthis isとか紹介してる場合かよ!

586 :名無しさん@英語勉強中 (ワッチョイ 53c1-Q0z2):2023/10/28(土) 01:08:59.02 ID:Ytlei18j0.net
What a big dumb ball 英語職人 is!!!!
How stupid he is!!!

587 :名無しさん@英語勉強中 (ワッチョイ 1b96-EGHm):2023/10/28(土) 01:22:41.98 ID:YPEeAK2a0.net
>>583
正しいかどうかじゃなくて日常的に使うかどうかが問題じゃなかったのか?
正しいかどうかなら a too big ball でも問題ないな。

588 :英語職人 ◆azN58fWNzw (ワッチョイ 6113-vgKx):2023/10/28(土) 05:30:32.60 ID:QC5dkYFv0.net
>>586
根拠も示さないで、悪口だけで反論するなんて、頭わるそう。まあ、反論するだけの能力がないから仕方ないか (^O^)

589 :英語職人 ◆azN58fWNzw (ワッチョイ 6113-vgKx):2023/10/28(土) 05:31:03.07 ID:QC5dkYFv0.net
■□■ 今回の問題のまとめ ■□■ みなさんもどちらが正しいかよく考えてみてね 🤠

どれがもっとも正しいですか?
① This is a too big ball.
② This is too a big ball.
③ This is too big a ball.

私の意見: ③番がもっとも正しい。
反論者の意見: ③番も日常会話でよく使うから、①と③だよ

検証
&#9679;ジーニアス辞書とウィズダム辞書は、「通例 too 形容詞 a 名詞 の語順になる」と言っている。通例とは、「ふつう」という意味である。
&#9679;https://books.google.com/ngrams/graph?content=a+too+big,too+big+a&year_start=1800&year_end=2019&corpus=en-2019&smoothing=3
これ↑によれば、「a too big」を使っている作家はほぼ皆無。
&#9679;「a too long story」なんて使ってる作家も誰もいないよ。↓
https://books.google.com/ngrams/graph?content=too+long+a+story,a+too+long+story&year_start=1800&year_end=2019&corpus=en-2019&smoothing=3

よって、③がもっとも正しいとする私の意見は、正しいことが証明される。

590 :英語職人 ◆azN58fWNzw (ワッチョイ 6113-vgKx):2023/10/28(土) 05:31:30.43 ID:QC5dkYFv0.net
■□■ 作家も a too 形容詞 名詞の語順を使う場合 (例外) ■□■

作家も「a too 形容詞 名詞」の語順をふつうに使う場合がある。それは、ある特定の形容詞が使われた場合である。これらの形容詞が使われている場合は、「a too 形容詞 名詞」も結構使われる。これは、例外事項である。

《例》
vivid
https://books.google.com/ngrams/graph?content=too+vivid+a%2Ca+too+vivid&year_start=1800&year_end=2019&corpus=en-2019&smoothing=3

literal
https://books.google.com/ngrams/graph?content=too+literal+a%2Ca+too+literal&year_start=1800&year_end=2019&corpus=en-2019&smoothing=3

hearty
https://books.google.com/ngrams/graph?content=too+hearty+a%2Ca+too+hearty&year_start=1800&year_end=2019&corpus=en-2019&smoothing=3

ジーニアスが「時にa too 形容詞 名詞」とか、ウィズダムが「まれにa too 形容詞 名詞」と言っているのは、これら特別な形容詞を使った表現のことを言っているのである。ただ、ジーニアスの「時に」という表現は、日常会話でも、どんな形容詞でも、「a too 形容詞 名詞」が自然である場合があるかのような誤解を与えかねないので、あんまりよくないと思う。実際、そう誤解している人がいるし。(>>547が「a too big ball」が自然な表現であると誤解している。)

591 :英語職人 ◆azN58fWNzw (ワッチョイ 6113-vgKx):2023/10/28(土) 05:33:53.30 ID:QC5dkYFv0.net
>>589の訂正
■□■ 今回の問題のまとめ ■□■ みなさんもどちらが正しいかよく考えてみてね 🤠

どれがもっとも正しいですか?
① This is a too big ball.
② This is too a big ball.
③ This is too big a ball.

私の意見: ③番がもっとも正しい。
反論者の意見: ①番も日常会話でよく使うから、①と③だよ

592 :英語職人 ◆azN58fWNzw (ワッチョイ 6113-vgKx):2023/10/28(土) 05:40:35.48 ID:QC5dkYFv0.net
訂正その2

&#9679;「a too long story」なんて使ってる作家も誰もいないよ。↓
https://books.google.com/ngrams/graph?content=too+long+a+story,a+too+long+story&year_start=1800&year_end=2019&corpus=en-2019&smoothing=3

1900年くらいに数人の作家が使ってそうなレベル (^O^)

593 :英語職人 ◆azN58fWNzw (ワッチョイ 6113-vgKx):2023/10/28(土) 05:42:41.64 ID:QC5dkYFv0.net
■□■ 今回の問題のまとめ (改定版) ■□■ みなさんもどちらが正しいかよく考えてみてね 🤠

どれがもっとも正しいですか?
① This is a too big ball.
② This is too a big ball.
③ This is too big a ball.

私の意見: ③番がもっとも正しい。
反論者の意見: ①番も日常会話でよく使うから、①と③だよ

検証
&#9679;ジーニアス辞書とウィズダム辞書は、「通例 too 形容詞 a 名詞 の語順になる」と言っている。通例とは、「ふつう」という意味である。
&#9679;https://books.google.com/ngrams/graph?content=a+too+big,too+big+a&year_start=1800&year_end=2019&corpus=en-2019&smoothing=3
これ↑によれば、「a too big」を使っている作家はほぼ皆無。
&#9679;「a too long story」なんて使ってる作家もほぼ皆無。↓
https://books.google.com/ngrams/graph?content=too+long+a+story,a+too+long+story&year_start=1800&year_end=2019&corpus=en-2019&smoothing=3

よって、③がもっとも正しいとする私の意見は、正しいことが証明される。

594 :The OED Loves Me Not :2023/10/28(土) 09:28:56.04 ID:VUjQEHpf0.net
黄緑色の絵を見せて、
「これは、緑に近いか?それとも黄色に近いか?」
という質問に対して、
「緑に近い」と言っている人と「黄色に近い」って言っている人がいて、
いつまでも「私の方が正しく、あなたは間違い」と言い続ける人がいる。

どっちも正しいというのが正解なんじゃないかな?あるいは、そんなくだらないことは
どうでもいいのだ、と言ってもいい。ある観点から見れば緑に近くも感じられる。
別の観点から見れば、黄色に近く感じる。それぞれの人の人生経験などによっても、
感じ方は違う。

これはあくまで黄色と緑のあいだであって、どっちに近いかなんて所詮は言葉の曖昧さとか
人間の感性の曖昧さ(その曖昧さこそが、実は人間の感性の素晴らしいところ)を利用して、
押しの強い人は相手を言い負かしたように見せかけることができる。

根負けした人、というか、別に最初から勝負なんてするつもりなんてなく、単に
「両方とも正しいと言ってもいい」と思っている人は、「まあ、いいか」と引き下がる。そんなことが繰り返される。

でも私自身は、そういう議論を目の当たりにして、いろんな証拠を見せてもらって、
「なるほど、両方とも使われているんだな。ただ、その頻度が違っているだけなんだな」と思って、勉強させてもらう。

595 :名無しさん@英語勉強中 :2023/10/28(土) 14:15:07.81 ID:DN63HVZG0.net
This is too big a ball.
などと言う場面はほぼないと思います。
強いて言うなら、
Picking up a ball from the shelf, he mumbled "This is too big for my purpose."
くらいの言い方ではないでしょうか?

596 :名無しさん@英語勉強中 :2023/10/28(土) 14:21:14.06 ID:kqHn6zQLr.net
This is the pen.

597 :The OED Loves Me Not :2023/10/28(土) 14:22:34.56 ID:VUjQEHpf0.net
"a too gorgeous woman" など、"a too gorgeous + 名詞" という
phrase は、もちろん "too gorgeous a 名詞" に比べると
はるかに使用頻度が低いし、文学作品だけに限られる傾向があるんだろうけど、
一応その用例を google,books などで見ていると面白いですね。

https://books.google.com/ngrams/graph?content=a+too+gorgeous%2Ctoo+gorgeous+a&year_start=1700&year_end=2019&corpus=en-2019&smoothing=3

この上のリンク先を見ていると、どういうわけか 1880 年あたりに(つまり今から 140 年も前に)
"too gorgeous a" よりも使用頻度の高かったころがあっように見えます。

598 :The OED Loves Me Not :2023/10/28(土) 14:28:10.56 ID:VUjQEHpf0.net
https://www.google.co.jp/books/edition/Sparrow_Nights/arLRSzJGVB4C?hl=en&gbpv=1&dq=%22a+too+gorgeous%22&pg=PT90&printsec=frontcover

この上のリンク先のページ上の、黄色く表示されたところの "a too gorgeous woman" という phrase などは、
面白いですね。

もちろん、いろいろと解説してくれた英語職人さんの考察をすべて尊重した上での話ですけど。
(決してここで私は、英語職人さんに反論しているわけではないので、誤解しないでくださいね。)

599 :名無しさん@英語勉強中 :2023/10/28(土) 14:33:12.56 ID:DN63HVZG0.net
>>595
自己レスですが「強いて言」ってないですね。ま、土曜の昼下がりと言うことでご容赦。

600 :The OED Loves Me Not (ワッチョイ eb89-H9h+):2023/10/28(土) 17:02:35.62 ID:VUjQEHpf0.net
興味本位だけど、"a too perfect + 名詞"という形が一カ所に3つも並んでいる
用例を見つけた。2015 年の Supersymmetry という小説の中での
一節で、ご丁寧にも too perfect のところに三つとも hyphen がついて
"a too-perfect + 名詞" となっている。

a too-perfect vase from a too-perfect table and smashed it on the too-perfect floor.
https://www.google.co.jp/books/edition/Supersymmetry/HeOKDwAAQBAJ?hl=en&gbpv=1&dq=%22a+too+perfect%22&pg=PT140&printsec=frontcover

リンク先に表示されるページの、下から3行目あたりに、こういう用例が三つも立て続けに出ているので、注目。

601 :The OED Loves Me Not (ワッチョイ eb89-H9h+):2023/10/28(土) 17:09:17.23 ID:VUjQEHpf0.net
... a too-perfect childhood, a too-perfect school experience may actually atrophy the kids' abilities to become problem-solvers and initiative-takers.
https://www.google.co.jp/books/edition/Childhood_is_a_Verb_Why_a_Virtual_Childh/C7BXDwAAQBAJ?hl=en&gbpv=1&dq=%22a+too+perfect%22&pg=PA183&printsec=frontcover

この上の用例では、"a too-perfect + 名詞" という形で、しかも too と perfect との間に hyphen をつけて、
それが2つも並んでいる。

もしかしたら、too perfect という言葉を連発することが日常会話では多いのかもしれない。しかも
それが頻繁に使われているうちに、too-perfect のように hyphen つきで書き表したくなるくらいに
too と perfect とが密接につながっているようにネイティブたちには感じられているのかもしれないと
思ったりもしたくなる。

602 :The OED Loves Me Not :2023/10/28(土) 18:17:05.54 ID:VUjQEHpf0.net
もう一つ、少し面白い用例。

He thinks I'm ★a too-weird, too-gross girl★ - just the way my classmates saw me in third grade.
https://www.google.co.jp/books/edition/The_Beautiful_Something_Else/HWR-EAAAQBAJ?hl=en&gbpv=1&dq=%22a+too+weird%22&pg=PT19&printsec=frontcover

上のリンク先のページの一番下に、これが出てくる。小説らしいのだが、
そのタイトルなどは、この下に示す通り。2023 年のもの。

The Beautiful Something Else
Ash Van Otterloo · 2023

603 :英語職人 :2023/10/28(土) 21:01:34.76 ID:QC5dkYFv0.net
>>595
それを言ったら、「Her pen is very long.」なんて言う場面もないぞ。でも、そんなペンがあったら、「Her pen is very long.」と言っても文法学的に問題はない。

https://www.heritageprofessional.com/blog/the-summer-of-yes

このページの8段落あたりに、「If this is too big a question」という英語がある。必要があれば、「This is too big a 名詞」は普通に問題なく使われる。

604 :名無しさん@英語勉強中 :2023/10/28(土) 21:13:01.19 ID:pqMI6Ygj0.net
I hope it wasn't too much of a trouble.

605 :英語職人 :2023/10/28(土) 21:13:18.10 ID:QC5dkYFv0.net
例外的に「 a too 形容詞 名詞」になれるかどうかは、どの形容詞が使われているかが大きく影響するのである。この指摘をしたのは、日本で私が最初ではなかろうか。

<論拠>
>>593
>>590

形容詞によって、こんなにも差が出るのである。🤠

606 :名無しさん@英語勉強中 (ワッチョイ 0996-c8RC):2023/10/29(日) 02:22:28.41 ID:nwT1nsYe0.net
ネットの評判を見て買った参考書にある英文です。

I know no statesman in the world who with greater right than I can say that he is the representative of his people.
 ↑
この文の関係詞のあたりの解釈が難しいです。
who (with greater right than I) can say that he is the representative of his people.
という構造らしいのですが、この文章はネイティブでも読解が難しいレベルなんでしょうか?

また、この文の意味は、
世界で一番政治家に向いているのはこの文の筆者という理解でよいのでしょうか?
he is the representative of his peopleの部分がheになっているけど、これは間接話法なんだという理解でよいですよね?

607 :名無しさん@英語勉強中 :2023/10/29(日) 03:33:16.45 ID:ZwdWAmTU0.net
私は、世界中の政治家の中で、私ほど自分が国民の代表
であると言い切れる人物を知らない。がDeepLの訳
with greater right than I が副詞句
whoの動詞がcan say

608 :名無しさん@英語勉強中 :2023/10/29(日) 03:33:22.10 ID:ZwdWAmTU0.net
私は、世界中の政治家の中で、私ほど自分が国民の代表
であると言い切れる人物を知らない。がDeepLの訳
with greater right than I が副詞句
whoの動詞がcan say

609 :名無しさん@英語勉強中 :2023/10/29(日) 03:37:10.53 ID:ZwdWAmTU0.net
私は、世界中の政治家の中で、私ほど自分が国民の代表
であると言い切れる人物を知らない。がDeepLの訳
with greater right than I が副詞句
whoの動詞がcan say

610 :The OED Loves Me Not :2023/10/29(日) 05:03:57.67 ID:hylKqUhZ0.net
>>606
その英文は、Adolf Hitler が 1936 年に言った、または書いた(おそらくドイツ語による)言葉
の英訳に過ぎないと思います。

https://books.google.co.jp/books?id=hMUPDgAAQBAJ&pg=PA298&lpg=PA298&dq=%22I+know+no+statesman+in+the+world+who+with+greater+right+than+I+can+say%22+that+he+is+the+representative+of+his+people.&source=bl&ots=cx7xKLtPdq&sig=ACfU3U2aD5-1rTCLTevpNMjoSVq3OSLCOA&hl=en&sa=X&ved=2ahUKEwj26trCwpmCAxU7sFYBHZz8DrgQ6AF6BAgUEAM#v=onepage&q=%22I%20know%20no%20statesman%20in%20the%20world%20who%20with%20greater%20right%20than%20I%20can%20say%22%20that%20he%20is%20the%20representative%20of%20his%20people.&f=false

上のリンク先に表示されるページの真ん中あたりに出てきます。

80年前の文章であり、しかも Hitler が勢い込んで言った(あるいは書いた)ドイツ語による文章の
英訳なのだから、少し堅くても不思議はないと思います。しかし現代の人でも十分に読める文章だと思います。

このような who with... can say... のような言い回しは。

who with
which with
などを検索すれば、いくらでも出てきます。そういう文章を普段から読み慣れていれば、今回のように
with のあとが少し長くなっていても、ちゃんとその文の構造が理解できるのです。

質問者さんの読んでいる本は、おそらくは伊藤という有名な人の英文解釈書だと思いますが、
少し古い時代の、日本人が誤読しやすい、少し複雑な構文ばかりを集めて
徹底的に英文解釈の仕方を教える本ですから、普通の人にとって難しいものばかりであっても
仕方がないと思います。

611 :名無しさん@英語勉強中 (ワッチョイW 29ab-Vp97):2023/10/29(日) 06:08:45.98 ID:4sLpOQ3p0.net
>>606
なるほど…やべぇ、俺自力ではわかんなかったな
でもこれ元々この文なんだろうか?
わざとわかりにくく改変したりしてないのかな

612 :The OED Loves Me Not (ワッチョイ eb89-H9h+):2023/10/29(日) 06:37:07.13 ID:hylKqUhZ0.net
>>606 にある課題文については、別の人がネット上で次のように解説しています。

******* 以下は引用 *****

9.1.1
?I know no statesman in the world who with greater right than I can say that he is the representative of his people.
自分が国民の代表であると、私よりも大きな権利をもって言うことのできる?政治家を私は世界中に知らない。

?「政治家を私は世界中に知らない」:誤差
あまりにも直訳的。I know that 〜 の形で、「〜という事実に気づいている」「〜ということをはっきり自覚している」の意味。
ここ、文の骨子はI can say with the greatest right that I am the representative of my people.

修正訳:「政治家は世界中にいないはずである」

http://www.wayaku.jp/monthly_review/review8.html

613 :The OED Loves Me Not (ワッチョイ eb89-H9h+):2023/10/29(日) 06:43:02.81 ID:hylKqUhZ0.net
>>606
この上に示した柴田氏による批評を私はきちんと読んでいませんでした。
単に伊藤氏による元々の和訳だけを参考にすればいいと思っておりました。

よく読むと、伊藤氏の和訳の方が正しくて、柴田氏による解釈が
間違っているように思います。柴田さんには申し訳ないですが。

>>I know that 〜 の形で、「〜という事実に気づいている」「〜ということをはっきり自覚している」の意味。
>>ここ、文の骨子はI can say with the greatest right that I am the representative of my people.

この上の部分が間違っていると思います。

614 :名無しさん@英語勉強中 :2023/10/29(日) 09:42:00.46 ID:mH1kk3i/0.net
次のout は形容詞ですか?自動詞ですか?
Can you send someone here?
こちらにどなたか派遣してもらえますか。
Absolutely. I'll have one of our technicians out there today.
もちろんです。本日中に、当社の技術者を1名そちらに向かわせます。
使役のhave OCだと思うのですが、形容詞の前にbe を補って考えたらよいのでしょうか?

615 :名無しさん@英語勉強中 :2023/10/29(日) 09:50:07.33 ID:9oUZeDZR0.net
文法考えずにみると副詞にしか見えないですけど

ぱっと見で形容詞とか動詞には見えないですよね

616 :名無しさん@英語勉強中 :2023/10/29(日) 10:58:53.55 ID:iYFO0X+E0.net
以下は、Michael Spivak著『Calculus on Manifolds』という数学の本に書いてある一文です。

「If ε > 0, let P be a partition of A such that U(f, P) - L(f, P) < ε/n.」

この「if」について質問です。
この文の意味は、「εを正であるとしたとき、PをU(f, P) - L(f, P) < ε/nであるようなAの分割とする。」というような意味だと思います。

「if」がこのように使われることは普通ですか?

617 :The OED Loves Me Not :2023/10/29(日) 11:02:22.59 ID:hylKqUhZ0.net
>>614
>>I'll have him out there today.

それはたぶん、次のような構文なんでしょうね。

have
SVOC〈人が〉O〈人・物など〉をCにしておく《◆Cは形容詞・名詞・副詞》
I want you to have this room clean and tidy.
この部屋をきれいに片付けておいてもらいたい.
(ジーニアス英和、6)

*********

今回の英文での out の品詞なんてどうでもいいと思いますが、
あえて言うなら副詞でしょうね。

over there
over here
right here
out here
out there

これらの over, right, out はすべて副詞ということでいいでしょう。

618 :名無しさん@英語勉強中 :2023/10/29(日) 11:02:50.85 ID:iYFO0X+E0.net
あるいは、「εが正であるとき、PをU(f, P) - L(f, P) < ε/nであるようなAの分割とする。」というような意味だと思います。

619 :The OED Loves Me Not :2023/10/29(日) 11:10:15.26 ID:hylKqUhZ0.net
>>616
>>「if」がこのように使われることは普通ですか?

「このように」って、「どのように」なのですか?

If *** is larger than ***, let ***."
というふうに、あとで let がつくような形式が普通かどうか、という意味なのか、
それとも
If *** is larger than***,"
という形式が普通かどうかという意味なのか。

それとも、どういう意味なのか、私にはわかりません。

いずれにしても私が昔に数学の本を英文でいくらか読んだ内容をうっすらと覚えている範囲内では、
こういうのは普通だと思いますが、どこが変だ(珍しい)と思うのですか?

620 :名無しさん@英語勉強中 :2023/10/29(日) 11:16:25.56 ID:FCrjVEkk0.net
>>616
普通だよ

621 :名無しさん@英語勉強中 :2023/10/29(日) 11:17:58.70 ID:6db0Z4+J0.net
>>613

「骨子」を言ってるので
柴田さんの方があってるんじゃない?

622 :名無しさん@英語勉強中 :2023/10/29(日) 11:18:10.12 ID:6db0Z4+J0.net
>>613

「骨子」を言ってるので
柴田さんの方があってるんじゃない?

623 :The OED Loves Me Not :2023/10/29(日) 11:57:39.13 ID:hylKqUhZ0.net
>>621
いや、骨子を言っているだけだから柴田さんが正しいかな、と私も少し思いましたが、
よく読んで見ると、やっぱり間違っているのではないかと思います。

>>@I know no statesman in the world who with greater right than I can say that he is the representative of his people.
>>ここ、文の骨子はI can say with the greatest right that I am the representative of my people.

I を仮に he に言い換えたとしても
He can say with the greatest right that he is the representative of his people.
となりますが、原文では

>>no statesman ... who with greater right than I (話者) can say that he is the representative of his people.

となっています。つまり原文の than I can say は greater right にかかっているので、それを消すと

no statesman ,... who can say that he is the representative of his people

となります。その骨子を和訳すると
「自国(自分の住んでいる国)の国民の代表者だと言える政治家(を私は知らない)」
ということです。これは決して、柴田さんの言うような
I can say with the greatest right that I am the representative of my people.
とは違っていると思いませんか?柴田さんは

>>I know that 〜 の形で、「〜という事実に気づいている」「〜ということをはっきり自覚している」の意味。

だと言っているのです。

つまり柴田さんは、ここでの骨子の構文が
I know that S V.
だと言っていますが、元の原文はあくまで
I know no statesman who can say that SV.
が骨子になっているのです。

624 :名無しさん@英語勉強中 :2023/10/29(日) 14:56:32.39 ID:iYFO0X+E0NIKU.net
>>620
ありがとうございました。

625 :名無しさん@英語勉強中 :2023/10/29(日) 16:41:44.93 ID:cNNHh/5jMNIKU.net
英文を前から読む訓練が出来る本はありますか?

626 :名無しさん@英語勉強中 :2023/10/29(日) 16:41:55.52 ID:cNNHh/5jMNIKU.net
英文を前から読む訓練が出来る本はありますか?

627 :名無しさん@英語勉強中 :2023/10/29(日) 18:52:07.10 ID:ZwdWAmTU0NIKU.net
上の方にある
onlyが被限定語句の直前ない場合だけど、
onlyが直後の言葉を限定して非限定語句の意味を
変えてしまう
というのが英語と日本語の違いかと思った。
few 名詞なんちゃらかんちゃら
でなんちゃらかんちゃらの名詞がほとんどない
というのに似てるのかも。

前にonlyが被限定後の直前にのみつく、というの
ではない、と答えてくださった方達ありがとう。
被限定語というのも、英語と日本語で違って
くるのかもしれない

628 :英語職人 :2023/10/29(日) 20:02:48.32 ID:DkmiePvX0NIKU.net
>>612
「greater right」のrightは、「権利」じゃなく「道理」ですね。🤠

I know no statesman in the world who with greater right than I can say that he is the representative of his people.
↑これは、ありふれた表現を使うと次のように言える。
I can rightfully say I represent my people better than any other leader in the world.
道理に鑑みて、私こそが世界のどの指導者よりも国民を代表していると言えるのだ。

rightfully の定義
https://www.oxfordlearnersdictionaries.com/us/definition/english/rightfully
according to the law or to what is right or correct 『法律または道理に基づいて』 

&#9679;with+抽象名詞は、副詞と言い換えられる。with care = carefully, with ease = easily

629 :The OED Loves Me Not :2023/10/29(日) 20:22:07.83 ID:hylKqUhZ0NIKU.net
>>628
確かにこの場合は、「権利」よりも「道理」が近いでしょうね。
ただここで「資格」と伊藤氏が訳したとき、法律上の「資格」とか
その他の手続き上の「資格」を言っているのではなく、もっと
日常的な意味で言っていたのだと私は解釈しています。

たとえば、「あんたにそんな権利があるのかよ」とかいうとき、
法律上の権利じゃなくて「そんなことをやってもいいのか」と
いう緩い意味で言っているのだと思います。そうであるなら、
ここでも「資格」と訳してもそう変ではないと思います。

さらには、「道理」と言うとあまりに堅い日本語であり、
英語の right という単語が持つ日常的な気楽な
意味合いを考えると、私などは「道理」という言葉をここでは
使いたくないと思ってしまいます。(ただし、ここでの本当の
意味合いは、たとえ堅い言葉だとはいえ「道理」に最も
近いということは認めます。)

私ならここで、「資格」と訳したくなります。日常会話でよく
軽い意味合いで「私にはその資格がある」とか
「お前にそんな偉そうなことを言う資格があるのかよ」
というときの、きわめて気楽な緩い意味合いでの「資格」です。

>>@I know no statesman in the world who with greater right than I can say that he is the representative of his people.
>>自分が国民の代表であると、私よりも大きな ★権利★ をもって言うことのできる@政治家を私は世界中に知らない。

私ならここで、あえて直訳調ではなくかなり崩して意訳的になりますが、伊藤氏による和訳を全体的に変えて、次のようにでも訳したいです。

私訳:
"自分が国民の代表だ" と言う資格を私よりもたくさん持っている政治家は、世界中を探しても私には見つからない。

630 :The OED Loves Me Not :2023/10/29(日) 20:28:49.29 ID:hylKqUhZ0NIKU.net
英語職人さん。私は決してあなたに反論してケチをつけようとは思っておりませんので、
どうかムキにならないでください。あなたはこの種の議論が好きで、2週間くらい毎日
数時間にわたって一つの問題について、たとえ水掛け論みたいになってもいいから
互いに批判し合うのもお好きなようですが、私はそういうのが大嫌いで、
そんなのが始まると、つくづく人間社会が嫌になり、どうか早く人類が滅亡してくれるよう
願いますが、どうせ人類の滅亡なんていう素晴らしい未来はやってこないから、
せめて私だけでも早く死にたいと思ってしまうタイプです。

だからどうか、私に対してはムキにならないでください。もし私が言いすぎたり、
あなたに失礼なことを言ってしまったりしたら、私の方から謝罪しますので。

私も、あなたの貴重なご意見を伺い続けたいので、あなたを NG 登録したくはないのです。
他の連中に対しては、即座に NG 登録しますけど。

631 :🤭英ちゃん🫢 :2023/10/29(日) 20:42:45.24 ID:N8uTEVv50NIKU.net
白人娘のcreampie動画が狂おしいほど好きなのに?!

632 :名無しさん@英語勉強中 :2023/10/29(日) 21:05:03.30 ID:4sLpOQ3p0NIKU.net
>>629
>ただここで「資格」と伊藤氏が訳したとき、法律上の「資格」とか
>その他の手続き上の「資格」を言っているのではなく、もっと
>日常的な意味で言っていたのだと私は解釈しています。

OEDさんが貼った柴田のサイトを見る限り伊藤訳も柴田も資格なんて言葉使ってないけど、どういうことなん?

633 :The OED Loves Me Not :2023/10/29(日) 21:19:48.70 ID:hylKqUhZ0NIKU.net
>>632
すみません。「資格」は間違いで、「権利」でした。
「資格」と私は訳したいとずっと思っていたので、
書き間違えたのです。

634 :英語職人 :2023/10/29(日) 21:50:30.58 ID:DkmiePvX0NIKU.net
>>630
ムキになりませんので、ご安心ください。🤠  「資格」と意訳してもいいと思います。

ここでのwith rightはrightfullyの意味を持っています。だから、rightfullyの例文を調べると、どう意訳したらよいかのヒントになるでしょう。
https://www.goodreads.com/quotes/517100-a-mother-cat-is-rightfully-called-a-queen-personally-i
このページに「A mother cat is rightfully called a Queen. 」という英文があります。この「rightfully」はどう訳しますか?
「道理に鑑みて、母猫はクイーンと呼ばれている」は、わかりやすく訳すと「(そう呼ばれるのにふさわしいちゃんとした理由があって)母猫はクイーンと呼ばれている」という意味になると思います。(母猫をクイーンと呼ぶ人が結構いる)証拠↓
https://petcube.com/blog/11-fact-about-mommy-cats/#:~:text=1)-,A%20mother,-cat%20is%20called

よって、I know no statesman in the world who with greater right than I can say that he is the representative of his people.
も「私よりも多くの十分な根拠をもって『国民を代表しているよ』なんて言ってる政治家なんてわしゃ知らん」という意味が根底に流れていることがわかっていれば、「資格」と意訳しても、なんら問題はありません。🤠

635 :名無しさん@英語勉強中 (ワッチョイ 6b51-CP9B):2023/10/30(月) 00:39:15.46 ID:LX2f3Kfi0.net
権利・・・ある物事をしてよい、またはしないでよいという資格

「資格」がいいなら「権利」でも問題ない。
who can say ~ with right
のような英文なら、rightを「権利」や「資格」と訳しても、
「根拠をもって〜と言うことが可能な」という意味として通じる。

636 :名無しさん@英語勉強中 (ワッチョイ 6b51-CP9B):2023/10/30(月) 01:05:07.96 ID:LX2f3Kfi0.net
意味は通じるけど、やはり「権利」はまずいですね。可算名詞として使われていない。
「正当性」という意味合いを強く出して訳すべきで、英語職人さんが正しいです。
失礼しました。

637 :名無しさん@英語勉強中 :2023/10/30(月) 01:25:11.94 ID:FbVYYLAF0.net
資格だね。

638 :名無しさん@英語勉強中 :2023/10/30(月) 01:25:27.05 ID:FbVYYLAF0.net
資格だね。

639 :The OED Loves Me Not :2023/10/30(月) 05:53:53.13 ID:ISEsyz2J0.net
>>636
right が可算名詞の時にのみ「権利」という意味になるとは言えないと思いますよ。
この下に「ジーニアス英和、6」の解説を抜き書きしますが、不可算名詞のときの
right が「権利」という意味にもなるということを示しているように思える部分だけを
示してみます。

right
CU[通例 a/the 〜]〔…する/…に対する/…の〕権利〔to do / to / of〕
《◆法律・道徳・生得・伝統・感情などのいずれに由来する場合にも用いる;法律に由来する場合はしばしばC》(⇔duty)

(私からの注釈:この上の C U というのは、文字化けしていますが、ジーニアス英和によると C は可算名詞で、U は不可算名詞です。
さらにそのあとに、電灯や感情などに由来するときのも使う、と書いてあります。法律に由来する場合はしばしば可算名詞なんだと書いてあります。
ということは、結論として、不可算名詞としての right を「権利」という意味で使うこともある、ということになります。)


the right to education [free speech]
教育を受ける権利[言論の自由]

(私からの注釈:上のように the right となると、不可算名詞なのか可算名詞なのかわからなくなります。)

protect [defend] religious right
宗教上の権利を保護する[擁護する]

(私からの注釈:この上の right には a がついていませんので、明らかに不可算名詞です。)

She has no right to talk to you like that.
君に対してそんな口をきく資格は彼女にはない.

(私からの注釈:こういう気楽な、ゆるい日常会話での right を「資格」と訳したらピッタリ来そうな用例。)

640 :The OED Loves Me Not :2023/10/30(月) 06:12:32.14 ID:ISEsyz2J0.net
今度は、英英辞典で不可算名詞としての right を「権利」のような意味で解説している
ように見える部分だけを抜き書きしてみます。

The Oxford Advanced Learner's Dictionary (OALD) のネット版より

right
moral/legal claim
​ [countable, uncountable] a moral or legal claim to have or get something or to behave in a particular way

by right The property belongs to her by right.

as of right Education is provided by the state as of right (= everyone has a right to it).

https://www.oxfordlearnersdictionaries.com/definition/english/right_3

**********

right に no, the, my などが付くと、可算名詞なのか不可算名詞なのかわからなくなりますので、
それを除外してもやはり、上のように不可算名詞の「権利」という意味らしき言葉として right
が使われているように見えるものがあります。そして冒頭に、uncountable つまり「不可算名詞」と
書いてあります。

ただし可算名詞になる用例があまりに多いので、それに圧倒されて、ついつい私たちは
「権利」という意味のときには可算名詞なのだと思ってしまいます。現に私も、今の今まで
そう思い込んでました。

でも可算名詞か不可算名詞なのかについては、英語ネイティブたちはけっこう流動的に使っているらしく、
決して「この単語は必ず不可算名詞だ、そしてあの単語は必ず可算名詞だ」などと峻別しているのではない
ようなのです。

641 :The OED Loves Me Not :2023/10/30(月) 06:16:58.73 ID:ISEsyz2J0.net
私が常に「この単語は必ず可算名詞だ」と思い込んでいた problem という言葉も、
実は不可算名詞として使うこともあるのだということについては、つい最近になって
英語ネイティブたちに質問し、膨大な回答をもらったことがあります。全部で
100個くらいの回答をもらったかもしれません。あまりに多かったので、その半分以上は
別の所に管理者によって移動させられてしまって、1ページ目には表示されていません。
リンクをクリックして別の所に行かないと、残りの膨大な回答を読むことができないくらいです。

https://english.stackexchange.com/questions/611717/we-have-little-problem

この上のリンク先に、
I have little problem with [something].
という言い回しにおける problem がここでは可算名詞ではなく不可算名詞なのだという
ことについて、みんなが回答してくれています。しかし英和辞典でも英英辞典でも、これについては
触れていないということにも、私や回答者たちも気づいています。

642 :The OED Loves Me Not :2023/10/30(月) 06:23:09.80 ID:ISEsyz2J0.net
>>634
英語職人さん、ありがとうございます。with right (あるいは with + 形容詞 + right) を
rightfully と解釈できそうなことを教えてもらって、大いに勉強させてもらいました。

The Road to Avonlea というカナダあたりのテレビ番組を昔に英語音声だけで
追っかけていたことがありますが、そのときに気難しい中年の女性教師が妹に対して
This is rightfully mine.
とか何とか言っていたように記憶していますが、その rightfully の使い方を思い出します。

643 :名無しさん@英語勉強中 (ワッチョイ 13f0-DSfs):2023/10/30(月) 08:54:28.46 ID:k0PbeUEX0.net
>>623
確かに、文法的というか、文章の構造的な視点からの
言い換えなら、OEDさんの詳しい解説に一部の隙も
誤りもないと思います。 ・・・・・ただ、

伊藤氏の「政治家を私は世界中に知らない」という
表現は、柴田氏同様にもっと熟れた言い回しの方が
いいのではないかと思ったのに加えて、

柴田氏は、文章の言い換えというよりも、ヒトラーが
スピーチ(?)のあの部分で言わんとした趣旨(骨子
と言うよりも趣旨)をあの英文に言い換えて説明した
のではないかと、私には思えたのです。
"with greater right than I" と”with the greatest right"
という、それぞれの文章での表現からも。

つまり、かなり込み入った物言いによる、言わば”遠回
しのヒトラーの最初の言葉”をより直截にそのまま表し
て説明したのではないかと、勝手に私が考えたのです。

あくまで、私の思い込みですので、これ以上のこの
議論はやめときます。・・・・・というより、

right「権利」「資格」「道理」「正当性」「ふさわしさ」、
さらに、名詞の「可算、不可算」などについての職人、
達人、名人レベルの諸氏の書き込みは今回も大いに
参考になりました。

644 :The OED Loves Me Not (ワッチョイ eb89-H9h+):2023/10/30(月) 10:03:43.97 ID:ISEsyz2J0.net
>>643
なるほど、わかりました。伊藤氏の和訳は、確かに堅いですよね。
それを何とか少しでも自然な日本語に変えたかったというのが
柴田氏の意図なのかもしれませんね。

I know no statesman who... は、直訳すれば伊藤氏の言う通りですが、
その直訳には私も違和感を感じます。

でもだからと言って、自然さを重視して意訳すると、今度は英文解釈の
勉強から外れてしまって、文法とか構文なんてどうでもいいのだ、と
思ってしまう学習者もいるから困ります。伊藤氏はそれを恐れて、あえて
不自然な直訳を提示したのでしょう。

私がもしこういう解説をするとしたら、直訳と意訳と、思い切った意訳との
3通りくらいの和訳を提示したいところです。時間とか気力があるときには、
私はいつもそうしているのです。

645 :名無しさん@英語勉強中 :2023/10/30(月) 14:10:29.08 ID:OaNgas9o0.net
英語って男言葉、女言葉がほとんど無いよな
使う単語に偏りは有るだろうけど

646 :名無しさん@英語勉強中 :2023/10/30(月) 19:38:22.05 ID:c3ZkSA2E0.net
なぜオカマは万国共通のなにかがあるんだろう

647 :名無しさん@英語勉強中 (ワッチョイW 13f0-UViH):2023/10/30(月) 20:48:28.45 ID:DsEHC5S00.net
英語って男言葉、女言葉がほとんど無いわよね
使う単語に偏りは有るんでしょうけど

648 :🤭英ちゃん🫢 (ワッチョイW 1386-ihTc):2023/10/30(月) 21:00:36.36 ID:tgT5pmPq0.net
it's so fab!!😍😍😍

649 :名無しさん@英語勉強中 (ワッチョイ 5320-Q0z2):2023/10/30(月) 21:26:25.69 ID:Ym/6lEmq0.net
https://www.bustle.com/articles/4198-5-ways-women-use-words-differently
Women's speech differs from men's. (Oh wait, that's not a feeling — that's a fact.) Linguists have long recognized that women and men deploy words differently.

[1] Women are more likely to use words like "divine", "adorable" and "gorgeous" — so-called "empty adjectives", defined by Harvard linguist Robin Lakoff as "adjectives that seem devoid of all but a vague positive emotive sense."

[2] Women swear less frequently. According to one study, we use expletives only 61 percent as often as men.

[3] Women tend to use excessively polite expressions, like "Would you mind…?" or "Would it be ok if…?" We're more likely to make an indirect request than issue a straightforward command.

[4] Women are less likely to use hostile verbs, like "murder" and "stab." (I’m okay with this one.)

[5] Women use more intensifying adverbs, like "very", "really" and "so." Because everyone knows that peppering your speech with "very"s is the cornerstone of effective communication.

650 :名無しさん@英語勉強中 :2023/10/30(月) 22:23:07.88 ID:OaNgas9o0.net
オカマ言葉って男が使うってだけで女言葉じゃないか?

651 :名無しさん@英語勉強中 :2023/10/30(月) 22:41:00.96 ID:Ym/6lEmq0.net
昔の英和辞典には
fabulous は女言葉であり、
男が使ってい場合は、その男はゲイである
という記述があったのだが、
LGBT運動の影響で
その記述はなくなった

652 :名無しさん@英語勉強中 :2023/10/30(月) 23:04:17.33 ID:JNWieJz/0.net
辞書の和訳の女言葉はたまに笑ってしまう
特に仏和
原文からはわからない情報がw

日本語の「わ」はニュアンスが難しい
女でもありおっさんくさくもあり

653 :名無しさん@英語勉強中 :2023/10/31(火) 01:39:36.95 ID:THi5/7a10HLWN.net
chamberにはいろんな意味があると思うのですが、
日本人の英語上級者がよく目にする意味として覚えるべき4つの意味を挙げるとしたらどの意味でしょうか?
立法機関、司法機関、議場、(特定の)部屋、の4つでしょうか?
部屋を覚えれば他は不要だ、などの回答は結構です

654 :名無しさん@英語勉強中 :2023/10/31(火) 02:06:33.16 ID:jny0W5wP0HLWN.net
36th Chamber Of Shaolin
gun chamber

655 :名無しさん@英語勉強中 :2023/10/31(火) 08:05:47.41 ID:roylCdnQ0HLWN.net
ウィズダムによれば頻度はこのタイプが多いので

1(動植物・機械などの内部の)空間, 小室; (銃の)薬室
▶the four chambers of our heart
人間の心臓の4つの小室(心室・心房)
▶The engine has combustion chambers.
エンジンには燃焼室がある

機構上の内部スペース
部屋
会議所
議員

でしょうね

656 :名無しさん@英語勉強中 (中止 134f-CP9B):2023/10/31(火) 13:15:00.66 ID:70eInElL0HLWN.net
>>651
O-LEX英和には「主に女性が好んで用いる」
ジーニアス英和大辞典「女性略式」

とあるな

657 :名無しさん@英語勉強中 :2023/10/31(火) 18:07:34.89 ID:E4NBBiQq0HLWN.net
単語ごとに韻を踏む ライミングディクショナリーはwebにいくつもありますが

フレーズ単位で印を探してくれるサイトはないでしょうか?おうちゃくすぎますか?

It's been a whileというフレーズ全体を印を踏んだいくつかの単語で言い換えてくれるサイトです

658 :名無しさん@英語勉強中 :2023/10/31(火) 18:07:41.48 ID:E4NBBiQq0HLWN.net
単語ごとに韻を踏む ライミングディクショナリーはwebにいくつもありますが

フレーズ単位で印を探してくれるサイトはないでしょうか?おうちゃくすぎますか?

It's been a whileというフレーズ全体を印を踏んだいくつかの単語で言い換えてくれるサイトです

659 :🤭英ちゃん🫢 (中止W 1386-ihTc):2023/10/31(火) 18:39:56.12 ID:KhVUd5/S0HLWN.net
そーゆー韻の踏み方って英文学史上誰がやったの🤔

660 :名無しさん@英語勉強中 (中止 9b76-CP9B):2023/10/31(火) 22:13:56.01 ID:1gZKcTLv0HLWN.net
Vallentrate managers client's accounts more_____ than most other brokerage firms.

a)conservative
b)conservatively

答えばbですが a ではいけない理由を文法的に教えてください

661 :ブタ耳 (中止W 2997-9vZe):2023/10/31(火) 22:20:37.16 ID:nOZGAZQ70HLWN.net
>>658
脚韻を踏む行を生成すれば良いのだとすれば、これはAIが得意とする課題だと思う。
とりあえずこういうプロンプトを考えた。

You are a poet

# first line
"It's been a while"
This is one line of a poem.

# Step 1
Add another line.
The next line must rhyme at the end.

# Step 2
Repeat Step 1 ten times.

662 :ブタ耳 (中止W 2997-9vZe):2023/10/31(火) 22:22:05.91 ID:nOZGAZQ70HLWN.net
ChatGPT (GPT-3.5):

"It's been a while,"
A distant, fading smile.

"It's been a while,"
Moonlight on the Nile.

"It's been a while,"
A secret, hidden aisle.

"It's been a while,"
In your eyes, I'd reconcile.

"It's been a while,"
Lost in thoughts, versatile.

"It's been a while,"
A journey mile by mile.

"It's been a while,"
In nature, we defile.

"It's been a while,"
In memories, we compile.

"It's been a while,"
Life's mysteries beguile.

663 :名無しさん@英語勉強中 (中止 e19b-czOC):2023/10/31(火) 22:37:21.96 ID:jny0W5wP0HLWN.net
>>660
おかしいと思ったら書き写し間違い酷すぎ。
ケアレスミスじゃなく、こういう写し間違いをする時点で文を根本的に理解できてない証拠。

664 :名無しさん@英語勉強中 (中止 e19b-czOC):2023/10/31(火) 22:43:40.31 ID:jny0W5wP0HLWN.net
Vallentrade manages clients' accounts more (   ) than most other brokerage firms.

動詞accountsの修飾で副詞conservatively一択。

665 :ブタ耳 (中止W 2997-9vZe):2023/10/31(火) 22:43:59.62 ID:nOZGAZQ70HLWN.net
>>661
改良版
https://www.phind.com/search?cache=gqmpb25nyj40xadbj1sc5daf

666 :ブタ耳 (中止W 2997-9vZe):2023/10/31(火) 22:56:01.52 ID:nOZGAZQ70HLWN.net
>>664
動詞はmanagesでしょ

667 :名無しさん@英語勉強中 (中止 e19b-czOC):2023/10/31(火) 22:57:31.59 ID:jny0W5wP0HLWN.net
>>666
ああ、そうだった。失敬。

668 :ブタ耳 (中止W 2997-9vZe):2023/10/31(火) 23:04:20.49 ID:nOZGAZQ70HLWN.net
>>667
managersをmanagesに訂正した人がその点分かってないはずかないことは分かっていましたが念のため

669 :名無しさん@英語勉強中 (中止 b19b-AkzC):2023/10/31(火) 23:34:14.11 ID:roylCdnQ0HLWN.net
good size

The room is a good size. [quite large] (English Collocations in Use Advanced)

[SIZE] The kitchen is a good size (= not small). (OALD10)
[GOOD] [only before noun] great in number, amount or degree
The kitchen is a good size. (OALD10)

That living room is a good size. あの居間はちょうどいい大きさだ (W4)
The living room is a good [fair, nice] size. その居間はかなりの大きさだよ (W4)

どういうことなんだぜ?

670 :名無しさん@英語勉強中 (中止 d197-nwkA):2023/10/31(火) 23:35:50.20 ID:qlr9nb360HLWN.net
「1日おきに」というのは「2日ごとに」と同じ意味ですから
every second day や every 2 days とするのは分かるのですが
一番よく使われる every other day だと、この場合の other とは
どのような意味を示して「1日おきに / 2日ごとに」となるのでしょうか?

671 :名無しさん@英語勉強中 (ワッチョイW 1ba5-A5qO):2023/11/01(水) 00:49:33.17 ID:ujESSNW40.net
PHRASE
If something happens every other day or every second day, for example, it happens one day, then does not happen the next day, then happens the day after that, and so on. You can also say that something happens every third week, every fourth year, and so on. 1日おきに、 起こる時期について ~おきに
▶ I went home every other week.
私は1週間おきに帰宅した.

今日じゃないother dayということかな?
every second dayと並列されてるので

672 :名無しさん@英語勉強中 (ワッチョイ 0996-c8RC):2023/11/01(水) 00:58:42.18 ID:i29xBKrt0.net
教えてください。

(1) The Thames would not be the fairyland it is without its flower-decked locks.
(2) Really good talk is one of the greatest pleasure there are, and yet how rarely one comes across it.

(1)のit is、(2)のthere areは、文法的には同じ働きだと思うのですが、どういうことかわかりません。
どういう意味があるのでしょうか?その後に何か省略しているのでしょうか?

673 :The OED Loves Me Not :2023/11/01(水) 03:26:58.92 ID:UoUz198V0.net
>>670
すでによい回答が他のサイトにあります。

https://oshiete.goo.ne.jp/qa/13636730.html

https://oshiete.goo.ne.jp/qa/1869729.html

674 :The OED Loves Me Not :2023/11/01(水) 03:48:34.91 ID:UoUz198V0.net
>>672
「英文解釈教室」か何かの参考書に出ていた英文でしょう?そういうものについては、
そのまま検索すれば誰か別の人たちが何度もすでに質問し、回答しているのです。
あちこちで似たような回答があると思いますが、たとえば

https://detail.chiebukuro.yahoo.co.jp/qa/question_detail/q1284732892

https://detail.chiebukuro.yahoo.co.jp/qa/question_detail/q13115469913

私は真面目には読まず斜め読み(走り読み)しかしていませんが、おそらくは
この上のようなサイトに正しそうな回答があるはずです。それを読んで、それでも
納得がいかなかったら、再び質問しなおしてください。

675 :The OED Loves Me Not :2023/11/01(水) 04:20:07.19 ID:UoUz198V0.net
>>669
good という言葉があまりによく使われてきたので、意味の範囲がどんどん
広がってきて、そのうちに、はたから見れば矛盾しているように見える
ことも出てくるということでしょう。

英語だけじゃなくて、日本語でも、「その言葉を使ったとき、A という意味じゃなくて
B という意味で使ったんだよ」と言い直さないといけないこともあるでしょう?
相手は B という意味で解釈し、自分は A という意味で使っていた。
それなのに喧嘩が続いてしまうということがよくあります。

676 :英語職人 :2023/11/01(水) 09:14:12.70 ID:s4Y+P+pd0.net
>>672のpleasureは、可算名詞で使われているので、「pleasures」と複数形にするのが正しいよ。

677 :名無しさん@英語勉強中 (ワッチョイ 5328-Q0z2):2023/11/01(水) 12:02:38.01 ID:GSyZ4kLP0.net
Really good talk is one of the greatest pleasures there are, and yet how rarely one comes across it!
https://detail.chiebukuro.yahoo.co.jp/qa/question_detail/q10114344035

ベストアンサー

質問者からのお礼コメント

なるほど、ありがとうございます。m(_ _)m
勉強頑張りますm(_ _)m

678 :名無しさん@英語勉強中 (ワッチョイ e19b-iHz2):2023/11/01(水) 12:13:42.36 ID:fFOMG+4V0.net
one of the greatest pleasures there ISじゃないの?

679 :名無しさん@英語勉強中 (ワッチョイ 5387-Q0z2):2023/11/01(水) 16:29:34.39 ID:GSyZ4kLP0.net
あそこに複数の人たちが立っていて、そのうちの一人
One of the people who are standing over there is Ms. XYZ.
あそこに複数の人たちがいて、立っているのは一人
One of the people who is standing over there is Ms. XYZ.

実際に複数の娯楽が存在し、そのうちの一つ
XYZ is one of the greatest pleasures there are, but ...
複数の娯楽の中で、実際に存在するのは一つ
XYZ is one of the greatest pleasures there is, but ...

680 :名無しさん@英語勉強中 (ワッチョイ e19b-iHz2):2023/11/01(水) 16:46:18.04 ID:fFOMG+4V0.net
文法の理屈以前にThe best there isの類は普通に会話で良く使う表現。

681 :名無しさん@英語勉強中 :2023/11/01(水) 17:24:54.54 ID:GSyZ4kLP0.net
the best there is の中で is が単数形なのは、the best が単数の意味だからです。

https://redkiwiapp.com/ja/english-guide/questions/hwu6arT4NmQs632L0Y4s
この「the best there is」の意味を教えてください。

ネイティブによる回答 (Rebecca)

「the best there is」とは、この仕事については最高の人材たちという意味です。

例: We're going to get the best performers there are for your wedding reception.(結婚式のレセプションのために最高のパフォーマー(複数)を集めるつもりです。)
例: Jane's the best there is when it comes to bakeries in this city. People visit from all over the country.(この街のパン屋なら、ジェーンのところが一番(単数)だ。国中の人が訪れているよ。)

682 :The OED Loves Me Not :2023/11/01(水) 17:35:24.25 ID:UoUz198V0.net
決して誰かを煽るつもりなんてないのですが、この下の理屈は変だと思います。

(1) あそこに複数の人たちが立っていて、そのうちの一人
One of the people who are standing over there is Ms. XYZ.

この (1) の英文とその和訳(解釈)は正しいと思います。
いっそのこと who are を消して、次のように書いたらすっきりすると思います。
One of the people standing over there is ...

(2) あそこに複数の人たちがいて、立っているのは一人
One of the people who is standing over there is Ms. XYZ.

これが変です。「あそこに複数の人たちがいて、立っているのは一人なんだったら、
上のような英文は書かず、次のように言うはずです。

The one standing over there is Ms. XYZ.

もしも、どうしても「あそこに(たくさんいる)人たちのうち、
立っているのが XYZ さんです」というふうにくどく説明したいのなら、
次のように言うはずです。
Of all those people there, the one standing over there is Ms. XYZ.
または
Among rthose people there, the one standing...

仮に (2) のような英文を書いている人がいるとしたら、それは one に
ひきずられて is と言ってしまっているだけであって、文法的に正しいのは
are のはずです。

(続く)

683 :The OED Loves Me Not :2023/11/01(水) 17:36:21.13 ID:UoUz198V0.net
(続き)

(3) 実際に複数の娯楽が存在し、そのうちの一つ
XYZ is one of the greatest pleasures there are, but ...

この上の (3) は正しいと思います。

(4) 複数の娯楽の中で、実際に存在するのは一つ
XYZ is one of the greatest pleasures there is, but ...

この英文と和訳(解釈)は変であり、(2) と同様に、
XYZ is one of the greatest pleasures there are, but...
と書くのが文法的に正しいはずであり、is と書いてしまっている
人がいれば、それは (2) で書いたことと同じ理由からそうしてしまった
だけだと思います。

そもそも、実際に存在しない pleasures についてここで書く必要はないはずです。
もしどうしても複数の娯楽の中で、実際に存在するのが一つなんだということが
言いたいのなら、

Of all the possible pleasures in the world,
XYZ is the one available now.
とか何とか言うはずです。

684 :名無しさん@英語勉強中 :2023/11/01(水) 17:45:33.77 ID:fFOMG+4V0.net
普通に主語の物が最高だという表現で、主語が単数ならbe動詞もisでそれを受ける主格補語も単数でそれにかかる関係詞もしかり。文法の理屈的に可能かどうかではなく、よく見る表現だからそれと違えば違和感を感じる。

https://www.google.com/search?q=%22is+one+of+the+greatest+pleasures+there%22&tbm=bks

685 :名無しさん@英語勉強中 :2023/11/01(水) 17:53:08.37 ID:fFOMG+4V0.net
原文これか:

Arthur Christopher Benson, From a College Window

“Really good talk is one of the greatest pleasures there is, and yet how rarely one comes across it!”

686 :名無しさん@英語勉強中 :2023/11/01(水) 20:22:31.45 ID:QZA+bsIS0.net
すみませんが、以下の文を当たり障りの内容に英語に訳してもらえないでしょうか。
よろしお願いします。

「この為、すみませんが、商品の提供のも申し出は結構でございます。」

687 :名無しさん@英語勉強中 :2023/11/01(水) 20:35:57.77 ID:XWUArm7B0.net
日本語でおk

688 :672 :2023/11/01(水) 21:55:42.42 ID:i29xBKrt0.net
>>674
ありがとうございました。承知しました。

689 :名無しさん@英語勉強中 :2023/11/01(水) 22:51:59.13 ID:GSyZ4kLP0.net
>>686

この為、すみませんが、商品の提供の申し出は結構でございます。

SmartJ: すみませんが、この理由で商品の提供の申し出はお断りします。

We apologize for the inconvenience, but for this reason
we cannot accept your offer to provide the product.

690 :英語職人 :2023/11/02(木) 00:06:07.48 ID:DkPUTeu90.net
>>686

For that reason, I'm afraid there is no need for the offer to provide the product.
(そういうわけで、申し訳ございませんが、商品の提供の申し出は結構でございます。)

※ I'm afraid SVで、「あいにく(申し訳ございませんが)、SはVなのです。」

691 :名無しさん@英語勉強中 (ワッチョイ 29a9-Q0z2):2023/11/02(木) 01:03:37.96 ID:32x0ipUn0.net
>>689 >>690
ご回答有難うございます。
組織へのご返答の場合は>>689いいですが、すみませんが今回私自身のお断りの
ご返答なので690がいいですね。皆さんの英訳参考になりました。

692 :名無しさん@英語勉強中 :2023/11/02(木) 11:12:08.52 ID:gshwSn3M0.net
この為、すみませんが、商品の提供の申し出は結構でございます。
SmartJ: すみませんが、この理由で商品の提供の申し出はお断りします。

この為、すみませんが、商品の提供の申し出は結構でございます。
<すみませんが今回私自身のお断りのご返答なので>
SmartJ: すみませんが、この理由で商品の提供の依頼はお断りします。

I apologize for the inconvenience, but for this reason
I cannot accept your request to provide the product.

693 :名無しさん@英語勉強中 :2023/11/03(金) 11:05:44.56 ID:18zRlabn0.net
pinterestにはられてる英文を簡単に翻訳する方法ないでしょうか?
pdf化してなんとかっていうのやってみたけど
pdfは通常のdeepLで翻訳できませんよね?

694 :名無しさん@英語勉強中 :2023/11/03(金) 11:34:45.24 ID:XezhA/y80.net
>>693
「簡単」がどの程度のものなのかは知らんけど、pdfなら一旦Google Driveに上げてGoogle Docsで開ければOCRでテキスト化してくれるから、その後は翻訳処理でもなんでも普通にできる。

695 :🤭英ちゃん🫢 (オッペケ Srdd-ihTc):2023/11/03(金) 12:05:07.03 ID:vIPZZfbqr.net
スマホで写真撮って、Google translate とかDeepL にアップロードすれば🤔

696 :名無しさん@英語勉強中 (ワッチョイ 0996-H9h+):2023/11/03(金) 12:28:01.41 ID:5o+wdNiu0.net
英語圏では知らぬ者がないほど有名なスカイリムの冒頭シーン、主人公が頭を乗せる
石の台のことを何と言うのでしょう?
辞書にはblockとしか出てこないようなので、もしかすると特別な名称は無かったりする?

ちなみに日本語訳はちゃんとあって「断頭台」というものですが

697 :The OED Loves Me Not (ワッチョイ eb89-H9h+):2023/11/03(金) 13:10:22.25 ID:7VlcRC6g0.net
>>696
私は Skyrim なんてまったく知りませんでしたが、

skyrim "put his head on it" the stone block

というキーワードで検索したら、いきなり

https://www.fanfiction.net/s/10756789/11/A-Song-For-Skyrim

上のような webpage が出てきて、それを見ると

chopping block
put his head on it

なんてことを書いているから、もしかして「断頭台」に相当する
英語表現はこの chopping block じゃないでしょうかね?

698 :ブタ耳 (ワッチョイW 29f7-9vZe):2023/11/03(金) 13:27:01.53 ID:RTTaA3xG0.net
AIに聞いてみた。

Prompt:
You are a professional executioner in the medieval era who is good at swinging your sword.

Now you are going to cut off the head of a prisoner.
You place his head on a heavy rock, which supports the head temporally before the moment of death.
What is the name of the rock?

ChatGPT (GPT-3.5):
The rock used to support the prisoner's head temporarily before execution is often referred to as a "block" or "executioner's block."

phind (GPT-4):
https://www.phind.com/search?cache=izgsvklomgwe4napwdw6hq4l

要約
the execution blockに似てる。
ただしこれはたいてい木製だった。

699 :ブタ耳 (ワッチョイW 29f7-9vZe):2023/11/03(金) 13:31:12.92 ID:RTTaA3xG0.net
promptの
"temporally"は"temporarily"に訂正したい。

700 :ブタ耳 (ワッチョイW 29f7-9vZe):2023/11/03(金) 13:39:42.32 ID:RTTaA3xG0.net
"execution block"
"executioner’s block"
で画像検索すると多数ヒットする。

ほとんど木製。

「あ、これ石だ」と思ったらまさにスカイリム
https://www.nexusmods.com/skyrimspecialedition/mods/62963

701 :ブタ耳 (ワッチョイW 29f7-9vZe):2023/11/03(金) 13:42:19.04 ID:RTTaA3xG0.net
スカイリムのも、よく見ると木製だね。

702 :名無しさん@英語勉強中 :2023/11/03(金) 16:14:30.05 ID:5o+wdNiu0.net
>>697-701
みなさんありがとうございます
その辺までは出てきたんですが、書き手によって表現が揺れるということは
あくまで各人が一般名詞を組み合わせて表現しているに過ぎず、固有の名称と
いうわけではないのかな?と思った次第です。
とりあえず英語圏ではblockの部分を共通に、ある程度自由に呼ぶようですね

703 :名無しさん@英語勉強中 :2023/11/03(金) 17:24:30.24 ID:HbviyS690.net
ちなみに悪魔将軍の地獄の断頭台はHell's Guillotineだった
https://i.imgur.com/48YcdA2.png

704 :名無しさん@英語勉強中 :2023/11/03(金) 17:34:13.82 ID:VJgn9z7P0.net
ソースキン肉マンは流石に草www

705 :名無しさん@英語勉強中 (アウアウウー Saa5-dFLh):2023/11/04(土) 01:15:27.86 ID:91LX15wTa.net
質問
英語教室に通ってるけど頻繁に生徒に聞き返される。なぜ?
皆「ばっと、あいどん~」って発音してるけど自分は「バライドン~」って発音してる
発音が雑すぎるのかな
早く上達したくて、先生の真似してるけど聞き返されると「あ、下手なんだ自分」と落ち込む
先生に聞いてみたいけど、聞き方がわからない

706 :名無しさん@英語勉強中 (ワッチョイW 6d0d-2HkR):2023/11/04(土) 01:38:36.43 ID:wIp9N68D0.net
>>705
状況がよくわからんけど、先生には
Do you understand my pronunciation “バライドン〜”?
My pronunciation isn’t understood by them(生徒) very often.
とでも聞いてみればいいんじゃないの

707 :名無しさん@英語勉強中 (ワッチョイW 6d0d-2HkR):2023/11/04(土) 01:40:00.79 ID:wIp9N68D0.net
ああ、生徒多人数で先生ネイティブみたいな状況なのか

708 :名無しさん@英語勉強中 (ワッチョイW fd42-kfR0):2023/11/04(土) 01:51:17.81 ID:lftzbePp0.net
大意
オレ以外の生徒の発音悪すぎ耳悪すぎ勘弁してくれよ

709 :名無しさん@英語勉強中 (ワッチョイW 651c-dFLh):2023/11/04(土) 02:19:14.96 ID:YOF367aG0.net
>>706
プロナンチエイション?
大人数なクラス
先生と個人的に話してみたいけど緊張。帰り際に一言話しかけるだけで汗ダラダラ。

>>708
自分も自信ないんだけどでも上達したい
けど聞き返されるたび授業止まるから自分が足引っ張ってる感じになっておりどうしようかなとオモタ
先生から注意は受けてない
けどクラスで浮きそうな気がしてる
なんていうか調和を取るべきなのか欲望のままがつがつ勉強するべきか

710 :The OED Loves Me Not :2023/11/04(土) 04:30:09.92 ID:GWi91P9R0.net
>>705
発音に関して本気で質問したいなら、
自分の発音している様子を出来ればビデオ
(だめなら音声ファイル)で投稿して、
みんなに聴いてもらうしかありません。

どうしてもそれが嫌なら、発音に関しては質問しても
まったく無駄だと思った方がいいです。

さらに、もしあなたが英語を勉強し始めたのが
12歳の時で、今は20歳を超えている
(あるいはもしかして30歳)なんだったら、
発音を直そうとしてもほとんど無駄です。

ある程度に歳を取ってから発音を直そうと
毎日のように練習しても、練習すればするだけ
余計に発音が悪くなります。

以上が、私の周囲にいる人たちの発音練習の
様子を見た上での私自身の感想です。

「夢をつぶすな」と言われるかもしれませんが、
事実は事実です。

歳を取ってから発音を直すのvが本当に無理だとしたら、
あとは読み書きに徹した方がいいと思います。

711 :ブタ耳 :2023/11/04(土) 08:55:48.23 ID:Jxe/JRxg0.net
>>710
>毎日のように練習しても、練習すればするだけ
>余計に発音が悪くなります。

それは練習の仕方がまずいからでしょう。
カタカナ英語レベルで話してるような人が、シャドウイングやったりするのはまずい練習の典型です。

正しい練習の仕方をすれば、ネイティブ並みとまではいかないまでも、ネイティブに誤解されないレベルの発音技術は獲得できると考えます。

一例を挙げると、
practiceを/purakutisu/のように発音してる人の発音を/prǽktis/に近づけることは充分可能です。

712 :The OED Loves Me Not (ワッチョイ 2e89-wdyj):2023/11/04(土) 10:18:55.92 ID:GWi91P9R0.net
He was a long time in healing.
この場合の was は、took という意味だそうだ。

.... we were a long time finding the place.
上のような英文が、私の今さっき読んでいた
Don Delillo, The Names
という小説に出て来たのだった。これに似た phrase は
ときどき出てくるのだが、10年くらい前に見かけた時は戸惑った。

よく考えてみると、
Don't be long.
と言えば「あんまり手間取らないで、早く帰って来いよ」
という意味ですよね。だからこのときもやはり、be は take と
同じ意味だと思っていいわけですよね。

さらには
It was a long time before this thing happened.
っていうときの It was にしても、It took と解釈すれば簡単に理解できますよね。

そんなことに、今になってやっと気づいた。(馬鹿な私ね〜。)

というわけで、OED Online を引いてみたら、ジャンジャンそれに似た
用例を紹介しています。40個くらいの用例を紹介しているので、
そのうちのごく一部だけをここに引用します。

(続く)

713 :The OED Loves Me Not (ワッチョイ 2e89-wdyj):2023/11/04(土) 10:20:22.80 ID:GWi91P9R0.net
(続き)
take
II.7.
c1175–
With an adverb or adverbial phrase of time indicating the period that elapses in the course of an activity or process: to occupy, use up, or take (a certain amount of time). Frequently with before, in, gerund, or †infinitive.
to be all day: see to be all day at all adj. A.1e. to be long: see long adv.1 I.ii.6.


1825
We were thirty days in making Antigua.
C. Waterton, Wanderings in South America iv. 275Citation details for C. Waterton, Wanderings in South America
1862
Town-Officer is some considerable time before he can return.
T. Carlyle, History of Friedrich II of Prussia vol. III. xii. iv. 211Citation details for T. Carlyle, History of Friedrich II of Prussia
1906
The mail-trains would be hours loading up at the depot of the little Kansas town.
U. Sinclair, Jungle xxx. 390Citation details for U. Sinclair, Jungle
1972
Excuse me, Albert, but I must see to the plumbing. Won't be a minute.
G. Bell, Villains Galore iii. 31Citation details for G. Bell, Villains Galore
1998
It could even be a named op that's been months in the planning.
R. Newman, Manners 226Citation details for R. Newman, Manners
2005
It's been a long time coming and there's still no firm release date.
Digit October 16Citation details for Digit

(OED Online)

714 :名無しさん@英語勉強中 (ワンミングク MM92-aLKc):2023/11/04(土) 10:51:51.06 ID:0NgrwsWuM.net
>>705
そのままでいいと思うけど、それ先生が悪いよ
他の人がそういう発音してるのよしとしてるんでしょ?
場所変えたら?w

715 :ブタ耳 (ワッチョイW 6d9f-SP+M):2023/11/04(土) 12:41:45.12 ID:Jxe/JRxg0.net
そのままでいいかどうかは実際の発音を聞いてみないと分からない。
「自己流のねいてぃぶ風発音」より、日本語訛り丸出しのカタカナ発音の方が伝わりやすい
場合もままある。

716 :名無しさん@英語勉強中 :2023/11/04(土) 13:43:29.49 ID:CQucF6F/0.net
>>712
be 時間 (in) doing
という構文は昔から気になってたのですが
be動詞がtakeの意味を表わすのは不思議ですね。

717 :名無しさん@英語勉強中 :2023/11/04(土) 13:49:03.21 ID:GlYw+vzK0.net
>>705
「ばっと、あいどん~」より「バライドン~」みたいな発音の方が「うまい」という認識がそもそも根本的に間違っている。

718 :名無しさん@英語勉強中 (ワッチョイW 6d1d-2HkR):2023/11/04(土) 13:53:09.31 ID:wIp9N68D0.net
先生「バライドンスィンクソゥ。バライドンイズベラープロナンスィエイシュン」

719 :名無しさん@英語勉強中 (ワッチョイW 6515-dFLh):2023/11/04(土) 14:32:28.38 ID:0wWLikry0.net
>>714,715,717,718
先生はネイティブでとてもゆっくり話してくれて、大多数に合わせてる感じで悪いとか悪くないって感じではない

私のスタンスは、まあ文法や発音なんて間違っててもいいからとりあえず声出してみよう!という心持ちで通ってるんだけど
自分が勘違いしてそうだから、よかったらぜひ正しい考え方を教えてくれませんか?

なんか何度も聞き返されるたびに気持ちがしゅんとなり、尻込みしてきました。
発音どうこうというより、感情面のご相談がメインなんだと今自覚しました。

720 :名無しさん@英語勉強中 :2023/11/05(日) 06:57:00.16 ID:WaVjMT+D0.net
ここで聞いていいかわからないのですが
イギリス英語得意な方
訳お願いできますか
https://youtu.be/3W72qk7fYWA?si=J3WBmjOmAKSLow_U
3.17からの井上選手のところからです

721 :名無しさん@英語勉強中 :2023/11/05(日) 07:02:06.81 ID:WaVjMT+D0.net
文字起こしアプリではなくブリティッシュ得意な方聞き取っていただけると嬉しいです
すいませんわがままいって

722 :名無しさん@英語勉強中 :2023/11/05(日) 08:50:58.85 ID:9UFnSM3Z0.net
「ジョジョの奇妙な冒険」で英語を学ぶ!で

You trying to humiliate me? 生き恥をかかせる気か て文がありました

trying to は調べたら be trying to てのはありましたが trying to 単独て用法は見当たりませんでした
これってbe動詞が省略てことでしょうか?

723 :The OED Loves Me Not :2023/11/05(日) 11:50:19.88 ID:m8B3Npv90.net
>>722
>>You trying to humiliate me?

Are you trying to humiliate me?
文頭の Are が省略されているのです。このように英語、特に口語英語においては、
文法の1語または2語が省略されることがよくあります。

The problem is that S V.
これを口語ではよく
Problem is, S V.
というふうに省略します。文頭の The だけではなく、that も省略し、さらには comma をつけるのです。

The trouble is that S V.
も同じで、
Trouble is, S V.

The bottom line is that S V.
も同じで、
Bottom line is, S V.
となることが多いようです。

Have you ever been to Paris?
この文頭の2語を省略して
Ecver been to Paris?
と言うことが多いです。

(続く)

724 :The OED Loves Me Not :2023/11/05(日) 11:50:43.83 ID:m8B3Npv90.net
(続き)
同じように、
Do you ever think of...?
というとっきに、Ever think of ...?
とかいうふうに、やはり2語を省略するのです。

Did she come here today?
これも
She come here today?
とかいうふうに言います。

かといって、何でもかんでも文頭の1語か2語を省略していいかというとそういうわけではなさそうで、
Do you, will you, have you, are you など、省略してもすぐに頭の中で補えそうなものだけを省略します。
さらに、The や A という冠詞が冒頭にあれば、それを口語では片っ端から省略してしゃべっているようです。

さらには、たとえば
It's neat.
と言いたいとき、英語ネイティブは
'S neat.
というふうに言うことが多いです。これは私たちには「スニート」二しか聞こえませんが、そのように
聞えたら実はこれは It's neat. のことなのだと脳内で補充しないといけないのです。

このような省略については、文法書の「省略」または Ellipsis という項目に詳しい解説が載っていることがあります。
たとえば、
Quirk et al., A Comprehensive Grammar of the English Language
という2,000ページくらいある英語で書かれた文法書が有名ですが、その巻末には数十ページもこの Ellipsis についての
解説があります。

725 :名無しさん@英語勉強中 :2023/11/05(日) 12:56:55.30 ID:rU/jYFMX0.net
She come here today?
彼女は今日ここに来ますか?来ましたか?
曖昧な表現?
She came here today?
彼女は今日ここに来ましたか?

726 :The OED Loves Me Not :2023/11/05(日) 13:06:27.43 ID:m8B3Npv90.net
(1) She come here today?
>>彼女は今日ここに来ますか?来ましたか?
>>曖昧な表現?

この英文を見れば、
(1) Did she come here today?
(2) Has she come here today?
(3) Does she come here today?
この三つのうちのいずれかでしょうけど、置かれた状況を考えたら
そのうちのいずれが正しいかはわかります。状況がなくて英文だけが
空中に浮かぶなんてことは、日本人の脳内だけで起こることです。


(2) She came here today?
こういう言い方を、普通はしないと思います。
She came here today, didn't she?
She came here today. Right?
とかいうふうに言うはずです。そしてそういう状況にあることが
わかっているなら、
Did (OR Has) she come here today?
とも言うはずです。

727 :The OED Loves Me Not (ワッチョイ 2e89-wdyj):2023/11/05(日) 15:22:58.22 ID:m8B3Npv90.net
「今日、彼女は来たか?」と尋ねたいとき、
She come today?
ならば
Has she come today?
Did she come today?
Will she come today?
などいろいろ考えることができて曖昧なので、

Has she come yet today?
を少し省略して
She come yet today?
と言いそうな気がします。

現に
Have your parents come yet today?
https://www.google.co.jp/books/edition/Extended_Love/gMSNDwAAQBAJ?hl=en&gbpv=1&dq=%22come+yet+today%22&pg=PT12&printsec=frontcover

上のような用例が、リンク先の13行目あたりに出てきます。yet がついているから、
Have または Did あたりが省略されているのだろうとすぐにわかるからこそ、
Your parents come yet today?
というふうに省略できるのだと思います。

(続く)

728 :The OED Loves Me Not (ワッチョイ 2e89-wdyj):2023/11/05(日) 15:23:30.83 ID:m8B3Npv90.net
(続き)
普通は話者と聴き手の置かれた状況が互いにわかっているので、
少しくらい省略しても意味が曖昧になるはずがありません。
しかしそれでも意味がどうしても曖昧になるときには、
省略しなかったらおしまいなのです。

いずれにしても、どのような言い方をしたらどういう状況で使えるのかなんていう
机上の空論は無駄だと思います。すべての英文にはその置かれた状況の中で
使われるのですから、状況(文脈)も説明しないで
英文だけを取り上げてその意味や文法を論議することが日本人の間で流行っていますが、
その質問をそのままネイティブに投げかけると、馬鹿にされます。
そんなの当たり前ですよね。言語はそれが使われている状況と切り離しては考えられないからです。

729 :名無しさん@英語勉強中 (ワッチョイ 4989-9NWm):2023/11/05(日) 15:40:33.40 ID:9UFnSM3Z0.net
>>723,724
そういえば疑問文でしたね
You と tryingの間のbe動詞が省略?とこんなんもあるんかと思ってました
詳しい解説ありがとうございました

730 :名無しさん@英語勉強中 (ワッチョイ 06e5-/9BA):2023/11/06(月) 23:35:07.75 ID:bXtD6ik40.net
As far as the reccomended restaurants, I have been to the three listed on the Web site.

これのas far as の用法ですが、後ろが名詞句になっています
as far as I know〜 みたいに SV にならずに名詞句の場合

〜の範囲では という意味になり 推薦されるレストランの範囲では〜、という和訳でいいでしょうか?
それとも As far as the reccomended restaurantsの中で、SV などが省略されているのでしょうか?

731 :The OED Loves Me Not (ワッチョイ 2e89-wdyj):2023/11/07(火) 00:11:10.51 ID:sQuYxje10.net
>>730
As far as 名詞句, S V.
の意味合いは、前後関係から見てわかるように、
As far as 名詞句 is concerned, S V.
の is concerned, が省略されただけでしょうね。
それ以外に考えられますか?

あくまでその
言い回しが使われている用例を片っ端から読んでいって
自分なりに判断すればいいんじゃないでしょうか?

732 :名無しさん@英語勉強中 (ワッチョイ 3145-ubk5):2023/11/07(火) 06:28:12.82 ID:MRJS2tx50.net
次のas をつかった構文について質問です。
Ms. Ku's command of French is excellent, as is her to interact with other employees.
クーさんのフランス語の能力は高く、ほかの社員と交流する能力にも優れている。
解説には as is S で、Sも〜。と書いてあります。
疑問点1.このas の品詞と意味が分かりません。
疑問点2.asの後ろの倒置は省略(excellentという形容詞の省略)発生による倒置でしょうか?
以上2点についてわかりやすい解説をお願いします。

733 :The OED Loves Me Not (ワッチョイ 2e89-wdyj):2023/11/07(火) 07:08:01.33 ID:sQuYxje10.net
>>732
>>Ms. Ku's command of French is excellent, as is her to interact with other employees.

ただの typo です。

Ms. Ku's command of French is excellent, as is her ★ability★ to interact with other employees.

★の部分を補いさえすれば、高校1年生でもわかる標準文法通りの、しかもおそらくは自然な英文になります。

734 :ブタ耳 :2023/11/07(火) 10:30:55.24 ID:+b3W8E6K0.net
接続詞andの並列関係についての質問です。

Migrant farm workers in the United States — the majority of whom are Latino ★and★ too often living in comparably crowded, unsanitary quarters — may similarly end up at the center of the next Covid-19 hot spots.

この文のダッシュで囲まれた挿入部分中にある and は、どの部分を並列しているのでしょうか?

A and B という形でAとBが並列されるとして、以下の(1)と(2)の解釈、どちらが適切でしょう?

(1)
A: Latino
B: too often living in comparably crowded, unsanitary quarters

(2)
A: the majority of whom are Latino
B: too often living in comparably crowded, unsanitary quarters

出典:
https://amp.cnn.com/cnn/2020/04/23/opinions/farm-workers-vulnerable-coronavirus-sepkowitz/index.html

735 :名無しさん@英語勉強中 :2023/11/07(火) 11:46:01.74 ID:O7h6kqMI0.net
She discreetly slipped the surprise gift into his bag without him noticing.

without his noticengではなく
without her doing
without me doing

というのが文法的に正しいのでしょうか?
解説サイトなどがあれば教えてください

736 :名無しさん@英語勉強中 :2023/11/07(火) 11:48:44.36 ID:O7h6kqMI0.net
>>731
as far as A is concerned
これは知りませんでした、ありがとうございました

737 :The OED Loves Me Not :2023/11/07(火) 11:55:43.84 ID:sQuYxje10.net
>>734
私にはちょっと難しい質問ですが、いろいろと考え直した末、
(2) つまり

(2)
A: the majority of whom are Latino
B: too often living in comparably crowded, unsanitary quarters

これが正しいと思います。なぜなら、もし (1) が正しいとすると

The majority of them are:
- Latino
and
- too often living in (... places).
ということになりますが、それが正しいとすると、

They are too often living in (... places).
という文が正しいものとして成立しないといけないことになります。しかし
私が思うには、too often と進行形とは両立しないような気がするのです。
それを裏付ける参考文献などを示すことは私にはできません。私が
直感的にそう感じているだけです。

(続き)

738 :The OED Loves Me Not :2023/11/07(火) 11:56:20.74 ID:sQuYxje10.net
>>734 への回答の続き

(?) They are too often [doing something].
は不自然で、
They are always [doing something].
They always do something.
They too often do something.
ならば自然だと私は感じています。

しかし (2) ならば

the majority of whom are Latino
too often living in (... quarters).
この二つが両方とも形容詞句みたいにその前の
Migrant farm workers in the United States
にぶら下がっていて、too often living in (... quarters) という
形は形容詞句としては (... ing) の形で正しいのだということになります。

739 :ブタ耳 :2023/11/07(火) 12:00:09.13 ID:+b3W8E6K0.net
>>735
動名詞の意味上の主語
https://www.eibunpou.net/08/chapter21/21_3.html

without his noticing
without him noticing
どちらも可。
上記サイトの説明によれば、後者の方が口語的。

740 :The OED Loves Me Not :2023/11/07(火) 12:05:49.79 ID:sQuYxje10.net
>>735
without him noticing
これは、もともとは
「彼が気づくことなしに」
ということだから
without his noticing
つまり
without + 所有格 (his, my, your, their, our, etc.) + 動名詞
が主流だったけど、そのうち
without + 目的格 (him, them, us, me, etc.) + 動名詞
が会話やくだけた文章の中では頻繁に使われるようになった
だけのことです。

つまり
without him noticing
withouit his noticing
この両方が現代では大いに使われているのです。そういうことは、
たくさん英文を読んだり聴いたりしていれば、自然とわかってきます。

741 :The OED Loves Me Not :2023/11/07(火) 12:06:54.68 ID:sQuYxje10.net
>>740 は、ブタ耳さんと被りました。
Cross-posted with Pig's Ears.

742 :The OED Loves Me Not :2023/11/07(火) 12:16:32.21 ID:sQuYxje10.net
>>734
"are too often [doing something]" が不自然だと思っていたけど、
頭を冷やして検索してみると、そういう用例がいくらでも見つかることがわかりました。

Such families ★are too often living★ on hope, with a profound faith in the ever advancing world of medical technology to keep loved ones alive with little thought to the consequences.
https://pubmed.ncbi.nlm.nih.gov/16961105/

こういう用例は、おそらくは例の有名な
He's always arguing...
などというのと同じく、話し手のいら立ちを示すのだろうと私は感じています。

というわけで、ブタ耳さんの質問に対しては、
「両方とも正しいと思います」
と言っておきたいと思います。

743 :ブタ耳 :2023/11/07(火) 12:27:54.87 ID:+b3W8E6K0.net
>>737
レスありがとうございました。
私も(2)の解釈をとります。

majority とoftenは、思考の流れ的に類似したものがあって、この二つが並列される方が自然だろうと考えました。
日本語で言うならば、
移民労働者、
A: 彼らの「大多数が」〜
B: 彼らは「多くの場合に」〜

(1)の解釈をとると、移民労働者の集合を二重に限定していくことになります。
例えば、移民労働者の総数が100人だとすると、それがAで90人に絞られ、Bで更に80人に絞られる。
こういう直列的な思考の流れもあり得るだろうと考えての質問でした。

too often と現在進行形be livingについて今調べみたところ、こういう例が見つかりました。
COVID-19 presents unique risks to displaced people, who are often living in densely populated settlements where communicable diseases can spread quickly.
https://www.unhcr.org/media/covid-19-displacement-and-climate-change

744 :ブタ耳 :2023/11/07(火) 12:35:01.73 ID:+b3W8E6K0.net
この進行形は、一時的に住んでいる、仮の住まいである、というニュアンスかと思います。移民労働者なので。

745 :The OED Loves Me Not :2023/11/07(火) 12:41:11.69 ID:sQuYxje10.net
>>743-744
なるほど。あれから考え直して、ブタ耳さんの言う通りだと考えなおしております。

746 :名無しさん@英語勉強中 :2023/11/07(火) 13:03:09.02 ID:mVsy9aTw0.net
>>734

(1)だと思います。

〜 [are Latino] and [(are) (too often) living in 〜 ]

あるいは、

are [Latino and (too often)living in 〜 ]

という構造だと考えられるので。

747 :名無しさん@英語勉強中 :2023/11/07(火) 13:23:02.68 ID:VoJBXQ4j0.net
almost all of の直後には定冠詞のthe か 所有格の語を必ず置きますが、
almost all の場合にはそれらはなくてもあってもどちらでもかまわないと
考えていいでしょうか?

748 :名無しさん@英語勉強中 :2023/11/07(火) 15:58:46.97 ID:P2BgSf5s0.net
アルクの教材の評価ってここだとどんなものですか?
アルクの日本語学習の教材使って良くないと言ってる人を見たんですが、アルク出版自体がもともと微妙なんですか?
ネットでレビューみる限り悪い意見はあまり見ないですが。。。

749 :The OED Loves Me Not :2023/11/07(火) 16:18:17.54 ID:sQuYxje10.net
>>747
そういうことを自分で調べたければ、簡単に調べられます。
この下の2種類のグラフを見て、ご自分で判断し、
次回からは自分でこのグラフ機能を使って調べてください。

(1) https://books.google.com/ngrams/graph?content=almost+all+the+countries+are%2Calmost+all+countries+are&year_start=1800&year_end=2019&corpus=en-2019&smoothing=3

(2) https://books.google.com/ngrams/graph?content=all+countries+are%2Call+the+countries+are&year_start=1800&year_end=2019&corpus=en-2019&smoothing=3

750 :名無しさん@英語勉強中 :2023/11/07(火) 17:19:31.98 ID:qp0gsUOf0.net
買収されて終わったよ

751 :名無しさん@英語勉強中 :2023/11/07(火) 18:13:50.39 ID:P2BgSf5s0.net
>>750
見てみた所、5年前に買収されたり3年まえに譲渡されてたり忙しいようですね。。

752 :英語職人 :2023/11/07(火) 21:40:47.50 ID:3CiEWOvF0.net
>>746 Good opinion! 🤠 746が正解です。

論拠:
① A and BのABは、同じ品詞であるのがふつうです。the majorityとtoo oftenでは品詞が異なります。
② この文は、ラテン系以外の移民もラテン系と同じように「混雑した不衛生な場所で暮らしている」と主張していません。中国やインドからの裕福な移民は当然この中には含まれないと考えるのが自然です。「混雑した不衛生な場所で暮らしていることが頻繁に見られる」のは移民の大半を占めるラテン系移民のみの特徴です。

753 :英語職人 :2023/11/07(火) 21:43:51.27 ID:3CiEWOvF0.net
追記
ここは、
「are Latino and (are) too often」と考えることによって、AとBの品詞(are)が一致します。

754 :名無しさん@英語勉強中 :2023/11/07(火) 22:01:05.06 ID:MYHAbfAz0.net
>>750
ほんとだ。ありがとう。
それでイングリッシュジャーナル休刊か

https://www.itmedia.co.jp/business/spv/1808/21/news100.html

755 :名無しさん@英語勉強中 :2023/11/07(火) 22:37:27.72 ID:1x1IB8uq0.net
えーと文章的にはラティーノと生活場所の話はどっちも労働者の説明をしてるよ

くどい言い方になるけど分かりやすく書き直すとこんな感じ
the majority of whom are Latino.
the majority of whom are too often living in comparably crowded, unsanitary quarters.

だから「労働者の多くはラティーノ」で「労働者の多くは良くない環境で暮らしてる」って言ってるかな
and too oftenの後の部分はラティーノの説明じゃないって言ったら分かるかな

記事全体をみてもラティーノの環境を切り出してそこにフォーカスしてる記事じゃないよね

756 :英語職人 :2023/11/07(火) 23:16:54.71 ID:3CiEWOvF0.net
>>755 その通りだね。 ちょっとボケてました。 

訂正版
>>746 Good opinion! 🤠 746が正解です。

論拠:
A and BのABは、同じ品詞であるのがふつうです。the majorityとtoo oftenでは品詞が異なります。
   are Latino and (are) too oftenと考えれば、品詞(are)は一致します。よって、「are Latino」と「 (are) too often」の並列だと考えられます。

だから、ここは、「the majority of whom are Latino」and 「the majority of whom are too often living in comparably crowded, unsanitary quarters」の2つの部分が同じ立場で、「Migrant farm workers in the United States」の意味を補足説明していると考えるのが正しいというわけだ。

757 :名無しさん@英語勉強中 :2023/11/08(水) 00:05:16.14 ID:uS8u2KUv0.net
>>734 の(1)の並列関係が適切だと考えることは可能ですが、その場合、are を補う必要はない、というか勝手に補って考えちゃだめでしょう。

the majority whom are
A:Latino
and
B: too often living in comparably crowded, unsanitary quarters

Aは形容詞。Bは形容詞句。ABともに the majority whom (=S) are (=V) に続く (C)であり、too often の前に and を補わなくても並列関係は成立します。

あと、”the majority" と "too often" の品詞が異なるから並列不可だという理屈は噴飯ものです。たぶん言葉足らずなだけなのかもしれませんが。

758 :ブタ耳 :2023/11/08(水) 00:15:44.65 ID:uS8u2KUv0.net
>>734の(2)の場合は、
A: the majority of whom are Latino
B: too often living in comparably crowded, unsanitary quarters
Aは挿入的な形容詞節。Bは挿入的な形容詞句。
形容詞節と形容詞句で、ともに挿入的に前の "Migrant farm workes in the United States" を説明しているのだから、並列関係は成立するはずだと思うのですが。

759 :ブタ耳 :2023/11/08(水) 00:28:51.28 ID:uS8u2KUv0.net
「品詞が異なるものは and で並列できない」のかということに関して、以下、安藤「現代英文法講義」から引用 (p.587)。

*****
等位に連結されるものは, 基本的に名詞句と名詞句, 前置詞句と前置詞句のように, 同一の統語 [まれに意味] 範疇でなければならない。
(6) の2例は, and で連結されている語句が同一の範疇ではないので, 非文法的である。

(6) a. *John is [clever] and [a salesman]. [形容詞と名詞句]
b. *Mary is [a schoolgirl] and [in the garden]. [名詞句と前置詞句]

次の例は, 副詞と前置詞句が等位接続されているのだろうか.

(7) We're here and out of that bloody place. [Hemingway, A Farewell to Arms]
(われわれは, ここにいて, あのいまいましい場所から抜け出している)

いや, そうではない. here は, 統語的には in this place という意味の前置詞句の代用形 (pro-form) であり, 意味的には, here も前置詞句も, ともに「場所」を表していると見ることができるからである。
*****

この(7)の説明は今一つ腑に落ちない。
ともに場所を表している [副詞]と[副詞句(あるいは前置詞句)]が並列されている、と考えればそれで良いのではないだろうか?                               
            

760 :ブタ耳 :2023/11/08(水) 00:35:39.05 ID:uS8u2KUv0.net
"John is [clever] and [a salesman]" は確かに不自然だと感じるが、次のような文はどうだろうか?

? He was [a professional wrestler] and [gentle to all his fans]. [名詞句と形容詞句]

この文は文法的に間違っていないように感じられるのだが…

この文が正しいとすれば、次のような文も可能なのでは?

? Antonio Inoki, [a professional wrestler] and [gentle to all his fans], died in 2022.

761 :ブタ耳 :2023/11/08(水) 00:44:46.58 ID:uS8u2KUv0.net
こういう問題に関してAIに正誤判断をゆだねるのは危険なのですが、とりあえず尋ねてみました。
https://www.phind.com/search?cache=cr76re0zs8pcfyymepuqnd1v

なかなかおもしろいやりとりになっていて、例文もいろいろ出してくれてますので、興味のある方は参照してみてください。

連投スマソ

762 :ブタ耳 :2023/11/08(水) 01:27:32.05 ID:uS8u2KUv0.net
>>757
自己レス。
あ、Bは形容詞句じゃないな。現在進行形 are living の一部で、 これに too often が割り込んだものと見るべきだろうな。
まあ実質、形容詞句とみなしても問題ないようにも思うが。

しかし、現在進行形の一部だとすると、「品詞の違うものは並列できない」という立場からは、この並列は成立しないね。
あ、だから、are を補って考えたのかな。

しかし、補っちゃだめだよね。もともと書かれてないんだから。

763 :英語職人 :2023/11/08(水) 07:17:37.83 ID:0E2EYnWk0.net
>>757
え?>>756が理解できないの? ちょっとびっくり 😲

areは勝手に補ったんじゃなく、>>734の文では、もともとあったareが省略されているのです。それを復活させてあげただけなのです。
「the majority of whom are Latino and the majority of whom are too often living in comparably crowded, unsanitary quarters」が本来の形であり、その後半の 「the majority of whom are」が冗長さを避けるために省略されてしまっているのです。

>>755(外人さんぽい)や>>746の言うとおり、756が正しい解釈です。 あきらめましょう 🤠

764 :英語職人 :2023/11/08(水) 07:37:08.20 ID:0E2EYnWk0.net
>>760

「Antonio Inoki, a professional wrestler and gentle to all his fans, died in 2022.」は正しいよ。

これは、「Antonio Inoki, who was a professional wrestler and who was gentle to all his fans, died in 2022.」の「who was」が省略されたものだと考えることができるからだ。そしてこれは、「who was a professional wrestler」と「who was gentle to all his fans」の並列と考えることができる。

He was a professional wrestler and gentle to all his fans.も正しく、これはHe was a professional wrestler and (was a) gentle to all his fans. の(  )内の省略と考えることができるので、品詞(are)は一致すると考える。

>>762 「もともと書かれてない」ではなくて、「もともと書かれていたものが省略されていた」と発想しましょう🤠

765 :英語職人 :2023/11/08(水) 07:40:39.09 ID:0E2EYnWk0.net
訂正
He was a professional wrestler and gentle to all his fans.も正しく、これはHe was a professional wrestler and (was a) gentle to all his fans. の(  )内の省略と考えることができるので、品詞(was)は一致すると考える。

766 :ブタ耳 :2023/11/08(水) 08:12:56.14 ID:i7kVW7xk0.net
そんな風に補って並列関係を成立させられらなら、
John is [clever] and [a salesman].
も、
John is [clever] and [(is) a salesman].
あるいは更に補って、
John is [clever] and [(he is) a salesman].

と考えて、正しい文だと言えてしまうのでは?

andの並列関係を考えるときには、勝手に語を補っちゃダメなんです。

767 :ブタ耳 :2023/11/08(水) 08:54:26.76 ID:i7kVW7xk0.net
"John is clever and a salesman.”
が文法的に正しいのかどうか、AIに尋ねてみた結果、「正しい」との返答。
ツッコミを入れてもやっぱり「正しい」と。
https://www.phind.com/search?cache=ncp9gh6wizxyxxs2v7nhurxg

繰り返すが、こういう問題の正誤判断をAIに委ねるのは危険。

768 :名無しさん@英語勉強中 :2023/11/08(水) 09:07:13.69 ID:STs8zu5l0.net
The storm caused him late.
は口語だとありそうですけど、間違いなんですか?
cause him to be lateは知ってるのですがーcause him lateがないかと思っての質問です

769 :ブタ耳 :2023/11/08(水) 10:44:31.33 ID:i7kVW7xk0.net
John is clever and a salesman. が不自然に感じられるのは、安藤が言うように and の並列関係が非文法的だからではなく、単にこの文の内容が too primitive であるからであるような気がしてきた。

770 :名無しさん@英語勉強中 :2023/11/08(水) 10:52:02.93 ID:GHT4TLdS0.net
>>768
文法的には間違ってますね

771 :英語職人 :2023/11/08(水) 11:16:10.28 ID:0E2EYnWk0.net
再度訂正
He was a professional wrestler and gentle to all his fans.も正しく、これはHe was a professional wrestler and (was) gentle to all his fans. の(  )内の省略と考えることができるので、品詞(was)は一致すると考える。

772 :英語職人 :2023/11/08(水) 11:34:56.65 ID:0E2EYnWk0.net
>>767
"John is clever and a salesman.”は正しいよ。John is clever and is a salesman.のisが省略されたものと考えられるから。
ブタ耳ちゃんは、なにか根本的に勘違いをしている。↑上記のようにisを補って同じ品詞になるなら、andで結べるのだよ。

証拠
https://books.google.co.jp/books?id=cnUWBAAAQBAJ&pg=PT19&lpg=PT19&dq=%22is+talented+and+a+born+performer%22&source=bl&ots=lRYU194YKT&sig=ACfU3U0GoRtwCZwcTbrzi2yHaouq6l-qKw&hl=ja&sa=X&ved=2ahUKEwjCg7L7rLOCAxVBCYgKHVx_BAMQ6AF6BAgIEAM#v=onepage&q=%22is%20talented%20and%20a%20born%20performer%22&f=false

このページの3段落目に「is talented and a born performer. とある。
このようなandの使い方は、ネット上に無数にあるよ。
実際の英語を見てきてるAIが"John is clever and a salesman.”を間違いだと言うはずがない。あきらめましょう。🤠

773 :英語職人 :2023/11/08(水) 11:40:37.70 ID:0E2EYnWk0.net
「主語 is 形容詞 and a 名詞」が許容される証拠 その2 ↓

https://books.google.com/ngrams/graph?content=is+intelligent+and+a%2C+is+beautiful+and+a&year_start=1800&year_end=2019&corpus=en-2019&smoothing=3

774 :名無しさん@英語勉強中 :2023/11/08(水) 11:43:52.31 ID:zOyu9i+r0.net
>>762
形容詞句の挿入ではなく、現在進行形の一部だとするなら
(1)の解釈とならざるを得ないのでは?
主語はthe majority of whomですねよ?

私は(1)だと思います。
形容詞句だと解しても、関係詞節と形容詞句が並列の関係というのは無理があると思う。

学校文法では、現在進行形は通常SVCとは分類されませんが、
ネイティブの感覚では
She is very cute.

She is living in tokyo.
も構文的には同じようなものとして使っていると思う。
つまり、現在進行形の現在分詞を形容詞のような感覚で使っている。
(実際、現在進行形を第二文型あるいはそれに準じるものと分類する本がある。高校総合英語『FACTBOOK』、『英文法を哲学する』、『5文型の底力』)

775 :ブタ耳 (ワンミングク MM92-SP+M):2023/11/08(水) 11:57:52.80 ID:0KWpPoNEM.net
John is clever and a salesman. が非文法的だと言ったのは安藤(>>759)ですよ。

最初、私は「この文は非文法的てある」という安藤説を素直に受け取って考察を進めていました。
確かに非常に不自然に感じられる。
しかし、それは安藤が言うように形容詞と名詞句が並列されているからではなく、単に文の意味内容が primitive であるからではないかと、疑い始めたところです。

この文の並列関係が正しいとすれば、それは形容詞と名詞句が共に補語として機能し、述語動詞 is を共有しているからであって、「後者に is を補えば is が同じ品詞となるから」という理由ではないと、考えています。

776 :ブタ耳 (ワンミングク MM92-SP+M):2023/11/08(水) 12:11:26.08 ID:0KWpPoNEM.net
>>774
(1)の解釈をとると、too often living は現在進行形の一部であり、これは仰るように、形容詞句が補語になっていると捉えても良いかもしれない。

(2)の解釈をとると、too often living 以下は形容詞句。
確かに、関係代名詞の形容詞説と、too often 以下の形容詞句が並列するのは多少違和感がある。
しかし、
Migrant farm workers, the majority of whom are Latino, may end up …
とも、
Migrant farm workers, too often living in crowded quarters, may end up …
とも、言えるとすれば、
Migrant farm workers, the majority of whom are Latino and too often living in crowded quarters, may end up …
の並列関係を(2)のようにも解釈できるのではないか、ということです。

777 :名無しさん@英語勉強中 (ワッチョイ c584-HiwA):2023/11/08(水) 12:21:12.76 ID:zOyu9i+r0.net
同じように挿入として機能するにしても
やはり関係代名詞節と現在分詞句を並列に結ぶのはとても違和感がある。

778 :746 :2023/11/08(水) 12:30:36.35 ID:1aeudSBH0.net
1.
>>757
> 〜 Aは形容詞。Bは形容詞句。ABともに the majority whom (=S) are (=V) に続く (C)であり、too often の前に and を補わなくても並列関係は成立します。

それは確かにそうです。
そのことは、自分も、>>746の後の方の構造図で示しました。

いずれにしても、Latino と(too often)living が述語動詞の are とワンセットと
なることは同じです。
文章の意味の解釈上は、前と後の構造図のどちらでも同じことになるので、自分はあまり
気にしませんでしたが。

2.
それとちょっと気になることが。

>Bは形容詞句

とおっしゃっているが、自分の考え方から言えば、living は現在進行形の一部、live
という動詞がたまたま進行形を作るために変化した形にすぎない。動詞の変化形にすぎない。
これを形容詞句と表現するやり方に自分は慣れていないが、現在学校教育の現場では
こういう説明の仕方が標準的なものになっているのだろうか。進行形の ing の部分
だけを形容詞句と説明するやり方は珍しくはないのだろうか。

「ちょっと気になる」というのは、自分が立てたスレで文法・構文の解説をすることが
あり、その際に、自分の従来の説明の仕方を変えた方がいいのかなと考えたりするから
です。

と、ここまで書いてスレを見直すと、>>774もこの点を取り上げていた。

779 :名無しさん@英語勉強中 :2023/11/08(水) 12:40:44.10 ID:zOyu9i+r0.net
>>778
>進行形の ing の部分だけを形容詞句と説明するやり方は珍しくはないのだろうか。

ブタ耳さんが、形容詞句と言っているのは
現在分詞の形容詞的用法のことで、これは昔も今も学校ではそう教えていると思います。

780 :746 (ワッチョイ c17f-ubk5):2023/11/08(水) 13:25:33.12 ID:1aeudSBH0.net
>>779

そうですか。
うーん。
しかし、従来の教え方では、現在分詞の形容詞的用法と進行形は別々のものとして
捉えているように思います。
進行形とは「be動詞 + 現在分詞の形容詞的用法」のことである、みたいな説明の
仕方は聞いた覚えがありません。 (´·ω·`)

まあ、別にそういう教え方はそれはそれでかまわないと思いますが、自分が人に
説明する際に、今の人がなじみがあるのはどちらであろうかというのが自分の疑問です。
>>774の言葉を借りれば「現在進行形を第二文型あるいはそれに準じるもの」として
教えている学校が多いのだろうか。

ま、大した問題じゃありませんから、無視していただいて結構です。

781 :746 (ワッチョイ c17f-ubk5):2023/11/08(水) 13:30:33.49 ID:1aeudSBH0.net
ちなみに、この英文を実際に読んでいく際は、

〜 the majority of whom are Latino and too often living in 〜 を読んだ瞬間に、
are が 「Latino と living をつなぐ」と即座に感じます。living だけであると
意味をなさないからです。
(あるいは、即座に、and の後に 「are があったけど、直前に are があって重複に
なるから省略した」と判断します。and の後で前と重複する語句を省略するのはごく
普通のこと。大学受験英語でもおなじみです)

また、
A 〜 〜 〜 〜 〜 B and C 〜 〜 〜 .
というような文があった場合、ここの and はBとCをつなぐと感じるのが自然な反応
でしょう。
このCは and によってAとつながれているのだとは普通考えません。
(名詞語句を列挙する A, B, C and D などはまた別の話です)
Aとつなぐつもりなら最初から
A and C 〜 〜 〜 〜 〜 B . とするでしょう。
接続詞の and などは「近接するものをつなげる」のが普通です。

だからこそ、上の部分を読んだ瞬間に are [Latino and(too often)living]と
即座にひとまとまりに捉えて読むことになります。

782 :ブタ耳 (ワッチョイW 6d3f-SP+M):2023/11/08(水) 14:06:27.35 ID:i7kVW7xk0.net
>>781
私の場合、この英文を実際に声に出して読む際には、"Latino" の直後に少し間を置き、>>734 (2)の並列関係が際立つように音読する方が、リズム的なバランスの上でも、思考の流れのスムーズさにおいても、より自然であるように感じます。

後半の、
A ~ ~ ~ ~ ~ B and C ~ ~ ~ .
という説明は、いささか的を外しています。

734の(2)は、単純に示すならば、
S,A and B,V ~
という並列関係で、AとBが共にSを説明している、というだけのことです。

783 :名無しさん@英語勉強中 (ワッチョイ c584-HiwA):2023/11/08(水) 14:09:51.07 ID:zOyu9i+r0.net
>>780
うまく伝わってないようです。

現在でも、現在分詞の形容詞的用法と進行形は別々のものとして
教えていると思います。
「現在進行形を第二文型あるいはそれに準じるもの」として
教えている学校もほとんどないでしょう。
(『FACTBOOK』を採用している学校は、そういう捉え方もあるくらいの解説をしているかと想像します)

で、ブタ耳さんが「形容詞句」と言っているのは
the boy standing here のような、現在分詞の形容詞用法のことを言っているのでしょう。

で、それに対して私が
進行形を「be動詞 + 現在分詞の形容詞的用法」と捉える見方もあると
主張しているということです。

784 :ブタ耳 (ワッチョイW 6d3f-SP+M):2023/11/08(水) 14:17:22.93 ID:i7kVW7xk0.net
皆さん、>>759で引用した安藤の説についてはどう思われますか?
特に、
John ls clever and a salesman.
という文を、「形容詞と名詞句が並列しているから文法的に正しくない」とする説についての賛否をお聞かせいただきたい。

785 :746 (ワッチョイ c17f-ubk5):2023/11/08(水) 14:38:21.06 ID:1aeudSBH0.net
>>782
> 〜 という説明は、いささか的を外しています。
>734の(2)は、単純に示すならば、
>S,A and B,V 〜
>という並列関係で、AとBが共にSを説明している、というだけのことです。

うーん。それは英文全体を読み直してから話ですね。
自分が言っているのは、この英文を読み進めている最中での感覚的な捉え方を
言っているんです。

文法的、理屈的には複数の考え方が存在するでしょう。
しかし、ネイティブがこの文章を実際に読んでいく際には、上記のように
捉えていくであろうということです。

また、
>〜 "Latino" の直後に少し間を置き 〜
とおっしゃっていますが、それなら、直後にコンマを打ちたいところです。

786 :746 (ワッチョイ c17f-ubk5):2023/11/08(水) 14:46:14.63 ID:1aeudSBH0.net
>>783
丁寧なレスありがとうございます。

>現在でも、現在分詞の形容詞的用法と進行形は別々のものとして
>教えていると思います。
>「現在進行形を第二文型あるいはそれに準じるもの」として
>教えている学校もほとんどないでしょう。

そうですか。それを聞いて安心しました。

787 :ブタ耳 :2023/11/08(水) 15:08:31.51 ID:i7kVW7xk0.net
>>786
その件についてですが、20年ほど前、ネイティブの子供向けの英語学習テキストで、"He is singing." のような現在進行形を "He is happy." のようなbe動詞の後に形容詞が来るパターンと同じユニットで扱っているものを見たことがあります。
このテキストは日本の教育現場では使いにくいだろうな、と思うと同時に、ネイティブはやはりこういう感覚で進行形を捉えているのか、といささか目の覚めるような思いもしたので、記憶に残っています。
テキストのタイトルは忘れました。同様の子供向けテキストは、もしかしたらまだ流通しているかもしれません。

788 :名無しさん@英語勉強中 :2023/11/08(水) 15:20:58.97 ID:QXaWRbvl0.net
>>784
Is the following sentence natural? John is clever and a salesman.
ChatGPT
ChatGPT
The sentence "John is clever and a salesman" is grammatically correct but may sound a bit unnatural or awkward. The conjunction "and" is typically used to link phrases or clauses of similar grammatical structure, but in this sentence, it links an adjective ("clever") with a noun phrase ("a salesman"), which can be jarring to the reader or listener.

A more natural way to express the same idea could be:

"John is a clever salesman."
"John is both clever and a salesman."
"John is clever as well as being a salesman."
These alternatives maintain the intended meaning while improving the sentence's flow and coherence.

789 :名無しさん@英語勉強中 :2023/11/08(水) 15:23:07.71 ID:QXaWRbvl0.net
文法的には正しいが、少し不自然でぎこちない。

790 :ブタ耳 :2023/11/08(水) 15:45:50.33 ID:i7kVW7xk0.net
>>788
それ、GPT-4ですか? それともGPT-3.5ですか?

791 :ブタ耳 :2023/11/08(水) 15:49:02.21 ID:i7kVW7xk0.net
GPT-4でしょうね。
GPT-3.5は、もっと素っ気ないです。

ChatGPT (GPT-3.5):
The sentence "John is clever and a salesman" is grammatically correct, but it may be more natural to say "John is clever and works as a salesman" to clarify that being a salesman is his occupation.

792 :名無しさん@英語勉強中 :2023/11/08(水) 17:38:12.35 ID:QXaWRbvl0.net
>>791
GPT-4です。

793 :英語職人 :2023/11/08(水) 19:59:34.75 ID:0E2EYnWk0.net
実際、「was beautiful and a」で検索するとごまんとヒットするからね。
https://www.instagram.com/p/CqJG5A-PEks/?hl=ja&img_index=1

ここに「Mexico City was beautiful and a ton of fun!」とある。

794 :英語職人 :2023/11/08(水) 20:00:36.20 ID:0E2EYnWk0.net
ChatGPTに聞いてみよう。
質問 Is the following sentence natural?

Mexico City was beautiful and a ton of fun!


回答 ChatGPT
Yes, the sentence "Mexico City was beautiful and a ton of fun!" is natural and grammatically correct. It effectively conveys the speaker's positive experience in Mexico City.

795 :英語職人 :2023/11/08(水) 20:02:38.13 ID:0E2EYnWk0.net
https://twitter.com/MsPattiPatti/status/1305663601752133633

ここに Last night was beautiful and a memory I will forever treasure!  とある。
(deleted an unsolicited ad)

796 :英語職人 :2023/11/08(水) 20:04:09.80 ID:0E2EYnWk0.net
ChatGPTに聞いてみた。
質問 Is the following sentence natural?

Last night was beautiful and a memory I will forever treasure!


回答 ChatGPT
Yes, the sentence "Last night was beautiful and a memory I will forever treasure!" is natural and well-formed. It conveys a clear and positive message about a memorable experience.

797 :英語職人 :2023/11/08(水) 20:11:06.42 ID:0E2EYnWk0.net
>>793 >>795 に違和感を感じるほうが間違っているのである。 ChatGPTは、自然で良い文だとほめてるぞ。🤠

つまり、省略されている語(ここではwas)が一致していれば、形容詞と名詞をandで結んでも許容される。それは下記↓からも明らかである。
https://books.google.com/ngrams/graph?content=is+beautiful+and+a%2Cwas+beautiful+and+a&year_start=1800&year_end=2019&corpus=en-2019&smoothing=3

正確には、「was beautiful」と「was 名詞」という同質のものを結び付けていると言えるのである。

798 :英語職人 :2023/11/08(水) 20:15:18.24 ID:0E2EYnWk0.net
>>784
その文が正しくないのは、ふつうは「He is a clever salesman.」というからだよ。なぜわざわざ2つに分ける?おかしいだろということ。🤠
andが原因ではなく、単語が問題。「is 形容詞 and (is) 名詞」の並列は、文法的に合法だよ。

799 :英語職人 :2023/11/08(水) 20:26:29.50 ID:0E2EYnWk0.net
■□■ まとめ ■□■
下記の&#9733;内のandは、なにとなにをつなげているか。
Migrant farm workers in the United States — the majority of whom are Latino &#9733;and&#9733; too often living in comparably crowded, unsanitary quarters — may similarly end up at the center of the next Covid-19 hot spots.

結論
「the majority of whom are Latino」と「the majority of whom are too often living in comparably crowded, unsanitary quarters」を結合している。冗長さを避けるため、後半の「the majority of whom are」は省略されている。この2つが同じ立場で、Migrant farm workers in the United Statesの意味を補足している。この2つは、同じ性質を持つ存在なので、andでつなげることができる。

ブタ耳ちゃんの「下記のAとBの並列である」という説は・・・
A: the majority of whom are Latino
B: too often living in comparably crowded, unsanitary quarters
両者が文法的に同じ立場に成り得ないので、つまり、「名詞句 VS 進行形の動詞部」なので、文法的にandを使って結びつけることができません。よって、残念ですが、間違いなのです。🤠

800 :ブタ耳 :2023/11/08(水) 20:36:55.33 ID:uS8u2KUv0.net
>>793
そのようにごまんとヒットする例は、「A and BのABは、同じ品詞であるのがふつうです(>>756)」という説に対する反証であるとは考えないのですか?
安藤は、より厳密に、「等位に連結されるものは, 基本的に名詞句と名詞句, 前置詞句と前置詞句のように, 同一の統語 [まれに意味] 範疇でなければならない。(>>759)」と規定していますが、そういう説に対する反証となるものだと私は考えますけど。

801 :ブタ耳 :2023/11/08(水) 20:48:48.28 ID:uS8u2KUv0.net
次の文の and は、どの部分を並列していると思いますか?
It would be an opportunity to do something quite new for me &#9733;and&#9733; in which I believed much more strongly than in our government's economic policy.

ある文法書からの引用です。
and について、興味深い記述がありますので順次報告していく予定。

802 :ブタ耳 :2023/11/08(水) 20:57:26.81 ID:uS8u2KUv0.net
>>799
>両者が文法的に同じ立場に成り得ないので、つまり、「名詞句 VS 進行形の動詞部」なので、文法的にandを使って結びつけることができません。

何か誤解されているようですが、>>734の(2)の解釈をとった場合、並列関係は「名詞句 VS 進行形の動詞部」などではなく、「形容詞節(関係代名詞節) VS 形容詞句(分詞の形容詞的用法)]
です。

803 :名無しさん@英語勉強中 :2023/11/08(水) 21:24:09.49 ID:6Hh1gbPs0.net
two weeks と a two weeksの違いを教えてください

804 :名無しさん@英語勉強中 :2023/11/08(水) 21:38:15.26 ID:STs8zu5l0.net
>>803
二週間が単位になっているんじゃない?

805 :ブタ耳 :2023/11/08(水) 22:33:55.81 ID:uS8u2KUv0.net
The Cambridge Grammar of the English Language (CamGEL)から引用していきます。
引用1 (p.1280)

■ Semantic equivalence
In the absence of special factors, a subclausal coordination is semantically equivalent to the corresponding clausal coordination.

[14]
[subclausal]
i a. There is a copy [<on the desk> and <in the top drawer>].
i b. They arrived [on <Tuesday> or <Wednesday>].
i c. He told me [<who she was> but <not what she wanted>].

[clausal]
ii a. <There is a copy on the desk> and <there is a copy in the top drawer>.
ii b. <They arrived on Tuesday> or <they arrived on Wednesday>.
ii c. <He told me who she was> but <he didn't tell me what she wanted>.

... 略

It can also be convenient to speak of [i] as a reduction of [ii], or of [ii] as an expansion of [i], with the understanding that &#9733;this does not imply that [i] is syntactically derived by ellipsis from [ii]&#9733;.

[i]を[ii]の短縮として、あるいは[ii]を[i]の拡大として説明することは便利であるかもしれないが、&#9733;これは[i]が[ii]から省略によって統語的に派生することを意味するものではない&#9733;ということを理解しておく必要がある。

806 :ブタ耳 :2023/11/08(水) 22:35:00.33 ID:uS8u2KUv0.net
引用2 (pp.1326-1327)
§3 The range of coordination: what can be coordinated with what

3.2 Coordination of unlike categories
In the great majority of cases, coordinates belong to the same syntactic category, but &#9733;a difference of category is generally tolerated where there is likeness of function.&#9733; This section surveys the main functions allowing coordinations of this kind.

多くの場合、等位接続詞によって並列される要素は同じ統語カテゴリーに属するが、&#9733;機能の類似性がある場合には、カテゴリーの違いは一般的に許容される。&#9733;

(a) Predicative complement
[13]
i It was [<extremely expensive> and <in bad taste>]. [AdjP + PP]
ii He became [<very forgetful> and <an embarrassment to his family>]. [AdjP + NP]

It is also possible to coordinate an AdjP, NP, or PP with a no-finite clause.
[14]
i He's [<in love> and <behaving quite irrationally>]. [PP + gerund-participal]
ii He is [<known to have a gun> and <likely to use it>]. [past-participal + AdjP]
iii This process [is <perfectly natural> and <to be welcomed>]. [AdjP + infinitival]

807 :ブタ耳 :2023/11/08(水) 22:37:20.08 ID:uS8u2KUv0.net
引用3 ( p.1328)

(§3-2の続き)
(d) Modifier in NP structure
&#9733;The most usual case involves post-head modi?ers: PPs, AdjPs, participial clauses, and ?nite clauses, as in [17i].&#9733; Mixed coordinations in pre-head (attributive) modi?erposition, however, are not common. As illustrated in [17ii], they tend to involve adjectives and nominals belonging within a single semantic set.

[17]
i a. They still won’t recommend grants for people [<over the age of65> or <who have retired>].
i b. She won in a match [<interrupted by showers> but <which lasted under an hour>].
i c. A man [<in singletand shorts> and <wearing a green baize apron>] ?nally appeared.
i d. It would be an opportunity to do something quite new for me and in which I believed much more strongly than in our government's economic policy. (>>801)

ii 以下省略

808 :ブタ耳 :2023/11/08(水) 22:42:44.88 ID:uS8u2KUv0.net
(>>807) OCRの読み取りミスが残っていたので修正

[17] i c. A man [<in singlet and shorts> and <wearing a green baize apron>] finally appeared.

[17] i d の並列関係の記述は省略してあります。考えてみてください。面白い問題では?

809 :ブタ耳 :2023/11/08(水) 23:25:46.79 ID:uS8u2KUv0.net
(>>807) 読み取りミスが残ってましたので修正。

&#9733;The most usual case involves post-head modifiers: PPs, AdjPs, participial clauses, and finite clauses, as in [17i].&#9733; Mixed coordinations in pre-head (attributive) modifier position, however, are not common.

810 :名無しさん@英語勉強中 (ワッチョイW 6dae-2HkR):2023/11/09(木) 01:36:41.48 ID:coLYre110.net
>>801
普通にquite newちゃうん

811 :ブタ耳 (ワッチョイW 6ddf-R+yP):2023/11/09(木) 01:43:47.41 ID:n/a9W7Ri0.net
>>810
A and B という形で並列される A と B が、それぞれこの文ではどこからどこまでかという問いです。

812 :The OED Loves Me Not (ワッチョイ 2e89-wdyj):2023/11/09(木) 01:56:27.34 ID:Jt/lCwEF0.net
>>808
ブタ耳さんは、OCR まで駆使しておられたのですね。

ともかくブタ耳さんと英語職人さんは、
これまでも、そして今回も、議論のレベルがとてつもなく高い。

頑張ってください。それから、お二人とも YouTube とか Twitter 上で
課金しながら情報を提供していけば、いくらでもお金が稼げることは間違いないですね。
ただ、おそらくはお二人ともそういう世俗的で下らないことには興味がないんでしょうね。

813 :The OED Loves Me Not (ワッチョイ 2e89-wdyj):2023/11/09(木) 02:00:34.12 ID:Jt/lCwEF0.net
>>803
two weeks は、ごく普通の誰でもわかる two weeks です。

a two weeks は、これだけを取り出しても埒があきません。
たとえば

Need to leave your current job but unsure how to write ★a two weeks notice letter★?
https://www.entrepreneur.com/growing-a-business/how-to-write-a-two-weeks-notice-letter/452707

のようになっているはずだから、質問者さんが抱えている文の全体を示さないと、回答者は答えたくても答えられません。

814 :ブタ耳 (ワッチョイW 6ddf-R+yP):2023/11/09(木) 02:02:35.67 ID:n/a9W7Ri0.net
例えば>>806の[14]iで、
He's in love ★and★ behaving quite irrationally.

★and★ によって並列される部分は、
A: in love
B: behaving quite irrationally
です。

これをCamGELでは、
He's [<in love> and <behaving quite irrationally>].
と表記しています。
(書籍では、<…>の部分は下線で示されています)

815 :名無しさん@英語勉強中 (ワッチョイW 6dae-2HkR):2023/11/09(木) 02:32:26.04 ID:coLYre110.net
>>811
quite new for me

in以下全部
ちゃうん

816 :ブタ耳 :2023/11/09(木) 10:09:36.28 ID:n/a9W7Ri0.net
>>815
正解です。
It would be an opportunity to do something [<quite new for me> and <in which I believed much more strongly than in our government's economic policy>].

A: quite new for me [形容詞句]
B: in which I believed much more strongly than in our government's economic policy [形容詞節(関係代名詞節)]
something A and B という形の後置修飾。

ここ(>>807)に記載されている「混淆タイプの後置修飾」の例文中に、次のような記載があってもおかしくないのでは。
[17] i e. Migrant farm workers in the United States — [<the majority of whom are Latino> and <too often living in comparably crowded, unsanitary quarters>] — may similarly end up at the center of the next Covid-19 hot spots.

と書きつつ、はたと自説に対する反論を思いつきました。

817 :ブタ耳 :2023/11/09(木) 10:16:16.39 ID:n/a9W7Ri0.net
? Antonio Inoki, [<a professional wrestler> and <gentle to all his fans>], died in 2022. (>>760)
仮にこの並列関係が成立しているとしても、次のように書いたらどうなる?

Antonio Inoki, who was a professional wrestler ★and★ gentle to all his fans, died in 2022.

これを、>>734と同様に、andの並列は(1)と(2)どちらが適切かと問うとしたら?
(1)
A: a professional wrestler
B: gentle to all his fans

(2)
A: who was a professional wrestler
B: gentle to all his fans

(2)の[形容詞節(関係代名詞節)] + [形容詞句] という並列も理屈上考えられなくはないかもしれない。
しかし、これは圧倒的に(1)が自然に感じられる。
これとまったく同様に、>>734でも(1)の方を圧倒的に自然だと感じるべきだったのではないか。

>>781>>785の指摘は、まさにこういう感覚のことを言っていたのではないか。

818 :英語職人 :2023/11/09(木) 10:18:07.95 ID:A7zIBVxt0.net
>>800
同じ品詞というより「同じ立場の存在」と言ったほうが正確でしたね。見た目「形容詞 and 名詞」でも、それが下記のようなケースでは「同じ立場」になるのです。

The park is beautiful and a good place to a good place to spend a holiday. ←これはなぜ文法的に正しいか?
それは、この文↑が次の文↓の省略形だからです。
The park is beautiful and it is a good place to a good place to spend a holiday.

結局は、文と文をつなげているのです。両者とも同じ立場ですね。

819 :英語職人 :2023/11/09(木) 10:19:31.09 ID:A7zIBVxt0.net
>>799が正しいのです。

ブタ耳ちゃんは、あきらめましょう 🤠

820 :ブタ耳 :2023/11/09(木) 10:27:13.72 ID:n/a9W7Ri0.net
>>818
>それは、この文↑が次の文↓の省略形だからです。

そういう考え方が間違っていることを裏付けるために、>>805でCamGELから引用しました。
>>805を100回読み直してください。特に★で囲った部分。

821 :英語職人 :2023/11/09(木) 10:33:51.34 ID:A7zIBVxt0.net
>>802
>何か誤解されているようですが、>>734の(2)の解釈をとった場合、並列関係は「名詞句 VS 進行形の動詞部」などではなく、「形容詞節(関係代名詞節) VS 形容詞句(分詞の形容詞的用法)]

「the majority of whom are Latino」って形容詞節じゃないでしょ。コンマに相当する横線がついているから、非制限用法だよね。非制限用法って、修飾しない用法だよね? なら、形容詞節じゃないでしょ。

これは、Lucy, my friend, is kind.のように同格として理解されるべき用法なのです。よって、「the majority of whom are Latino」は、名詞節として働いているのです。

822 :英語職人 :2023/11/09(木) 10:37:15.88 ID:A7zIBVxt0.net
■□■ まとめ 訂正版 ■□■
下記の&#9733;内のandは、なにとなにをつなげているか。
Migrant farm workers in the United States — the majority of whom are Latino &#9733;and&#9733; too often living in comparably crowded, unsanitary quarters — may similarly end up at the center of the next Covid-19 hot spots.

結論
「the majority of whom are Latino」と「the majority of whom are too often living in comparably crowded, unsanitary quarters」を結合している。冗長さを避けるため、後半の「the majority of whom are」は省略されている。この2つが同じ立場で、Migrant farm workers in the United Statesの意味を補足している。この2つは、同じ性質を持つ存在なので、andでつなげることができる。

ブタ耳ちゃんの「下記のAとBの並列である」という説は・・・
A: the majority of whom are Latino
B: too often living in comparably crowded, unsanitary quarters

実際には両者が文法的に同じ立場に成り得ないので、つまり、実際には「名詞節(同格) VS 進行形の動詞部」なので、文法的にandを使って結びつけることができません。よって、残念ですが、間違いなのです。🤠

823 :ブタ耳 :2023/11/09(木) 10:41:34.56 ID:n/a9W7Ri0.net
>>812
励ましの言葉ありがとうございます。

今回も、英語職人さんが執拗に反論してくださるおかげで、この論争を通じて私も非常多くのことを学び、知識を深めることができたように思います。
では、英語職人さんの方は、私が執拗に反論するせいで何かしら学んでいただけているのでしょうか? この点について甚だ疑問に感じざるをえないとしたら、その責任の一端は私にあるのかもしれませんね。

824 :ブタ耳 :2023/11/09(木) 10:45:10.12 ID:n/a9W7Ri0.net
>>821
>これは、Lucy, my friend, is kind.のように同格として理解されるべき用法なのです。よって、「the majority of whom are Latino」は、名詞節として働いているのです。

さすがに、それはないわー。
致命的な誤解ですよ。

825 :英語職人 :2023/11/09(木) 10:50:50.68 ID:A7zIBVxt0.net
>>820
>It can also be convenient to speak of [i] as a reduction of [ii],

「[i]を[ii]の短縮として説明することが便利である場合もあるが」という意味だよ。否定はしていないことに着目。
これは、『短縮としても説明可能だが、文法上厳密には別物として分類されるべきものである』と言っている。

そして、それは、今回の問題、つまり、「形容詞+名詞」がなぜ正しいかを説明するくだりではない。
「形容詞+名詞」がなぜ正しいかを説明するには、省略されていることを示すしかないのです。

あきらめましょう 🤠

826 :英語職人 :2023/11/09(木) 10:52:40.40 ID:A7zIBVxt0.net
>>824
非制限用法の関係詞節は、andなどで書き換えられるのだよ。つまり、修飾してないので、形容詞節だとする発想こそ、噴飯ものです。🤠

827 :ブタ耳 :2023/11/09(木) 10:52:54.59 ID:n/a9W7Ri0.net
┐(´д`)┌ヤレヤレ

828 :ブタ耳 (ワッチョイW 6ddf-R+yP):2023/11/09(木) 11:37:27.36 ID:n/a9W7Ri0.net
https://www.phind.com/search?cache=wp129bcsitsfdzytg5nkma7p

829 :746 (ワッチョイ c125-ubk5):2023/11/09(木) 11:49:39.50 ID:Vdh1TwvX0.net
>>787
そうですね。
自分も初めてこの考え方を知った時は「なるほど!」と思いました。

ただ、旧来の教え方でも間に合うと(自分には)感じられるので、
教え方を変えようという気にまではなりませんね。
学校の現場の大半では教えられていないのが現状なら、自分も当分、
従来の文法説明に則って解説を続けたいと思います。 (´·ω·`)

830 :746 (ワッチョイ c125-ubk5):2023/11/09(木) 11:52:41.97 ID:Vdh1TwvX0.net
>>781に補足。

英文を書く際は、暗黙の了解として以下のルールがあると考えられます。

1. 主語と述語はなるべく離さない。
2. 修飾語と被修飾語はなるべく離さない。
3. and などの接続詞でつなぐ2つの要素はなるべく離さない。

などです。
なぜなら、これらの要素を離すと意味が取りにくくなる、誤解が生まれる
余地が大きくなるからです。
円滑なコミュニケーションを図るにはこれらのルールは大事なものであり、
無意識にせよ多くの人がこれに従っています(また、英語でも日本語でも
作文の教本ではこれらが大抵、明確に推奨されています)。

ですから、>>781で書いたように、
「A 〜 〜 〜 〜 〜 B and C 〜 〜 〜 .というような文があった場合、ここの
and はBとCをつなぐと感じるのが自然な」(ネイティブの)反応であり、
第一感の反応でしょう。
この時、もし B and C が意味上おかしい、違和感があると感じた場合に
だけ、もっと離れた位置にあるAがCとつながれているのではないかと
考え直すことになります。
しかし、今回の文章では普通通り B and C で読んで意味的にすんなり通る
ので、A と結びつける動機は最初から生まれないでしょう。

831 :ブタ耳 (ワッチョイW 6ddf-R+yP):2023/11/09(木) 11:54:27.35 ID:n/a9W7Ri0.net
promptのミス修正
1)
"Antonio Inoki, who was loved by ★every one★, died in 2022."

Question about the function of who-clause.
In this sentence, how does the who-clause "who was loved by ★every one★" function?
Is it a noun clause or an adjective clause?

→ everyone

2)
If someone should say it is a noun clause and ★insist in★ this interpretation, how will you persuade him?

→ insist on または persist in

3)
If I have ★failef★ to persuade him with all the arguments above, then what can I do as a decent, reasonable person?

→ failed

832 :746 (ワッチョイ c125-ubk5):2023/11/09(木) 11:55:35.94 ID:Vdh1TwvX0.net
>>784
安藤貞夫氏は「 〜 基本的に 〜でなければならない」という言い方ですね。
これは、裏の意味は
「基本的にはそうであるべきであるが、実際にはこれに反した用例が多数
あるのはもちろんである」
ではないでしょうか。

また、同様に、「非文法的」という言葉遣いにしても、これは「文法的に
間違いである」と断定しているわけではないのでは?
だから、「非文法的」の意味は、「形式的整合性・文法的整合性を備えて
いない」程度のことなのではないでしょうか。

(忙しいので今日の書き込みはここまでとさせていただきます)

833 :ブタ耳 (ワッチョイW 6ddf-R+yP):2023/11/09(木) 12:01:18.91 ID:n/a9W7Ri0.net
>>830
ずいぶん遠回りしましたが、仰っていることがようやく腑に落ちました。ありがとうございました。

834 :ブタ耳 (ワッチョイW 6ddf-R+yP):2023/11/09(木) 12:06:06.60 ID:n/a9W7Ri0.net
>>832
なるほど。
しかし、そういう意図であったのなら、せめて反例をいくつか挙げておいて欲しかったですね。
CamGELほど網羅的に、とは言わないまでも。

835 :英語職人 ◆azN58fWNzw (ワッチョイ 09e0-0SSA):2023/11/09(木) 12:41:13.54 ID:A7zIBVxt0.net
ところで、ブタ耳ちゃんは、次のandの使い方が正しいと言っていると思っていいんですね?

"This is a good TV show, showing various celebrities and which is very popular."

836 :英語職人 ◆azN58fWNzw (ワッチョイ 09e0-0SSA):2023/11/09(木) 12:51:19.08 ID:A7zIBVxt0.net
■□■ まとめ 訂正版2 ■□■
下記の&#9733;内のandは、なにとなにをつなげているか。
Migrant farm workers in the United States — the majority of whom are Latino &#9733;and&#9733; too often living in comparably crowded, unsanitary quarters — may similarly end up at the center of the next Covid-19 hot spots.

結論
「the majority of whom are Latino」と「the majority of whom are too often living in comparably crowded, unsanitary quarters」を結合している。冗長さを避けるため、後半の「the majority of whom are」は省略されている。この2つが同じ立場で、Migrant farm workers in the United Statesの意味を補足している。この2つは、同じ性質を持つ存在なので、andでつなげることができる。

ブタ耳ちゃんの「下記のAとBの並列である」という説は・・・
A: the majority of whom are Latino
B: too often living in comparably crowded, unsanitary quarters
(Migrant farm workers in the United States←修飾←too often以下という説)

ブタ耳ちゃんは、「This is a good TV show, showing various celebrities and which is very popular.」が正しい英語だと言っているのと同様です。
しかし、こんなandの使い方は自然ではありません。🤠

837 :ブタ耳 (ワッチョイW 6d73-R+yP):2023/11/09(木) 14:01:59.11 ID:n/a9W7Ri0.net
The TV show "Antonio’s Round", featuring various celebrities and which was one of the most popular TV shows in Japan, had sadly come to an end due to the sudden death of Antonio. The show was famous for a legendary episode, widely broadcasted live and in which Antonio slapped Lady Dewi so hard that she was knocked down and lost her consciousness.

838 :名無しさん@英語勉強中 :2023/11/09(木) 15:04:04.00 ID:M95BcMIo0.net
It would be an opportunity to do something [<quite new for me> and <in which I believed much more strongly than in our government's economic policy>].

の例示は大変参考になりました。
形容詞句と関係代名詞節が並列に接続されるなんて知りませんでした。

それを知らずにこの文に接していたら、自分なら、コンマの代わりにandが使われたwhichの非制限用法だと誤読したと思います。

ところで、非制限用法の関係詞節って従属節じゃなく独立節と習うと思いますが、
形容詞節って言うのでしたっけ?
独立節かつ形容詞節って、変ですよね。

で、当該英文のように、挿入節の場合はどうなんだろう。
挿入節の場合は、さすがに独立節って言わないですよね?
混乱してきた・・・

839 :名無しさん@英語勉強中 :2023/11/09(木) 15:44:55.67 ID:coLYre110.net
>>806
これ見ると結構何でもありって感じだよね
そう考えていいのかしら?

840 :名無しさん@英語勉強中 :2023/11/09(木) 15:58:12.87 ID:1/VdLCDW0.net
>>839
横だが、受動態の be の場合は be を繰り返すことが
好ましい、または要求される、とも書いてあるな。

He was invited but was unable to accept.
He was insolvent and was dismissed.

そして、

*He got insolvent and sacked.
*He kept awake and listening for her.

などは容認されないらしい。

841 :名無しさん@英語勉強中 :2023/11/09(木) 16:14:18.39 ID:1/VdLCDW0.net
>>838
Patrice Talon, the president of Benin, won a second term in office after an election
marred by violence and in which popular opposition figures were barred from running.

のような分詞節と関係詞節の並列なら時折見掛けることがあるが、
形容詞句と関係詞節の並列は自分も余り見た記憶がない。

後置修飾の分詞節や形容詞句は「縮約関係詞節」だと主張する人たちもいるから、
このような並列は不合理ではないのかも知れない。

非制限用法の関係詞節は、同書では「補充(足)節」となっていて、
先行詞は anchor と呼ばれている。
樹形図では、anchor に tether されているような感じなっているが、
学校文法的にはどうなんだろう?

独立節だとされているとして、たとえそうだとしても、
どちらにせよ、関係詞節だけでは文になれないのだから、
形容詞節という分類でも良いような気がする。

842 :名無しさん@英語勉強中 :2023/11/09(木) 19:40:45.73 ID:/fwHXxoq0.net
・A is about the size of B
・A is twice the size of B
about も twice も the size (=名詞) を修飾する特殊なケースの副詞だと思いますが
・A is one-third the size of B
この場合のone-third も副詞でしょうか?

843 :🤭英ちゃん🫢 (ワッチョイW 4286-Kzch):2023/11/09(木) 20:23:43.19 ID:okkHB5o/0.net
about副詞
twice, one third限定詞じゃない

844 :英語職人 :2023/11/09(木) 21:53:23.05 ID:A7zIBVxt0.net
>>837 ソースは、日本人が書いた英語っぽいんだが🤠  ちゃんとネイティブが書いたというソースがあるものを出してみて。

私は、「現在分詞 and 関係詞節」のandの使い方は誤用だと思う。誰か、ネイティブで使っている例 (文法書や辞書で使っているものがあれば最高)を知ってるなら教えてほしい。

845 :英語職人 :2023/11/09(木) 21:55:47.39 ID:A7zIBVxt0.net
■□■ まとめ 訂正版3 ■□■
下記の&#9733;内のandは、なにとなにをつなげているか。
Migrant farm workers in the United States — the majority of whom are Latino &#9733;and&#9733; too often living in comparably crowded, unsanitary quarters — may similarly end up at the center of the next Covid-19 hot spots.

結論
「the majority of whom are Latino」と「the majority of whom are too often living in comparably crowded, unsanitary quarters」を結合している。冗長さを避けるため、後半の「the majority of whom are」は省略されている。この2つが同じ立場で、Migrant farm workers in the United Statesの意味を補足している。この2つは、同じ性質を持つ存在なので、andでつなげることができる。

ブタ耳ちゃんの「下記のAとBの並列である」という説は・・・
A: the majority of whom are Latino
B: too often living in comparably crowded, unsanitary quarters
(Migrant farm workers in the United States←修飾←too often以下という説)
>>830により、否定されました。これにて一件落着。🤠

846 :英語職人 ◆azN58fWNzw (ワッチョイ 09e0-0SSA):2023/11/09(木) 23:12:52.85 ID:A7zIBVxt0.net
"This is a good TV show, showing various celebrities and which is very popular among young people."

私には、どうしても、この英文が自然なものに見えないだよね。これ本当に自然な英語? 

自分なら次のように描くよ。
"This is a good TV show featuring various celebrities, which is very popular among young people."

847 :名無しさん@英語勉強中 (ワッチョイ c1d0-/UqF):2023/11/09(木) 23:20:35.81 ID:/fwHXxoq0.net
「彼の年収はわたしの年収の5倍です」を
as ... as を明記して表すなら
His annual income is five times as high as mine.
His annual income is five times as much as mine.
のどちらが一般的な英語でしょうか?

848 :名無しさん@英語勉強中 (ワッチョイ c218-HiwA):2023/11/09(木) 23:39:19.34 ID:M95BcMIo0.net
>>847
highが正しいと書いてあるけど、どうなんだろう。

http://blog.livedoor.jp/eg_daw_jaw/archives/51773798.html

849 :名無しさん@英語勉強中 (ワッチョイ c1d0-/UqF):2023/11/09(木) 23:41:32.16 ID:/fwHXxoq0.net
>>848
ありがとうございます
勉強になりました

850 :名無しさん@英語勉強中 (ワッチョイ c218-HiwA):2023/11/09(木) 23:45:40.98 ID:M95BcMIo0.net
In 2015, a family in the top 1 percent nationally received, on average, 26.3 times as much income as a family in the bottom 99 percent.

muchがincomeを修飾

851 :ブタ耳 (ワッチョイW 6d73-R+yP):2023/11/09(木) 23:53:54.47 ID:n/a9W7Ri0.net
>>844
その英文書いたのは私ですよ。

>>760以降、Antonioが出てくる英文は全部私が考えたものなので、>>837も当然私が書いたものだと理解してもらえると思っていました。

ひょっとして私のレスちゃんと読んでなかったりする?

852 :英語職人 ◆azN58fWNzw (ワッチョイ 09e0-0SSA):2023/11/09(木) 23:57:59.56 ID:A7zIBVxt0.net
>>847
どちらでもいいですよ。同じくらい使われている。

ここを見ると↓muchに異議を唱える人がいないことからも、「"muchは誤り"は間違い」
https://hinative.com/questions/12631968

853 :英語職人 :2023/11/10(金) 00:03:12.33 ID:J36pqK9Z0.net
>>851 ごめん、今回はすべてのレスを読まなくても私の説のほうが正しいことは明らかだったので、私への反論部しか読んでませんでした。

でも、他の人達は、>>837をネイティブの書いた文だと誤解しているみたいですよ😄

854 :名無しさん@英語勉強中 (ワッチョイ c218-HiwA):2023/11/10(金) 04:51:51.79 ID:My1NVmXj0.net
英語職人氏は
CamGELが挙げている例文
It would be an opportunity to do something [<quite new for me> and <in which I believed much more strongly than in our government's economic policy>].
などをどう考えているのだろ。

855 :名無しさん@英語勉強中 :2023/11/10(金) 05:32:34.13 ID:IqboNzW10.net
something に全部がかかっているよね

believe in….がin which I believeに変わった

856 :英語職人 :2023/11/10(金) 06:51:15.67 ID:J36pqK9Z0.net
>>854
コンマがなかったら、<quite new for me>も<in which I believed --->も、両方とも形容詞的存在だから、andで結んでも合法だよ。

しかし、非制限用法の関係詞節は、文法上、何かを修飾する存在ではないわけ。文法上、形容詞節とみなすと矛盾が生じる。だから、現在分詞とandで結びつけるのは、間違いとは言えないまでも、とても不自然で、ほとんど誰も使わないというお話さ 🤠

"This is a good TV show, showing various celebrities and which is very popular among young people."は、まともなネイティブなら、"This is a good TV show featuring various celebrities, which is very popular among young people."と書くでしょう。

857 :英語職人 :2023/11/10(金) 07:05:11.41 ID:J36pqK9Z0.net
そして、This is a good TV show featuring various celebrities, which is very popular among young people.は、This is a good TV show featuring various celebrities, and it is very popular among young people.の意味なのである。

非制限用法の関係詞節は、基本的に接続詞の意味を内在しているので、接続詞の意味を内在していない現在分詞と「同等の立場である」とはいいがたい。だから、等位接続詞のandの使用は、「現在分詞」と「非制限用法関係詞節」では、はばかれるのだ。

私の意見に反対の人は、「現在分詞」と「非制限用法関係詞節」をandで結んだネイティブによる使用例がどれだけの頻度で見られるのか調べてほしい。

858 :英語職人 :2023/11/10(金) 07:26:52.51 ID:J36pqK9Z0.net
>>857の訂正
非制限用法の関係詞節は、基本的に接続詞の意味を内在しているので非修飾語である。だから、修飾語として働いている現在分詞と「同等の立場である」とはいいがたい。だから、等位接続詞のandの使用は、「現在分詞」と「非制限用法関係詞節」では、はばかれるのだ。

私の意見に反対の人は、「現在分詞」と「非制限用法関係詞節」をandで結んだネイティブによる使用例がどれだけの頻度で見られるのか調べてほしい。

859 :英語職人 :2023/11/10(金) 07:40:17.19 ID:J36pqK9Z0.net
ただ、「The TV show "XXX", showing various celebrities and which is very popular among young people, is a lot of fun for me.」のshowingをandの意味を持った付帯状況の分詞構文だと主張することも不可能ではない。だから、完全に間違いだとは言えないのだ。しかし、多くのネイティブはこの文を見て不自然さを感じることは確かである。

860 :名無しさん@英語勉強中 (ワッチョイ c291-HiwA):2023/11/10(金) 08:56:06.96 ID:My1NVmXj0.net
英語職人氏の言うことは説得的だ。
確かに関係節が制限用か非制限用法かで話は違ってくるなあ。
関係詞節を非制限用法で挿入して、主語の説明をすることはできるけど
現在分詞の形容詞的用法でそのようなことはできない。

Migrant farm workers in the United States — the majority of whom are Latino — may similarly end up at the center of the next Covid-19 hot spots.
は可能だが
Migrant farm workers in the United States — too often living in comparably crowded, unsanitary quarters— may similarly end up at the center of the next Covid-19 hot spots.
だと、現在分詞句は分詞構文と解するしかなく、主語の説明をする挿入された関係詞節とは同等な関係ではない。

挿入の場合は
It would be an opportunity to do something [<quite new for me> and <in which I believed much more strongly than in our government's economic policy>].
のような例とは違うわけだ。

861 :名無しさん@英語勉強中 (ワッチョイ c291-HiwA):2023/11/10(金) 09:07:59.57 ID:My1NVmXj0.net
>>859
>「The TV show "XXX", showing various celebrities and which is very popular among young people, is a lot of fun for me.」のshowingをandの意味を持った付帯状況の分詞構文だと主張することも不可能ではない。
>だから、完全に間違いだとは言えないのだ。

それには異議がある。
付帯状況の分詞構文と、主語の説明をする挿入された関係詞節とは同等の関係にない
よって、私は「完全な間違い」だと主張しておく。

862 :名無しさん@英語勉強中 :2023/11/10(金) 09:51:58.06 ID:MGJMMXF70.net
名詞を後置修飾する分詞の非制限用法(カンマ入り)もあるけどね

863 :ブタ耳 :2023/11/10(金) 10:10:11.56 ID:8FkGm+Ij0.net
[<現在分詞の形容詞句> and <関係代名詞節>] が、名詞の後に挿入されている実例を見つけるのは確かに難しい。
めっちゃ頑張れば見つかるのかもしれんが、そんなに頑張る気にもなれない。
それならとりあえず作ってみよう、というわけでAIに作らせてみる。

とりあえずこういうpromptを考えた。

“In my early years of school, I appreciated the science fair, showcasing innovative projects and in which ( )"

Fill in the blank with appropriate words to complete this sentence.
Show me five sentences completed in this way.

showcasingの意味上の主語は" I "ではなく" the science fair "なので、これは分詞構文とは考えられない。
(続く)

864 :ブタ耳 :2023/11/10(金) 10:16:21.81 ID:8FkGm+Ij0.net
出力結果はこちら
https://www.phind.com/search?cache=nskmdtrvoh1z6iaveom0rw3j

生成された文だけ転記する。

1. "In my early years of school, I appreciated the science fair, showcasing innovative projects and in which I developed critical thinking, presentation and public speaking, research and writing skills."

2. "… in which I was able to explore the forces of friction that affect slippery slopes and sticking surfaces."

3. "… in which I learned about the role planktons play to sustain marine life.

4. "… in which I investigated the effect of salt concentration on the freezing point of water."

5. "… in which I experimented on how to move water with the Archimedes screw pump”

865 :ブタ耳 :2023/11/10(金) 10:30:47.18 ID:8FkGm+Ij0.net
これらの文について、
"Do these sentences sound naural?"
(naural は natural のtypo)
と尋ねると、次のような回答が得られた。
(ただし、こういう判断に関してはAIはあまり当てにならないので注意)

Yes, the sentences you provided are grammatically correct, and they sound natural in the context of recalling experiences from a science fair. They are written in past tense, which is appropriate for sharing past experiences. The structure of the sentences is coherent and each sentence is complete, having a subject ("I") and a verb (e.g. "developed", "was able to explore", "learned", "investigated", "experimented").

この後、個々の文についての分析が続くのだが省略。
省略された部分を確認したい人は>>864のリンク先を見てください。

で、私個人としては、生成されたこれらの文を自然だと感じる。とは言えそれは、これまでの議論の流れからして、そのように感じるバイアスがかかっているせいかもしれない。

この件に関してニュートラルな立場にいる人、また私とは逆のバイアスが(強烈に)かかっている人の感想も聞いてみたい。

866 :🤭英ちゃん🫢 :2023/11/10(金) 10:37:17.02 ID:kvlxiuEF0.net
第二のエロリストじゃん

867 :ブタ耳 :2023/11/10(金) 10:48:34.17 ID:8FkGm+Ij0.net
>>866
外野は引っ込んでろ

868 :🤭英ちゃん🫢 :2023/11/10(金) 11:04:07.85 ID:kvlxiuEF0.net
最終的な判断を人間に頼るなら、新たに問題を増やしただけで何の解決もしてない
いい加減に自分のレベルの低さに気づけ

869 :🤭英ちゃん🫢 :2023/11/10(金) 11:07:22.84 ID:kvlxiuEF0.net
最終的な判断をしてくれる人間をコマに持ってないのがブタ耳じゃん
あってるかどうかはわからないけど、AI回答貼ってみましたって、誰得なわけ?

870 :ブタ耳 :2023/11/10(金) 11:10:08.37 ID:8FkGm+Ij0.net
>>868
正しいAIの活用法だど思うが何か?

お前は安全地帯から石を投げるばかり。
>>864の文をどう判断するのか自分の見解を述べてみろ。この卑怯者めがッ

871 :ブタ耳 :2023/11/10(金) 11:12:37.27 ID:8FkGm+Ij0.net
おっと思わずなまっちまっだど。

872 :🤭英ちゃん🫢 :2023/11/10(金) 11:14:21.02 ID:kvlxiuEF0.net
英語に質問回答に安全地帯もへったくれもない

分からなかったら黙ってる、変だったらツッコミ入れる、でいいじゃん

つーか、AIに問題丸投げしてる人間がよく言うわ😂

873 :ブタ耳 :2023/11/10(金) 11:17:53.32 ID:8FkGm+Ij0.net
>>864の文が自然かどうかあなたには判断がつかない、ということでよろしいか?

874 :🤭英ちゃん🫢 :2023/11/10(金) 11:23:20.66 ID:kvlxiuEF0.net
AI絵師(笑)が「お前も自分で絵を描いてみろ!」って説教してる感じ🤭

>>864
自然かどうかは、ネイティブかそれに準ずるレベルの人間が判断すべきであって、中間言語の段階の人間が軽々しく判断すべきでない。エロリストがネイティブイタコやって、何度も間違えてる

てか、どこかのフォーラム行って聞いてくりゃいいじゃん

875 :🤭英ちゃん🫢 :2023/11/10(金) 11:30:25.15 ID:kvlxiuEF0.net
私、なんか間違ったこと言ってますか?
AIにお伺い立てますか?

876 :The OED Loves Me Not :2023/11/10(金) 11:33:44.69 ID:rh2Smrx30.net
(1) 方言は大好きなので、大いに訛りを披露してください。日本全国の方言に触れたいです。

(2) comma のあとは必ず非制限用法になるとか、あるいは comma があればその前の
Parting は分詞構文であるはずだとか、そういうふうに comma のあるなしで大々的に意味が
違うはずだという固定観念に囚われ過ぎている人がいるように思いますが、
英語ネイティブはその辺は日本人よりもはるかに緩い判断をしているような気がします。

comma は単に意味の小さな切れ目を示しているだけで、たとえば一つのセンテンスに
5つくらいの comma をつけようと思えばつけられるけれども、片っ端からすべて
comma をつけていくと煩雑になるので、たいていの文では comma の数を
1つとか2つくらいに抑えているだけなんだろうと思います。

今回の問題においても、comma があるから非制限用法であるはずだ。だから
これこれはこうなるはずだ、と思わないで、comma は単に長い phrase が続くと
読んでいると苦しくなるし息を継ぐ場所もわかりにくなるから、便宜上、つけているだけだ
と考えたら、もっと柔軟に英文が読めると思うのです。

そういう観点から読んでいたら、>>864 の英文はすべて、私には自然に感じられます。
仮に自然でないとしたら、それがどうしたんだ、と私は思います。おそらくはこういう英文を
書いている英語ネイティブはたくさんいるだろうと思います。

prescriptive grammar 二ばかり拘泥しないで、descriptive grammar を
見つめていきたいと思っています。つまり、文法学者や教壇の教師たちの大多数が
どのように言っているかなんてどうでもよくて、何割くらいの英語ネイティブがどのように
書いていて、どういう意味で使っているように見えるか、それだけが少なくとも私にとっては
重要です。

877 :ブタ耳 :2023/11/10(金) 11:38:54.48 ID:8FkGm+Ij0.net
>>874
> 自然かどうかは、ネイティブかそれに準ずるレベルの人間が判断すべきであって、中間言語の段階の人間が軽々しく判断すべきでない。

それは確かにそうだ。学門的に厳密な結果を得ようとするならば。

では、「自然かどうか」ではなく、「文法的に正しいかどうか」を尋ねてみよう。

「文法的に正しいかどうか」という問まで否定するとしたら、このスレの存在自体無意味になる。

★[<現在分詞の形容詞句> and <関係代名詞節>] が、名詞の後にコンマ付きで挿入されることがありえるのか?
*例えば、>>864の文は文法的に正しいのか?

こういう問いに対しても、あなたは何の回答も、何の感想も出せない、という理解でよろしいか?

878 :The OED Loves Me Not :2023/11/10(金) 11:42:23.00 ID:rh2Smrx30.net
とはいえ、中高生や浪人生に英語を教えている人たちは、
その学生たちには prescriptive grammar を教える必要があり、
学生たちやその親たちにとっては、実際にはどのように英語が使われているかよりも、
ともかくマークシート方式の試験で高得点を取れさえすればそれでよいわけですから、
まずは主流の文法学者たちがどう言っているかだけが問題なんだ、という立場を
取らざるを得ないんだろうと想像しています。

第一、そんな細かい問題を無視していても、英検1級や TOEIC 満点や、
企業におけるビジネス英語を書いたり読んだり聴いたりしゃべったりして
金を稼ぐ際には、まるで問題がないということになりますので、大多数の人は
そういう問題には無頓着でしょう。

879 :🤭英ちゃん🫢 :2023/11/10(金) 11:50:41.15 ID:kvlxiuEF0.net
>>877
コミュニカティブ重視なら、意味は通じるからいいんじゃないの?
よろしくなければ、ネイティブが訂正してくれるでしょ

言語学的、もしくはスタイル的な精確さを追求するならネイティブの判断を入れる
一見精確さを追求してるように見えて、全部AI任せだからダメなの

880 :🤭英ちゃん🫢 :2023/11/10(金) 12:00:04.13 ID:kvlxiuEF0.net
というか、英語職人と何十レスも使ってバトルできる人は、英語職人並にヤバいよ
自覚したほうがいいよ

レッスンがあるので、以上です🤭

881 :ブタ耳 :2023/11/10(金) 12:01:07.07 ID:8FkGm+Ij0.net
>>879
あなた、この一連の議論の流れ把握できてないでしょ?
把握できていたら、私が「全部AI任せだ」などという言葉は出てこないはずだと思うが?

邪魔だから黙っててくれませんか?

以降このスレでのあなたへは返信は控えて、無駄にスレを消費しないようにしたいと思います。あしからず。

882 :🤭英ちゃん🫢 :2023/11/10(金) 12:10:01.22 ID:kvlxiuEF0.net
ブタ耳さんのAIコピペと職人さんのバトルは無価値なので、基本的に読んでません
お互いさまだから無視してもらって結構です🤭

883 :名無しさん@英語勉強中 :2023/11/10(金) 13:55:32.41 ID:My1NVmXj0.net
>>876
たしかにカンマの有無だけで、制限か非制限かを決めることは危険だと思います。
しかし、関係詞節がカンマで挟まれて主語のと述部に挿入されているときは
非制限用法でしょう。
主語を制限するのではなく、説明している。
それ以外の用例には出会ったことがありません。
当該英文の場合は、コンマではなくダッシュです。
なおさら、制限用法は考えにくい。追加説明をしているのです。
更にいうなら、この関係詞節は、the majority of whomで始まります。
数量代名詞+関係代名詞で始まる関係詞節が制限用法になることはありませんので、この場合も違うでしょう。

ただ、
>名詞を後置修飾する分詞の非制限用法(カンマ入り)もあるけどね
という意見がありました。
主語の後にコンマ等で囲まれて挿入された現在分詞句が、分詞構文ではなく、主語の補足説明として用いられることがあるのなら、
再考する必要があります。

たとえば
Glasgow Subway in Scotland, opening in December 1896, is the world’s third oldest metro system.
のような例は、分詞構文だと思っていたのだけど、違うのだろうか。

884 :名無しさん@英語勉強中 :2023/11/10(金) 14:05:54.41 ID:My1NVmXj0.net
あ、でも
Tom, standing over ther, is my best friend.
とか普通にありそうだな。

すみません。私の主張は全面的に撤回します。

885 :名無しさん@英語勉強中 :2023/11/10(金) 14:07:02.78 ID:My1NVmXj0.net
over there の間違いでした。

886 :名無しさん@英語勉強中 :2023/11/10(金) 14:23:15.82 ID:My1NVmXj0.net
結論
後置修飾の分詞節や形容詞句は「縮約関係詞節」だとする説を採用し
制限用法か非制限用法かを問わず
関係詞節と分詞節(分詞句)及び形容詞句が並列で結ばれることは正しい。
ただし、関係詞節が前置する場合は、分詞節や形容詞句は関係詞節の述部だと解されるので
避けられるはず。

887 :名無しさん@英語勉強中 :2023/11/10(金) 14:32:57.17 ID:My1NVmXj0.net
訂正
be動詞が述語動詞の関係詞節が前置する場合は、
分詞節や形容詞句は関係詞節内の述部だと誤解されるので
賢明な書き手なら避けるはず。

888 :ブタ耳 :2023/11/10(金) 14:40:45.82 ID:8FkGm+Ij0.net
>>886-887
同意します。

「結論」という出だしで始まると、なぜか生理的に反発したくなるのですが。

889 :名無しさん@英語勉強中 :2023/11/10(金) 14:58:40.80 ID:HpoMnz5sd.net
the earth is planet to have ocean.
このto V O ってどういう用法ですか?

890 :The OED Loves Me Not :2023/11/10(金) 15:14:22.96 ID:rh2Smrx30.net
>>889
>>the earth is planet to have ocean.

それはデタラメの英語ですが、それを書いた人はどうせ
The earth is a planet with oceans on it.
というような意味で書いたのでしょう。

デタラメの英文を見て、その厳密な文法構造なんて
云々しても仕方がありません。

This a is pen pineapple strawberry study at.
この英文の文法構造なんて、、解説できますか?
それと同じです。

891 :The OED Loves Me Not :2023/11/10(金) 15:18:27.82 ID:rh2Smrx30.net
確かに、「結論」とか「まとめ」なんてことを
何度も強調されると、反発したくなりますね。

「私自身としての中間結論」
「私自身の話の、私個人としてのまとめ」
ならいいですが、
「結論」とか「まとめ」と書かれると、
これが最終的勝決定的なもので、
それ以外の考えは許さねえぜ、とでも
言っているかのように見えてしまいます。

892 :名無しさん@英語勉強中 :2023/11/10(金) 15:26:12.98 ID:My1NVmXj0.net
私は紆余曲折して前に述べた主張を撤回しているので
現時点での私の主張という意味で書きました。
反論は許さないという意味合いはありません。

893 :名無しさん@英語勉強中 :2023/11/10(金) 15:32:08.12 ID:3cOIVG4q0.net
>>892
テテンさんへ。

非定形節による非制限後置修飾について
自分なりにまとめてみたので
もし興味があればお読みください。

https://twitter.com/leviathan_99999/status/1722860897579020351
https://twitter.com/leviathan_99999/status/1722861228744434001
 
(deleted an unsolicited ad)

894 :名無しさん@英語勉強中 :2023/11/10(金) 15:37:57.55 ID:My1NVmXj0.net
>>893
ありがとうございます。
とても参考になります。
例示も的確で素晴らしいですね。

895 :ブタ耳 :2023/11/10(金) 15:40:42.40 ID:8FkGm+Ij0.net
「結論」というタイトルを使用するのも、本来まったく問題ないはずなのです。

しかしどういうわけか、「結論」とか「まとめ」とか書かれると気分が悪くなるんですよね。これに顔文字が加わるとさらに気分が悪くなります。

何故なんでしょう。不思議ですね(棒

896 :ブタ耳 :2023/11/10(金) 15:44:01.64 ID:8FkGm+Ij0.net
>>893
す、素晴らしい!

897 :The OED Loves Me Not :2023/11/10(金) 15:49:11.20 ID:rh2Smrx30.net
>>892
HiWA さんは、テテンさんだったのですね。それならそうと
「テテン」と書いてくださっていれば、私は前々からテテンさんの
性格をある程度はよく知っているつもりなので、変な誤解はしませんでしたよ。

みんな謙虚な態度を貫くつもりで名無しであり続けているのかもしれませんが、
何も知らずにいろんな名無しさんの文章を読んでいると、まるで
無数の人たちが互いに攻撃し合っているように見えます。そして
実際に罵倒したり嫌味を言ったりする人もいます。それに対して
こちらが反論しても、他の無数の名無しさんたちと同じく、
その人は謝罪もせずに名無しの海の中で消えて行き、二度と現れません。
そんなことがいつも続くので、いつまたどこの名無しさんが攻撃をかけてくるのかと
ひやひやしながらこの掲示板を読むことになるのです。

だから言っているのです。できることならコテハンを名乗ってほしいのです。
そうすれば、昔からのそれぞれの人の性格がある程度はわかるので、
「この人はこう言っているけど、これは決して悪気て言っているのではないな」と
大目に見たり、言葉足らずの説明の深い意味合いを過去のその人の書き込みと
照らし合わせて、読者は読み込むこともできます。

とはいえ、テテンさんの名無しでありたいという意思も尊重したいので、決して無理強いするつもりはありません。
しかし傍から読んでいると、無数の名無しの人たちの文章を読んでいると、いろんな人たちの不可解な
思惑が絡み合っていて、実に疲れることは確かです。

898 :ブタ耳 :2023/11/10(金) 15:55:55.07 ID:8FkGm+Ij0.net
■□■ まとめ ■□■

いやー、ここまで面白かったなー。
実に実に勉強になったわー。

899 :The OED Loves Me Not :2023/11/10(金) 15:58:44.97 ID:rh2Smrx30.net
>>893
ブタ耳さんのいみじくも仰せしが如く、
藤田五郎さんは素晴らしいですね。
ただ私は文法用語に慣れていないので、
ついついチンプンカンプンになってしまいますが。

それはともかく、TOEIC を受け続け、満点を
獲得し続けておられるのですね。

900 :名無しさん@英語勉強中 :2023/11/10(金) 16:01:24.60 ID:My1NVmXj0.net
>>897
いや、私は結構攻撃的になりますよw

5chは卒業すると言った手前、恥ずかしくて
あえて名乗ることは控えます。
https://lavender.5ch.net/test/read.cgi/english/1688264526/764
で、一度名乗りましたがw

就職決まり、大学の方も落ち着いてきて、暇なので
また少し参加させてもらっています。
でも、前みないにムキになったバトルをするのは止めようと思っています。
(某スレを除く)

今更ですが、>>716 も私です。
このbe動詞の使い方は面白いですね。

901 :名無しさん@英語勉強中 :2023/11/10(金) 16:05:42.78 ID:ZvkLmHjt0.net
>>893
五郎か…いい名前だね

902 :名無しさん@英語勉強中 (ワッチョイ 42e2-KRVY):2023/11/10(金) 16:58:02.90 ID:aB3pdmeX0.net
ほたる…

903 :名無しさん@英語勉強中 :2023/11/10(金) 17:29:20.51 ID:ZvkLmHjt0.net
まだ藤田五郎がTOEIC受けてる途中でしょうが!!!

904 :ブタ耳 :2023/11/10(金) 18:04:58.13 ID:143NYjGZM.net
Eastwoodの「分詞関係詞節」については、美誠社のサイトに丁寧に説明されているようです。
https://www.biseisha.co.jp/lab/qa/57

905 :英語職人 (ワッチョイ 09e0-jfG6):2023/11/10(金) 22:09:56.79 ID:J36pqK9Z0.net
>>865 現在分詞と関係詞をandで結んだ文を作れと命令したら、そのような駄文を作るのは当然でしょ。😄

ChatGPTに聞いてみたよ
============
Q:下記の英文に改善の余地はありますか?
Is there room for improvement in the following English text?

In my early years of school, I appreciated the science fair, ★showcasing innovative projects and in which I developed critical thinking, presentation and public speaking, research and writing skills.

ChatGPT
Certainly! Here's a slightly improved version of your sentence:

"During my early school years, I valued the science fair for its display of innovative projects. Participating in it helped me cultivate critical thinking, presentation and public speaking abilities, as well as honing my research and writing skills."

This version maintains the essence of your original sentence while making it a bit smoother and more concise.
================

見事に、現在分詞と関係詞のandの結合を投げ捨てた文を提示してきたぞ🤠
現在分詞と関係詞のandの結合は、間違いではないけれど、達筆者が使わない言い方なのだよ。まあ、このセンスがわかるのは、このスレで私だけのようだね。ブタ耳ちゃんは、あきらめましょう🤠

906 :英語職人 (ワッチョイ 09e0-jfG6):2023/11/10(金) 22:11:59.57 ID:J36pqK9Z0.net
sikiが「送信した内容が破損しています」エラーになって投稿できなくなったぞ。5ちゃんのやめどきか。

907 :ブタ耳 :2023/11/11(土) 01:39:50.47 ID:nBmSd2kO0.net
>>906
「あきらめましょう」というのは、何をあきらめればいいのでしょう?
>>734の(2)の方が適切だと主張することをあきらめましょう、ということですか?

だとすると、あなた本当に私のレス読んでないんですね。だいぶ前に諦めてますよ。

(1)の方が適切だろうということは、>>816-817のアントニオシリーズで、はたと気づき、
>>833で納得しています。
>>888でも同意していますよ。

何というか、こちらが抱いていた様々な疑問が氷解し、隅々までクリアーに理解し尽くしたような、深い満足感に浸っています。

あなたの見解については、省略についての考え方や、"the majority of whom ~" を名詞節とする説など、まだまだ突っ込みどころは残ってますけど、そんなことはもうどうでもいいや、という心境に至りました。

まずはめでたしめでたし。

908 :名無しさん@英語勉強中 :2023/11/11(土) 04:43:08.34 ID:gwyR4GA20.net
ここで質問することかどうか分からないがグーグル翻訳で日本語を英語に翻訳したんだけどほぼ完ぺきに思える はじめは参考にしておかしなところは自分で手直ししようかと思ったが必要なかった
簡単な文章だからかどうかは知らないが驚いた。 みなさんの評価はどうなのか?
普段、簡単な韓国語や中国語の翻訳に使っていたがおかしな訳が結構多かったので驚いた。この違いは英語の翻訳はレベルが高いということなのか?

909 :名無しさん@英語勉強中 :2023/11/11(土) 04:50:45.74 ID:gwyR4GA20.net
補足
韓国語や中国語は日本語への翻訳

910 :名無しさん@英語勉強中 (ワッチョイ 7ff0-ugN5):2023/11/11(土) 06:09:55.48 ID:l8kFGoJ10.net
>>908
英語の場合、他言語よりは精度が高いのは確か。
また報告文や新聞雑誌などの間違いのない標準的な
文章も十分な精度が得られる。
(従って、多少の間違いがあっても構わない用途、
例えば厳密な法律や契約、重要なビジネス案件に
関わることがない状況での利用、あるいはその文章の
”概要”を把握する目的など限られた利用の場合は
既に十分に有用)

それでも、文章に間違いはなくても込み入った構造
や内容のものは、予想通り・・・間違うことが多い。

また素人が書いたSNSなどの間違いが混じった文章、
誤字や綴りの間違いの多いもの、同音語や多義語、
皮肉や反語、嫌味、婉曲など、その解釈に内容や文脈
の理解が不可欠なケース、あるいは人間でも人により
解釈が分かれるものも当然曖昧な訳文になる。

自分が読んで”手強そうな”文章、”一読しても理解で
きない”文章で実際に試してみて、どういう場合に
間違うのか(又は精度が落ちるのか)、その原因や
傾向を分析してみるとその後の利用に役立つと思う。

911 :英語職人 :2023/11/11(土) 08:02:33.47 ID:2YH7jt3j0.net
やっと設定変えて、sikiで書き込みできるようになったよ・・・

912 :名無しさん@英語勉強中:2023/11/11(土) 10:32:50.18 ID:4Fw3qLcup
国会でいい感じに質問してる優秀な議員を見かけて維新かと思ったら立憲だったわ
立憲は憲法の下の平等も世代による公平もガン無視の児童給付た゛の税金泥棒主張をやめて藤岡隆雄氏みたいなのだけにしろよ
子を育てられるだけの金もないのに孑を産み落としたら遺棄罪で逮捕懲役、日当5千円で塀の中から子に送金させるのが筋た゛ろうに
犯罪者に追い銭とかモラ儿ハザードいい加減にしとけ、風俗で働いて孑育てしてる自立した女なんていくらでもいるだろうにそんな女と
陳情寄生虫女と温室効果ガスに騒音にコロナにとまき散らして他人の権利を強奪して儲けてる空飛ぶ強盗殺人女、クズっふ゜り比較しろ
国が介入するならひとり産み落とすごとに数千万課税して払える見込みがなければ逮捕懲役にしたり親権廃止するくらいた゛ろ
1か月も男と暮らしてた某JKなんて家出だろうに問答無用で逮捕されたんだろな哀れ、その男が充分な金を持ってて責任を持てるなら
毒親と居るより幸せなのは明らかなんだし親を捨てて余裕ある家に行くことを合法化するのが税金泥棒より理に適ってるのは明白
(羽田)ttρs://www.call4.jp/info.php?type=items&id=I0000062 , тtps://haneda-project.jimdofrее.Сom/
(成田)ttps://n-souonhigaisosyoudan.amebaownd.com/
(テロ組織〕ttps://i.imgur.com/hnli1ga.jpеg

913 :名無しさん@英語勉強中 :2023/11/11(土) 09:22:26.99 ID:c/InNh2w0.net
TikTok LiteでPayPayやAmazonギフトなどに交換可能な4000円分のポイントをプレゼント中!
※既存TikTokユーザーの方はTikTokアプリからログアウトしてアンインストールすれば参加できる可能性があります。

1.SIMの入ったスマホ・タブレットを用意する
2.以下のTikTok Litのサイトからアプリをダウンロード(ダウンロードだけでまだ起動しない)
https://lite.tiktok.com/t/ZSNfJPhEu/
3.ダウンロード完了後、もう一度上記アドレスのリンクからアプリを起動
4.アプリ内でTikTok未使用の電話番号かメールアドレスを使用して登録
5.10日間連続チェックインで合計で4000円分のポイントゲット

ポイントはPayPayやAmazonギフト券に交換可能!
家族・友人に紹介したり通常タスクをこなせば更にポイントを追加で獲得できます。

914 :名無しさん@英語勉強中 :2023/11/11(土) 10:08:57.76 ID:m6Ul5zbp0.net
>>913
そんなに貰えるの?

915 :英語職人 :2023/11/11(土) 10:26:25.34 ID:2YH7jt3j0.net
>>905の修正
(×)現在分詞と関係詞のandの結合は、間違いではないけれど、達筆者が使わない言い方なのだよ。
(〇)現在分詞と非制限用法関係詞とのandの結合は(とくにandの前にコンマを置かない用法は)、間違いではないけれど、達筆者の多くは使わない用法なので、一般的であるとは言えない。

916 :名無しさん@英語勉強中 :2023/11/11(土) 11:32:35.59 ID:+MxDIcd901111.net
みなさんにお知らせです

いままで英語に関する質問スレが2つ存在していました
どちらに質問していいか迷うこともありました

[English] -- 英文法・語法に関する質問 Part 27
https://lavender.5ch.net/test/read.cgi/english/1688264526/

スレッドを立てるまでもない質問スレッド Part 368
https://lavender.5ch.net/test/read.cgi/english/1696138309/

そこでこれら2つのスレを1つにまとめることにしました

<英語職人 The OED ブタ耳>の英文法講座 Part 1
https://lavender.5ch.net/test/read.cgi/english/1699500346/

917 :🤭英ちゃん🫢 :2023/11/11(土) 11:52:11.60 ID:fszfT8fb01111.net
>>916
良いスレですね!
外野は黙ってろのブタ耳はぜひあちらで、じっくりコトコト濃厚な議論してください😉👍

918 :🤭英ちゃん🫢 :2023/11/11(土) 11:54:01.02 ID:fszfT8fb01111.net
*ブタ耳さん

自動修正がひどくて、じっくりってタイプしたら「じっくりコトコト」になっちゃいました

919 :名無しさん@英語勉強中 :2023/11/11(土) 12:01:27.17 ID:dmjo4yJE01111.net
>>913
まじか

920 :ブタ耳 :2023/11/11(土) 12:02:05.35 ID:nBmSd2kO01111.net
>>916
そのスレに書き込みしといた。

11:ブタ耳 ◆2wLzAFYxl2 (N_7/0W 9fb4-dq4Y):2023/11/11(土) 11:57:57.06 ID:nBmSd2kO01111
どこぞのアホが勝手に立てたスレはここですか?

私は今後いっさいここには書き込みませんのでご了承ください。
このスレで「ブタ耳」というコテが書き込みをしたとしたら、それは私のコテを騙る偽物です。

921 :ブタ耳 :2023/11/11(土) 12:06:54.46 ID:nBmSd2kO01111.net
>>920
誤解を招きかねないので、補足。

「私は今後いっさいここには書き込みませんのでご了承ください。
このスレで「ブタ耳」というコテが書き込みをしたとしたら、それは私のコテを騙る偽物です。」

までが向こうのクソスレからの引用です。

922 :黒ブタ :2023/11/11(土) 12:21:33.43 ID:nBmSd2kO01111.net
>>916
スレタイに勝手に他人のコテ付けんな、このボケがッ
英文法スレでブタ耳騙ったのもお前やろ?
いっぺん氏んでこいや

923 :名無しさん@英語勉強中 :2023/11/11(土) 15:09:28.87 ID:gwyR4GA201111.net
>>910
サンクス 色々試してみる

924 :英語職人 ◆azN58fWNzw (ポキッー 1fe0-Lcu7):2023/11/11(土) 19:01:22.26 ID:2YH7jt3j01111.net
>>916
誰もが回答できるようなスレじゃないとダメじゃん 🤠  

みなさん、>>916は、いつもの(コテハンをうとましく思う)掲示板アラシなので、無視してください。

925 :英語職人 ◆azN58fWNzw (ポキッー 1fe0-Lcu7):2023/11/11(土) 19:05:57.94 ID:2YH7jt3j01111.net
>>918 「じっくりコトコト」でもちゃんと違和感なく読めるところがおもしろい 😆

926 :名無しさん@英語勉強中 :2023/11/12(日) 11:01:52.96 ID:XJTpo+sw0.net
>>897で書いていることは、
少しでも不愉快な場合名無しは自分の敵と見做してハエのように叩く。だがその中にはかつての行動から友好的と考えられる人が含まれていることも事実。その一部の人は攻撃する対象から外したい。
そうすることで自分の名無しに対する攻撃をより先鋭化し、
名無しの撲滅を効果的に達成できる。
ということかな

927 :名無しさん@英語勉強中 :2023/11/12(日) 11:07:36.38 ID:XJTpo+sw0.net
oed さんはこの小さな板世界のすべての住民に対する聖戦を
宣告し戦う聖戦士

928 :名無しさん@英語勉強中 :2023/11/12(日) 12:52:28.26 ID:hR+YDyL/0.net
『英文解釈教室』にある文章です。
英語が苦手な人の思考傾向についての文章です。

Partly because they would like to have a fixed standard to settle all questions of usage,
they come to believe that one exists and operates, and constantly repeat judgements
and rules without examining the basis for them.

最後のbasis for them を basis of them とする方が違和感がないのですが、ダメなんでしょうか?
「それらの根拠」という意味なのだから、「〜の」という意味でofの方がしっくりきます。
英語の感覚的にはforが妥当なんでしょうか?

929 :名無しさん@英語勉強中 :2023/11/12(日) 14:09:22.20 ID:FhxXOlhF0.net
TEDからの引用です
While it's too easy to say Van Gogh's turbulent genius enabled him to depict turbulaence, it's also difficult to accurately express the rousing beauty of the fact that in a period of intense srffering, van Gogh was some how able to perceive and represent one of the most supremely difficult concepts nature has ever brought before mankind, and to unite his unique mind's eye with the deepest mysteries of movement, fluid and light.
私としてはthat以下の文章が全部the factにかかっていると考えたのですが、上記の日本語字幕ではit's also〜の部分を下記のように訳されてました
彼が激しい苦しみに苛まれながらも真実に潜む美を正しく表現することは至難の業です。ゴッホは〜出来たのです(〜の部分は省略)
しかしそれだとthe fact thatのthatは文法的にどう解釈すればよいのかわかりません
どなたかご教授お願いします

930 :名無しさん@英語勉強中 :2023/11/12(日) 14:26:15.62 ID:miW6oYFO0.net
>>928
なんかムズイ文章だな 学習ならもっとマンガとかカンタンなの読めばいいのにっていつも思うw
で、違和感がないならofで全然いいよ 実際どっちでも通じる
なんでforなのかって言われるとまあそういうもんっていうのが正直だけど、しいて言えばofって言っちゃうと
なんかもう決まり切ってるもんの性質を言ってる感じが超若干する 誰に聞いても間違いない性質みたいな
例えばまだ定まってなくてそれこそ話し合ってこれからそれを決めるみたいな感じがforだと出る気がする
ofはかなりはっきり、forはふわっとみたいな でも確かにこれofって言わないな

931 :名無しさん@英語勉強中 :2023/11/12(日) 14:41:18.76 ID:+b1DaID50.net
>>928

ChatGPT
The difference between "the basis of them" and "the basis for them" lies in the prepositions used ("of" vs. "for") and their subtle impact on the meaning of the phrase.

"The basis of them": This phrase suggests a focus on the origin or source of "them" (in this case, 'judgements and rules'). It emphasizes what these judgements and rules consist of or are made from. The preposition "of" points to a relationship where "them" is integral to "the basis."

"The basis for them": This phrase implies a reason or justification for "them" (again referring to 'judgements and rules'). It's more about the grounds or rationale behind these judgements and rules. The preposition "for" indicates a purpose or reason behind "the basis."

While the overall difference in meaning is subtle, "the basis for them" is more commonly used when discussing reasons or justifications, whereas "the basis of them" tends to focus more on the composition or substance of something.

932 :名無しさん@英語勉強中 :2023/11/12(日) 15:01:20.52 ID:+b1DaID50.net
>>929

ChatGPT
Yes, in the given sentence, the phrase "in a period of intense suffering, van Gogh was somehow able to perceive and represent one of the most supremely difficult concepts nature has ever brought before mankind, and to unite his unique mind's eye with the deepest mysteries of movement, fluid, and light" does indeed modify "the fact."

This is a complex sentence where the initial part ("While it's too easy to say Van Gogh's turbulent genius enabled him to depict turbulence, it's also difficult to accurately express the rousing beauty of the fact") sets up a statement that is further expanded and explained by the latter part. The latter part provides specific details about what this "fact" is: it describes Van Gogh's ability to capture complex natural concepts in his art, especially during a time of personal hardship. The structure of the sentence connects these detailed observations directly to "the fact," showing what aspect of Van Gogh's work the author finds so compelling and difficult to articulate.

933 :🤭英ちゃん🫢 :2023/11/12(日) 16:07:03.13 ID:2EhvXOhh0.net
文法的には the fact は that ...以下だけど、意味的には??だよね
the fact...が the rousing beautyなの?
講演だから、字におこしてみると非論理的なこと言ってこともあるよね

934 :🤭英ちゃん🫢 :2023/11/12(日) 16:12:54.08 ID:2EhvXOhh0.net
ゴッホの渦巻きは彼の内なる世界の表出であって、自然界の光学とか流体力学的ななんかを解き明かすものではない
トンチンカンだね

935 :The OED Loves Me Not :2023/11/12(日) 16:18:37.06 ID:hOPRoJ400.net
>>929
(1) While it's too easy to say Van Gogh's turbulent genius enabled him to depict turbulaence,
---> While S V, という形

(2) it's also difficult to accurately express the rousing beauty of the fact that ...
the fact that S V という構文で、that 以下のすべてが the fact と同格になっています。

(3-a) in a period of intense srffering,

(3-b) van Gogh was somehow able:

(i) to perceive and represent one of the most supremely difficult concepts [that] nature has ever brought before mankind,

and

(ii) to unite his unique mind's eye with the deepest mysteries of movement, fluid and light.

質問者さんの言う通り、この英文は上のような構造となっています。なお、TED に添えられていた和訳は無視したらいいと思います。
英文の構造がしっかり理解していて、その解釈が正しいと確信できるのであれば、他人の和訳なんて無視すればいいのです。
なお、わかりやすくするために思いっきり直訳した結果として、本来の英文の構造とは少し外れた解釈をしているかのように見えることもあるとは思います。

936 :ブタ耳 :2023/11/12(日) 16:32:44.08 ID:VfwpR9y40.net
>>928
長くなりそうなので、別スレでこの件に関してAI (phind) に尋ねてみました。

ChatGPTに質問してみた
https://lavender.5ch.net/test/read.cgi/english/1673484651/617-621

937 :ブタ耳 :2023/11/12(日) 17:02:40.90 ID:VfwpR9y40.net
>>933
私も "beauty of the fact" という言い方には疑問を覚えたので、ちょっと検索してみたところ、こんな引用文を見つけた。

“You and I aren’t going to spend our lives together,” Jesse says.
“But I finally understand that that doesn’t take away any of the beauty of the fact that we were right for each other once.”

Tylor Jenkins Reid

「感動的な事実」くらいの意味合いで、”beatutiful fact" などと言うこともできるのかもしれない。

938 :ブタ耳 :2023/11/12(日) 17:05:46.60 ID:VfwpR9y40.net
beautiful fact の類義語。意外な表現多し。
https://www.powerthesaurus.org/beautiful_fact/synonyms

939 :The OED Loves Me Not :2023/11/12(日) 17:06:09.91 ID:hOPRoJ400.net
>>928
the basis for/of rules
the basis for/of judgments
the basis for/of them

この中で言えば、私なら for を使うと思いました。しかし
Ngram Viewer で見る限りは、of の方がむしろよく使われているように
見えます。とはいえ、for も悪くはなく、of でも for でもどちらでもよい
ように見えます。

以上は、あくまで Ngram Viewer での結果です。他の方法で調べたり、
英語ネイティブたち数人に尋ねたら、まったく違う結果が出るかもしれません。

(1) the basis of/for them
https://books.google.com/ngrams/graph?content=the+basis+of+them%2Cthe+basis+for+them&year_start=1800&year_end=2019&corpus=en-2019&smoothing=3

(2) the basis of/for jdugments
https://books.google.com/ngrams/graph?content=the+basis+for+judgments%2Cthe+basis+of+judgments&year_start=1800&year_end=2019&corpus=en-2019&smoothing=3

(3) the basis of/for rules
https://books.google.com/ngrams/graph?content=the+basis+for+rules%2Cthe+basis+of+rules&year_start=1800&year_end=2019&corpus=en-2019&smoothing=3

940 :The OED Loves Me Not :2023/11/12(日) 17:19:26.99 ID:hOPRoJ400.net
the beauty of the fact (that...) には、私自身は別に違和感を感じません。
単に、★「〜ということ(〜という事実)の見事さ」★ くらいの意味だろうと思っています。
beauty を「美」とか「美しさ」などというふうに解釈するから、何となく違和感を感じてしまう
だけではないかと思ってしまいます。

>>While it's too easy to say Van Gogh's turbulent genius enabled him to depict turbulaence,
it's also difficult to accurately express the rousing beauty of the fact that in a period of intense srffering,
van Gogh was some how able to perceive and represent one of the most supremely difficult concepts
nature has ever brought before mankind, and to unite his unique mind's eye with the deepest mysteries of movement, fluid and light.

この文の大雑把な趣旨を、少し大胆に意訳的に述べると、
「ゴッホがひどく苦しみながらも ●見事に● 〜することができた(見事に達成した)ということを正確に示すことは難しいとも言える」
という感じだと思います。

この上の★印のついた「見事に」は、この the beauty の部分を意訳的に添えたものです。

941 :The OED Loves Me Not :2023/11/12(日) 17:21:49.41 ID:hOPRoJ400.net
第一、口語では beauty という言葉を、きわめて casual な文脈で使うことがありますよね。

That's the beauty of it.
そこがそれのいいところだ。
(ジーニアス英和、6より)

942 :ブタ耳 :2023/11/12(日) 17:29:07.76 ID:VfwpR9y40.net
>>938
このサイト、どこまで信用できるのか疑問。
musical fact なんて言わんだろ。言うとしたら「音楽に関する事実」という意味だろう。

943 :827 :2023/11/12(日) 17:41:28.21 ID:hR+YDyL/0.net
>>827です。
ご回答いただいた皆様、ありがとうございました。
英文での回答は解読が難しかったですが、>>930様の話から言っても、ofではまあ問題ないということはわかりました。
ありがとうございました。

944 :名無しさん@英語勉強中 :2023/11/12(日) 17:45:01.97 ID:hR+YDyL/0.net
もうひとつ質問お願いいたします。

He said that twenty-five years ago in India. 

という文章なのですが、これは、
「彼は”25年前インドにいた”と言った」という解釈と、
「彼は25年前、インドでそれを言った」という解釈の両方が可能で、
どちらが正しいとも言えないと思うのですが、それでいいですか?

945 :943 :2023/11/12(日) 17:47:03.47 ID:hR+YDyL/0.net

間違えました!

ただしくは、
He said that twenty-five years ago when in India.
です。

946 :名無しさん@英語勉強中 :2023/11/12(日) 17:49:36.39 ID:vaqXXfq90.net
「インドにいた」なんてどこにも書いてないでしょ

947 :The OED Loves Me Not :2023/11/12(日) 17:56:39.94 ID:hOPRoJ400.net
>>944-944
>>He said that twenty-five years ago when in India.

二通りの意味なんてありません。ただ一つです。

彼はそれを、25年前にインドにいるときに言った。

それだけ。それ以外の意味はあり得ません。

948 :名無しさん@英語勉強中 (ワッチョイW 1ff0-eA1i):2023/11/12(日) 20:13:41.17 ID:YaFIsy+u0.net
928です
皆様の解説とても参考になりました
私としてはゴッホが神経を病んでいた時だけ何故かこのような乱流の構造を描写できたという科学的な事実がbeautyなのかと考えておりました
そして上記の事柄や人間の視覚と処理に関する内容はこのスレで出した文章の前の段落でしか書かれておりませんでした
この情報不足が皆様の解釈に若干の支障をきたしてしまったであろうことを謝罪します
そして改めてお礼申し上げます

949 :ブタ耳 :2023/11/12(日) 23:44:23.89 ID:VfwpR9y40.net
>>928
> Partly because they would like to have a fixed standard to settle all questions of usage,
> they come to believe that one exists and operates, and constantly repeat judgements
> and rules without examining the basis for them.

的外れ、かつ不遜な言いがかりになるかもしれないが、他の人の感想も聞いてみたい。
この文の、"one exists and operates" の one が、文体的にいかにも軽すぎるような気がする。
そう思いませんか?

950 :The OED Loves Me Not :2023/11/13(月) 01:20:56.77 ID:miKhwT2/0.net
>>928 の英文の文脈を示す少し長い英文を見つけたので、ここに
引用しておきます。

People who lack confidence in their knowledge of good English
would like to have each spelling, pronunciation, meaning,
and grammatical usage decided for them, once an for all,
so that they could know that one is right and another is wrong.
They feel uncertain when choices are clearly permissible.
Partly because they would like to have a fixed standard
to settle all questions of usage, they come to believe
that one exists and operates, and constantly
repeat judgments and rules without examining the basis for them.
https://power-of-awareness.com/entry/2022/09/05/030000

951 :The OED Loves Me Not :2023/11/13(月) 01:23:40.15 ID:miKhwT2/0.net
>>949
one の代わりに、どういう phrase ならよいと思いますか?
私自身は、こういうときに one だけで済ませることができるのだ
ということを知りませんでした。

もし私がこの英文を書いていたら、きっと one じゃなくて
such a standard とでも書いていたと思います。

952 :ブタ耳 :2023/11/13(月) 01:38:05.91 ID:f98FlyiX0.net
>>951
うーん、前後も書き換えて、
they come to believe in the existence of such an absolute standard,
とか?
operates が消えちゃってますけど

953 :名無しさん@英語勉強中 :2023/11/13(月) 07:01:34.21 ID:UG6a9flT0.net
意味をとるのが難しい。
one is right
another is wrong
のoneとかanotherは何を指すか、だけど、
questions of usage だと思う

the basis for themのthemは
judgements and rules

954 :名無しさん@英語勉強中 :2023/11/13(月) 07:06:04.22 ID:UG6a9flT0.net
questionsにrightとかwrong とか言えないから
愛ちゃんfixed standard to settle all questions of usage となるのか。

955 :名無しさん@英語勉強中 :2023/11/13(月) 07:11:41.66 ID:UG6a9flT0.net
aが愛ちゃんに変換されたのは、福原愛スレに書き込みを以前したことがあるから

956 :名無しさん@英語勉強中 :2023/11/13(月) 07:34:38.64 ID:UG6a9flT0.net
fixed standard は信じ込んでいるものなので
one is right another is wrong のone anotherではない。

each spelling, pronunciation, meaning,
and grammatical usage
がone another だな

957 :名無しさん@英語勉強中 :2023/11/13(月) 07:39:46.62 ID:UG6a9flT0.net
peopleから始まるものと

they feel uncertainから始まるものは、
後者が前者を受けたものになってるんだよ。
起承の承。

958 :名無しさん@英語勉強中 :2023/11/13(月) 07:51:50.31 ID:UG6a9flT0.net
have each spelling, pronunciation, meaning,
and grammatical usage decided


settle all questions of usage
のこと

959 :The OED Loves Me Not :2023/11/13(月) 08:23:27.02 ID:miKhwT2/0.net
>>953
>>one is right
another is wrong
のoneとかanotherは何を指すか、だけど、
questions of usage
===========

となると、
One question is right, while another (question) is wrong.
などと言うことになりますが、question に right とか wrong とかいうことは
言えなくて、This answer to the question is right, while that one is wrong.
とかいうふうに言うものではないでしょうか?

だから私には、one is right / another is wrong の one や another は
question を指しているのではないように感じられてしまいます。

となると、あとは standard しかないというふうに、私には感じられるのです。

960 :The OED Loves Me Not :2023/11/13(月) 08:24:32.68 ID:miKhwT2/0.net
>>question に right とか wrong とかいうことは
言えなくて、
====

すみません。>>954 ですでにあなたはそのように言ってましたね。

961 :The OED Loves Me Not :2023/11/13(月) 08:27:12.16 ID:miKhwT2/0.net
>>956
そういうわけで、私にはどうしても

>>each spelling, pronunciation, meaning,
>>and grammatical usage
>>がone another だな

のようには思えず、one / another は
standard を指しているように思えてしまいます。

962 :The OED Loves Me Not :2023/11/13(月) 08:37:14.68 ID:miKhwT2/0.net
>>950 で引用した長い文脈を土台にして、そこに主な語句の意味合いについて
私の解釈を書き加えていきます。私の書き加えた部分には、★などの印をつけます。

> People who lack confidence in their knowledge of good English
> would like to have each spelling, pronunciation, meaning,
> and grammatical usage decided for them ★(= for those people)★, once and for all,
> so that they could know that one ★(= one spelling, one pronunciation, one meaning, etc.)★ is right and another is wrong.
> They feel uncertain when choices are clearly permissible.

> Partly because they would like to have a fixed standard
> to settle all questions of usage, they come to believe
> that one ★(= a standard, such a standard, such a fixed standard?)★ exists and operates, and constantly
> repeat judgments and rules without examining the basis for them ●(=the judgments and rules)●.

> https://power-of-awareness.com/entry/2022/09/05/030000

963 :名無しさん@英語勉強中 (ワッチョイW ffd2-ZjZa):2023/11/13(月) 08:52:44.18 ID:UG6a9flT0.net
one exists and operatesの
ほうなら
oneは
a fixed standard
to settle all questions of usage
でいいんじゃないですか。
>>962さんの書かれておられる通り。

one anotherの方については、
>>954を取り消します。そちらについては>>958
イキで、
each spelling…usageが
all questions of usage
と等価で、それがone anotherのこと

964 :名無しさん@英語勉強中 (ワッチョイW ffd2-ZjZa):2023/11/13(月) 08:55:42.46 ID:UG6a9flT0.net
文の中に起承転結を見るというのは佐々木の英文解釈考に
書いている考え方。
英文が難しいという時にはそういう着眼点がいる場合もあると
いう例に英文解釈教室の上の例題はなってる

965 :名無しさん@英語勉強中 (ワッチョイ 1f6a-vZMC):2023/11/13(月) 12:07:50.46 ID:gbrE4Seh0.net
「〜〜な景色の中で僕たちは」って英訳したらどうなりますか?

景色の中っていうだけですこし意訳っぽい(文語的?比喩的)日本語なので

landscapeやviewやsceneryは仕えなさそうな気がするんですが、、、

966 :ブタ耳 ◆2wLzAFYxl2 (ワッチョイW 9f8f-dq4Y):2023/11/13(月) 12:17:56.70 ID:f98FlyiX0.net
>>962
私の解釈も全く同じです。

>>963
>one anotherの方については、

>each spelling…usageが
>all questions of usage
>と等価で、それがone anotherのこと

ここ、どういうことなのか理解できません。
"one is right and another is wrong"
というのは、至近の例をあげれば、前置詞の用法に関して、
"the basis of them”がrightで、”the basis for them"はwrongだ、と言うようなことでしょう。

967 :ブタ耳 ◆2wLzAFYxl2 (ワッチョイW 9f8f-dq4Y):2023/11/13(月) 12:23:54.08 ID:f98FlyiX0.net
>>965
circumstanceとかsituationが使えそうな気もするけど、そんな断片だけ示されても訳しようがないです。
せめて一文として完結していて、できれば前後の文脈が与えられないと、「景色」をどう解釈しどう訳すかは決められません。

968 :名無しさん@英語勉強中 :2023/11/13(月) 12:46:38.69 ID:MFCInENg0.net
>>965
例えばwe sang in a beautiful settingとかamid beautiful sceneryとかかな

969 :名無しさん@英語勉強中 :2023/11/13(月) 12:59:22.56 ID:tYfiwA010.net
>>965
landscapeやviewやsceneryにも、比喩的はあるよ
landscapeが詩的でいいかも

970 :ブタ耳 :2023/11/13(月) 12:59:23.14 ID:f98FlyiX0.net
こんな文脈を予想。

Looking around, we see unacceptable violence everywhere. In such a tragic situation, we tend to lose the meaning of life.
どっちを向いても酷い暴力ばかりだ。そんな悲惨な景色の中で僕たちは生きる意味を見失いがちになる。

971 :名無しさん@英語勉強中 :2023/11/13(月) 13:02:17.90 ID:tYfiwA010.net
>>969 訂正
>>965
landscapeやviewやsceneryにも、比喩的意味はあるよ
landscapeが詩的でいいかも

972 :名無しさん@英語勉強中 :2023/11/13(月) 13:47:06.15 ID:gbrE4Seh0.net
ども!
でも in a 〜〜landscapeって 英語のgoogleでは一見もヒットしないんですよ〜
使われない表現なのかもって思って。

あと関係ないですが自分OS古くて表示されないんですが
🫠

↑ って 溶けた顔ですか?それとも涙をこらえた顔?

973 :ブタ耳 :2023/11/13(月) 15:21:12.35 ID:f98FlyiX0.net
絶対に嘘
https://sl.bing.net/eI69BqN0iCi

974 :ブタ耳 :2023/11/13(月) 15:27:37.68 ID:f98FlyiX0.net
正解はこれか
https://emojigraph.org/ja/melting-face/

975 :名無しさん@英語勉強中 :2023/11/13(月) 16:09:29.72 ID:jiHYKe+50.net
溶けた顔

976 :名無しさん@英語勉強中 :2023/11/13(月) 16:15:48.15 ID:1W2Vngre0.net
すみません、京大の英語から質問させてください

the cycle of birth,reproduction,and death is fundamentally the same for us
as it was for our distant ancestors.

出生・生殖・死亡のサイクルは、遠い昔の先祖にとっても我々にとっても
基本的に変わっていない。

この文のas以下の構造と訳がいまいち分かりません

itはthe cycle〜deathを指すのは分かりますが、その後のwas for our distant ancestorsと
合わせて訳すと、

出生・生殖・死亡のサイクルは/であった/我々の遠い祖先にとって

になると思いますが、これでは意味が通らないので、何かが省略されていると
考えるのでしょうか?その場合は前にあるsameだと思うのですが…
回答お願いします

977 :名無しさん@英語勉強中 :2023/11/13(月) 16:17:58.75 ID:jiHYKe+50.net
>>976
「そう(≒同じ)」であったように、so

978 :The OED Loves Me Not :2023/11/13(月) 16:26:51.67 ID:miKhwT2/0.net
>>976
>>977 の言う通りだけど、もしも省略をいっさいせずにくどくどとすべてを書くとしたら、次のような意味だということになります。

The cycle of birth, reproduction, and death is fundamentally the same for us
as it was for our distant ancestors.

= ... as it was ◆so◆ for our distant ancestors

= .The cycle of birth, reproduction, and death is fundamentally the same for us ★today★
●as the cycle of birth, reproduction, and death was fundamentally the same for our distant ancestors●.

979 :The OED Loves Me Not (ワッチョイ ff89-Zoup):2023/11/13(月) 16:38:18.69 ID:miKhwT2/0.net
>>972
ただ単に機械的に in a * landscape を検索すればいいのであれば、

in a beautiful landscape
in a magical landscape
in a romantic landscape
in a sensual landscape
in a picturesque landscape
in a dreamy landscape
in a provocative landscape
in a medieval landscape
in a futuristic landscape
in a philosophical landscape
in a literary landscape
in a lovely landscape

などをキーワードにすれば、いくらでも用例が出てきますけど、
もしかしてこれは質問者さんの目的には合わないのでしょうか?

980 :名無しさん@英語勉強中 :2023/11/13(月) 17:28:32.83 ID:gbrE4Seh0.net
あらほんと!
それって普通の日本版googleでアスタリスク検索ででましたか?
米国版?
もしかして自分はgoogle booksで検索してしまったかも
あっちはアスタリスク検索できないかもって思って
スペース入れて。
でも山ほど出るとおもたんですが。出なかったんです。
だから inがちがうのか? 「〜〜な景色の中で」っていう発想の際の語彙として相応しくないのかもって
思ってしまいました。
でもsceneryより多く出てきますよね
in the majestic landscape なんかもちょいちょい
でもmajestic landscapeはたくさんヒットするけど
[in a ]majestic landscape [in athe]majestic landscape
[in the]lovely landscapeが滅多に出ないので
「in the]が不自然なのかなって思ったんです。

981 :975 :2023/11/13(月) 17:42:37.78 ID:1W2Vngre0.net
>>977
ありがとうございます
soが省略可能だとは気づきませんでした

>>977
詳しい回答をありがとうございました
これからは省略箇所だと思ったなら前を見て
比較対象を考えて補うようにします

982 :975 :2023/11/13(月) 17:42:37.74 ID:1W2Vngre0.net
>>977
ありがとうございます
soが省略可能だとは気づきませんでした

>>977
詳しい回答をありがとうございました
これからは省略箇所だと思ったなら前を見て
比較対象を考えて補うようにします

983 :The OED Loves Me Not :2023/11/13(月) 17:45:56.68 ID:miKhwT2/0.net
>>980
私はいつも日本版の Google を使っています。そして
asterisk はなるべく使わないようにしています。
asterisk を使うと、自分の望み通りの検索結果が
出にくくなるので、なるべく asterisk を使わないで、
asterisk の代わりに、例えば今回の例で言えば
自分で適当に形容詞を考え出して
この上のリストのように
in a romantic landscape
in a medieval landscape
など、そのまま検索したのです。


in a ならたくさん出てきますが、
in the [形容詞] landscape なら
検索結果はぐっと減るのは当然でしょう?

984 :The OED Loves Me Not :2023/11/13(月) 17:54:02.49 ID:miKhwT2/0.net
in a 形容詞 landscape
in the 形容詞 landscape

この二つを比べると、出現頻度は a の方がはるかに多いはずだと
思っていましたが、Ngram Viewer で確かめると、the の方が多くなる
傾向にあることがわかりました。間違ったことを言って済みませんでした。

985 :英語職人 :2023/11/13(月) 18:10:28.32 ID:iHkdlhpS0.net
>>977 >>978

ちょっとまった! 田原総一朗風

じゃあ、そのit と soが、それぞれ何を指してるか言ってみて

986 :The OED Loves Me Not :2023/11/13(月) 18:38:40.65 ID:miKhwT2/0.net
>>985
英語職人さんが言いたいのは、as が一種の関係代名詞みたいな
働きもしてるはずだから、as it was [so] の so なんてものは
省略されているわけではないのだ、ということですよね。

それは意識しています。でも、わかりやすくするために
わざと so をつけたのです。そのように考えた方が
わかりやすいと思ったのです。

実際には、英語ネイティブは決してこういうときに
as it was... so なんていうふうに so を補った英文なんて
書きません。

987 :英語職人 :2023/11/13(月) 19:02:43.86 ID:iHkdlhpS0.net
>>986
まあ、OEDさんは、前の回答者につられたという部分もありますので、これ以上は、なにも言いません。🤠

>>977
itとsoのそれぞれ意味するものを挙げてみてください。挙げられないでしょう。
次のスレでこのit wasが何なのかを説明するので、ご参考に。

988 :The OED Loves Me Not :2023/11/13(月) 19:05:36.52 ID:miKhwT2/0.net
>>976
しまった。間違ってました。大間違いでした。

The cycle of birth, reproduction, and death is fundamentally the same for us
as it was for our distant ancestors.

the same as ...
これで「〜と同じ」という意味ですね。だから、もしも省略をまったくしなくてくどい英文を書くとしたら
(普通はネイティブはそんな風にくどくは書かないけど)

The cycle of birth, reproduction, and death is fundamentally the same for us
as the cycle of birth, reproduction, and death was for our distant ancestors.

(今日の私たちにとっての〜の周期は、私たちの遠い祖先にとっての〜の周期と、基本的には同じである。)

何がどのように省略されてるかなんて説明しようとすると、ややこしくなるので、
このまま覚えた方が早いですね。

989 :英語職人 :2023/11/13(月) 19:18:23.73 ID:iHkdlhpS0.net
(京都大学の入試問題文から)
The cycle of birth, reproduction, and death is fundamentally the same for us as it was for our distant ancestors.
この英語は、同じ構文の簡単な文に書き換えると、
The mountain is the same as it was. になります。この「as it was」は、慣用表現で、「昔のそれ」と訳します。

文法的には、The mountain is the same as what it was.のwhatの省略とも考えられます。whatはあってもなくてもいい。

京都大学の問題文は、次のように書いてもよいということです。
The cycle of birth, reproduction, and death is fundamentally the same for us as &#9733;what&#9733; it was for our distant ancestors.

ちなみに、what I was 10 years agoで「10年前の自分」と訳します。

990 :The OED Loves Me Not :2023/11/13(月) 19:20:02.45 ID:miKhwT2/0.net
というわけなので、

(1) The cycle of birth, reproduction, and death is fundamentally the same for us
as it was for our distant ancestors.

これは paraphrase すると

(2) We today have fundamentally the same cycle of birth, reproduction, and death
as the one our distant ancestors had.

(3) We today get born, get reproduced, and die in almost the same cycle as our distant ancestors did.

というふうにでもなるでしょうね。

991 :The OED Loves Me Not :2023/11/13(月) 19:22:04.54 ID:miKhwT2/0.net
>>989
なるほど。素晴らしい説明です。

992 :英語職人 :2023/11/13(月) 19:23:11.14 ID:iHkdlhpS0.net
>>987 訂正

(×)次のスレで
(〇)下記のレスで

993 :英語職人 :2023/11/13(月) 19:25:05.41 ID:iHkdlhpS0.net
>>991 ありがとうございます 🤠

994 :The OED Loves Me Not :2023/11/13(月) 19:32:12.09 ID:miKhwT2/0.net
英語職人さんが説明してくれた as it was に似た台詞として、

He is not as he was (OR used to be).
He is not what he was (OR used to be).

という言葉があって、
「彼は昔の彼ならず」
と訳されたりしていますが、もしかしたら、これは昔からの決まり文句なのかもしれません。

995 :英語職人 :2023/11/13(月) 20:02:49.18 ID:iHkdlhpS0.net
「what I was 10 years ago」という言い方も、asの後ろではwhatを省いてもよい証拠。

① whatがある例
https://www.worldsquash.org/i-thought-i-wouldnt-be-back-subramaniam-makes-winning-return-seven-months-after-accident/#:~:text=I%E2%80%99m%20the%20same-,as%20what%20I%20was,-%5Bbefore%20the%20accident

② whatを省いた例
https://www.youtube.com/watch?v=nJ4WgCJ-6Ug

996 :975 :2023/11/14(火) 00:44:37.80 ID:Oz2knSNx0.net
>>988
>>989
お二方とも詳しく回答していただいてありがとうございます
勉強になりました。もう一度文法と熟語を確認します
また分からない点があったら、ぜひお願いします

997 :名無しさん@英語勉強中 (ワッチョイ 7f74-gzdM):2023/11/15(水) 11:49:13.89 ID:U8+nU11y0.net
このスレもそろそろ終わりだな

998 :名無しさん@英語勉強中 (ワッチョイ 7f74-gzdM):2023/11/15(水) 11:50:18.72 ID:U8+nU11y0.net
何か書き忘れたことがたくさんある

999 :名無しさん@英語勉強中 (ワッチョイ 7f74-gzdM):2023/11/15(水) 11:51:19.53 ID:U8+nU11y0.net
これも書きたいし、あれも書きたい

1000 :名無しさん@英語勉強中 (ワッチョイ 7f74-gzdM):2023/11/15(水) 11:52:13.90 ID:U8+nU11y0.net
それでもスレは終わってしまう

1001 :名無しさん@英語勉強中 (ワッチョイ 7f74-gzdM):2023/11/15(水) 11:53:21.00 ID:U8+nU11y0.net
これで本当に終わりでいいのか

1002 :2ch.net投稿限界:Over 1000 Thread
2ch.netからのレス数が1000に到達しました。

総レス数 1002
498 KB
掲示板に戻る 全部 前100 次100 最新50
read.cgi ver 2014.07.20.01.SC 2014/07/20 D ★